542

Cfa Ques Answer

Embed Size (px)

Citation preview

Page 1: Cfa Ques Answer
Page 2: Cfa Ques Answer

   

Table of Contents Link       

Page 3: Cfa Ques Answer

    

 

ExamWise® Volume 1

CFA 2008 Level I Certification

With Preliminary Reading Assignments The Candidates Question And Answer Workbook For

Chartered Financial Analyst

Authors Jane Vessey, CFA

M. Afdal Pamilih, CFA David Stewart

Published by   

TotalRecall Publications, Inc.  1103 Middlecreek Friendswood, TX 77546 281‐992‐3131 

 

Page 4: Cfa Ques Answer

    

 

TotalRecall Publications, Inc. This Book Sponsored by The Center For Financial Certification, Inc. Portions Copyright © 1999‐2008 by TotalRecall Publications, Inc.. Portions Copyright © 2005‐2006 by Pegasus, Inc.. All rights reserved.  Printed in the United States of America. Except as permitted under the United States Copyright Act of 1976, No part of this publication may be reproduced, stored in a retrieval  system,  or  transmitted  in  any  form  or  by  any  means  electronic  or  mechanical  or  by photocopying, recording, or otherwise without the prior permission of the publisher. The views expressed in this book are solely those of the author, and do not represent the views of any other party or parties.  Printed in United States of America, Canada, and England   Paper Back:  ISBN:   978‐1‐59095‐945‐9     UPC:    6‐43977‐93703‐2 EBook:    ISBN:   978‐1‐59095‐948‐0 

    UPC:    6‐43977‐93763‐6  The sponsoring editor for this book is Bruce Moran and the production supervisor is Corby R. Tate.  This publication  is not  sponsored by,  endorsed by, or  affiliated with CFA  Institute™, CFA®,  and their  logo  are  trademarks  or  registered  trademarks  of CFA  Institute.org  in  the United  States  and certain other countries. All other trademarks are trademarks of their respective owners. Throughout this book, trademarked names are used. Rather than put a trademark symbol after every occurrence of  a  trademarked  name,  we  used  names  in  an  editorial  fashion  only  and  to  the  benefit  of  the trademark owner.  No intention of infringement on trademarks is intended.   The CFA Institute™ does not  endorse, promote or  review  the accuracy of  the products or  services offered by organizations  sponsoring or providing CFA® Exam preparation materials or programs, nor  does  CFA Institute™  verify  pass  rates  or  exam  results  claimed  by  such  organizations.  Any warranty regarding the offered products or services is made solely by TotalRecall Publications, Inc., which are not in any way affiliated with CFA Institute™, the Institute of Chartered Financial Analysts (ICFA),  or  the  Financial Analysts  Federation  (FAF).  If  you  are  dissatisfied with  the  products  or services provided, please contact, TotalRecall Publications, Inc. 1103 Middlecreek, Friendswood, TX 77546 (888‐237‐7849). CFA® is a licensed service mark of CFA Institute™. Used by permission.  Disclaimer Notice: Judgments as to the suitability of the information herein for purchaser’s purposes are  necessarily  the  purchaser’s  responsibility.  TotalRecall  Publications,  Inc.  and  The  Financial Certification  Center,  Inc.  extends  no  warranties,  makes  no  representations,  and  assumes  no responsibility as to the accuracy or suitability of such information for application to the purchaser’s intended purposes or for consequences of its use.  

Page 5: Cfa Ques Answer

    

 

This book is dedicated to our fantastic children Adam and Julia who we love very much.  

Jane Vessey & M. Afdal Pamilih     This Study Guide is dedicated to the widow(er)s and orphans of the “Silent Spring”.  Those who have sacrificed  loved  ones  to  the  obscurity  of  quiet  study  and  endured weekend  sacrifice  above  and beyond  the call of continuing education.   On  the alter of a profession’s highest accreditation  these unsung heroes have sacrificed time with their spouse, shopping with Mom, and pitch and catch with Dad.   These patient  supporters have  endured  tense  attitudes, unfinished  chores,  extra duties,  and received the respect and appreciation of all who have studied throughout the “Silent Spring”. In particular, never ending thanks to: Carol Lee, Mary Elizabeth, Sophia Victoria, David Todd II 

David Stewart

Page 6: Cfa Ques Answer

    

 

ExamWise® Volume 1

CFA 2008 Level I Certification With

Preliminary Reading Assignments

The Candidates Question And Answer Workbook For Chartered Financial Analyst

BY

Authors Jane Vessey, CFA

M. Afdal Pamilih, CFA

David Stewart Jane Vessey Jane Vessey manages a  training company  in  the United Kingdom specializing  in  financial analysis and  investment.    She  is  a  visiting  lecturer  at  Cass  Business  School  teaching  classes  in  asset management and valuation. She also teaches a CFA®  revision course at ISMA (the business school at Reading University) and  is an associate at a  leading London  financial  training company where she teaches  courses  covering  investment management  and  related  topics.    She  has  developed  online training programs for students taking the CFA examinations and teaches CFA courses for UKSIP (the UK Society of Investment Professionals).  Jane  graduated  in Mathematics  from Oxford University, United Kingdom,  and  is  a  CFA  charter holder.  She has some eighteen years experience working in the investment industry,  starting out as an equity analyst before becoming an investment manager.  She was based in London and Tokyo and took responsibility for managing equity portfolios invested in the Japanese and other Asian markets. In 1990, Jane moved to Indonesia and established and ran an investment management operation on behalf of Mees Pierson.   She  took responsibility  for all areas of  the business,  including  investment, operations, marketing and administration.  While in Asia, Jane was involved in providing training to capital market participants and state officials and teaching in courses provided by local universities. 

Page 7: Cfa Ques Answer

    

 

M. Afdal Pamilih Afdal  has  18  yearsʹ  experience  working  in  the  finance  industry.  He  started  his  career  with  J.P. Morgan,  and  then with County NatWest Government  Securities,  in New York  specializing  in  the development  of  quantitative  products  for  foreign  exchange  and  fixed  income  markets.  After returning  to  Indonesia  in 1989 he was  responsible  for  the development of  investment services and subsequently treasury management for leading banks in Jakarta.  Afdal has developed web‐based training programs for the CFA examinations and has wide teaching experience,    including  instructing  at  the  School  of  Management,  University  of  Surrey,  United Kingdom. He  obtained  a  MSc  in  Mathematics  from  the  University  of  Texas  at  Arlington  and  holds  the Chartered Financial Analyst (ʺCFAʺ) qualification.     

David Stewart: David Stewart has extensive experience in venture capital and business structural reorganizations. As president  of  a  private  client  broker  dealer  firm,  he  has  business  valuation  and  project  valuation experience on the venture capital side and portfolio management on the asset management side. His analysis,  commentary,  books,  and  study  guides  have  appeared  in  the  financial  management, securities, and exam prep industries.  David has collaborated with experts  in  the  field  to produce  the 2001  through 2006 editions of  this study guide. His extensive research into the CFA exam program and past exam histories, field work, and  consistent  review of CFA  Institute  information allows him  and his  co‐authors  to deliver high quality and up to date information. 

Page 8: Cfa Ques Answer

    

 

About the Book: ExamWise Volume  1 For CFA Level  I Concept Check Q&A Workbook With Preliminary Reading Assignments is designed to give you plenty of practice questions to test your readiness for the CFA exam.  It offers 400+ concept check questions based 18 exam study sessions  that cover  the Learning Outcome Statements and their associated CFA Assigned Readings. For additional practice, there is an accompanying free download test engine that generates multiple mock exams similar in design and difficulty to the real CFA exam. The questions and explanations have references to the page number in the related Reading and to the related LOS. Use this workbook to test your understanding of the basic concepts covered in the CFA Readings and identify your strengths and weaknesses. Then you can move on to more advanced study materials to sharpen your weakest knowledge areas.  This book is divided into Study Sessions (1 – 18) that cover the 76 Learning Outcome Statements and the associated Assigned Readings. Appendix A  (Exhibits 1 – 4),  is a collection of exhibits and  flow charts  for condensed reference and review,  including examples of accounting statements, puts and calls, PE breakdown, and financial ratios.   

The 18 2008 CFA Level I Study Sessions breakout is as follows: Ethical and Professional Standards

1. Study Session 1: Ethical and Professional Standards 

Investment Tools 2. Study Session 2. Quantitative Methods: Basic Concepts 3. Study Session 3. Quantitative Methods: Application 4. Study Session 4. Economics: Microeconomic Analysis 5. Study Session 5. Economics: Market Structure and Macroeconomic Analysis  6. Study Session 6. Economics: Monetary and Fiscal Economics 7. Study Session 7. Financial Statement Analysis: Introduction  8. Study Session 8. Financial Statement Analysis: Income Statement, Balance Sheet, Cash 9. Study Session 9. Financial Statement Analysis: Inventories, Assets, Taxes, and Debt  10. Study Session 10. Financial Statement Analysis: Techniques, Apps, & International 11. Study Session 11. Corporate Finance 

Portfolio Management 12. Study Session 12. Portfolio Management 

Asset Valuation 13. Study Session 13. Analysis of Equity Investments: Securities Markets 14. Study Session 14. Analysis of Equity Investments: Industry and Company Analysis 15. Study Session 15. Analysis of Fixed Income Investments: Basic Concepts 16. Study Session 16. Analysis of Fixed Income Investments: Analysis and Valuation 17. Study Session 17. Derivative Investments 18. Study Session 18. Alternative Investments Equity Investments: Securities Markets 

Page 9: Cfa Ques Answer

    

 

Online Information: 1. What is CFA Institute

http://www.cfainstitute.org/aboutus/index.html

2. CFA Program: http://www.cfainstitute.org/cfaprogram

3. The Code of Ethics (Full Text) http://www.cfainstitute.org/centre/ethics/code/

The Standards of Professional Conduct

Standard I: Fundamental Responsibilities

Standard II: Relationships with and Responsibilities to the Profession

Standard III: Relationships with and Responsibilities to the Employer

Standard IV: Relationships with and Responsibilities to Clients and Prospects

Standard V: Relationships with and Responsibilities to the Public

4. Why The CFA Designation Matters to You: Individual Investor FAQ

http://www.cfainstitute.org/aboutus/investors/articles/cfamatters.html

5. Soft Dollar Standards http://www.cfainstitute.org/centre/ethics/softdollar/

6. CFA Institute-PPSTM AIMR Performance Presentation Standards

http://www.cfapubs.org/doi/ref/10.2469/faj.v57.n2.2433

7. Global Investment Performance Standards http://www.cfainstitute.org/centre/ips/

Click the picture and link to a free CFA Candidates online glossary.

Page 10: Cfa Ques Answer

    

 

List of Chapters Study Session 01: Ethical and Professional Standards: 14 Study session 02: Quantitative Methods: 34 Study Session 03: Quantitative Methods: 60 Study Session 4: Introduction 84 Study Session 04: Economics: 112 Study Session 05: Economics: 136 Study Session 06: Economics: 160 Study Session 7: Introduction 184 Study Session 07: Financial Statement Analysis: 198 Study Session 08: Financial Statement Analysis: 222 Study Session 09: Financial Statement Analysis: 248 Study Session 10: Financial Statement Analysis: 274 Study Session 11: Corporate Finance: 300 Study Session 12: Portfolio Management: 330 Study Session 13: Equity Investments: 354 Study Session 14: Equity Investments: 378 Study Session 15: Fixed Income Investments: 402 Study Session 16: Fixed Income Investments: 426 Study Session 17: Derivative Investments: 450 Study Session 18: Alternative Investments: 474 Terminology: 501 Appendix A: 503 Download Instructions 526

With the purchase of this book you get FREE Author Collaberation:

Now that you have purchased this product you have access to the Instructors/Authors that authored this book. Send you questions to us and we will answer them for you.   

Page 11: Cfa Ques Answer

Table of Contents   IX 

 

Table of Contents About the Book:........................................................................................................................... VI Online Information:..................................................................................................................... VII

Study Session 01: Ethical and Professional Standards: 14 Reading 1: Code of Ethics and Standards of Professional Conduct ..........................................14 Reading 2: “Guidance” for Standards I–VII.................................................................................14 Reading 3: Introduction to the Global Investment Performance Standards (GIPS) ...................14 Reading 4: Global Investment Performance Standards (GIPS) .................................................14

Study session 02: Quantitative Methods: 34 Basic Concepts ................................................................................................................................34

Reading 5: The Time Value of Money ........................................................................................34 Reading 6: Discounted Cash Flow Applications .........................................................................34 Reading 7: Statistical Concepts and Market Returns .................................................................34 Reading 8: Probability Concepts.................................................................................................34

Study Session 03: Quantitative Methods: 60 Application .......................................................................................................................................60

Reading 9: Common Probability Distributions ............................................................................60 Reading 10: Sampling and Estimation........................................................................................60 Reading 11: Hypothesis Testing .................................................................................................60 Reading 12: Technical Analysis ..................................................................................................60

Study Session 4: Introduction 84 Introductory Readings......................................................................................................................84 Supply, Demand, and the Market Process CH 5.............................................................................84

Introduction..................................................................................................................................84 Consumer choice and the Law of Demand.................................................................................85 Producer choice and the Law of Supply .....................................................................................85 Price changes and demand and supply......................................................................................86 Shifts in demand .........................................................................................................................87 Shifts in supply ............................................................................................................................88 Impact of changes in demand and supply ..................................................................................88

Supply and Demand: Applications and Extensions CH 4................................................................90 Introduction..................................................................................................................................90 Resources ...................................................................................................................................90 Elasticity and the incidence of tax...............................................................................................91

Taking the Nation’s Economic Pulse CH 7......................................................................................92

Page 12: Cfa Ques Answer

X   Table of Contents 

 

Introduction ................................................................................................................................. 92 Gross domestic product.............................................................................................................. 92

Working with Our Basic Aggregate Demand/ Aggregate Supply Model CH 10.............................. 96 Introduction ................................................................................................................................. 96 Aggregate demand ..................................................................................................................... 96

Keynesian Foundations of Modern Macroeconomics CH 11........................................................ 100 Introduction ............................................................................................................................... 100 Keynesian economics............................................................................................................... 100 Introductory Readings Concept Check Questions ............................................................... 104 Introductory Readings Concept Check Answers.................................................................. 108

Study Session 04: Economics: 112 Microeconomic Analysis................................................................................................................ 112

Reading 13: Elasticity ............................................................................................................... 112 Reading 14: Efficiency and Equity............................................................................................ 112 Reading 15: Markets in Action.................................................................................................. 112 Reading 16: Organizing Production.......................................................................................... 112 Reading 17: Output and Costs ................................................................................................. 112

Study Session 05: Economics: 136 Market Structure and Macroeconomic Analysis............................................................................ 136

Reading 18: Perfect Competition.............................................................................................. 136 Reading 19: Monopoly.............................................................................................................. 136 Reading 20: Monopolistic Competition and Oligopoly.............................................................. 136 Reading 21: Demand and Supply in Factor Markets................................................................ 136 Reading 22: Monitoring Cycles, Jobs, and the Price Level ...................................................... 136 Reading 23: Aggregate Supply and Aggregate Demand ......................................................... 136

Study Session 06: Economics: 160 Monetary and Fiscal Economics ................................................................................................... 160

Reading 24: Money, Banks, and the Federal Reserve............................................................. 160 Reading 25: Money, Interest, Real GDP, and the Price Level ................................................. 160 Reading 26: Inflation................................................................................................................. 160 Reading 27: Fiscal Policy ......................................................................................................... 160 Reading 28: Monetary Policy.................................................................................................... 160

Study Session 7: Introduction 184 Introductory Readings ................................................................................................................... 184 Measuring Business Income ......................................................................................................... 184

Introduction ............................................................................................................................... 184

Page 13: Cfa Ques Answer

Table of Contents   XI 

 

Accounting methods..................................................................................................................184 Financial Reporting and Analysis ..................................................................................................185

Introduction................................................................................................................................185 Balance Sheet ...........................................................................................................................185 Income statement......................................................................................................................186

Inventories .....................................................................................................................................187 Introduction................................................................................................................................187 Inventory....................................................................................................................................187 Inventory cost ............................................................................................................................187 Effect of inventory accounting method......................................................................................188

Current Liabilities and the Time Value of Money...........................................................................190 Introduction................................................................................................................................190 Liabilities....................................................................................................................................190

Contributed Capital ........................................................................................................................191 Introduction................................................................................................................................191 Contributed capital ....................................................................................................................191 Accounting for dividends ...........................................................................................................191 Common stock ..........................................................................................................................191 Preferred stock..........................................................................................................................191 Stock issuance ..........................................................................................................................192 Treasury stock...........................................................................................................................192

The Corporate Income Statement and the Statement of Stockholders’ Equity .............................192 Introduction................................................................................................................................192 Retained earnings .....................................................................................................................192 Accounting for stock dividends and stock splits........................................................................192 Introduction Concept Check Questions................................................................................193 Introduction Concept Check Answers..................................................................................195

Study Session 07: Financial Statement Analysis: 198 An Introduction...............................................................................................................................198

Reading 29: Financial Statement Analysis: An Introduction .....................................................198 Reading 30: Financial Reporting Mechanics ............................................................................198 Reading 31: Financial Reporting Standards .............................................................................198

Study Session 08: Financial Statement Analysis: 222 The Income Statement, Balance Sheet, and Cash Flow Statement .............................................222

Reading 32: Understanding the Income Statement..................................................................222 Reading 33: Understanding the Balance Sheet........................................................................222

Page 14: Cfa Ques Answer

XII   Table of Contents 

 

Reading 34: Understanding the Cash Flow Statement ............................................................ 222 Study Session 09: Financial Statement Analysis: 248

Inventories, Long-Term Assets, Deferred Taxes, and On- and Off-Balance Sheet Debt ............. 248 Reading 35: Analysis of Inventories ......................................................................................... 248 Reading 36: Analysis of Long-Lived Assets: ............................................................................ 248 Part I—The Capitalization Decision.......................................................................................... 248 Reading 37: Analysis of Long-Lived Assets: ............................................................................ 248 Part II—Analysis of Depreciation and Impairment.................................................................... 248 Reading 38: Analysis of Income Taxes .................................................................................... 248 Reading 39: Analysis of Financing Liabilities ........................................................................... 248 Reading 40: Leases and Off-Balance-Sheet Debt ................................................................... 248

Study Session 10: Financial Statement Analysis: 274 Techniques, Applications, and International Standards Convergence ......................................... 274

Reading 41: Financial Analysis Techniques............................................................................. 274 Reading 42: Financial Statement Analysis: Applications ......................................................... 274 Reading 43: International Standards Convergence ................................................................. 274

Study Session 11: Corporate Finance: 300 Reading 44: Capital Budgeting................................................................................................. 300 Reading 45: Cost of Capital...................................................................................................... 300 Reading 46: Working Capital Management.............................................................................. 300 Reading 47: Financial Statement Analysis ............................................................................... 300 Reading 48: The Corporate Governance of Listed Companies: .............................................. 300 A Manual for Investors.............................................................................................................. 300

Study Session 12: Portfolio Management: 330 Reading 49: The Asset Allocation Decision.............................................................................. 330 Reading 50: An Introduction to Portfolio Management............................................................. 330 Reading 51: An Introduction to Asset Pricing Models .............................................................. 330

Study Session 13: Equity Investments: 354 Securities Markets ......................................................................................................................... 354

Reading 52: Organization and Functioning of Securities Markets ........................................... 354 Reading 53: Security-Market Indexes ...................................................................................... 354 Reading 54: Efficient Capital Markets ...................................................................................... 354 Reading 55: Market Efficiency and Anomalies......................................................................... 354

Study Session 14: Equity Investments: 378 Industry and Company Analysis.................................................................................................... 378

Reading 56: An Introduction to Security Valuation: Part I ........................................................ 378

Page 15: Cfa Ques Answer

Table of Contents   XIII 

 

Reading 57: Industry Analysis...................................................................................................378 Reading 58: Equity: Concepts and Techniques........................................................................378 Reading 59: Company Analysis and Stock Valuation...............................................................378 Reading 60: An Introduction to Security Valuation: Part II........................................................378 Reading 61: Introduction to Price Multiples ..............................................................................378

Study Session 15: Fixed Income Investments: 402 Basic Concepts ..............................................................................................................................402

Reading 62: Features of Debt Securities ..................................................................................402 Reading 63: Risks Associated with Investing in Bonds ............................................................402 Reading 64: Overview of Bond Sectors and Instruments .........................................................402 Reading 65: Understanding Yield Spreads...............................................................................402 Reading 66: Monetary Policy in an Environment of Global Financial Markets .........................402

Study Session 16: Fixed Income Investments: 426 Analysis and Valuation...................................................................................................................426

Reading 67: Introduction to the Valuation of Debt Securities ...................................................426 Reading 68: Yield Measures, Spot Rates, and Forward Rates ................................................426 Reading 69: Introduction to the Measurement of Interest Rate Risk........................................426

Study Session 17: Derivative Investments: 450 Reading 70: Derivative Markets and Instruments .....................................................................450 Reading 71: Forward Markets and Contracts ...........................................................................450 Reading 72: Futures Markets and Contracts ............................................................................450 Reading 73: Option Markets and Contracts..............................................................................450 Reading 74: Swap Markets and Contracts ...............................................................................450 Reading 75: Risk Management Applications of Option Strategies ...........................................450

Study Session 18: Alternative Investments: 474 Reading 76: Alternative Investments ........................................................................................474

Terminology: 501 Appendix A: 503 Download Instructions 526

 

Page 16: Cfa Ques Answer

14   Study Session 01:  

 

Study Session 01: Ethical and Professional Standards:  

The  readings  in  this  study  session  present  a  framework  for  ethical  conduct  in  the  investment profession by focusing on the CFA Institute Code of Ethics and Standards of Professional Conduct as well as the Global Investment Performance Standards (GIPS®). 

The principles and guidance presented in the CFA Institute Standards of Practice Handbook (SOPH) form  the  basis  for  the CFA  Institute  self‐regulatory  program  to maintain  the  highest  professional standards  among  investment  practitioners.  “Guidance”  in  the  SOPH  addresses  the  practical application of the Code of Ethics and Standards of Professional Conduct. The guidance reviews the purpose  and  scope  of  each  standard,  presents  recommended  procedures  for  compliance,  and provides examples of the standard in practice. 

The Global  Investment Performance Standards  (GIPS)  facilitate  efficient  comparison of  investment performance  across  investment managers  and  country  borders  by  prescribing methodology  and standards  that  are  consistent  with  a  clear  and  honest  presentation  of  returns.  Having  a  global standard for reporting investment performance minimizes the potential for ambiguous or misleading presentations. 

 

Reading 1: Code of Ethics and Standards of Professional Conduct 

Reading 2: “Guidance” for Standards I–VII 

Reading 3: Introduction to the Global Investment Performance Standards (GIPS) 

Reading 4: Global Investment Performance Standards (GIPS) 

 

 

Page 17: Cfa Ques Answer

Ethical and Professional Standards   1 

 

1.  Jason Vasco, CFA, is the director for a major Talia‐owned investment management firm branch in Rasen. Talia is known as the world’s centre of  investment management with securities  laws stricter than  the CFA  Institute Code  and  Standards,  and Vasco  is governed by Talia’s  laws.  In Rasen,  an emerging market,  the  local  securities  laws and  regulations are  lenient. They are very vague  in  the definition of insider trading and have no provision regulating soft‐dollars. Which of the following is most accurate? 

A.  Vasco must comply with Talia’s law.  

B.  Vasco only has to comply with Rasen’s law and therefore can take the fullest advantage of soft‐dollar arrangements. 

C.  Vasco should not worry about Rasen’s law, it is an early stage emerging market and the law enforcement will be lax, if any at all. 

D.  As a CFA Institute member, Vasco must only comply with the Code and Standards regarding insider trading and soft‐dollar arrangements. 

 

 

 

2.  As an expression of gratitude, Tracy Blanc, CFA, a portfolio manager, is invited to spend a three‐week vacation valued at $10,000 with her spouse  in a  luxurious resort owned by a wealthy private client after she skillfully protected the value of the client’s capital during a severe market downturn. The private client is a fee‐paying client of Blanc’s firm. According to Standard IV(B) – Disclosure of Additional Compensation Arrangements: 

A.  Blanc must refuse the invitation as it may jeopardize her investment judgment. 

B.  Blanc is recommended to donate the monetary value of the vacation to a charity of her choice. 

C.  Blanc may accept such an invitation as long as she reports it in writing to her employer and gains their approval. 

D.  Blanc may accept the invitation if she reports it in writing to CFA Institute citing the full monetary value of the vacation. 

 

Page 18: Cfa Ques Answer

2   Study Session 01:  

 

1.  Jason Vasco, CFA, is the director for a major Talia‐owned investment management firm branch in Rasen. Talia is known as the world’s centre of  investment management with securities  laws stricter than  the CFA  Institute Code  and  Standards,  and Vasco  is governed by Talia’s  laws.  In Rasen,  an emerging market,  the  local  securities  laws and  regulations are  lenient. They are very vague  in  the definition of insider trading and have no provision regulating soft‐dollars. Which of the following is most accurate? 

A. Vasco must comply with Talia’s law.

B. Vasco only has to comply with Rasen’s law and therefore can take the fullest advantage of soft-dollar arrangements.

C. Vasco should not worry about Rasen’s law, it is an early stage emerging market and the law enforcement will be lax, if any at all.

D. As a CFA Institute member, Vasco must only comply with the Code and Standards regarding insider trading and soft-dollar arrangements.

Correct Answer:  A  LOS: Reading 2‐b 

Standard I (A) stipulates that in foreign jurisdictions members must comply with the stricter of the applicable laws and the Code of Standards, in this case Talia’s law is the strictest.

Reference: CFA® Program Curriculum, Volume 1, pp. 15‐17. 

 

 

2.  As an expression of gratitude, Tracy Blanc, CFA, a portfolio manager, is invited to spend a three‐week vacation valued at $10,000 with her spouse  in a  luxurious resort owned by a wealthy private client after she skillfully protected the value of the client’s capital during a severe market downturn. The private client is a fee‐paying client of Blanc’s firm. According to Standard IV(B) – Disclosure of Additional Compensation Arrangements: 

A. Blanc must refuse the invitation as it may jeopardize her investment judgment.

B. Blanc is recommended to donate the monetary value of the vacation to a charity of her choice.

C. Blanc may accept such an invitation as long as she reports it in writing to her employer and gains their approval.

D. Blanc may accept the invitation if she reports it in writing to CFA Institute citing the full monetary value of the vacation.

Correct Answer:  C ...................................................................................................... Reading 2‐b Blanc needs to report in writing the additional compensation so her supervisor and the firm can assess whether it is potentially a conflict of interest. If there is no objection she is free to accept the invitation.

Reference: CFA® Program Curriculum, Volume 1, pp. 75‐76. 

Page 19: Cfa Ques Answer

Ethical and Professional Standards   3 

 

3.  Kevin  Dudman,  CFA,  has  just  been  offered  an  exciting  new  position  with  Walton  Asset Management and decides that he will resign from his current position with Trust Asset Management. Before he resigns he decides to ensure that he uses some of the skills and materials he has developed at Trust Asset Management. He is least likely to violate the Code and Standards, if he takes:  

A. stock market analysis prepared by Dudman when he was working at Trust Asset Management.

B. internal contact information on Trust Asset Management‘s major clients which is available from other eternal sources.

C. computer models developed to identify mispriced securities developed by Dudman and a colleague at Trust Asset Management.

D. experience in pricing unlisted securities which he gained while attending training courses which were paid for by Trust Asset Management.  

 

 

4.  Joseph  Morgon,  CFA,  is  a  research  analyst  covering  the  Bourgogne  Vineyard  Corporation. Morgon’s parents bought $50 worth of Bourgogne Vineyard Corporation shares for his two‐year old son on his birthday. Under Standard VI(A), Disclosure of Conflicts, Morgon: 

A. must file a report with the SEC.

B. must sell the shares immediately.

C. must disclose the ownership of the shares by a member of his immediate family.

D. does not need to disclose the fact that his son owns the shares of Bourgogne Vineyard Corporation.

Page 20: Cfa Ques Answer

4   Study Session 01:  

 

3.  Kevin  Dudman,  CFA,  has  just  been  offered  an  exciting  new  position  with  Walton  Asset Management and decides that he will resign from his current position with Trust Asset Management. Before he resigns he decides to ensure that he uses some of the skills and materials he has developed at Trust Asset Management. He is least likely to violate the Code and Standards, if he takes:  

A. stock market analysis prepared by Dudman when he was working at Trust Asset Management.

B. internal contact information on Trust Asset Management‘s major clients which is available from other eternal sources.

C. computer models developed to identify mispriced securities developed by Dudman and a colleague at Trust Asset Management.

D. experience in pricing unlisted securities which he gained while attending training courses which were paid for by Trust Asset Management.

Correct Answer:  D........................................................................................... LOS: Reading  2‐b Models and research which he worked on when employed by Trust Asset Management belong to Trust Asset Management. Client contact details should not be taken from his employer, although he is not prohibited from collecting client information from outside sources. However skills and experience gained at Trust Asset Management can be used in his new job, so D is the correct answer.

Reference: CFA® Program Curriculum, Volume 1, pp. 69‐74. 

 

 

 

4.  Joseph  Morgon,  CFA,  is  a  research  analyst  covering  the  Bourgogne  Vineyard  Corporation. Morgon’s parents bought $50 worth of Bourgogne Vineyard Corporation shares for his two‐year old son on his birthday. Under Standard VI(A), Disclosure of Conflicts, Morgon: 

A. must file a report with the SEC.

B. must sell the shares immediately.

C. must disclose the ownership of the shares by a member of his immediate family.

D. does not need to disclose the fact that his son owns the shares of Bourgogne Vineyard Corporation.

Correct Answer:  D........................................................................................... LOS: Reading  2‐b The share ownership is not likely to be material and therefore will not reasonably affect Morgon’s ability to make unbiased and objective recommendation according to Standard VI(A) Disclosure of Conflicts.

Reference: CFA® Program Curriculum, Volume 1, pp. 89‐94. 

Page 21: Cfa Ques Answer

Ethical and Professional Standards   5 

 

5.  Wimpy Greenback, CFA,  is  the  research  analyst  responsible  for  following  Brown Appliances Company. This analysis suggests  the stock should be rated a “sell” because  the market outlook  for the firm’s new products  is bleak compared with that of the closest competition. Greenback  lives on the  same  street  as  the  CFO  of  Brown  Appliances.  During  a  recent  neighborhood  gathering, Greenback’s wife overheard the wife of the Chief Financial Officer of Brown Appliances complaining that her husband had been working  late due  to a hostile  takeover  threat  from a  foreign appliances group. This  fact has not yet been made public by Brown Appliances. Upon  returning  to his office, Greenback  released a  strong  “buy”  recommendation  to  the public based on  this new  information. Greenback: 

A.  was in full compliance with the Code and Standards. 

B.  did not violate the Code and Standards because he used mosaic theory to arrive at his recommendation. 

C.  violated the Code and Standards by failing to distinguish between facts and opinions in his recommendation. 

D.  violated the Code and Standards because he did not have a reasonable and adequate basis for his recommendation. 

Page 22: Cfa Ques Answer

6   Study Session 01:  

 

5.  Wimpy Greenback, CFA,  is  the  research  analyst  responsible  for  following  Brown Appliances Company. This analysis suggests  the stock should be rated a “sell” because  the market outlook  for the firm’s new products  is bleak compared with that of the closest competition. Greenback  lives on the  same  street  as  the  CFO  of  Brown  Appliances.  During  a  recent  neighborhood  gathering, Greenback’s wife overheard the wife of the Chief Financial Officer of Brown Appliances complaining that her husband had been working  late due  to a hostile  takeover  threat  from a  foreign appliances group. This  fact has not yet been made public by Brown Appliances. Upon  returning  to his office, Greenback  released a  strong  “buy”  recommendation  to  the public based on  this new  information. Greenback: 

A. was in full compliance with the Code and Standards.

B. did not violate the Code and Standards because he used mosaic theory to arrive at his recommendation.

C. violated the Code and Standards by failing to distinguish between facts and opinions in his recommendation.

D. violated the Code and Standards because he did not have a reasonable and adequate basis for his recommendation.

Correct Answer:  D........................................................................................... LOS: Reading  2‐b Standard V(A) Diligence and Reasonable Basis, states that members must have a reasonable and adequate basis for a recommendation. Greenback should have reinvestigated the company’s situation and not only relied on unofficial information. This may well be a misappropriation of material nonpublic information as stated in Standard V(A) Prohibition against Use of Material Nonpublic Information, if a tender offer to Brown Appliances follows.

Reference: CFA® Program Curriculum, Volume 1, pp. 80‐84. 

Page 23: Cfa Ques Answer

Ethical and Professional Standards   7 

 

6.  The fixed‐income corporate finance department of Golden Brothers, an investment banking firm, has  decided  to  compete  for  the  advisory  and  underwriting  bond  offering  of Kia  Telcom,  a  ‘hot’ telecommunications  company.  The  firm’s  equity  brokerage  unit  is  about  to  publish  a  “sell” recommendation on Kia Telcom due to an unexpected announcement of cost overruns. The head of fixed‐income  investment  banking  has  asked  the  head  of  the  equity  brokerage  unit  to  change  the recommendation from “sell” to “buy” before distributing the research report to clients. According to the Code and Standards, the best course of action for the equity brokerage unit is to: 

A.  place Kia Telcom on a restricted list and publish only factual information about the company. 

B.  immediately re‐rate the stock to a “buy” since the firm’s overall interest supersedes that of the client. 

C.  assign a more senior analyst to decide if the stock deserves a higher rating for the sake of objectivity since less senior analysts may err in judgment. 

D.  increase the rating by no more than one increment (in this case, to a “hold” recommendation) since little harm is done by being a bit more positive, while the firm’s overall interest is served. 

 

 

 

7.  Fiona Griffiths, CFA, is an equity sales manager at a London‐based Tiger Securities branch in an emerging market. Initial public offerings are often oversubscribed making it difficult to ensure a fair allocation.  Griffiths  understands  the  local  environment  so  she  is  able  to  influence  the  allocation process so that she can personally subscribe to the maximum she can afford and then allocate the rest to her clients. Her clients never complain because they have almost always profited from investing in the  emerging  market  over  the  last  couple  of  years. Which  of  the  following  describes  Griffiths’ situation? 

A. Griffiths is in compliance with the Code and Standards since her clients are satisfied.

B. Griffiths violates the Code and Standards due to the priority she gives to transactions.

C. Griffiths violates the Code and Standards since she lacks independence and objectivity.

D. Griffiths violates the Code and Standards since she does not maintain client, confidentiality.

Page 24: Cfa Ques Answer

8   Study Session 01:  

 

6.  The fixed‐income corporate finance department of Golden Brothers, an investment banking firm, has  decided  to  compete  for  the  advisory  and  underwriting  bond  offering  of Kia  Telcom,  a  ‘hot’ telecommunications  company.  The  firm’s  equity  brokerage  unit  is  about  to  publish  a  “sell” recommendation on Kia Telcom due to an unexpected announcement of cost overruns. The head of fixed‐income  investment  banking  has  asked  the  head  of  the  equity  brokerage  unit  to  change  the recommendation from “sell” to “buy” before distributing the research report to clients. According to the Code and Standards, the best course of action for the equity brokerage unit is to: 

A. place Kia Telcom on a restricted list and publish only factual information about the company.

B. immediately re-rate the stock to a “buy” since the firm’s overall interest supersedes that of the client.

C. assign a more senior analyst to decide if the stock deserves a higher rating for the sake of objectivity since less senior analysts may err in judgment.

D. increase the rating by no more than one increment (in this case, to a “hold” recommendation) since little harm is done by being a bit more positive, while the firm’s overall interest is served.

Correct Answer:  A ............................................................................................LOS: Reading  2‐a In this case, any action to accommodate the interest of the investment banking department that may compromise the independence and objectivity of the brokerage research efforts can violate Standard I(B) and the Code of Ethics.

Reference: CFA® Program Curriculum, Volume 1, pp. 21‐25. 

 

7.  Fiona Griffiths, CFA, is an equity sales manager at a London‐based Tiger Securities branch in an emerging market. Initial public offerings are often oversubscribed making it difficult to ensure a fair allocation.  Griffiths  understands  the  local  environment  so  she  is  able  to  influence  the  allocation process so that she can personally subscribe to the maximum she can afford and then allocate the rest to her clients. Her clients never complain because they have almost always profited from investing in the  emerging  market  over  the  last  couple  of  years. Which  of  the  following  describes  Griffiths’ situation? 

A. Griffiths is in compliance with the Code and Standards since her clients are satisfied.

B. Griffiths violates the Code and Standards due to the priority she gives to transactions.

C. Griffiths violates the Code and Standards since she lacks independence and objectivity.

D. Griffiths violates the Code and Standards since she does not maintain client, confidentiality.

Correct Answer:  B............................................................................................ LOS: Reading  2‐b Griffiths is in violation as Standard VI(B) Priority of Transactions, since she puts her personal investment ahead of her clients, regardless of whether the clients are pleased with her services.

Reference: CFA® Program Curriculum, Volume 1, pp. 94‐99. 

Page 25: Cfa Ques Answer

Ethical and Professional Standards   9 

 

8.  Victoria Anderson, CFA, works  for  Pluto Capital,  a  newly  established  investment  counseling firm.  The  founding  partners  of  Pluto Capital  came  from Vulcan  Investments which was  recently taken over by a large financial services group. Jonathan Beecham, a prospective client of the firm, is meeting with Anderson for the first time. Beecham has been a client of Vulcan Investments for years, but is now considering switching his account to Pluto Capital because he has been disappointed by Vulcan’s  underperformance  following  the  takeover. At  the  beginning  of  their meeting, Anderson sympathized with  his  situation,  then  immediately  explains  to Beecham  that  she  has discovered  a highly undervalued stock that offers large potential gains. Anderson then promises Beecham that she can buy  the  stock  for his  account  at  the  current price  if he  switches  the  account within  48 hours. Anderson’s  actions  violated  the  Code  and  Standards.  Which  of  the  following  statements  best describes the action Anderson should have taken? Anderson should have: 

A.  elaborated on the technical features of Pluto’s standard valuation method used to identify the undervaluation. 

B.  avoided the meeting with Beecham in the first place because the founding partners of Pluto came from Vulcan. 

C.  given Beecham a longer time period to take advantage of the offer price when switching his account to Pluto. 

D.  determined Beecham’s investment needs, objectives, and tolerance for risk before making any investment recommendation. 

Page 26: Cfa Ques Answer

10   Study Session 01:  

 

8.  Victoria Anderson, CFA, works  for  Pluto Capital,  a  newly  established  investment  counseling firm.  The  founding  partners  of  Pluto Capital  came  from Vulcan  Investments which was  recently taken over by a large financial services group. Jonathan Beecham, a prospective client of the firm, is meeting with Anderson for the first time. Beecham has been a client of Vulcan Investments for years, but is now considering switching his account to Pluto Capital because he has been disappointed by Vulcan’s  underperformance  following  the  takeover. At  the  beginning  of  their meeting, Anderson sympathized with  his  situation,  then  immediately  explains  to Beecham  that  she  has discovered  a highly undervalued stock that offers large potential gains. Anderson then promises Beecham that she can buy  the  stock  for his  account  at  the  current price  if he  switches  the  account within  48 hours. Anderson’s  actions  violated  the  Code  and  Standards.  Which  of  the  following  statements  best describes the action Anderson should have taken? Anderson should have: 

A. elaborated on the technical features of Pluto’s standard valuation method used to identify the undervaluation.

B. avoided the meeting with Beecham in the first place because the founding partners of Pluto came from Vulcan.

C. given Beecham a longer time period to take advantage of the offer price when switching his account to Pluto.

D. determined Beecham’s investment needs, objectives, and tolerance for risk before making any investment recommendation.

Correct Answer:  D........................................................................................... LOS: Reading  2‐b Prior to recommending any investments, Anderson should determine Beecham's investment needs, objectives, and tolerance for risk as stated in Standard III(C) Suitability.

Reference: CFA® Program Curriculum, Volume 1, pp. 60‐64. 

Page 27: Cfa Ques Answer

Ethical and Professional Standards   11 

 

9.  Ken  Janzen, CFA,  is  an  economist  at  a  large  bank  and  he  has  never made  direct  investment decisions.  Jenzen  is  the  latest winner of  a well‐publicized portfolio management  competition  in  a national newspaper. On the recommendation of his friends, he  is  launching an  investment fund. In the prospectus he tells the prospective clients, “The fund has no long‐term track record as yet, but the investment  manager  has  shown  considerable  skills  in  managing  hypothetical  portfolios.  In  a competition  the  manager  has  demonstrated  a  portfolio  total  return  above  26  percent  per  year annualized, and that is more than 12 percent above the benchmark for the same period.” He managed to  raise a  significant amount of money  from  retail  investors who are  interested  in  investing  in  the fund. Has Janzen violated the Code and Standards? 

A.  Yes, because the statement misrepresents Janzen’s track record. 

B.  Yes, because he cannot quote performance for a hypothetical portfolio.  

C.  Yes, because the statement about return ignores the risk preferences of his clients. 

D.  No, because the statement is a true and accurate description of Janzen’s track record. 

Page 28: Cfa Ques Answer

12   Study Session 01:  

 

9.  Ken  Janzen, CFA,  is  an  economist  at  a  large  bank  and  he  has  never made  direct  investment decisions.  Jenzen  is  the  latest winner of  a well‐publicized portfolio management  competition  in  a national newspaper. On the recommendation of his friends, he  is  launching an  investment fund. In the prospectus he tells the prospective clients, “The fund has no long‐term track record as yet, but the investment  manager  has  shown  considerable  skills  in  managing  hypothetical  portfolios.  In  a competition  the  manager  has  demonstrated  a  portfolio  total  return  above  26  percent  per  year annualized, and that is more than 12 percent above the benchmark for the same period.” He managed to  raise a  significant amount of money  from  retail  investors who are  interested  in  investing  in  the fund. Has Janzen violated the Code and Standards? 

A. Yes, because the statement misrepresents Janzen’s track record.

B. Yes, because he cannot quote performance for a hypothetical portfolio.

C. Yes, because the statement about return ignores the risk preferences of his clients.

D. No, because the statement is a true and accurate description of Janzen’s track record.

Correct Answer:  D........................................................................................... LOS: Reading  2‐b Although Janzen’s experience in managing investments is only based on his winning a hypothetical portfolio management competition, he does not misrepresent his capabilities and experience as described in Standard III(D) Performance Presentation. Whether it is appropriate for an investor to subscribe to his investment fund is a different matter. The role of the Code and Standards is to guide self-regulation of CFA Institute members, not to certify the merit of an investment.

Reference: CFA® Program Curriculum, Volume 1, pp. 64‐67. 

Page 29: Cfa Ques Answer

Ethical and Professional Standards   13 

 

10.  Martha Pierpont, CFA, works  for  the securities custody department of North Pole Trust Bank. She  makes  a  reciprocal  referral  fee  arrangement  with  Robert  Underhill,  CFA,  an  adviser  at BestAdvice.com. She does not disclose  the  referral arrangement but Underhill does so by  inserting one clause  in BestAdvice.com’s  investment advisory agreement that  includes “… from time to time referral fees may be arranged with a number of selected securities custodians.” Clients of BestAdvice regularly use North Pole’s services and pay referral fees. Which of the following is most accurate? 

A.  Only Pierpont complies with the Code and Standards. 

B.  Only Underhill complies with the Code and Standards. 

C.  Both Pierpont and Underhill comply with the Code and Standards. 

D.  Neither Pierpont nor Underhill comply with the Code and Standards. 

 

 

 

11.  Charles Chaplane, who is not a member of CFA Institute, is a senior partner of a small brokerage firm,  Blue  Moon  Securities,  which  recently  participated  in  a  large  stock  offering.  The  offering company has been given an unfavorable recommendation by his research department in the past two quarters  due  to  lacklustre  performance.  Chaplane  immediately  calls  his  junior  analyst  John Blumenberg, CFA, and instructs him to upgrade his recommendation. Blumenberg comes up with a more  favorable  recommendation within  a  short period of  time. Blumenberg  is  least  likely  to have violated the Standards because he failed: 

A.  to avoid a conflict of interest.  

B.  to maintain independence and objectivity. 

C.  to make a fair statement of investment performance. 

D.  to exercise due diligence and thoroughness in making an investment recommendation. 

Page 30: Cfa Ques Answer

14   Study Session 01:  

 

10.  Martha Pierpont, CFA, works  for  the securities custody department of North Pole Trust Bank. She  makes  a  reciprocal  referral  fee  arrangement  with  Robert  Underhill,  CFA,  an  adviser  at BestAdvice.com. She does not disclose  the  referral arrangement but Underhill does so by  inserting one clause  in BestAdvice.com’s  investment advisory agreement that  includes “… from time to time referral fees may be arranged with a number of selected securities custodians.” Clients of BestAdvice regularly use North Pole’s services and pay referral fees. Which of the following is most accurate? 

A. Only Pierpont complies with the Code and Standards.

B. Only Underhill complies with the Code and Standards.

C. Both Pierpont and Underhill comply with the Code and Standards.

D. Neither Pierpont nor Underhill comply with the Code and Standards.

Correct Answer:  D........................................................................................... LOS: Reading  2‐b The best choice is D since any referral fee arrangement that a client ultimately pays must be disclosed in terms of the nature of the consideration or the benefit together with the estimated monetary value, by both the payer and recipient of the fee. See Standard VI (C) Referral Fees.

Reference: CFA® Program Curriculum, Volume 1, pp. 99‐101. 

 

 

11.  Charles Chaplane, who is not a member of CFA Institute, is a senior partner of a small brokerage firm,  Blue  Moon  Securities,  which  recently  participated  in  a  large  stock  offering.  The  offering company has been given an unfavorable recommendation by his research department in the past two quarters  due  to  lacklustre  performance.  Chaplane  immediately  calls  his  junior  analyst  John Blumenberg, CFA, and instructs him to upgrade his recommendation. Blumenberg comes up with a more  favorable  recommendation within  a  short period of  time. Blumenberg  is  least  likely  to have violated the Standards because he failed: 

A. to avoid a conflict of interest.

B. to maintain independence and objectivity.

C. to make a fair statement of investment performance.

D. to exercise due diligence and thoroughness in making an investment recommendation.

Correct Answer:  C ........................................................................................... LOS: Reading  2‐b The best answer is C, because there is no evidence to suggest that performance presentation is relevant to the question.

Reference: CFA® Program Curriculum, Volume 1, pp. 64‐67. 

Page 31: Cfa Ques Answer

Ethical and Professional Standards   15 

 

12.  Patricia Lualua, CFA,  is a portfolio manager of Raven Asset Management. Recently she won a mandate  from  the Flemish Widows pension  fund  trustees  to manage  the  investments of  the  fund. One  of  the  Flemish Widows  trustees  privately mentions  that  Lualua  should  direct  her  trades  to Churner Securities, which is owned by a relative of one of the trustees. Lualua, for fear of losing the account, directs 50% of the trades to Churner Securities. She is pleased to find that Churner’s quality of  execution  is  good  and  the  emerging  market  research  quality  is  excellent.  Although  Flemish Widows does not invest in emerging markets, Lualua finds the research useful for the other funds she manages. Lualua decides not  to  inform anyone regarding  the situation. According  to  the Code and Standards: 

A.  Lualua should stop trading with Churner Securities. 

B.  Lualua may continue trading with Churners Securities. 

C.  Lualua should disclose this arrangement to Flemish Widows. 

D.  Lualua should disclose this arrangement to the CFA Institute. 

Page 32: Cfa Ques Answer

16   Study Session 01:  

 

12.  Patricia Lualua, CFA,  is a portfolio manager of Raven Asset Management. Recently she won a mandate  from  the Flemish Widows pension  fund  trustees  to manage  the  investments of  the  fund. One  of  the  Flemish Widows  trustees  privately mentions  that  Lualua  should  direct  her  trades  to Churner Securities, which is owned by a relative of one of the trustees. Lualua, for fear of losing the account, directs 50% of the trades to Churner Securities. She is pleased to find that Churner’s quality of  execution  is  good  and  the  emerging  market  research  quality  is  excellent.  Although  Flemish Widows does not invest in emerging markets, Lualua finds the research useful for the other funds she manages. Lualua decides not  to  inform anyone regarding  the situation. According  to  the Code and Standards: 

A. Lualua should stop trading with Churner Securities.

B. Lualua may continue trading with Churners Securities.

C. Lualua should disclose this arrangement to Flemish Widows.

D. Lualua should disclose this arrangement to the CFA Institute.

Correct Answer:  C ........................................................................................... LOS: Reading  2‐b Under most securities laws this situation is acceptable but under Standard III(A), Loyalty, Prudence and Care, Lualua’s trading relationship does not put her client’s interest first.

Lualua should disclose the arrangement to the Board of Trustees of Flemish Widows and let the Board give the direction.

Reference: CFA® Program Curriculum, Volume 1, pp. 48‐53. 

Page 33: Cfa Ques Answer

Ethical and Professional Standards   17 

 

13.  Carlina Paparazzi, a fund manager with Abbotswood Advisors, has just been given the authority to manage a newly acquired client which has a retirement benefit plan, when she realizes that a US Government Bond belonging to the account matures the next day. The bond comprises 5% of the total assets.  Abbotswood  Advisors  is  still  in  the midst  of  a  discussion  with  the  client  regarding  the formulation of a new investment policy and portfolio objectives. Looking at what the current market has  to  offer,  there  are  a  number  of  attractive  opportunities. One  opportunity  that  stands  out  is  a corporate bond of a major oil company that went out of favor due to an environmental accident that occurred the week before. She has followed the oil company for a number of years and knows that its fundamentals are sound. The prospect of an improved credit rating in the next six months is not yet reflected  in the current price. Her supervisor asks Paparazzi to invest the proceeds in the corporate bond. Paparazzi prefers however to invest them in 3‐month Treasury Bills, albeit with a much lower yield, until the new investment policy and objectives are formulated. What is the best course of action for Paparazzi? 

A.  Invest in the Treasury Bills until the new investment policy and objectives are established. 

B.  Split the investment between the corporate bond and the Treasury Bills to diversify the risk. 

C.  Revert to the client for a decision and do nothing until the client’s direction is received.  

D.  Follow her supervisor’s direction as the corporate bond opportunity will benefit the overall performance of the fund. 

Page 34: Cfa Ques Answer

18   Study Session 01:  

 

13.  Carlina Paparazzi, a fund manager with Abbotswood Advisors, has just been given the authority to manage a newly acquired client which has a retirement benefit plan, when she realizes that a US Government Bond belonging to the account matures the next day. The bond comprises 5% of the total assets.  Abbotswood  Advisors  is  still  in  the midst  of  a  discussion  with  the  client  regarding  the formulation of a new investment policy and portfolio objectives. Looking at what the current market has  to  offer,  there  are  a  number  of  attractive  opportunities. One  opportunity  that  stands  out  is  a corporate bond of a major oil company that went out of favor due to an environmental accident that occurred the week before. She has followed the oil company for a number of years and knows that its fundamentals are sound. The prospect of an improved credit rating in the next six months is not yet reflected  in the current price. Her supervisor asks Paparazzi to invest the proceeds in the corporate bond. Paparazzi prefers however to invest them in 3‐month Treasury Bills, albeit with a much lower yield, until the new investment policy and objectives are formulated. What is the best course of action for Paparazzi? 

A. Invest in the Treasury Bills until the new investment policy and objectives are established.

B. Split the investment between the corporate bond and the Treasury Bills to diversify the risk.

C. Revert to the client for a decision and do nothing until the client’s direction is received.

D. Follow her supervisor’s direction as the corporate bond opportunity will benefit the overall performance of the fund.

Correct Answer:  A ........................................................................................... LOS: Reading  2‐b Regardless of whether it is the best investment decision, choices C or D will violate Standard III(C), Suitability, because the overall investment policy and objectives are not yet established. Choice A is not correct because the client pays a fee to hire expertise in investment decision making. So the best choice is B, where the client’s interest is protected, as a Treasury Bill is a cash equivalent and is risk-free, as are the maturing Treasury Bonds.

Reference: CFA® Program Curriculum, Volume 1, pp. 60‐64. 

Page 35: Cfa Ques Answer

Ethical and Professional Standards   19 

 

14.  Muhammad  Taqdir,  CFA,  is  an  investment  manager  whose  clients  are  high‐net  worth individuals. Taqdir  is a member of a  local charity organization that supports children with asthma. During  a meeting  at  the  charity, Taqdir  recommends  that  the  organization  sends  a  letter  to Xara Corporation  requesting  they make  a donation  to  the  charity. Taqdir  knows of Xara Corporation’s involvement  in  this  cause  from  previous  discussions  with  a  colleague  in  the  office.  The  chief executive  and  owner  of  Xara  Corporation  is  a  client  of  the  firm.  The  charity,  citing  Taqdir’s recommendation, sent the letter and received a substantial donation. According to the CFA Institute Code and Standards: 

A.  Taqdir has done his best since the organisation received a substantial donation. 

B.  Taqdir should not have disclosed the identity of the chief executive without his prior approval. 

C.  Taqdir should have informed the chief executive of Xara that he is going to receive a letter from the organization. 

D.  Taqdir should have requested the approval of his colleague before disclosing the name of the chief executive of Xara. 

Page 36: Cfa Ques Answer

20   Study Session 01:  

 

14.  Muhammad  Taqdir,  CFA,  is  an  investment  manager  whose  clients  are  high‐net  worth individuals. Taqdir  is a member of a  local charity organization that supports children with asthma. During  a meeting  at  the  charity, Taqdir  recommends  that  the  organization  sends  a  letter  to Xara Corporation  requesting  they make  a donation  to  the  charity. Taqdir  knows of Xara Corporation’s involvement  in  this  cause  from  previous  discussions  with  a  colleague  in  the  office.  The  chief executive  and  owner  of  Xara  Corporation  is  a  client  of  the  firm.  The  charity,  citing  Taqdir’s recommendation, sent the letter and received a substantial donation. According to the CFA Institute Code and Standards: 

A. Taqdir has done his best since the organisation received a substantial donation.

B. Taqdir should not have disclosed the identity of the chief executive without his prior approval.

C. Taqdir should have informed the chief executive of Xara that he is going to receive a letter from the organization.

D. Taqdir should have requested the approval of his colleague before disclosing the name of the chief executive of Xara.

Correct Answer:  B............................................................................................ LOS: Reading  2‐b Regardless of the fact that that the organization finally received the substantial donation, Tariq has violated the preservation of confidentiality under Standard III(E), Preservation of Confidentiality, in disclosing the name of the chief executive and owner of Xara without prior knowledge of both the chief executive and his colleague.

Reference: CFA® Program Curriculum, Volume 1, pp. 67‐69. 

Page 37: Cfa Ques Answer

Ethical and Professional Standards   21 

 

15.  Marco  Maggio,  CFA,  is  scheduled  to  visit  the  corporate  headquarters  of  Venus  Industries. Maggio expects to use the information obtained there to complete his research report on Venus stock. The location of Venus Industries is within a 15‐minute drive of a prestigious golf course. On arrival at the Venus premises, Marco Maggio learns that Venus is offering Maggio an extension of his stay that weekend and invites him for a day of golf with all expenses paid. Venus Industries also offers to pay for all the expenses for the trip, including the cost of meals, hotel room, and air transportation back to Venus Industries. The total cost for the weekend is about $2,000. 

Which of  the  following  actions would be  the best  course  for Maggio  to  take under  the Code  and Standards? 

A.  Pay for all travel expenses, including costs of meals and incidental items and politely reject the golf outing offer. 

B.  Reject the golf outing offer but accept the reimbursement of the travel expenses since they are legitimate business‐related expenses. 

C.  Accept both the expenses‐paid trip and the golf outing as more information can often be extracted from the company in a more leisurely environment. 

D.  Accept the expenses‐paid trip and disclose the value of the trip in the report, but it is at Maggio’s discretion to take the golf outing offer without disclosing it as it occurs outside working hours. 

Page 38: Cfa Ques Answer

22   Study Session 01:  

 

15.  Marco  Maggio,  CFA,  is  scheduled  to  visit  the  corporate  headquarters  of  Venus  Industries. Maggio expects to use the information obtained there to complete his research report on Venus stock. The location of Venus Industries is within a 15‐minute drive of a prestigious golf course. On arrival at the Venus premises, Marco Maggio learns that Venus is offering Maggio an extension of his stay that weekend and invites him for a day of golf with all expenses paid. Venus Industries also offers to pay for all the expenses for the trip, including the cost of meals, hotel room, and air transportation back to Venus Industries. The total cost for the weekend is about $2,000. 

Which of  the  following  actions would be  the best  course  for Maggio  to  take under  the Code  and Standards? 

A. Pay for all travel expenses, including costs of meals and incidental items and politely reject the golf outing offer.

B. Reject the golf outing offer but accept the reimbursement of the travel expenses since they are legitimate business-related expenses.

C. Accept both the expenses-paid trip and the golf outing as more information can often be extracted from the company in a more leisurely environment.

D. Accept the expenses-paid trip and disclose the value of the trip in the report, but it is at Maggio’s discretion to take the golf outing offer without disclosing it as it occurs outside working hours.

Correct Answer:  A ........................................................................................... LOS: Reading  2‐b Maggio risks violating Standard I(B) Independence and Objectivity because accepting any significant gift may impede his independence and objectivity. He should pay, whenever possible, for his own travel expenses and not accept the golf outing.

Reference: CFA® Program Curriculum, Volume 1, pp. 21‐29. 

Page 39: Cfa Ques Answer

Ethical and Professional Standards   23 

 

16.  Simon Freud, CFA, is a private‐client investment manager at Super Echo investment firm based in  Vienna,  Austria.  One  of  his  clients  in  Monaco  offers  him  bonus  compensation  beyond  that provided by his  firm  if  the portfolio performance exceeds  the agreed benchmark. To make  it more attractive to Freud, his client will send the bonus compensation to a tax‐free account in a tax haven. Freud: 

A.  should report the situation to the compliance officer of the CFA Institute according to Standard I(B) Independence and Objectivity. 

B.  should turn down the additional compensation offer because it violates Standard IV(B) Additional Compensation Arrangements. 

C.  may accept the additional compensation subject to the approval of his employer as required by Standard IV(B) Additional Compensation Arrangements. 

D.  is free to accept the additional compensation in the tax‐free account, as long as the account is not under the jurisdiction of either Monaco or Austria, it will therefore also be outside the jurisdiction of the Code and Standards. 

Page 40: Cfa Ques Answer

24   Study Session 01:  

 

16.  Simon Freud, CFA, is a private‐client investment manager at Super Echo investment firm based in  Vienna,  Austria.  One  of  his  clients  in  Monaco  offers  him  bonus  compensation  beyond  that provided by his  firm  if  the portfolio performance exceeds  the agreed benchmark. To make  it more attractive to Freud, his client will send the bonus compensation to a tax‐free account in a tax haven. Freud: 

A. should report the situation to the compliance officer of the CFA Institute according to Standard I(B) Independence and Objectivity.

B. should turn down the additional compensation offer because it violates Standard IV(B) Additional Compensation Arrangements.

C. may accept the additional compensation subject to the approval of his employer as required by Standard IV(B) Additional Compensation Arrangements.

D. is free to accept the additional compensation in the tax-free account, as long as the account is not under the jurisdiction of either Monaco or Austria, it will therefore also be outside the jurisdiction of the Code and Standards.

Correct Answer:  C ........................................................................................... LOS: Reading  2‐b Standard IV(B) Additional Compensation Arrangements does not prohibit the acceptance of additional compensation as long as approval from the employer is obtained. The tax-free account is a separate issue and will have to be viewed in light of tax rules and regulations.

Reference: CFA® Program Curriculum, Volume 1, pp. 75‐76. 

Page 41: Cfa Ques Answer

Ethical and Professional Standards   25 

 

17.  Joseph Luny, CFA, is a bank analyst with London Fog Securities. On a recent trip to see a bank that he covers, he was presented with a rosy outlook  for  the bank’s earnings  in  the next  two years which  is above  the  consensus expectations. When probed  further about  the assumptions,  the CFO inadvertently mentioned that serious discussions are taking place for a tender offer of a smaller well‐managed bank  that Luny also covers. This  information has not been made public. Luny  feels very lucky to receive this unexpected tip and rushes back to his office to revise his projections and advise his major clients to buy the smaller bank’s stock. What should Luny have done instead? 

A. Luny should request his supervisor’s approval.

B. Luny is entitled to take advantage of the information as he did not misappropriate it.

C. Luny should refrain from taking any action on the smaller bank’s stock until the bank has made the tender offer information public.

D. Luny should encourage the bank to disclose the tender offer information to the public but is free to take advantage of the information in the meantime.

Page 42: Cfa Ques Answer

26   Study Session 01:  

 

17.  Joseph Luny, CFA, is a bank analyst with London Fog Securities. On a recent trip to see a bank that he covers, he was presented with a rosy outlook  for  the bank’s earnings  in  the next  two years which  is above  the  consensus expectations. When probed  further about  the assumptions,  the CFO inadvertently mentioned that serious discussions are taking place for a tender offer of a smaller well‐managed bank  that Luny also covers. This  information has not been made public. Luny  feels very lucky to receive this unexpected tip and rushes back to his office to revise his projections and advise his major clients to buy the smaller bank’s stock. What should Luny have done instead? 

A. Luny should request his supervisor’s approval.

B. Luny is entitled to take advantage of the information as he did not misappropriate it.

C. Luny should refrain from taking any action on the smaller bank’s stock until the bank has made the tender offer information public.

D. Luny should encourage the bank to disclose the tender offer information to the public but is free to take advantage of the information in the meantime.

Correct Answer:  C ........................................................................................... LOS: Reading  2‐b Under Standard II(A) Material Nonpublic Information Lung should not act or cause others to act on this information . If it is not appropriate to encourage public dissemination of the information he can only communicate the information to his supervisor or compliance department.

Reference: CFA® Program Curriculum, Volume 1, pp. 36‐45. 

Page 43: Cfa Ques Answer

Ethical and Professional Standards   27 

 

18.  Marianne Warner,  CFA,  is  a  portfolio manager  at  Creative  Investment Management  and  in charge of managing several discretionary portfolios. Her husband holds 25 percent of  the shares of Gurita  Corporation,  a  computer  services  company.  In  line  with  the  high  sector  growth,  Gurita Corporation  went  public  earlier  in  the  year.  The  share  price  skyrocketed  and  the  value  of  her husband’s holding went up from $1 million prior to the public offering to $8 million at the current market price. Warner believes that the current market price is too high and immediately advises her husband  to  sell  half  of  his  shares.  She  also  recommends  he  put  the  proceeds  into  one  of  the discretionary portfolios she is currently managing. Which one is the best answer? 

A.  Warner does not violate the Code and Standards. 

B.  Warner violates the Code and Standards for failing to disclose the conflicts of interest. 

C.  Warner violates the Code and Standards for possessing material non‐public information. 

D.  Warner violates the Code and Standards for failing to disclose additional compensation arrangements. 

Page 44: Cfa Ques Answer

28   Study Session 01:  

 

18.  Marianne Warner,  CFA,  is  a  portfolio manager  at  Creative  Investment Management  and  in charge of managing several discretionary portfolios. Her husband holds 25 percent of  the shares of Gurita  Corporation,  a  computer  services  company.  In  line  with  the  high  sector  growth,  Gurita Corporation  went  public  earlier  in  the  year.  The  share  price  skyrocketed  and  the  value  of  her husband’s holding went up from $1 million prior to the public offering to $8 million at the current market price. Warner believes that the current market price is too high and immediately advises her husband  to  sell  half  of  his  shares.  She  also  recommends  he  put  the  proceeds  into  one  of  the discretionary portfolios she is currently managing. Which one is the best answer? 

A. Warner does not violate the Code and Standards.

B. Warner violates the Code and Standards for failing to disclose the conflicts of interest.

C. Warner violates the Code and Standards for possessing material non-public information.

D. Warner violates the Code and Standards for failing to disclose additional compensation arrangements.

Correct Answer:  A ........................................................................................... LOS: Reading  2‐b Choice B would apply if the advice was given when one or more of the portfolios contain Gurita shares. There is no mention of material nonpublic information or additional compensation so Warner has not violated any of the Standards.

Reference: CFA® Program Curriculum, Volume 1, pp. 36‐45. 

Page 45: Cfa Ques Answer

Ethical and Professional Standards   29 

 

19.  The Professional Conduct staff under  the direction of CFA  Institute are  least  likely  to make an enquiry into a member’s conduct when:  

A.  they perform random checks on members’ professional conduct. 

B.  they receive a written complaint regarding a member’s professional conduct.  

C.  the media reports on a member whose professional conduct appears to have been unethical. 

D.  members self‐disclose on their Professional Conduct Statement that they are involved in litigation regarding their investment advice.  

 

 

 

20.  Jonathan Seller, CFA, works for an investment bank that is acting as the principal underwriter for an issue of stock of a large tire manufacturer. Seller found out that the prospectus has concealed an impending  product  recall  due  to  a  quality  control  error.  Since  the  number  of  items  affected  is relatively  small,  the  product  recall  is  planned  to  be  a  quiet  affair. However  Seller  is  aware  that recently a competitor’s product recall received a large amount of adverse publicity. The preliminary prospectus has been distributed. According to the Code and Standards: 

A. Seller should do nothing as it may jeopardize the success of the issue.

B. Seller should revise the preliminary prospectus to include the omitted information to avoid any possible misrepresentation.

C. Seller should inform CFA Institute of the violation of the Code and Standards so he can clear himself of the possible misrepresentation.

D. Seller should inform his supervisor and let him/her deal with the situation since Seller himself should not jeopardize the success of the issue.

Page 46: Cfa Ques Answer

30   Study Session 01:  

 

19.  The Professional Conduct staff under  the direction of CFA  Institute are  least  likely  to make an enquiry into a member’s conduct when:  

A. they perform random checks on members’ professional conduct.

B. they receive a written complaint regarding a member’s professional conduct.

C. the media reports on a member whose professional conduct appears to have been unethical.

D. members self-disclose on their Professional Conduct Statement that they are involved in litigation regarding their investment advice.

Correct Answer:  D.......................................................................................................Reading 1‐a There is no mention of CFA Institute performing random checks on members’ (or candidates’) behavior. The circumstances that might prompt an enquiry are self-disclosure by a member, written complaints, media or other public sources providing information, or whenever a candidate is suspected of comprising their professional conduct during an examination.

Reference: CFA® Program Curriculum, Volume 1, p. 9. 

 

20.  Jonathan Seller, CFA, works for an investment bank that is acting as the principal underwriter for an issue of stock of a large tire manufacturer. Seller found out that the prospectus has concealed an impending  product  recall  due  to  a  quality  control  error.  Since  the  number  of  items  affected  is relatively  small,  the  product  recall  is  planned  to  be  a  quiet  affair. However  Seller  is  aware  that recently a competitor’s product recall received a large amount of adverse publicity. The preliminary prospectus has been distributed. According to the Code and Standards: 

A. Seller should do nothing as it may jeopardize the success of the issue.

B. Seller should revise the preliminary prospectus to include the omitted information to avoid any possible misrepresentation.

C. Seller should inform CFA Institute of the violation of the Code and Standards so he can clear himself of the possible misrepresentation.

D. Seller should inform his supervisor and let him/her deal with the situation since Seller himself should not jeopardize the success of the issue.

Correct Answer:  B....................................................................................................... Reading 2‐b 

Standard V requires that members shall make reasonable and diligent efforts to avoid any material misrepresentation in any research report or investment recommendation. B is the best answer.

Reference: CFA® Program Curriculum, Volume 1, pp. 80‐81. 

Page 47: Cfa Ques Answer

Ethical and Professional Standards   31 

 

21.  Tamara Deneuve, CFA, is an investment manager in charge of Asian equity portfolios. Together with her colleagues, she has developed a new proprietary valuation model for emerging markets in Asia.  Back  testing  using  12‐month  earnings  data,  the  valuation model  produces  favorable  results particularly when applied to certain industries, but not to others. Deneuve has decided to implement the new model to those industries but use the usual model for the others. According to the Code and Standards: 

A.  Deneuve must inform her clients prior to implementing the model. 

B.  Deneuve has the sole right to any proprietary model she has developed. 

C.  Deneuve should not implement the model since it can only be applied to certain industries. 

D.  Deneuve may implement the new model without informing her private clients since they would be unlikely to understand the model. 

 

 

 

22.  Which of the following is not a concept covered by the CFA Institute Code of Ethics? 

A.  Competence. 

B.  Integrity and diligence. 

C.  Independent judgment. 

D.  Remuneration levels of investment professionals. 

Page 48: Cfa Ques Answer

32   Study Session 01:  

 

21.  Tamara Deneuve, CFA, is an investment manager in charge of Asian equity portfolios. Together with her colleagues, she has developed a new proprietary valuation model for emerging markets in Asia.  Back  testing  using  12‐month  earnings  data,  the  valuation model  produces  favorable  results particularly when applied to certain industries, but not to others. Deneuve has decided to implement the new model to those industries but use the usual model for the others. According to the Code and Standards: 

A. Deneuve must inform her clients prior to implementing the model.

B. Deneuve has the sole right to any proprietary model she has developed.

C. Deneuve should not implement the model since it can only be applied to certain industries.

D. Deneuve may implement the new model without informing her private clients since they would be unlikely to understand the model.

Correct Answer:  A ........................................................................................... LOS: Reading  2‐b The application of a new valuation model may constitute a significant change to the investment process. Her clients must be informed in advance and given sufficient time to evaluate and decide whether such changes have a significant impact to their situation. This falls under Standard V(B) Communication with Clients and Prospective Clients.

Reference: CFA® Program Curriculum, Volume 1, pp. 83‐87. 

Page 49: Cfa Ques Answer

Ethical and Professional Standards   33 

 

22.  Which of the following is not a concept covered by the CFA Institute Code of Ethics? 

A. Competence.

B. Integrity and diligence.

C. Independent judgment.

D. Remuneration levels of investment professionals.

Correct Answer:  D ........................................................................................... LOS: Reading  1‐c Remuneration of investment professionals is not explicitly covered in the Code of Ethics. Disclosure of compensation is stipulated in Standard IV(B) Additional Compensation Arrangements and in Standard VI(C) Referral Fees.

Reference: CFA® Program Curriculum, Volume 1, pp. 12‐14. 

 

Page 50: Cfa Ques Answer

34   Study Session 02:  

 

Study session 02: Quantitative Methods: Basic Concepts

 

This  introductory study session presents the fundamentals of those quantitative techniques that are essential in almost any type of financial analysis, and which will be used throughout the remainder of the CFA curriculum. This session introduces two main building blocks of the quantitative analytical tool kit: (1) the time value of money and (2) statistics and probability theory. 

The time value of money concept is one of the main principles of financial valuation. The calculations based on this principle (e.g., present value, future value, and internal rate of return) are the basic tools used to support corporate finance decisions and estimate the fair value of fixed income, equity, or any other type of security or investment. 

Similarly, the basic concepts of statistics and probability theory constitute the essential tools used in describing  the main statistical properties of a population and understanding and applying various probability concepts in practice. 

 

Reading 5: The Time Value of Money 

Reading 6: Discounted Cash Flow Applications 

Reading 7: Statistical Concepts and Market Returns 

Reading 8: Probability Concepts 

 

 

 

Page 51: Cfa Ques Answer

Quantitative Methods: Basic Concepts   35 

 

1.  The following income streams will be paid from an investment: 

End year 1   $15,000 

End year 2   $25,000 

End year 3   $10,000 

 

At the end of year 3 the investment will have no remaining value. If the discount rate is 8% the present value of the investment is closest to: 

A.  $38,580.  

B.  $39,692. 

C.  $43,260. 

D.  $46,721. 

 

 

 

2.  An analyst states “ …. the odds against the company increasing its dividend are twelve to one”.  This means that the analyst believes that the probability of it increasing the dividend is closest to:  

A.  0.0769. 

B.  0.0833. 

C.  0.9166. 

D.  0.9230. 

 

Page 52: Cfa Ques Answer

36   Study Session 02:  

 

1.  The following income streams will be paid from an investment: 

End year 1 $15,000

End year 2 $25,000

End year 3 $10,000

At the end of year 3 the investment will have no remaining value. If the discount rate is 8% the present value of the investment is closest to:

A. $38,580.

B. $39,692.

C. $43,260.

D. $46,721.

Correct Answer:  C ........................................................................................... LOS: Reading  5‐d 

( ) ( )260,43$938,7$433,21$889,13$

08.1000,10$

08.1000,25$

08.1000,15$PV 32

=++=

++=

Reference: CFA® Program Curriculum, Volume 1, pp. 197‐198. 

 

 

2.  An analyst states “ …. the odds against the company increasing its dividend are twelve to one”.  This means that the analyst believes that the probability of it increasing the dividend is closest to:  

A. 0.0769.

B. 0.0833.

C. 0.9166.

D. 0.9230.

Correct Answer:  A ............................................................................................LOS: Reading  8‐c 

Odds against of twelve to one, means the probability is 1/(12 + 1) = 0.0769, there is a one in thirteen chance it will happen.

Reference: CFA® Program Curriculum, Volume 1, pp. 320‐321. 

Page 53: Cfa Ques Answer

Quantitative Methods: Basic Concepts   37 

 

3.  A portfolio increases in value from $10 million to $12 million over the first year. New cash of $2 million is then invested in the fund and the fund increases in value to $15 million at the end of the second year. The money‐weighted and time‐weighted rates of return are closest to (respectively): 

A.  12.8%, 13.4%. 

B.  12.8%, 27.1%. 

C.  13.4%, 22.5%. 

D.  14.0%, 22.5%.  

 

 

 

4.  If  a  credit  card  company  charges  interest  at  a  rate  of  15%  compounded monthly,  then  the effective annual rate of interest is closest to: 

A.  10.03%. 

B.  14.04%. 

C.  15.86%. 

D.  16.08%. 

Page 54: Cfa Ques Answer

38   Study Session 02:  

 

3.  A portfolio increases in value from $10 million to $12 million over the first year. New cash of $2 million is then invested in the fund and the fund increases in value to $15 million at the end of the second year. The money‐weighted and time‐weighted rates of return are closest to (respectively): 

A. 12.8%, 13.4%.

B. 12.8%, 27.1%.

C. 13.4%, 22.5%.

D. 14.0%, 22.5%.

Correct Answer:  A ............................................................................................LOS: Reading  6‐c  

The money-weighted return is calculated by solving

So, R = 12.8%.

The time-weighted return is the geometric average of the returns in each period, 2)r1()0714.1)(20.1( += so r = 13.39%

Reference: CFA® Program Curriculum, Volume 1, pp. 222‐223.  

 

 

4.  If  a  credit  card  company  charges  interest  at  a  rate  of  15%  compounded monthly,  then  the effective annual rate of interest is closest to: 

A. 10.03%.

B. 14.04%.

C. 15.86%.

D. 16.08%.

Correct Answer:  D............................................................................................LOS: Reading  5‐c 

( )%08.16

10125.01

1mr1EAR

12

mS

=−+=

−⎟⎠⎞

⎜⎝⎛ +=

Reference: CFA® Program Curriculum, Volume 1, pp. 182‐183. 

( ) ( )2R115

R1210

+=

++

Page 55: Cfa Ques Answer

Quantitative Methods: Basic Concepts   39 

 

5.  A manager  is offered  two  investments projects X and Y with net cash  flows,  in $ million,  from each investment as shown below. The cost of X is $2 million and the cost of Y is $10 million. The cost of capital for X is 10% and for Y is 8%. Which should be accepted for investment? 

End of Year  X  Y 

1  1.1  3.0 

2  1.8  9.0 

 

A.  Both projects should be accepted. 

B.  X should be accepted and Y rejected. 

C.  Y should be accepted and X rejected. 

D.  Both projects should be rejected. 

 

 

 

6.  Which is the lowest yield on a 90‐day Treasury bill? 

A.  Bank discount yield. 

B.  Holding period yield. 

C.  Money market yield. 

D.  Effective annual yield. 

Page 56: Cfa Ques Answer

40   Study Session 02:  

 

5.  A manager  is offered  two  investments projects X and Y with net cash  flows,  in $ million,  from each investment as shown below. The cost of X is $2 million and the cost of Y is $10 million. The cost of capital for X is 10% and for Y is 8%. Which should be accepted for investment? 

End of Year  X Y1  1.1 3.02  1.8 9.0

A.  Both projects should be accepted. 

B.  X should be accepted and Y rejected. 

C.  Y should be accepted and X rejected. 

D.  Both projects should be rejected. 

 

Correct Answer:  A ............................................................................................LOS: Reading  6‐a 

( )

( )49.0

08.10.9

08.10.310

r1CF

)Y(NPV

49.010.1

8.110.11.12

r1CF

)X(NPV

2

N

0tt

t

2

N

0tt

t

=++−=∑+

=

=++−=∑+

=

=

=

Both investments have positive NPVs so they should both be accepted.

Reference: CFA® Program Curriculum, Volume 1, pp. 214‐216. 

 

 

6.  Which is the lowest yield on a 90‐day Treasury bill? 

A. Bank discount yield.

B. Holding period yield.

C. Money market yield.

D. Effective annual yield.

Correct Answer:  B............................................................................................ LOS: Reading  6‐d The holding period yield is not an annualized figure, so for 90-day paper it will be the lowest yield. The highest will be the effective annual yield, followed by the money market yield, followed by the bank discount yield. This is a result of the compounding in the effective annual yield calculation, and the money market yield being based on the purchase price, whereas the bank discount yield is based on the maturity value.

Reference: CFA® Program Curriculum, Volume 1, pp. 229‐233. 

Page 57: Cfa Ques Answer

Quantitative Methods: Basic Concepts   41 

 

7.  The probability of a customer  in a restaurant ordering potatoes  is 40%,  the probability of  them ordering rice  is 60% and  the probability of  them ordering both  is 10%. What  is  the probability of a customer, chosen at random, ordering neither potatoes nor rice? 

A.  0.00. 

B.  0.10. 

C.  0.14. 

D.  0.24. 

 

 

 

8.  If  a  credit  card  company  charges  interest  at  a  rate  of  15%  compounded monthly,  then  the effective annual rate of interest is closest to: 

A.  10.03%. 

B.  14.04%. 

C.  15.86%. 

D.  16.08%. 

Page 58: Cfa Ques Answer

42   Study Session 02:  

 

7.  The probability of a customer  in a restaurant ordering potatoes  is 40%,  the probability of  them ordering rice  is 60% and  the probability of  them ordering both  is 10%. What  is  the probability of a customer, chosen at random, ordering neither potatoes nor rice? 

A. 0.00.

B. 0.10.

C. 0.14.

D. 0.24.

Correct Answer:  B.............................................................................................LOS: Reading  8‐e Use the general rule of addition to calculate the probability that a customer orders either potatoes or rice:

P(A or B) = P(A) + P(B) – P(A and B)

= 0.4 + 0.6 – 0.1 = 0.9

The probability of a customer ordering neither is 1 – 0.9 = 0.1

Reference: CFA® Program Curriculum, Volume 1, pp. 325‐326. 

 

 

 

8.  If  a  credit  card  company  charges  interest  at  a  rate  of  15%  compounded monthly,  then  the effective annual rate of interest is closest to: 

A. 10.03%.

B. 14.04%.

C. 15.86%.

D. 16.08%.

Correct Answer:  D............................................................................................LOS: Reading  5‐c 

( )%08.16

10125.01

1mr1EAR

12

mS

=−+=

−⎟⎠⎞

⎜⎝⎛ +=

Reference: CFA® Program Curriculum, Volume 1, pp. 182‐183. 

Page 59: Cfa Ques Answer

Quantitative Methods: Basic Concepts   43 

 

9.  A shop which sells matches knows that 14 out of 20 boxes of matches will contain 100 matches exactly; the remainder will contain more than 100 matches. The probability of a customer picking up a box of matches that contains more than 100 matches, and then picking up a second box containing more than 100 matches is closest to: 

A.  0.06. 

B.  0.08. 

C.  0.09. 

D.  0.12. 

 

 

 

10.  If P(A|B) = P(A) then the events A and B are: 

A.   exhaustive. 

B.   independent. 

C.   mutually exclusive. 

D.   equally likely to occur. 

Page 60: Cfa Ques Answer

44   Study Session 02:  

 

9.  A shop which sells matches knows that 14 out of 20 boxes of matches will contain 100 matches exactly; the remainder will contain more than 100 matches. The probability of a customer picking up a box of matches that contains more than 100 matches, and then picking up a second box containing more than 100 matches is closest to: 

A. 0.06.

B. 0.08.

C. 0.09.

D. 0.12.

Correct Answer:  B.............................................................................................LOS: Reading  8‐e Use the general rule of multiplication, which says that:

P(A and B) = P(A) P(B|A)

= 6/20 × 5/19

= 0.08

Reference: CFA® Program Curriculum, Volume 1, pp. 323‐324. 

 

 

 

10.  If P(A|B) = P(A) then the events A and B are: 

A. exhaustive.

B. independent.

C. mutually exclusive.

D. equally likely to occur.

Correct Answer:  B............................................................................................. LOS: Reading  8‐f Two events are independent if the occurrence of one event does not affect the probability of the other event occurring.

Reference: CFA® Program Curriculum, Volume 1, p. 327. 

Page 61: Cfa Ques Answer

Quantitative Methods: Basic Concepts   45 

 

11.  A mortgage has an annual quoted  interest rate of 12 percent.    If mortgage payments are made monthly, then the effective annual interest rate is closest to: 

A.  11.40%. 

B.  12.36%. 

C.  12.55%. 

D.  12.68%. 

 

 

 

12.  An  investor puts $50,000 into a mutual fund at the end of each quarter and his purchase prices are $20, $25, $28, $23. The average price that he pays per share is closest to: 

A.  $23.12. 

B.  $23.64. 

C.  $24.00. 

D.  $24.50. 

Page 62: Cfa Ques Answer

46   Study Session 02:  

 

11.  A mortgage has an annual quoted  interest rate of 12 percent.    If mortgage payments are made monthly, then the effective annual interest rate is closest to: 

A. 11.40%.

B. 12.36%.

C. 12.55%.

D. 12.68%.

Correct Answer:  D............................................................................................LOS: Reading  5‐c The effective annual rate (EAR) is given by:

1mr

1EARm

S −⎟⎠⎞

⎜⎝⎛ +=

where

m = number of compounding periods per year

rs = quoted annual interest rate

EAR = (1.01)12 - 1

= 12.68%

Reference: CFA® Program Curriculum, Volume 1, pp. 179‐180. 

 

12.  An  investor puts $50,000 into a mutual fund at the end of each quarter and his purchase prices are $20, $25, $28, $23. The average price that he pays per share is closest to: 

A. $23.12.

B. $23.64.

C. $24.00.

D. $24.50.

Correct Answer:  B............................................................................................ LOS: Reading  7‐d The harmonic mean is the average price on a per share basis.

64.23$1692.04

231

281

251

201

4

X1

nXn

1i i

H ==⎟⎠⎞

⎜⎝⎛ +++

=

∑ ⎟⎟⎠

⎞⎜⎜⎝

⎛=

=

Reference: CFA® Program Curriculum, Volume 1, p. 272. 

Page 63: Cfa Ques Answer

Quantitative Methods: Basic Concepts   47 

 

13.  The value  of  a portfolio  starts  at  $100 million,  at  the  end  of  the  first  year  it has  fallen  to  $80 million, at the end of the second year it has risen to $105 million, and at the end of the third year it has risen to $115 million.  The geometric mean rate of return is closest to: 

A.  3.2%. 

B.  4.1%. 

C.  4.8%. 

D.  6.7%. 

 

 

 

14.  A deposit of $1,000,000 earns a return of 5% compounded continuously for 8 years.   The future value is closest to: 

A.  $1,400,000. 

B.  $1,477,000. 

C.  $1,492,000. 

D.  $1,500,000. 

Page 64: Cfa Ques Answer

48   Study Session 02:  

 

13.  The value  of  a portfolio  starts  at  $100 million,  at  the  end  of  the  first  year  it has  fallen  to  $80 million, at the end of the second year it has risen to $105 million, and at the end of the third year it has risen to $115 million.  The geometric mean rate of return is closest to: 

A. 3.2%.

B. 4.1%.

C. 4.8%.

D. 6.7%.

Correct Answer:  C ........................................................................................... LOS: Reading  7‐d 

1)R1).....(R1)(R1(R n21G −+++= =

= (0.80 x 1.313 x 1.095) 1/3 – 1

= 0.0476

Reference: CFA® Program Curriculum, Volume 1, pp. 269‐270. 

 

14.  A deposit of $1,000,000 earns a return of 5% compounded continuously for 8 years.   The future value is closest to: 

A. $1,400,000.

B. $1,477,000.

C. $1,492,000.

D. $1,500,000.

Correct Answer:  C ........................................................................................... LOS: Reading  8‐d 

The future value is given by

Note: to find the programmed value of e on your CFA Institute-approved financial calculator, use the keystrokes:

HP‐12C  1  g   ex   2.7183 BA II Plus  1  2nd  [ex]  2.7183  Reference: CFA® Program Curriculum, Volume 1, pp. 171‐182. 

825,491,1$e000,000,1$

ePVFV805.0

NrN

S

==

×

Page 65: Cfa Ques Answer

Quantitative Methods: Basic Concepts   49 

 

15.  There  is a competition  in which  there are six contestants and you need  to pick winners  for 1st, 2nd and 3rd places, how many ways can they be selected?  

A.  20. 

B.  36. 

C.  120. 

D.  720. 

 

 

 

16.  The following information was collected on the average numbers of hours worked per day by the 15 employees of a shop over the last month:  

5  5  8  4  4  6  7  6 

6  5  4  2  7  7  5     

The mean, median and mode are closest to: 

    Mean    Median   Mode  

A.    5.4    5.0    6.0  

B.    5.0    6.0    5.4 

C.    5.4    5.0    5.0 

D.    5.0    6.0    5.0 

Page 66: Cfa Ques Answer

50   Study Session 02:  

 

15.  There  is a competition  in which  there are six contestants and you need  to pick winners  for 1st, 2nd and 3rd places, how many ways can they be selected?  

A.  20. 

B.  36. 

C.  120. 

D.  720. 

 

Correct Answer:  C ........................................................................................... LOS: Reading  8‐n This requires the permutation formula since the order of the r objects matters, so apply:

Number of ways = )!rn(!n− = 6!/(6 – 3)! = 120

Reference: CFA® Program Curriculum, Volume 1, p. 356. 

 

 

16.  The following information was collected on the average numbers of hours worked per day by the 15 employees of a shop over the last month:  

5 5 8 4 4 6 7 6

6 5 4 2 7 7 5

The mean, median and mode are closest to:

Mean Median Mode

A. 5.4 5.0 6.0

B. 5.0 6.0 5.4

C. 5.4 5.0 5.0

D. 5.0 6.0 5.0

Correct Answer:  C ........................................................................................... LOS: Reading  7‐d 

Mean = X = ∑∑

i

ii

wXw

= [1(2) + 3(4) + 4(5) +3(6) +3(7) +1(8)]/15 = 5.4

The median is the middle observation, which is 5.

The mode is the most frequently occurring observation, which is 5.

Reference: CFA® Program Curriculum, Volume 1, pp. 255‐263. 

Page 67: Cfa Ques Answer

Quantitative Methods: Basic Concepts   51 

 

17.  Which of  the  following statements  is most accurate regarding a money‐weighted rate of return for a portfolio? 

A.  It is the internal rate of return. 

B.  It is always lower than the time‐weighted rate of return. 

C.  It is the arithmetic average of the periodic rates of return. 

D.  It is the geometric average of the periodic rates of return. 

 

 

 

18.  Conditional probability refers to: 

A.  the probability that an event will happen more than once.  

B.  the probability that two or more events will occur concurrently. 

C.  the probability that one of two mutually exclusive events will occur. 

D.  the probability of a particular event occurring given that another event has already occurred. 

Page 68: Cfa Ques Answer

52   Study Session 02:  

 

17.  Which of  the  following statements  is most accurate regarding a money‐weighted rate of return for a portfolio? 

A. It is the internal rate of return.

B. It is always lower than the time-weighted rate of return.

C. It is the arithmetic average of the periodic rates of return.

D. It is the geometric average of the periodic rates of return.

Correct Answer:  A ............................................................................................LOS: Reading  6‐c B will not be true if there have been cash flows into the portfolio before superior performance. C and D refer to arithmetic and geometric time-weighted rates of return.

Reference: CFA® Program Curriculum, Volume 1, pp. 221‐229. 

 

 

 

18.  Conditional probability refers to: 

A. the probability that an event will happen more than once.

B. the probability that two or more events will occur concurrently.

C. the probability that one of two mutually exclusive events will occur.

D. the probability of a particular event occurring given that another event has already occurred.

Correct Answer:  D........................................................................................... LOS: Reading  8‐d Conditional probability is the probability of an event occurring given another event has already occurred. It is ‘conditional’ on the other event.

Reference: CFA® Program Curriculum, Volume 1, p. 322. 

Page 69: Cfa Ques Answer

Quantitative Methods: Basic Concepts   53 

 

19.  The following data is provided on the quarterly performance of a fund.   

  1st Quarter 2nd Quarter 3rd Quarter  4th QuarterBeginning value $1,000,000 $1,200,000 $1,500,000  $1,500,000

Cash inflow at beginning $100,000 $100,000 ($200,000)  $300,000Ending value $1,200,000 $1,500,000 $1,500,000  $1,600,000

The time‐weighted return for the year is closest to: 

A.  6.6%. 

B.  28.7%. 

C.  29.1%. 

D.  61.4%. 

 

 

 

20.  Stock A has a coefficient of variation of 30% and  stock B has a coefficient of variation of 60%.  Which of the following statements is the most accurate? 

A. The dispersion of returns relative to the mean is lower for stock A than stock B.

B. The dispersion of returns relative to the mean is higher for stock A than stock B.

C. The standard deviation of stock A is double that of stock B, if the mean returns of both stocks are the same.

D. The variance of stock A is double that of stock B, if the mean returns of both stocks are the same.

Page 70: Cfa Ques Answer

54   Study Session 02:  

 

19.  The following data is provided on the quarterly performance of a fund.   

  1st Quarter 2nd Quarter 3rd Quarter 4th Quarter Beginning value $1,000,000 $1,200,000 $1,500,000 $1,500,000 

Cash inflow at beginning $100,000 $100,000 ($200,000) $300,000 Ending value $1,200,000 $1,500,000 $1,500,000 $1,600,000 

The time‐weighted return for the year is closest to: 

A. 6.6%.

B. 28.7%.

C. 29.1%.

D. 61.4%.

Correct Answer:  C ............................................................................................LOS: Reading  6‐c The first step is to calculate the returns for each quarter:

Q1, HPR = ($1,200,000 - $1,100,000)/$1,100,000 = 9.09%

Q2, HPR = ($1,500,000 - $1,300,000)/$1,300,000 = 15.38%

Q3, HPR = ($1,500,000 - $1,300,000)/$1,300,000 = 15.38%

Q4, HPR = ($1,600,000 - $1,800,000)/$1,800,000 = -11.11%

Then calculate the geometric average to get the time-weighted return:

(1.0909) x (1.1538) x (1.1538) x (0.8889) –1 = 0.2909 = 29.1%

Reference: CFA® Program Curriculum, Volume 1, pp. 223‐226. 

Page 71: Cfa Ques Answer

Quantitative Methods: Basic Concepts   55 

 

20.  Stock A has a coefficient of variation of 30% and  stock B has a coefficient of variation of 60%.  Which of the following statements is the most accurate? 

A. The dispersion of returns relative to the mean is lower for stock A than stock B.

B. The dispersion of returns relative to the mean is higher for stock A than stock B.

C. The standard deviation of stock A is double that of stock B, if the mean returns of both stocks are the same.

D. The variance of stock A is double that of stock B, if the mean returns of both stocks are the same.

Correct Answer:  A............................................................................................LOS: Reading  7‐h 

X 100

Answers C and D are incorrect since, if the mean return is the same the standard deviation of stock A is half that of B. The coefficient of variation measures the dispersion of returns relative to the mean return.

Reference: CFA® Program Curriculum, Volume 1, pp. 292‐294. 

Xs

=CV

Page 72: Cfa Ques Answer

56   Study Session 02:  

 

21.  A shop buys pens from two manufacturers, 55% from Mountain Pens and 45% from Valley Pens.  The shop knows  that 3% of  the pens supplied by Mountain Pens are defective and 5% of  the pens supplied by Valley Pens are defective.  The pens in the shops are mixed together. If a pen is chosen at random and found to be defective what is the probability that it was supplied by Mountain Pens? 

A.  33.3%. 

B.  42.3%. 

C.  55.0%. 

D.  60.0%. 

Page 73: Cfa Ques Answer

Quantitative Methods: Basic Concepts   57 

 

22.  A person invests $10,000 at the end of each year for the next ten years, if the investment earns 6% interest annually, the value of the investment at the end of ten years will be closest to: 

A. $131,808.

B. $134,350.

C. $139,708.

D. $149,708.

Page 74: Cfa Ques Answer

58   Study Session 02:  

 

21.  A shop buys pens from two manufacturers, 55% from Mountain Pens and 45% from Valley Pens.  The shop knows  that 3% of  the pens supplied by Mountain Pens are defective and 5% of  the pens supplied by Valley Pens are defective.  The pens in the shops are mixed together. If a pen is chosen at random and found to be defective what is the probability that it was supplied by Mountain Pens? 

A. 33.3%.

B. 42.3%.

C. 55.0%.

D. 60.0%.

Correct Answer:  B........................................................................................... LOS: Reading  8‐m 

Apply Bayes’ formula:

( ) ( ) ( )( ) ( ) ( ) ( )2211

111 A|BPAPA|BPAP

A|BPAPB|AP

+=

define A1 - Mountain Pens supply the pen

A2 - Valley Pens supply the pen

B - the information that the pen is defective

( ) ( ) ( )05.045.003.055.003.055.0B|AP 1 ×+×

×=

= 0.423

Reference: CFA® Program Curriculum, Volume 1, pp. 349‐353. 

Page 75: Cfa Ques Answer

Quantitative Methods: Basic Concepts   59 

 

22.  A person invests $10,000 at the end of each year for the next ten years, if the investment earns 6% interest annually, the value of the investment at the end of ten years will be closest to: 

A. $131,808.

B. $134,350.

C. $139,708.

D. $149,708.

Correct Answer:  A............................................................................................LOS: Reading  5‐d This is a question asking for the future value of an annuity, use the annuity formula or a financial calculator:

This is an annuity type question.

Or use a financial calculator.  

Explicit use of annuity formula: Calculator keystrokes for: 

( )

( )

808,131$=1808.13×000,=

06.0106.0+1

000,=

r1r+1

=

=N==r

=

10

N

$10

$10

AFV

100.066%

$10,000A

N  

HP‐12C 

f CLEAR FIN

10000 CHS PMT

-10,000.00

6 i 6.00

10 n 10.00

FV FV = 131,807.95

BA II Plus 

2nd [QUIT] 2nd [CLR TVM]

10000 +/- PMT = -10,000.00

6 I/Y I/Y = 6.00

10 N N = 10.00

CPT FV FV = 131,807.95

Reference: CFA® Program Curriculum, Volume 1, pp. 183‐185. 

 

Page 76: Cfa Ques Answer

60   Study Session 03:  

 

Study Session 03: Quantitative Methods: Application

 

This  study  session  introduces  the  discrete  and  continuous  probability  distributions  that  are most commonly used to describe the behavior of random variables. Probability theory and calculations are widely  applied  in  finance,  for  example,  in  the  field  of  investment  and  project  valuation  and  in financial risk management. 

Furthermore,  this  session  teaches  how  to  estimate  different  parameters  (e.g., mean  and  standard deviation)  of  a  population  if  only  a  sample,  rather  than  the whole  population,  can  be  observed. Hypothesis testing is a closely related topic. This session presents the techniques that can be applied to accept or reject an assumed hypothesis (null hypothesis) about various parameters of a population. Finally, you will also learn about the fundamentals of technical analysis. It is important that analysts properly  understand  the  assumptions  and  limitations when  applying  these  tools  as mis‐specified models or improperly used tools can result in misleading conclusions. 

 

Reading 9: Common Probability Distributions 

Reading 10: Sampling and Estimation 

Reading 11: Hypothesis Testing 

Reading 12: Technical Analysis 

 

Page 77: Cfa Ques Answer

Quantitative Methods: Applications   61 

 

1.  An analyst is selecting a sample from the orders received from a firm’s customers. The orders are time stamped and he decides to include in the sample every 5th order received by the firm. This is an example of: 

A.  cluster sampling. 

B.  simple random sampling. 

C.  stratified random sampling.  

D.  systematic random sampling. 

 

 

 

2.  Which of the following statements regarding normal distributions is most accurate?  

A.  The normal distribution is a good model for the distribution of asset prices.  

B.   The mean and standard deviation of the standard normal distribution are both 1. 

C.  A standard normal distribution with low standard deviation will be less peaked than a standard normal distribution with higher standard deviation. 

D.  If the returns on individual securities in a portfolio are normally distributed, the returns of a portfolio containing these securities are normally distributed. 

 

 

Page 78: Cfa Ques Answer

62   Study Session 03:  

 

1.  An analyst is selecting a sample from the orders received from a firm’s customers. The orders are time stamped and he decides to include in the sample every 5th order received by the firm. This is an example of: 

A. cluster sampling.

B. simple random sampling.

C. stratified random sampling.

D. systematic random sampling.

Correct Answer:  D......................................................................................... LOS: Reading  10‐b Systematic random sampling is when we select every kth member of a population, often used when we cannot number or label the members of a population.

Reference: CFA® Program Curriculum, Volume 1, pp. 422‐423. 

 

 

 

2.  Which of the following statements regarding normal distributions is most accurate?  

A. The normal distribution is a good model for the distribution of asset prices.

B. The mean and standard deviation of the standard normal distribution are both 1.

C. A standard normal distribution with low standard deviation will be less peaked than a standard normal distribution with higher standard deviation.

D. If the returns on individual securities in a portfolio are normally distributed, the returns of a portfolio containing these securities are normally distributed.

Correct Answer:  D............................................................................................ LOS: Reading  9‐f The normal distribution is a good model for many asset returns but not prices. By definition a standard normal distribution has a mean of 0 and standard deviation of 1, so statements B and C are not correct. Statement D is accurate.

Reference: CFA® Program Curriculum, Volume 1, pp. 389‐397. 

Page 79: Cfa Ques Answer

Quantitative Methods: Applications   63 

 

3.  A hypothesis is a statement about a: 

A.  sample statistic. 

B.  sample parameter. 

C.  population statistic. 

D.  population parameter. 

 

 

 

 

4.  A company is analyzing the days that employees take off as sick leave each year and is concerned that the number of days that employees are taking off has risen above the past average number of 4.0 days.  It  is  assumed  that  the  population  is  approximately  normally  distributed.  A  sample  of  20 employees is taken and the mean number of days taken is 4.5 with a standard deviation of 1.5 days. If the rejection point for a one‐tailed test with 19 degrees of freedom is 1.729 at the 5% significance level we can conclude that: 

A.  using the t‐statistic is not valid for a small sample. 

B.  the mean number of days is more than 4 days at the 5% significance level. 

C.  the mean number of days is still 4 days or less at the 5% significance level. 

D.  we should use the z‐test, not the t‐test, if the population is approximately normally distributed. 

Page 80: Cfa Ques Answer

64   Study Session 03:  

 

3.  A hypothesis is a statement about a: 

A. sample statistic.

B. sample parameter.

C. population statistic.

D. population parameter.

Correct Answer:  D..........................................................................................LOS: Reading  11‐a A hypothesis is a statement about a population parameter. We use data from a sample to test whether we should accept or reject the hypothesis.

Reference: CFA® Program Curriculum, Volume 1, pp. 456‐457. 

 

 

4.  A company is analyzing the days that employees take off as sick leave each year and is concerned that the number of days that employees are taking off has risen above the past average number of 4.0 days.  It  is  assumed  that  the  population  is  approximately  normally  distributed.  A  sample  of  20 employees is taken and the mean number of days taken is 4.5 with a standard deviation of 1.5 days. If the rejection point for a one‐tailed test with 19 degrees of freedom is 1.729 at the 5% significance level we can conclude that: 

A. using the t-statistic is not valid for a small sample.

B. the mean number of days is more than 4 days at the 5% significance level.

C. the mean number of days is still 4 days or less at the 5% significance level.

D. we should use the z-test, not the t-test, if the population is approximately normally distributed.

Correct Answer:  C ..........................................................................................LOS: Reading  11‐e 

Set the null hypothesis as H0: µ ≤ 4.0

We can use the t-test since we are assuming the population is normally distributed.

49.1335.05.0

205.10.45.4

ns

xt19 ==

−=

µ−=

This is within the confidence interval so we do not reject the null hypothesis.

Reference: CFA® Program Curriculum, Volume 1, pp. 466‐474. 

Page 81: Cfa Ques Answer

Quantitative Methods: Applications   65 

 

5.  The standard deviation of a population is 25 and a sample of 50 observations  is taken from the population.  The standard error of the sample mean is closest to: 

A.  0.10. 

B.  0.28. 

C.  3.54. 

D.  10.00. 

 

 

 

6.  Which of the following statements regarding lognormal distributions is most accurate? 

A.  The distribution is bell shaped. 

B.  The mean of the distribution is zero. 

C.  Y is a lognormal distribution if LnY is normally distributed. 

D.  Lognormal distributions are frequently used to reflect the distribution of stock returns.  

Page 82: Cfa Ques Answer

66   Study Session 03:  

 

5.  The standard deviation of a population is 25 and a sample of 50 observations  is taken from the population.  The standard error of the sample mean is closest to: 

A. 0.10.

B. 0.28.

C. 3.54.

D. 10.00.

Correct Answer:  C ..........................................................................................LOS: Reading  10‐e 

The standard error of the sample mean is given by nx

σ=σ

= 25/7.07 = 3.54.

Reference: CFA® Program Curriculum, Volume 1, p. 429. 

 

 

 

6.  Which of the following statements regarding lognormal distributions is most accurate? 

A. The distribution is bell shaped.

B. The mean of the distribution is zero.

C. Y is a lognormal distribution if LnY is normally distributed.

D. Lognormal distributions are frequently used to reflect the distribution of stock returns.

Correct Answer:  C ............................................................................................ LOS: Reading  9‐j A. is not correct, they are positively skewed distributions.

B. is not correct, the lower bound is zero.

D. is not correct, they are frequently used to show the distribution of stock prices.

Reference: CFA® Program Curriculum, Volume 1, pp. 400‐406. 

Page 83: Cfa Ques Answer

Quantitative Methods: Applications   67 

 

7.  A stock market rises by an average of 10% a year and the standard deviation of returns is 6%. The probability of the stock market falling by more than 2% in a year is closest to: 

A.  2.3%. 

B.  4.6%. 

C.  15.9%. 

D.  31.7%. 

 

 

 

8.  The  Central  Limit  Theorem  states  that  if  the  sampling  distribution  of  the  sample  mean  is calculated using samples of equal size from a population that is not normal, then: 

A.  it is approximately a normal distribution. 

B.  the standard error tends to one as the size of the samples increases.  

C.  the mean of the distribution will be smaller than the population mean. 

D.  the dispersion of the distribution is more than the dispersion of the population. 

Page 84: Cfa Ques Answer

68   Study Session 03:  

 

7.  A stock market rises by an average of 10% a year and the standard deviation of returns is 6%. The probability of the stock market falling by more than 2% in a year is closest to: 

A. 2.3%.

B. 4.6%.

C. 15.9%.

D. 31.7%.

Correct Answer:  A ............................................................................................LOS: Reading  9‐g A fall of 2% is two standard deviations from the mean. 95.4% of observations fall between the mean plus or minus two standard deviations so 4.6% lie outside this range. Therefore 2.3% will be below -2% and 2.3% will be above 22%.

Reference: CFA® Program Curriculum, Volume 1, pp. 389‐393. 

 

 

 

8.  The  Central  Limit  Theorem  states  that  if  the  sampling  distribution  of  the  sample  mean  is calculated using samples of equal size from a population that is not normal, then: 

A. it is approximately a normal distribution.

B. the standard error tends to one as the size of the samples increases.

C. the mean of the distribution will be smaller than the population mean.

D. the dispersion of the distribution is more than the dispersion of the population.

Correct Answer:  A .......................................................................................... LOS: Reading  10‐f B. is not correct, the standard error tends to zero as the sample size increases.

C. is not correct, the mean of the distribution will be the same as the population mean.

D. is not correct, the dispersion is less than that of the population, or equal if n = 1.

A is correct, the distribution will be approximately normal although the underlying population may not be normally distributed.

Reference: CFA® Program Curriculum, Volume 1, pp. 428‐431. 

Page 85: Cfa Ques Answer

Quantitative Methods: Applications   69 

 

9.  Historic  analysis  suggests  that  stocks  trading  on  a  low  price/book  value  have  tended  to outperform the market.  If the analysis has not included companies that have gone bankrupt then the analysis could be biased due to: 

A.  look‐ahead bias. 

B.  data‐mining bias. 

C.  survivorship bias. 

D.  data‐snooping bias. 

 

 

 

10.  Hypothesis  testing  is used  to determine  the mean return of mutual  funds  in a single period. A sample of 25 funds is taken and the null hypothesis is defined as the mean is equal to 12%.  Is this an example of a one or two‐tailed test, and should the z‐test or t‐test be used?  

  Hypothesis test  Test statistic 

A.  a one‐tailed test  t‐test 

B.  a one‐tailed test  z‐test 

C.  a two‐tailed test  t‐test 

D.  a two‐tailed test  z‐test 

Page 86: Cfa Ques Answer

70   Study Session 03:  

 

9.  Historic  analysis  suggests  that  stocks  trading  on  a  low  price/book  value  have  tended  to outperform the market.  If the analysis has not included companies that have gone bankrupt then the analysis could be biased due to: 

A. look-ahead bias.

B. data-mining bias.

C. survivorship bias.

D. data-snooping bias.

Correct Answer:  C ......................................................................................... LOS: Reading  10‐k 

It has been argued that if all companies had been included, the companies that went bankrupt would have been trading on low price/book multiples, and this would have lowered the average performance of the low price/book value stocks.

Reference: CFA® Program Curriculum, Volume 1, pp. 444‐446. 

 

 

 

10.  Hypothesis  testing  is used  to determine  the mean return of mutual  funds  in a single period. A sample of 25 funds is taken and the null hypothesis is defined as the mean is equal to 12%.  Is this an example of a one or two‐tailed test, and should the z‐test or t‐test be used?  

Hypothesis test Test statistic

A. a one-tailed test t-test

B. a one-tailed test z-test

C. a two-tailed test t-test

D. a two-tailed test z-test

Correct Answer:  C ..........................................................................LOS: Reading  11‐a and 11‐e If there is no direction (greater or less than) in the null hypothesis it is two-tailed. Give the sample size of less than 30 funds the t-test should be used. The z-test should only be used for large samples.

Reference: CFA® Program Curriculum, Volume 1, pp. 458 and 466‐474. 

Page 87: Cfa Ques Answer

Quantitative Methods: Applications   71 

 

11.  An analyst  is  studying  the monthly  income of workers at a  factory.   He  is  told  that  the mean income is $5,000 per month and he decided to look at a sample 100 workers to see if this is correct.  The  sample mean  is  $5,018 with  a  standard deviation of  80.    If he  sets  the null hypothesis  as  the population mean is $5,000 he should: 

A.  reject the null hypothesis at both the 1% and 5% significance level.  

B.  not reject the null hypothesis at both the 1% and 5% significance level.  

C.  not reject the null hypothesis at the 1% significance level but reject it at the 5% significance level.  

D.  not reject the null hypothesis at the 5% significance level but reject it at the 1% significance level.  

 

 

 

12.  In hypothesis tasting a p‐value of 0.1 indicates that:  

A.  there is extremely strong evidence that H0 should be rejected. 

B.  there is extremely strong evidence that H0 should not be rejected. 

C.  there is a probability of 0.1 of observing a sample value at least as extreme as the value observed, assuming H0 is correct. 

D.  there is a probability of 0.1 of observing a sample value at least as extreme as the value observed, assuming H0 is rejected. 

Page 88: Cfa Ques Answer

72   Study Session 03:  

 

11.  An analyst  is  studying  the monthly  income of workers at a  factory.   He  is  told  that  the mean income is $5,000 per month and he decided to look at a sample 100 workers to see if this is correct.  The  sample mean  is  $5,018 with  a  standard deviation of  80.    If he  sets  the null hypothesis  as  the population mean is $5,000 he should: 

A. reject the null hypothesis at both the 1% and 5% significance level.

B. not reject the null hypothesis at both the 1% and 5% significance level.

C. not reject the null hypothesis at the 1% significance level but reject it at the 5% significance level.

D. not reject the null hypothesis at the 5% significance level but reject it at the 1% significance level.

Correct Answer:  C ..........................................................................................LOS: Reading  11‐e Z = (5018-5000)/8 = 2.25. This is less than 2.58 (the critical value for the 1% significance level) so there is no evidence to reject the null hypothesis at the 1% level.. However it is more than 1.96 (the critical value for the 5% significance level) so we can reject the hypothesis at the 5% significance level.

Reference: CFA® Program Curriculum, Volume 1, pp. 466‐474. 

 

 

 

12.  In hypothesis tasting a p‐value of 0.1 indicates that:  

A. there is extremely strong evidence that H0 should be rejected.

B. there is extremely strong evidence that H0 should not be rejected.

C. there is a probability of 0.1 of observing a sample value at least as extreme as the value observed, assuming H0 is correct.

D. there is a probability of 0.1 of observing a sample value at least as extreme as the value observed, assuming H0 is rejected.

Correct Answer:  C ..........................................................................................LOS: Reading  11‐c The p-value is the probability of observing a value as extreme or more extreme than the value observed, for it to be extremely strong evidence it would need to be 0.001 or less.

Reference: CFA® Program Curriculum, Volume 1, pp. 465‐466. 

Page 89: Cfa Ques Answer

Quantitative Methods: Applications   73 

 

13.  You are analyzing the monthly returns from a fund over the last year and calculate that the mean return was 1.25% with a sample standard deviation of 1.0%.  You expected the fund to have achieved a return of 1.4% in line with the risk taken on by the fund and wish to decide at the 10% significance level whether the results are consistent with a population mean return of 1.4%.  Given that t0.05,11= 1.796 you can conclude that: 

A.  the null hypothesis is rejected and the results are consistent with a mean of 1.4%.  

B.  the null hypothesis is not rejected and the results are consistent with a mean of 1.4%.  

C.  the null hypothesis is rejected and the results are not consistent with a mean of 1.4% 

D.  the null hypothesis is not rejected and the results are not consistent with a mean of 1.4% 

 

 

 

14.  A chi‐square test statistic (2χ ) could be used for hypothesis tests for 

A.  the mean for a single normally distributed population. 

B.  the variance for a single normally distributed population. 

C.  the mean for a single non‐normally distributed population. 

D.  the variance for a single non‐normally distributed population. 

Page 90: Cfa Ques Answer

74   Study Session 03:  

 

13.  You are analyzing the monthly returns from a fund over the last year and calculate that the mean return was 1.25% with a sample standard deviation of 1.0%.  You expected the fund to have achieved a return of 1.4% in line with the risk taken on by the fund and wish to decide at the 10% significance level whether the results are consistent with a population mean return of 1.4%.  Given that t0.05,11= 1.796 you can conclude that: 

A. the null hypothesis is rejected and the results are consistent with a mean of 1.4%.

B. the null hypothesis is not rejected and the results are consistent with a mean of 1.4%.

C. the null hypothesis is rejected and the results are not consistent with a mean of 1.4%

D. the null hypothesis is not rejected and the results are not consistent with a mean of 1.4%

Correct Answer:  B...........................................................................................LOS: Reading  11‐e 

Set the null hypothesis as H0: µ = 1.4%. We need to apply the t-test because it is a small sample.

52.029.015.0

120.14.125.1

ns

xt 1n −=−=

−=

µ−=−

This is within the confidence interval so the null hypothesis is not rejected and the results are consistent with the mean return being 1.4%.

Reference: CFA® Program Curriculum, Volume 1, pp. 466‐474. 

 

 

14.  A chi‐square test statistic (2χ ) could be used for hypothesis tests for 

A. the mean for a single normally distributed population.

B. the variance for a single normally distributed population.

C. the mean for a single non-normally distributed population.

D. the variance for a single non-normally distributed population.

Correct Answer:  B........................................................................................... LOS: Reading  11‐f A chi-square test statistic is used to test for variance when we have a normally distributed population. It is always 0 or a positive number and has different distributions based on the number of degrees of freedom.

Reference: CFA® Program Curriculum, Volume 1, pp. 482‐484. 

Page 91: Cfa Ques Answer

Quantitative Methods: Applications   75 

 

15.  Large  samples  are  taken  from  a  normal  population;  the  sample mean  is  25  and  the  sample standard deviation is 5. The 95% confidence interval for the population mean is closest to: 

A.  between 12.1 and 37.9. 

B.  between 15.2 and 34.8. 

C.  between 20.0 and 30.0. 

D.  between 15.0 and 40.0. 

 

 

 

16.  When  selecting  a  sample  a  population  is  first  divided  into  subpopulations  and  then  random samples  are  taken  from  each  subpopulation,  the  number  taken  proportional  to  the  size  of  the subpopulation. This is an example of: 

A.  structured sampling. 

B.  systematic sampling. 

C.  simple random sampling. 

D.  stratified random sampling. 

Page 92: Cfa Ques Answer

76   Study Session 03:  

 

15.  Large  samples  are  taken  from  a  normal  population;  the  sample mean  is  25  and  the  sample standard deviation is 5. The 95% confidence interval for the population mean is closest to: 

A. between 12.1 and 37.9.

B. between 15.2 and 34.8.

C. between 20.0 and 30.0.

D. between 15.0 and 40.0.

Correct Answer:  B........................................................................................... LOS: Reading  10‐f The 95% confidence interval is given by sample mean plus or minus 1.96 standard deviations.

This is ( ) 8.925596.125 ±=×± which is between 15.2 and 34.8.

Reference: CFA® Program Curriculum, Volume 1, pp. 433‐435. 

 

 

 

16.  When  selecting  a  sample  a  population  is  first  divided  into  subpopulations  and  then  random samples  are  taken  from  each  subpopulation,  the  number  taken  proportional  to  the  size  of  the subpopulation. This is an example of: 

A. structured sampling.

B. systematic sampling.

C. simple random sampling.

D. stratified random sampling.

Correct Answer:  D......................................................................................... LOS: Reading  10‐b Stratified random sampling ensures that different subdivisions within a population are represented in the sample.

Reference: CFA® Program Curriculum, Volume 1, pp. 422‐425. 

Page 93: Cfa Ques Answer

Quantitative Methods: Applications   77 

 

17.  If a very large sample size is used it is least likely to lead to: 

A.  a smaller standard error of the sample means. 

B.  a larger dispersion in the distribution of the sample means. 

C.  the sampling distribution of the sample mean being approximately a normal distribution. 

D.  the mean of the sampling distribution of the sample mean being equal to the population mean. 

 

 

 

18.  Which of the following describe lognormal distributions? 

  Skew      Often used to describe asset 

A.  positively skewed     prices 

B.  positively skewed    returns 

C.  negatively skewed     prices 

D.  negatively skewed.    returns 

Page 94: Cfa Ques Answer

78   Study Session 03:  

 

17.  If a very large sample size is used it is least likely to lead to: 

A. a smaller standard error of the sample means.

B. a larger dispersion in the distribution of the sample means.

C. the sampling distribution of the sample mean being approximately a normal distribution.

D. the mean of the sampling distribution of the sample mean being equal to the population mean.

Correct Answer:  B...........................................................................................LOS: Reading  10‐e A large sample size will lead to a smaller standard error of the sample mean since the standard error is equal to the population standard deviation divided by the square root of the sample size. The central limit theorem says that the sampling distribution of the sample mean is close to a normal distribution if the sample size is large (greater than thirty). The mean of the sample means will be the same as the mean of the population.

However a large sample size will lead to sample means being close to the population mean and therefore less disperse, so B is the correct answer.

Reference: CFA® Program Curriculum, Volume 1, pp. 428‐431. 

 

 

 

18.  Which of the following describe lognormal distributions? 

Skew Often used to describe asset

A. positively skewed prices

B. positively skewed returns

C. negatively skewed prices

D. negatively skewed. returns

Correct Answer:  A ............................................................................................ LOS: Reading  9‐j A lognormal distribution is bounded by zero on the left and has a long tail on the right so it is positively skewed. The bound of zero means the distribution is often used to model asset prices.

Reference: CFA® Program Curriculum, Volume 1, pp. 400‐406. 

Page 95: Cfa Ques Answer

Quantitative Methods: Applications   79 

 

19.  An  investment manager  is managing  a  diversified  portfolio  of  international  equities  and  the performance benchmark is the MSCI World index. Shortfall risk for the portfolio refers to: 

A.   the standard deviation of returns. 

B.  the risk that the value of the portfolio falls below a critical level. 

C.   the standard deviation of returns relatives to the MSCI World Index. 

D.  the absolute deviation of returns relatives to the MSCI World Index. 

 

 

 

20.  The standard error of the sample mean is: 

A.  the average standard deviation of each sample. 

B.  the standard deviation of the sampling distribution of sample mean. 

C.  the standard deviation of the sample minus the standard deviation of the population. 

D.  the standard deviation of the sample divided by the standard deviation of the population. 

Page 96: Cfa Ques Answer

80   Study Session 03:  

 

19.  An  investment manager  is managing  a  diversified  portfolio  of  international  equities  and  the performance benchmark is the MSCI World index. Shortfall risk for the portfolio refers to: 

A. the standard deviation of returns.

B. the risk that the value of the portfolio falls below a critical level.

C. the standard deviation of returns relatives to the MSCI World Index.

D. the absolute deviation of returns relatives to the MSCI World Index.

Correct Answer:  B............................................................................................. LOS: Reading  9‐i Shortfall risk is the risk that the value of a portfolio falls below a certain level, it can be reduced by minimizing the probability that the return falls below a certain minimum level.

Reference: CFA® Program Curriculum, Volume 1, pp. 397‐ 400. 

 

 

 

20.  The standard error of the sample mean is: 

A. the average standard deviation of each sample.

B. the standard deviation of the sampling distribution of sample mean.

C. the standard deviation of the sample minus the standard deviation of the population.

D. the standard deviation of the sample divided by the standard deviation of the population.

Correct Answer:  B...........................................................................................LOS: Reading  10‐e The standard deviation of a sample mean is the standard error. The central limit theorem says that this is equal to the population standard deviation divided by the square root of the sample size.

Reference: CFA® Program Curriculum, Volume 1, pp. 428‐431. 

Page 97: Cfa Ques Answer

Quantitative Methods: Applications   81 

 

21.  A sample of 100 observations  is  taken  from a normally distributed population with a standard deviation of 2.  The sample mean is 6.  The 99% confidence level is closest to: 

A.  between 4.45 and 7.55. 

B.  between 5.48 and 6.52. 

C.  between 5.61 and 6.39. 

D.  between 5.95 and 6.05. 

 

 

 

22.  A  client wishes  to  protect  the  value  of  his  capital.   Which  of  the  following portfolios will  be optimal from a safety‐first analysis? 

Asset  Expected Return  Standard Deviation   

A  25  12 

B  15  8 

C  10  6 

D  8  5 

 

A.  Portfolio A. 

B.  Portfolio B. 

C.  Portfolio C. 

D.  Portfolio D. 

Page 98: Cfa Ques Answer

82   Study Session 03:  

 

21.  A sample of 100 observations  is  taken  from a normally distributed population with a standard deviation of 2.  The sample mean is 6.  The 99% confidence level is closest to: 

A. between 4.45 and 7.55.

B. between 5.48 and 6.52.

C. between 5.61 and 6.39.

D. between 5.95 and 6.05.

Correct Answer:  B........................................................................................... LOS: Reading  10‐j 

The confidence interval is given by nzx 2/

σ± α

where

x = sample mean, which is the point estimate of the population mean σ = population standard deviation

n = sample size

2/zα = reliability factor, the point where α/2 of the probability is in the right tail

The 99% confidence level is 6 ± (2.58 x 2)/10 which is between 5.48 and 6.52.

Reference: CFA® Program Curriculum, Volume 1, pp. 433‐439. 

Page 99: Cfa Ques Answer

Quantitative Methods: Applications   83 

 

22.  A  client wishes  to  protect  the  value  of  his  capital.   Which  of  the  following portfolios will  be optimal from a safety‐first analysis? 

 

Asset  Expected Return  Standard Deviation   

A 25 12

B 15 8

C 10 6

D 8 5

A. Portfolio A.

B. Portfolio B.

C. Portfolio C.

D. Portfolio D.

Correct Answer:  A.............................................................................................LOS: Reading  9‐i 

( )[ ]P

LP RRERatio F S σ−=

where

PR = portfolio return

LR = threshold level

Pσ = portfolio standard deviation

Calculate the S F Ratio for each portfolio:

S F Ratio(A) = 25/12 = 2.08

S F Ratio(B) = 15/8 = 1.875

S F Ratio(C) = 10/6 = 1.667

S F Ratio(D) = 8/5 = 1.600

A has the highest S F Ratio and is the correct answer.

Reference: CFA® Program Curriculum, Volume 1, pp. 397‐400. 

 

 

Page 100: Cfa Ques Answer

84   Study Session 04:  

 

Study Session 4: Introduction Introductory Readings

Economics:  Private  and  Public  Choice,  10th  edition,  James  D.  Gwartney,  Richard  L.  Stroup, Russell S. Sobel, and David A. Macpherson (South‐Western, 2003) “Supply, Demand, and the Market Process,” Ch. 3 “Supply and Demand: Applications and Extensions” Ch. 4 “Taking the Nation’s Economic Pulse,” Ch. 7 “Working with Our Basic Aggregate Demand/Aggregate Supply Model,” Ch. 10 “Keynesian Foundations of Modern Macroeconomics,” Ch. 11  

Supply, Demand, and the Market Process CH 5 Introduction

This chapter is centered on the laws of supply and demand. The law of supply says that a higher price means producers produce more of the product, and the law of demand says that consumers buy less of a good if the price rises, and vice versa. Candidates should know what type of factors cause shifts in the supply and demand curves and movements along the curves and understand the concept of market equilibrium.     

Page 101: Cfa Ques Answer

Introductory Readings   85 

 

Consumer choice and the Law of Demand Consumers are forced to make choices regarding how they spend their income in order to get the most  value  for  their money. The  Law  of Demand  states  that  there  is  an  inverse  relationship between  the price of a product and  the amount or quantity of  it  that consumers are willing  to purchase. 

 A consumer surplus is the difference between the maximum price that consumers are willing to pay and the price that they actually pay; this is a net gain to the buyers of the good. 

Producer choice and the Law of Supply All  economic participants  in an  economy  are aiming  to generate profit, which  is  the  excess of sales  revenue  over  the  production  costs.  The  Law  of  Supply  states  that  there  is  a  direct relationship between the price of a product and the amount of it that is offered for sale.  

Price Law of Demand

Demand

Quantity 

Page 102: Cfa Ques Answer

86   Study Session 04:  

 

Price changes and demand and supply Consumers buy less of a product as the price increases because of the availability of substitutes. The availability of substitutes is a major factor in deciding the sensitivity to the quantity demanded to a change  in price.  If  the demand  for a product  is elastic  it means a small price change will  lead  to a large change  in demand.  In  the diagram below when  the price moves  from P1  to P2,  the quantity demanded falls sharply from Q1 to Q2. If demand is inelastic it means that a change in price only has a small impact on the quantity demanded, so prices move from Q1 to Q2. Unitary elastic means that a percent change in quantity demanded leads to a percent change in price. 

Similarly a supply curve is elastic when the quantity supplied is very responsive to a change in price (a flat curve) and inelastic when quantity supplied is not very responsive to a change in price (a steep curve). 

Price Law of Supply

Supply

                      Quantity

Page 103: Cfa Ques Answer

Introductory Readings   87 

 

Shifts in demand A shift in the demand curve will be a result of a change in demand due to factors other than price. Factors  that  lead  to a shift  include  increases  in consumer  income, changes  in  taxes on  the product, changes in price or availability of competing products, and changes in expectations of future prices. It is important to differentiate between changes in demand (a shift of the demand curve) and changes in quantity demanded (a movement along the same demand curve). 

  

Page 104: Cfa Ques Answer

88   Study Session 04:  

 

Shifts in supply A change in supply indicates a shift in the supply curve. The following are examples of factors that could  lead  to  a  change  in  supply  –  changes  in  resource  costs,  technology  improvements,  natural disasters which limit supply of a product, changes in taxes on the producers of the product.  Equilibrium  is  defined  as  a  state  of  balance  between  forces  such  as  supply  and  demand.  It  is important to differentiate between short run (an insufficient time period for decision makers to fully adjust to changes in market conditions) and long run (a sufficient time for decision makers to make adjustments). 

High prices will tend to lead to an excess supply illustrated by the points a and b, producers will tend to  lower prices until  supply  and demand  are  in balance. On  the other hand  if prices are  too  low, demand will exceed supply,  illustrated by points c and d, and producers will  increase prices until equilibrium is reached. 

Impact of changes in demand and supply Increase in demand – the demand curve shifts to the right, which will increase both the equilibrium price and quantity.  Decrease in demand – the demand curve shifts to the left, which will decrease both the equilibrium price and quantity.  Increase in supply – the supply curve shifts to the right, which will decrease the equilibrium price and increase the equilibrium quantity. Decrease in supply – the supply curve shifts to the left, which will increase the equilibrium price and decrease the equilibrium quantity. An unanticipated cut in supply will, in the short run, lead to a sharp increase in price, but in the long run  demand will  respond  to  the  price  change.  Similarly  an  unanticipated  surge  in  demand  for  a product will initially push up prices but longer‐term supply will be increased. 

Page 105: Cfa Ques Answer

Introductory Readings   89 

 

The invisible hand principle says that market prices act as an inducement to individuals to pursue productive activities that also promote the economic well being of society. 

Page 106: Cfa Ques Answer

90   Study Session 04:  

 

Supply and Demand: Applications and Extensions CH 4 Introduction

This  chapter  examines  some  of  the  applications  of  supply  and  demand  analysis  covered  in  the previous Reading. First we  look at the  impact of wage rates,  interest rates and foreign exchange on supply and demand. Then we  look at the  impact of government action,  including price controls on markets,  and  of  black markets which  are  operating  outside  the  legal  system.  Finally we  consider taxation and how it relates to elasticity of supply and demand. 

Resources The  first  step  in  a  firm’s production  process  is  the purchase  of  resources. Resources  include  raw materials,  labor  etc.  Resource  markets  generally  have  downward‐sloping  demand  curves  and upward‐sloping supply curves. There is a close link between the markets for the end‐product and the resources  used  to make  the  product.  For  example, when  the  price  of  a  resource  increases,  costs increase leading to a reduction in supply and higher prices for the end‐product. The loanable funds market refers to the market that coordinates the borrowing and lending decisions of firms and households. Participants in the market include commercial banks, and stock and bond markets. The interest rate is the price of loanable funds.  The demand curve will slope downwards to the right since firms and households will borrow more at low interest rates. On the other hand, low interest rates will make it less attractive to save so the supply curve will be upward sloping to the right. Market forces will drive interest rates to a level, E, where the quantity of funds demanded will equal the quantity of funds supplied.  

The interest rate is important because it is the link between the price of something today with its price in the future. 

Page 107: Cfa Ques Answer

Introductory Readings   91 

 

The  foreign  exchange market  is  the market  in which  different  currencies  are  bought  and  sold. Exchange rates between currencies are very important since they determine the price of all goods and services  that are  traded  in  international markets. They will also  influence decision makers who are looking at producing goods in different countries, although other issues such as transport costs, legal issues will be factors to consider before a decision is made to move production overseas. A price ceiling is a legal restriction that establishes a maximum price that a good can be sold at. If the price  ceiling  is  set  below  the  equilibrium  price  it will  increase  demand  and  reduce  the  quantity supplied creating a shortage of the good. Non‐price factors (e.g. waiting lists) will determine who is able to buy the product. Suppliers will be tempted to reduce the quality of the good supplied if they cannot increase prices. Price floors establish a minimum price that can be charged for a good. If the price is fixed above the equilibrium price then a surplus of the good will appear. The minimum wage  is an example of a price  floor, and has  led  to  the  substitution of machines or more highly‐skilled workers for low‐paid workers. Also employers will have little incentive to offer non‐wage benefits to workers on the minimum wage.  A black market is a market that operates outside the legal system, either the sale of illegal goods or goods  at  illegal  prices  or  terms.  Black  markets  are  characterized  by  higher  profit  margins  for suppliers who do not get caught (to compensate the supplier for the higher risk), defective products and violence (to settle disputes). The point is made that a legal system that allows for settlement of disputes is essential for the smooth operation of markets. Tax incidence refers to how the burden of a tax is distributed between buyers and sellers and related parties (the actual incidence). This will often be quite different to the statutory incidence which is the legal  assignment  of  the  responsibility  to pay  the  tax.  It  can  be  shown  that  the  actual  incidence  is independent of its statutory incidence, i.e. whether it is imposed on the buyer or the seller. Looking at the example of when a tax is imposed on the seller of a product, this will shift the supply curve up by the amount of the tax. The intersection of the demand curve and supply curve will move, splitting  the  tax burden between  the buyer  and  seller. The  reduction  in overall  trade  (and  loss of benefit of this trade to both parties) results in a deadweight loss; this is the loss over and above the actual payment of tax to the government. 

Elasticity and the incidence of tax In the case that demand is inelastic and supply elastic the burden of tax will largely fall on the buyer (e.g. when  oil prices  rise). Conversely when demand  is  relatively  elastic  compared  to  supply,  the sellers will bear the largest burden. If either demand or supply is relatively inelastic fewer trades will be eliminated so a rise in tax will result in a relatively small deadweight loss. 

Page 108: Cfa Ques Answer

92   Study Session 04:  

 

Taking the Nation’s Economic Pulse CH 7 Introduction

The focus of this chapter  is on the measurement of GDP as the most commonly quoted measure of economic performance. Candidates need to know how to calculate GDP using both the expenditure and the resources cost‐income approaches. They also need to be familiar with the differences between GDP and GNP, and how to switch between nominal and real GDP given the rate of inflation. At the end of the chapter we look at alternative measures for the performance of the economy. 

Gross domestic product Gross domestic product  (GDP)  is defined as  the  total market value of all  final goods and  services produced within  a  country’s  borders  during  a  specific  time  period. GDP  is  a  broad measure  of current  production  of  goods  and  services,  and  GDP  calculations  exclude  second‐hand  goods, intermediate goods, and financial transactions. There are two main approaches to measuring GDP: 

19. 1.  Expenditure approach, where GDP is the sum of: Consumption (C) Gross private investment (Ig) Government expenditure (G) Net Exports (NX)

The expenditure approach is commonly stated as:   GDP = C + Ig + G + NX 

Gross  private  investment  includes  depreciation  expense;  gross  investment  is  the  sum  of  net investment (In) plus depreciation. Net exports (NX) is equal to exports minus imports (X – IM), and NX can be either positive or negative.  If exports are higher  than  imports, NX will be positive and added to GDP; if imports are higher than exports, NX will be negative and subtracted from GDP. 

20. 2.  Cost resource‐income approach, where GDP is the sum of: Wages, salaries, self-employed income Rents, profits, and interest Indirect business taxes Depreciation Net income of foreigners

Net income of foreigners can be positive or negative. If foreigners bring in more investment income into a country than the residents of the same country pay abroad, then net income will be positive. GDP calculated by the expenditure approach and GDP calculated by the cost‐income approach must be equal.  

Page 109: Cfa Ques Answer

Introductory Readings   93 

 

 

Example Int-1 Calculating GDP

The fictitious country of Euphoria has the following expenditure and income categories and amounts (in millions): 

• Imports $95 • Net investment $100 • Wages and salaries $295 • Government expenditure $70 • Depreciation $25 • Indirect business taxes $60 • Issuance of corporate bonds $35 • Self-employed income $95 • Exports $150 • Rents, profits, and interest $140 • Net income of foreigners -$40 • Consumption expenditure $325

The trick to calculating GDP is to identify expenditure versus cost‐income components. To use the GDP expenditure approach, we must first compute two preliminary items: 

  Ig = net investment + depreciation = $100 + $25 = $125   NX = exports – imports = $150 – $95 = $55   GDP = C + Ig + G + NX   GDP = $325 + $125 + $70 + $55   GDP = $575 

To use the GDP cost‐income approach, sum up all cost‐income components:   GDP  =  wages  and  salaries  +  self‐employed  income  +  rents,  profits,  and   interest + depreciation + indirect business taxes + net income of foreigners 

  GDP = $295 + $95 + $140 + $25 + $60 + (‐$40)   GDP = $575 

Notice  that GDP  is  equal using  either  approach. The  issuance of bonds  is a  financial flow, which is excluded from GDP calculations. (Assume that the asset financed by the bond flotation has already been counted under gross business investment). 

 

Page 110: Cfa Ques Answer

94   Study Session 04:  

 

GDP counts the value of goods and services produced inside a nation by residents of that nation and by foreigners living there, too. Whereas, GNP counts the value of goods and services produced by the citizens of a country, whether they are living in the country itself or living abroad. GDP and GNP will be equal only when net income of foreigners is zero. GDP  is measured  in  the U.S.  in dollars.  In  simple  terms, GDP  is  the  sum  of  price multiplied  by quantity (P × Q) of all final goods and services produced. Quantity represents real production within an economy;  if quantity  (or output) changes,  then economic activity  reported by GDP will change. However, GDP can change, even if there is no change in output, if prices by themselves change. Nominal  GDP  measures  economic  activity  by  multiplying  current  prices  with  current  output; nominal GDP can increase because of either increasing prices and/or increasing output. Whereas, real GDP measures economic activity by multiplying historical prices referenced  to a particular year by current  output;  because  prices  are  held  constant,  real  GDP  can  only  change  because  of  output changes. When newspapers or television report GDP forecasts, they almost always mean real GDP. Inflation  is the  increase  in prices over time, and there are two main ways  in which  inflation can be measured. The Consumer Price Index (CPI) measures narrow price changes in a typical “basket” of goods bought by consumers, captured in the consumption component of GDP calculations. The GDP deflator  measures  broad  price  changes  across  all  categories  of  economic  activity,  captured  by consumption, investment, government, and net exports. Both CPI and the GDP deflator are referenced to a base year, which is assigned a value of 100.0. As inflation rises, so does the index value of the CPI or the GDP deflator. The Equation below  is used  to calculate  real GDP when we are given nominal GDP and  the GDP deflator for any given year: 

 

Equation Int-1

tperiod

year base tperiod tperiod deflator GDP

deflator GDPGDP nominalGDP real ×=   

Recall that the GDP deflator in the base year is equal to 100.0. Therefore, real GDP will always equal nominal GDP in the base year. 

Page 111: Cfa Ques Answer

Introductory Readings   95 

 

 

Example Int-2 Calculating real GDP

Given the following information: • Nominal GDP in 2005 = $11,381.2 billion • GDP deflator in 2005 = 121.4 • GDP base year is 1986

Calculate real GDP in 2005: 

billion 03759$

121.4100.0billion $11,381.2

deflator GDPdeflator GDP

GDP nominalGDP real tperiod

year base tperiod tperiod

.,=

×=

×=

    

Although  GDP  attempts  to  capture  a  nation’s  economic  activity,  some  activities  will  elude statisticians and will be excluded from any measure. Such excluded activities include: 

• Leisure and human costs; • Unreported and illegal activities, such as the underground economy; • Value of unpaid household production, including chores; • Harmful side effects of production, including pollution of air, water, soil; and • Quality and variety of product improvements which occur through innovation.

GDP and GNP are the broadest measures of output. However, there are several alternative measures of economic performance: 

• National income (NI) is the earnings of all resource owners, which is the sum of employment compensation (salaries), rents, corporate profits, interest, and self-employment income. However, not all of this income is available for personal use.

• Personal income (PI) adjusts NI by subtracting corporate profits and social insurance taxes and adding transfer payments and dividends. PI can be spent on consumption, saving, or paying of personal taxes.

• Disposable income (DI) strips away personal taxes from PI. Out of the five output measures, DI is the narrowest. DI can either be consumed or saved.

Page 112: Cfa Ques Answer

96   Study Session 04:  

 

Working with Our Basic Aggregate Demand/ Aggregate Supply Model CH 10 Introduction

We now look at the effects of changes in aggregate demand and supply on other economic indicators, such as growth, prices and employment. First of all we examine the factors that change supply and demand  and  then  differentiate  between  the  short‐run  and  long‐run  impact  of  anticipated  versus unanticipated  changes on  the  economy. We  also  consider  the  self‐correcting mechanisms  that will help stabilise the economy. 

Aggregate demand There are, in fact, three separate curves: aggregate demand, short‐run aggregate supply, and long‐run aggregate supply The aggregate demand (AD) curve is a downward‐sloping curve, with the price level (P) plotted on the vertical axis and real GDP (Y) plotted on the horizontal axis. There are six factors that will shift AD to the right (AD1), also known as an increase in aggregate demand: 

• Increase in real wealth • Decrease in real interest rates • Increased optimism in the economy • Higher expected inflation in the future • Higher incomes abroad, thereby boosting exports • Domestic currency depreciation, thereby boosting exports and restricting imports

The opposite of these factors will cause the AD curve to shift to the left, also known as a decrease in aggregate demand  (AD2). The diagram below depicts  the  shape of  the original aggregate demand curve (AD0) and the shifted curves. 

Real GDP

AD0

Price level

AD1AD2

Aggregate Demand

Page 113: Cfa Ques Answer

Introductory Readings   97 

 

The  short‐run aggregate  supply  (SRAS) curve  is an upward‐sloping curve, with  the price  level  (P) plotted on the vertical axis and real GDP (Y) plotted on the horizontal axis. There are five factors that will shift SRAS to the right (SRAS1), also known as an increase in short‐run aggregate supply: 

• Increase in the stock of capital • Technological improvements • Reduction in input prices • Lower expected inflation in the future • Favourable shocks, such as good weather

The opposite of these factors will cause the SRAS curve to shift to the left, also known as a decrease in short‐run aggregate supply (SRAS2). The diagram below depicts the shape of the original short‐run aggregate supply curve (SRAS0) and the shifted curves. 

Real GDP

Price level Short-run aggregate supply

SRAS2 SRAS0 SRAS1

The long‐run aggregate supply (LRAS) curve is a vertical curve, with the price level (P) plotted on the vertical  axis  and  real GDP  (Y) plotted on  the horizontal  axis. There  are  two  factors  that will  shift LRAS to the right (SRAS1), also known as an increase in long‐run aggregate supply: 

• Increase in the stock of capital • Technological improvements

Observe that these factors are common to SRAS, which means that whenever the LRAS curve shifts, the SRAS curve will move with it in the same direction. The opposite of these factors will cause the LRAS curve to shift to the left, also known as a decrease in long‐run aggregate supply (LRAS2). The diagram below depicts  the shape of  the original  long‐run aggregate supply curve  (LRAS0) and  the shifted curves. 

Page 114: Cfa Ques Answer

98   Study Session 04:  

 

Real GDP

Price level Long-run aggregate supply

LRAS2 LRAS0

YF0YF2

LRAS1

YF1

The  LRAS  curve  plays  a  significant  role  in  regulating  the macro  economy:  the  position  of  LRAS determines the  level of economic activity corresponding to full employment (YF0). Full employment can only be achieved when an economy is operating somewhere along the LRAS curve. An  unanticipated  change  in  macroeconomics  suggests  that  people  are  caught  off  guard  by  the change, but over time, the market will react to it. When people initially react to unanticipated changes in  either AD  or  SRAS,  then  output  can  change,  but  only  in  the  short  run.  (Remember  that LRAS represents the limit of production in the economy in the long run.) Consider an unanticipated increase in government expenditure, a component of AD. If the AD curve shifts upwards, then the new short‐run equilibrium will occur at the intersection of the new AD curve and  the original SRAS  curve. Thus, output  (Y) and price  level  (P) will be higher  in  the  short  run. However,  the  economy  is  now  operating  beyond  its  capacity, which  exerts  pressure  on  resource prices. As  resource  prices  increase,  SRAS will  shift  to  the  left  until  equilibrium  is  restored  along LRAS. Thus, output (Y) is restored to full employment and the price level is higher. In the long run, unanticipated changes in AD result in changes to the price level only. 

AD0

Price levelLRAS0

SRAS0

AD1

E0

eSR

ELR

YF YSR

P0

PSR

PLR

SRAS1

Page 115: Cfa Ques Answer

Introductory Readings   99 

 

Unanticipated changes in SRAS will only affect the short‐run position of the SRAS curve itself; there will  be  no  affect  on  aggregate  demand. Consider  an  economy  dependent  on  agriculture,  and  an unexpected warm summer boosts the economy’s main crop. In the short run, the favorable boost in production will shift the SRAS curve to the right, leading to a lower price level but a higher level of output.  However,  the  economy  is  operating  beyond  its  long‐run  capacity,  and  there  is  upward pressure on  resource prices. As  resource prices  increase, SRAS will  shift  to  the  left until  long‐run equilibrium is restored along the LRAS curve. In the long run, unanticipated changes in SRAS result in neither a change to the price level nor change in output. 

AD0

Price levelLRAS0

SRAS0

E0

eSR

SRAS1

YF YSR

P0

PSR

Market economies  respond when  they operate at an output  level above or below  full employment (along  the  LRAS  curve).  There  are  two  self‐correcting mechanisms  that  tend  to  restore  long‐run equilibrium: 

• Changes in resource prices affect the SRAS curve. We have discussed this in the previous LOS. For example, when output is higher than full employment, increasing resource prices tends to decrease SRAS, causing higher price levels and eventually restoring the economy to full employment.

• Changes in real interest rates affect the AD curve. For example, when output is higher than full employment, real (inflation-adjusted) interest rates will increase as the demand for money increases. Higher real interest rates depress business investment, which is a component of AD. Thus, there is a tendency for AD to automatically decrease, which can help restore the economy to its full employment level of output.

The third self‐correcting mechanism relates to consumption expenditure, which is a large component of AD.  (Consumption  represents approximately 65 percent of GDP  in most developed economies.)  Compared with other components of AD, consumption  is  the most stable component of GDP. The inherent  stability  of  consumption  expenditure  helps  to  mitigate  changes  to  other  less  stable components of GDP, such as net exports (tied to the exchange rate) and business investment (tied to interest rates and fickle business sentiment). 

Page 116: Cfa Ques Answer

100   Study Session 04:  

 

Keynesian Foundations of Modern Macroeconomics CH 11 Introduction

In this section we explore the main principles of Keynesian theory which was developed by a British economist  to  explain  the  prolonged  unemployment  in  the  1930s;  it  has  had  a major  influence  on subsequent economic  thinking. Keynes  focused on demand  rather  than  supply and  the  role of  the government’s fiscal policy  in taking an economy out of recession. He believed that the government should  run  a  budget  deficit  in  order  to  stimulate  demand  and  bring  the  economy  back  to  full employment. This chapter examines the rationale behind his thinking including the importance of the expenditure multiplier.  

Keynesian economics The distinction between classical and Keynesian economics stems from the assumptions made on the length of  recessions. The classical school assumed  that  the economy operated mostly at or near  its long‐run level of output; recessions tended to be short lived as the economy automatically reverted to long‐run output and employment through the SRAS mechanism discussed in the previous Reading. Keynesians  are  followers  of  John Maynard Keynes,  a British  economist  of  the  early  20th  century. Writing during  the Great Depression, Keynes  (hence Keynesians) believed  that  the economy could get stuck  in prolonged recession, with reduced output and widespread unemployment. Keynesians do not believe that adjustments to SRAS will restore equilibrium. Rather, they believe that recessions stem from a deficiency of AD, and the government can only restore long‐run output and employment levels by boosting AD. According to the Keynesians, SRAS adjustments are too slow, if indeed they operate at all. The Keynesian model explains the output level at which an economy can operate, in the absence of either an explicit SRAS or LRAS curve. The  model  distinguishes  between  autonomous  verses  induced  expenditure.  (Recall  the  major components of  expenditure: C,  I, G, and NX.)   Autonomous  expenditure has no  relationship with levels of  income; whereas  induced expenditure  is a  function of  income. For example, consumption expenditure  increases  as  disposable  income  increases  (induced). However,  even when  disposable income is equal to zero, the economy will maintain a level of consumption by drawing down assets (autonomous). In  a  simplified  economy  consisting  of  consumers,  total  expenditure  is  equal  to  consumption expenditure. Total expenditure is an upward sloping function of GDP (output), and even when GDP is zero, there will be autonomous expenditure. The aggregate expenditure curve is shown as the AE curve in the graph below. 

Page 117: Cfa Ques Answer

Introductory Readings   101 

 

Businesses plan  for a certain  level of expenditure, and  they build  inventories  to meet consumption expenditure. In the graph below, businesses wish to operate along a 45‐degree  line, where planned aggregate  expenditure  is  equal  to  actual  output.  If  people  consume more  than  is  planned,  then businesses inventories will be depleted faster than expected. Businesses increase production to meet the unanticipated  expenditure, which boosts  output. Alternatively,  if people  consume  less  than  is planned,  then  businesses  inventories  will  be  unexpectedly  increased.  In  response,  businesses decrease their production, resulting in lower output. 

           Keynesian  macro‐equilibrium  occurs along  the  45‐degree line, when  real GDP is  equal  to  planned 

aggregate expenditure. The  marginal 

propensity to consume (MPC) is additional consumption related to an additional unit of disposable income, is shown in the Equation Int‐2 below: 

 

Equation Int-2

marginal propensity to consume = Δ consumption expenditure            Δ disposable income where Δ is uppercase Greek letter delta meaning change.  

The MPC is a number greater than zero but less than one (0 < MPC <1). Recall that disposable income can be either consumed or saved; therefore, we can define the marginal propensity to save (MPS) as 1 – MPC. Note that MPC + MPS = 1. 

Keynesian equilibrium

Real GDP 

Planned aggregate expenditure (AE

Actual AE

45‐degree lineAE = GDP

Unplannedinventoryreduction

Keynesian  equilibrium  (Planned AE Y)

Unplannedinventorybuild

‐up

Planned  AE  < GDPOutput increased

Planned  AE  > GDPOutput decreased

Page 118: Cfa Ques Answer

102   Study Session 04:  

 

The expenditure multiplier is related to the MPC and MPS, as shown below: 

 

Equation Int-3

expenditure multiplierMPS

1MPC-11

==     

 

 

Example Int-3 Marginal propensity to consume and expenditure multiplier

When  disposable  income  is  $150,  consumption  expenditure  is  $120;  and  when disposable income is $200, consumption expenditure is $157.50. Calculate the marginal propensity to consume and the expenditure multiplier. Use Equations Int‐2  and Int‐3, respectively: marginal propensity to consume 

75.050$

5.37$150$200$

120$50.157$income disposable ∆exp. consump. ∆

==−−

==  

Note that the MPS = 1 – MPC = 1 – 0.75 = 0.25 

expenditure multiplier 40.25

10.75-11

MPC-11

====     

The  expenditure multiplier  explains  the  total  increase  in  output  caused  by  an  initial  change  in autonomous expenditure. The expenditure multiplier is positively related to the MPC and negatively related to MPS. Intuitively,  if some of the  increase  in autonomous expenditure  is saved rather than spent, the effect of the autonomous stimulus on the economy will be reduced. Assume  that  the  MPC  =  0.75,  which  yields  an  expenditure  multiplier  of  4.  If  the  government increased  autonomous  expenditure  by  $10,  then  the  total  effect  of  the  government’s  increased expenditure would be a $10 × 4 = $40 increase in total output (Y). If, however, people had a tendency to consume more of the autonomous increase (MPC = 0.80), then the new expenditure multiplier would be: 

expenditure multiplier 50.20

10.80-11

MPC-11

====  

Page 119: Cfa Ques Answer

Introductory Readings   103 

 

If  the  government  increased  autonomous  expenditure  by  $10,  then  the  total  effect  of  the government’s increased expenditure would be a $10 × 5 = $50 increase in total output (Y). In terms of the Keynesian model, an increase in autonomous expenditure would shift the actual AE curve upwards, resulting in a multiplied increase in output (Y) until planned aggregate expenditure equals output. Keynesians believe that private investment is the most volatile component of aggregate expenditure. An  unexplained  decline  in  business  confidence  results  in  an  autonomous  decrease  in  private investment, which reverberates throughout the entire economy by the multiplier effect. The decrease in aggregate demand causes a decline in output and employment levels. The economy’s built‐in  stabilizers might not be  enough  to offset  the decline  in AD,  so Keynesian believe  that  the government ought to increase expenditure to restore full employment. Alternatively,  an  increase  in  business  confidence  would  result  in  an  autonomous  increase  in investment  expenditure,  which  would  reverberate  throughout  the  economy  via  the  expenditure multiplier. If output expands beyond full employment, inflation pressures would emerge. 

Page 120: Cfa Ques Answer

104   Study Session 04:  

 

Introductory Readings Concept Check Questions 1.  It is noticed that a small change in price has a big impact on demand for a product. This means the demand is:  

A. elastic, with a flat demand curve.

B. elastic, with a steep demand curve.

C. inelastic, with a flat demand curve.

D. inelastic, with a steep demand curve.

 

 

 

 

2.  A natural disaster will often cause:  

A. a shift in the supply curve to the left.

B. a shift in the supply curve to the right.

C. a move to the left along the supply curve.

D. a move to the right along the supply curve.

 

 

 

 

3.  If the GDP deflator in a country has risen from 100 in 1985, to 130 in 2005 and nominal GDP has risen from $85 billion to $125 billion then the change in real GDP over the period is closest to:  

A. a fall of 3.9%.

B. an increase of 13.1%.

C. an increase of 17.1%.

D. an increase of 62.5%.

Page 121: Cfa Ques Answer

Introductory Readings   105 

 

4.  Which of  the  following  statements  is CORRECT  regarding gross national product  (GNP)  and gross domestic product (GDP)? 

A. GNP and GDP are both the same, but different terms are used in different countries.

B. GNP is GDP less the income of foreigners in the country plus the income earned by the country’s citizens overseas.

C. GNP is GDP less depreciation costs.

D. GNP is calculated using the expenditure approach and GDP using the income approach.

 

 

5.  Which of the following would cause a shift in aggregate demand? 

I.  An increase in real interest rates. 

II.  New technology increasing productivity. 

III.  A change in inflationary expectations. 

IV.  A significant change in the exchange rate. 

A. II only.

B. I and III only.

C. I, III and IV only.

D. All of the above.

 

 

6.  When economic output is less than the economy’s potential, which of the following will act as a self‐correcting mechanism? 

A. Fiscal stimulus.

B. Real interest rates will decline.

C. Real resource prices will rise.

D. Rising inflation.

Page 122: Cfa Ques Answer

106   Study Session 04:  

 

7.  The Keynesian model implies that: 

A. market forces are sufficient to ensure that the economy operates at the full employment level in the long term.

B. adjusting money supply is the primary tool that should be used to stimulate aggregate demand.

C. adjusting interest rates is the primary tool that should be used to stimulate aggregate demand.

D. macroeconomic policy should focus on maintaining aggregate expenditures at the level that leads to full employment .

 

 

 

 

8.  If the marginal propensity to consume (MPC) is 0.7 and investment spending is increased by $3 billion then the additional income generated is closest to: 

A. $3.33 billion.

B. $4.28 billion.

C. $7.00 billion.

D. $10.00 billion

Page 123: Cfa Ques Answer

Introductory Readings   107 

 

Page 124: Cfa Ques Answer

108   Study Session 04:  

 

Introductory Readings Concept Check Answers 1.  It is noticed that a small change in price has a big impact on demand for a product. This means the demand is:  

A. elastic, with a flat demand curve.

B. elastic, with a steep demand curve.

C. inelastic, with a flat demand curve.

D. inelastic, with a steep demand curve.

 

Correct Answer: 1.  A Demand is elastic since it is highly responsive to a change in price, this will lead to a flat demand curve.

 

 

 

2.  A natural disaster will often cause:  

A. a shift in the supply curve to the left.

B. a shift in the supply curve to the right.

C. a move to the left along the supply curve.

D. a move to the right along the supply curve.

Correct Answer: 2.  A The supply curve shifts to the left as supply is decreased. This will in turn increase the equilibrium price.

Page 125: Cfa Ques Answer

Introductory Readings   109 

 

3.  If the GDP deflator in a country has risen from 100 in 1985, to 130 in 2005 and nominal GDP has risen from $85 billion to $125 billion then the change in real GDP over the period is closest to:  

A. a fall of 3.9%.

B. an increase of 13.1%.

C. an increase of 17.1%.

D. an increase of 62.5%.

 

Correct Answer: 3.  B Real GDP (2004) = $125 billion x (100/130) = $96.15 billion

This is an increase of 13.1%

 

 

 

4.  Which of  the  following  statements  is CORRECT  regarding gross national product  (GNP)  and gross domestic product (GDP)? 

A. GNP and GDP are both the same, but different terms are used in different countries.

B. GNP is GDP less the income of foreigners in the country plus the income earned by the country’s citizens overseas.

C. GNP is GDP less depreciation costs.

D. GNP is calculated using the expenditure approach and GDP using the income approach.

Correct Answer: 4.  B 

Page 126: Cfa Ques Answer

110   Study Session 04:  

 

5.  Which of the following would cause a shift in aggregate demand? 

I.  An increase in real interest rates. 

II.  New technology increasing productivity. 

III.  A change in inflationary expectations. 

IV.  A significant change in the exchange rate. 

A. II only.

B. I and III only.

C. I, III and IV only.

D. All of the above.

 

Correct Answer: 5.  C Item II would lead to a shift in the supply curve.

 

 

 

6.  When economic output is less than the economy’s potential, which of the following will act as a self‐correcting mechanism? 

A. Fiscal stimulus.

B. Real interest rates will decline.

C. Real resource prices will rise.

D. Rising inflation.

Correct Answer: 6.  B The main self-correcting mechanisms will be the fall in real interest rates and resource prices (including labor costs).

Page 127: Cfa Ques Answer

Introductory Readings   111 

 

7.  The Keynesian model implies that: 

A. market forces are sufficient to ensure that the economy operates at the full employment level in the long term.

B. adjusting money supply is the primary tool that should be used to stimulate aggregate demand.

C. adjusting interest rates is the primary tool that should be used to stimulate aggregate demand.

D. macroeconomic policy should focus on maintaining aggregate expenditures at the level that leads to full employment .

 

Correct Answer: 7.  D 

 

 

 

8.  If the marginal propensity to consume (MPC) is 0.7 and investment spending is increased by $3 billion then the additional income generated is closest to: 

A. $3.33 billion.

B. $4.28 billion.

C. $7.00 billion.

D. $10.00 billion

Correct Answer: 8.  D The expenditure multiplier is 1/(1 – 0.7) or 3.33. Therefore the additional income is $3 billion x 3.33 or $10 billion.

Page 128: Cfa Ques Answer

112   Study Session 04:  

 

Study Session 04: Economics: Microeconomic Analysis

 

This study session focuses on microeconomic concepts and how firms are affected by these concepts. One of the main concepts related to the equilibrium between demand and supply is elasticity, which measures  the dependency between demand and supply and  the  impact of changes  in either on  the equilibrium price level. A second key concept is efficiency, which is a measure of the firm’s “optimal” output  given  its  cost  and  revenue  functions.  Understanding  these  concepts  enables  analysts  to differentiate among various companies on an individual level, and to determine their attractiveness for an investor.  

 

Reading 13: Elasticity 

Reading 14: Efficiency and Equity 

Reading 15: Markets in Action 

Reading 16: Organizing Production 

Reading 17: Output and Costs 

 

 

 

 

Page 129: Cfa Ques Answer

Economics: Microeconomic Analysis   113 

 

1.  Economic profit is total revenue less:  

A.  explicit costs. 

B.  implicit costs. 

C.  normal profit. 

D.  opportunity costs. 

 

 

 

2.  The momentary supply curve of agricultural crops is generally:  

 

A.  vertical. 

B.  horizontal. 

C.  an upward sloping curve. 

D.  linear and passes through the origin. 

 

 

Page 130: Cfa Ques Answer

114   Study Session 04:  

 

1.  Economic profit is total revenue less:  

A. explicit costs.

B. implicit costs.

C. normal profit.

D. opportunity costs.

Correct Answer:  D..........................................................................................LOS: Reading  16‐a Economic profit is after opportunity costs which include both explicit and implicit costs. Normal profit is included in implicit costs.

Reference: CFA® Program Curriculum, Volume 2, pp. 94‐95. 

 

 

 

2.  The momentary supply curve of agricultural crops is generally:  

A. vertical.

B. horizontal.

C. an upward sloping curve.

D. linear and passes through the origin.

Correct Answer:  A ..........................................................................................LOS: Reading  13‐a The quantity of agricultural goods available will depend on decisions on planting of the crop made some time previously. Generally producers cannot rapidly change their output so supply is inelastic and the supply curve is vertical.

Reference: CFA® Program Curriculum, Volume 2, pp. 26‐27. 

Page 131: Cfa Ques Answer

Economics: Microeconomic Analysis   115 

 

3.  Which of the following are examples of implicit and explicit opportunity costs? 

  Implicit cost       Explicit cost 

A.  Utility expenses       Rental costs  

B.  Interest foregone        Accounting depreciation 

C.  Dividend payments     Salary costs 

D.  Economic depreciation     Bank interest 

 

 

 

4.  Making a market in a good illegal is most likely to:   

A.  increase value for buyers. 

B.  reduce costs for suppliers. 

C.  reduce the quantity traded. 

D.  create a surplus of the good. 

Page 132: Cfa Ques Answer

116   Study Session 04:  

 

3.  Which of the following are examples of implicit and explicit opportunity costs? 

Implicit cost Explicit cost

A. Utility expenses Rental costs

B. Interest foregone Accounting depreciation

C. Dividend payments Salary costs

D. Economic depreciation Bank interest

Correct Answer:  D..........................................................................................LOS: Reading  16‐a Implicit costs are when there is no payment made but another action has been foregone, these are often related to use of capital and use of owner’s resources. Interest forgone and economic depreciation are both forms of implicit costs. Explicit costs are paid for in money, such as bank interest, salaries, utilities, rental expense.

Reference: CFA® Program Curriculum, Volume 2, pp. 92‐95 

 

 

 

4.  Making a market in a good illegal is most likely to:   

A. increase value for buyers.

B. reduce costs for suppliers.

C. reduce the quantity traded.

D. create a surplus of the good.

Correct Answer:  C ......................................................................................... LOS: Reading  15‐d Making a market illegal is likely to push up costs for suppliers (if there is a penalty on suppliers) and/or reduce value for buyers (if there is a penalty on buyers). This will lead to a move in the equilibrium point to the left, reducing quantities traded.

Reference: CFA® Program Curriculum, Volume 2, p. 81‐84. 

Page 133: Cfa Ques Answer

Economics: Microeconomic Analysis   117 

 

5.  Which of the following curves is least likely to be U‐shaped?  

A.  The marginal cost curve. 

B.  The average total cost curve. 

C.  The average fixed cost curve. 

D.  The average variable cost curve. 

 

 

 

6.  If a government puts a price ceiling on a service which is below the equilibrium price, which of the following is least likely to happen?  

A.  Demand for the service will decline. 

B.  A black market in the service will develop. 

C.  There will be as shortage of supply of the service. 

D.  The service providers’ producer surplus will decline.  

Page 134: Cfa Ques Answer

118   Study Session 04:  

 

5.  Which of the following curves is least likely to be U‐shaped?  

A. The marginal cost curve.

B. The average total cost curve.

C. The average fixed cost curve.

D. The average variable cost curve.

Correct Answer:  C ..........................................................................................LOS: Reading  17‐c The average total, marginal and variable costs decrease at low outputs and increase at high outputs, they are U-shaped. The average fixed cost decreases steadily as output increases.

Reference: CFA® Program Curriculum, Volume 2, pp. 129‐134. 

 

 

 

6.  If a government puts a price ceiling on a service which is below the equilibrium price, which of the following is least likely to happen?  

A. Demand for the service will decline.

B. A black market in the service will develop.

C. There will be as shortage of supply of the service.

D. The service providers’ producer surplus will decline.

Correct Answer:  A ..........................................................................................LOS: Reading  15‐a The price ceiling will lead to a drop in supply, reducing the producer surplus. Demand will be unchanged, or increase, so “Demand for the service will decline.” is the correct choice. A black market may well develop to fulfill demand.

Reference: CFA® Program Curriculum, Volume 2, pp. 60‐66. 

Page 135: Cfa Ques Answer

Economics: Microeconomic Analysis   119 

 

7.   Technology and labor are usually considered to be which type of resources? 

  Technology   Labor 

A.  Long‐run    Long‐run 

B.  Long‐run    Short‐run 

C.  Short‐run    Long‐run 

D.  Short‐run    Short‐run 

 

 

 

8.  If the supply of a product is unit elastic it means that the supply curve is: 

A.  vertical. 

B.  horizontal. 

C.  an upward sloping curve. 

D.  linear and passes through the origin. 

Page 136: Cfa Ques Answer

120   Study Session 04:  

 

7.   Technology and labor are usually considered to be which type of resources? 

Technology Labor

A. Long-run Long-run

B. Long-run Short-run

C. Short-run Long-run

D. Short-run Short-run

Correct Answer:  B...........................................................................................LOS: Reading  17‐a Technology cannot usually be varied in the short run so is considered a long-run resource. Labor is usually a variable input so is a short-run resource.

Reference: CFA® Program Curriculum, Volume 2, pp. 122‐123. 

 

 

 

8.  If the supply of a product is unit elastic it means that the supply curve is: 

A. vertical.

B. horizontal.

C. an upward sloping curve.

D. linear and passes through the origin.

Correct Answer:  D..........................................................................................LOS: Reading  13‐a Unit elastic means that the percentage change in price equals the percentage change in quantity supplied, so the supply curve is a straight line and passes through the origin.

Reference: CFA® Program Curriculum, Volume 2, p. 25. 

Page 137: Cfa Ques Answer

Economics: Microeconomic Analysis   121 

 

9  When a government introduces a subsidy on an agricultural product, in the domestic market this is least likely to have the effect of: 

A.  increasing supply. 

B.  creating overproduction. 

C.  shifting the supply curve downwards to the right. 

D.  marginal cost on the original supply curve falling below marginal benefit. 

 

 

 

10.  Proprietorships in the U.S. are generally characterized by which of the following? 

A.  The entire wealth of owners is at risk. 

B.  They are owned by one or more people.  

C.  Profits are taxed at the proprietorship and owner level.  

D.  The cost of capital and labor are lower than for corporations. 

Page 138: Cfa Ques Answer

122   Study Session 04:  

 

9.  When a government introduces a subsidy on an agricultural product, in the domestic market this is least likely to have the effect of: 

A. increasing supply.

B. creating overproduction.

C. shifting the supply curve downwards to the right.

D. marginal cost on the original supply curve falling below marginal benefit.

Correct Answer:  D......................................................................................... LOS: Reading  15‐d A subsidy is a payment by the government to producers. The supply curve is the marginal cost curve and the demand curve the marginal benefit curve. A subsidy increases supply so original marginal cost rises above marginal benefit, creating overproduction and a deadweight loss.

Reference: CFA® Program Curriculum, Volume 2, pp. 79‐80. 

 

 

 

10.  Proprietorships in the U.S. are generally characterized by which of the following? 

A. The entire wealth of owners is at risk.

B. They are owned by one or more people.

C. Profits are taxed at the proprietorship and owner level.

D. The cost of capital and labor are lower than for corporations.

Correct Answer:  A ..........................................................................................LOS: Reading  14‐e A proprietor has unlimited liability, if a proprietorship cannot pay its debts then the parties owed money can claim the assets of the proprietor, so A is correct. Proprietorships can only be owned by a single person, so B is not correct. Profits are only taxed once as the owner’s income so C is not correct. The cost of capital is usually higher than that of a corporation so D is not correct.

Reference: CFA® Program Curriculum, Volume 2, pp. 101‐102. 

Page 139: Cfa Ques Answer

Economics: Microeconomic Analysis   123 

 

11.  Followers of utilitarianism are most likely to believe that: 

A.  people should be treated fairly and their property rights respected. 

B.  administrative costs reduce the impact of transferring wealth between different parties. 

C.  the state should not own property to ensure all participants have equal access to goods and services. 

D.  all participants should have the same income to maximize the benefits gained from the economic pie. 

 

 

 

12.  It is noted that the income elasticity of a good is slightly negative.  It is most likely that the good is: 

A.  a luxury item. 

B.  a normal good. 

C.  an inferior good. 

D.  a substitute good. 

Page 140: Cfa Ques Answer

124   Study Session 04:  

 

11.  Followers of utilitarianism are most likely to believe that: 

A. people should be treated fairly and their property rights respected.

B. administrative costs reduce the impact of transferring wealth between different parties.

C. the state should not own property to ensure all participants have equal access to goods and services.

D. all participants should have the same income to maximize the benefits gained from the economic pie.

Correct Answer:  D.......................................................................................... LOS: Reading  14‐f Utilitarianism says that all participants should have the same income as this maximizes the combined marginal benefit of consuming goods so D is correct. A and C are part of the symmetry principle, and B is an argument against utilitarianism.

Reference: CFA® Program Curriculum, Volume 2, pp. 49‐50. 

 

 

 

12.  It is noted that the income elasticity of a good is slightly negative.  It is most likely that the good is: 

A. a luxury item.

B. a normal good.

C. an inferior good.

D. a substitute good.

Correct Answer:  C ..........................................................................................LOS: Reading  13‐a Income elasticity measures the change in demand when income changes. For most goods this is positive, as when incomes rise, spending on goods usually rises. However, for inferior goods demand moves in the opposite direction to income, these are goods typically bought by people on low incomes.

Reference: CFA® Program Curriculum, Volume 2, pp. 21‐22. 

Page 141: Cfa Ques Answer

Economics: Microeconomic Analysis   125 

 

13.  The value of one more unit of a good is least accurately described as:  

A.  its marginal benefit. 

B.  the price of the good less the marginal cost of the unit. 

C.  the maximum price consumers are willing to pay for it. 

D.  the price paid for the unit plus the consumer surplus from it. 

 

 

 

14.  When a government imposes a quota on a product it is most likely to:  

A.  benefit consumers of the product.  

B.  benefit certain producers of the product.   

C.  increase the marginal cost of production of the good.  

D.  lead to the marginal cost being higher than marginal benefit. 

Page 142: Cfa Ques Answer

126   Study Session 04:  

 

13.  The value of one more unit of a good is least accurately described as:  

A. its marginal benefit.

B. the price of the good less the marginal cost of the unit.

C. the maximum price consumers are willing to pay for it.

D. the price paid for the unit plus the consumer surplus from it.

Correct Answer:  B.......................................................................................... LOS: Reading  14‐b The value is the same as the marginal benefit, which is the most consumers are willing to pay to buy another unit of the good. The consumer surplus for each good is the value or marginal benefit less the price. The price of the good less the marginal cost of the unit is the producer, not consumer, surplus.

Reference: CFA® Program Curriculum, Volume 2, pp. 38‐40. 

 

 

 

14.  When a government imposes a quota on a product it is most likely to:  

A. benefit consumers of the product.

B. benefit certain producers of the product.

C. increase the marginal cost of production of the good.

D. lead to the marginal cost being higher than marginal benefit.

Correct Answer:  B...........................................................................................LOS: Reading  15‐c A quota is likely to create a supply shortage, which pushes up the price creating extra profits for producers who are allocated the quota, so B is the best answer. The marginal cost will decline as supply drops and consumers will face shortages and higher prices so A, C and D are not correct.

Reference: CFA® Program Curriculum, Volume 2, pp. 77‐81. 

Page 143: Cfa Ques Answer

Economics: Microeconomic Analysis   127 

 

15.  An industry has five participants with market shares of 30%, 25%, 25% 10% and 10%.  The four firm concentration ratio and Herfindahl‐Hirschman Index are: 

 

Four firm concentration ratio   Herfindahl‐Hirschman Index 

A.  10%          470 

B.  90%          470 

C.  10%          2,350 

D.  90%          2,350 

 

 

 

16.  The law of diminishing returns implies that  

A.  total profits diminish as output increases. 

B.  marginal product is initially upward sloping then downward sloping as quantities increase. 

C.  as more units of a variable resource are added marginal product eventually decreases. 

D.  marginal product is eventually higher then average product when sufficient units of a variable resource are added.  

Page 144: Cfa Ques Answer

128   Study Session 04:  

 

15.  An industry has five participants with market shares of 30%, 25%, 25% 10% and 10%.  The four firm concentration ratio and Herfindahl‐Hirschman Index are: 

Four firm concentration ratio Herfindahl-Hirschman Index

A. 10% 470

B. 90% 470

C. 10% 2,350

D. 90% 2,350

Correct Answer:  D..........................................................................................LOS: Reading  16‐g 

Four-firm concentration ratio is (30% + 25% + 25% + 10%) = 90%

Herfindahl-Hirschman Index is 302 + 252 + 252 + 102 = 2350

Reference: CFA® Program Curriculum, Volume 2, pp. 105‐106. 

 

 

 

16.  The law of diminishing returns implies that  

A. total profits diminish as output increases.

B. marginal product is initially upward sloping then downward sloping as quantities increase.

C. as more units of a variable resource are added marginal product eventually decreases.

D. marginal product is eventually higher then average product when sufficient units of a variable resource are added.

Correct Answer:  C ..........................................................................................LOS: Reading  17‐c The law of diminishing returns sates that marginal product starts to decline as quantities increase due to technological constraints.

Reference: CFA® Program Curriculum, Volume 2, pp. 126‐127.  

Page 145: Cfa Ques Answer

Economics: Microeconomic Analysis   129 

 

17.  A manufacturing company has sales of $1 million per annum and explicit costs of $700,000.  The owner of the company, who also owns the factory, is charging an annual rent of only $25,000 for the factory whereas the rent he could receive in the open market is $75,000.   He also does not charge for his time (wages would be $60,000 per annum).  Interest forgone is $20,000 and economic depreciation is $40,000 per annum.  The owner could earn a normal annual profit of $50,000 in a similar business.  The economic profit of the manufacturing company is closest to: 

A. $80,000

B. $120,000.

C. $130,000

D. $180,000.  

 

 

18.  Company  ABC  provides  the  following  information  on  labor  (workers  per  day)  and  total production (units output per day).   

 

Labor  Total Production 

24  5,000 

25  5,200 

26  5,350 

When labor increase from 25 to 26 per day the marginal and average product are closest to: 

  marginal product    average product 

A.    150 units     150.0 units 

B.    150 units     205.8 units 

C.    200 units     208.0 units 

D.    350 units     207.0 units 

Page 146: Cfa Ques Answer

130   Study Session 04:  

 

17.  A manufacturing company has sales of $1 million per annum and explicit costs of $700,000.  The owner of the company, who also owns the factory, is charging an annual rent of only $25,000 for the factory whereas the rent he could receive in the open market is $75,000.   He also does not charge for his time (wages would be $60,000 per annum).  Interest forgone is $20,000 and economic depreciation is $40,000 per annum.  The owner could earn a normal annual profit of $50,000 in a similar business.  The economic profit of the manufacturing company is closest to: 

A. $80,000

B. $120,000.

C. $130,000

D. $180,000.

Correct Answer:  A ..........................................................................................LOS: Reading  16‐a The economic profit is therefore

$1,000,000 - $700,000 - $50,000 (additional implicit cost of rental)

- $60,000 (wages foregone)

- $20,000 - $40,000 - $ 50,000 (normal profit lost).

This equals $80,000.

Reference: CFA® Program Curriculum, Volume 2, pp. 93‐96. 

 

18.  Company  ABC  provides  the  following  information  on  labor  (workers  per  day)  and  total production (units output per day).   

Labor  Total Production24  5,00025  5,20026  5,350

When labor increase from 25 to 26 per day the marginal and average product are closest to:

marginal product average product

A. 150 units 150.0 units

B. 150 units 205.8 units

C. 200 units 208.0 units

D. 350 units 207.0 units

Correct Answer:  B.......................................................................................... LOS: Reading  17‐b The marginal product is the increase in total product for one additional unit of labor, this is 5,350 – 5,200 which equals 150 units. Average product is total product divided by the quantity of labor employed, which is 5,350 divided by 26 equals 205.8

Reference: CFA® Program Curriculum, Volume 2, pp. 124‐129. 

Page 147: Cfa Ques Answer

Economics: Microeconomic Analysis   131 

 

19.  In the market economy, if a good has external benefits it is least likely to lead to: 

A.  a social loss. 

B.  a deadweight loss.  

C.  overproduction of the good. 

D.  an inefficient quantity being produced. 

 

 

 

20.  The efficient quantity produced of a good is least likely to: 

A.  maximize the sum of the consumer and producer surpluses.   

B.  be the quantity to which free market forces drive production. 

C.  be the point where the marginal society benefit equals the marginal society cost.  

D.  disregard external benefits and external costs to the consumer and producer respectively.   

Page 148: Cfa Ques Answer

132   Study Session 04:  

 

19.  In the market economy, if a good has external benefits it is least likely to lead to: 

A. a social loss.

B. a deadweight loss.

C. overproduction of the good.

D. an inefficient quantity being produced.

Correct Answer:  C ..........................................................................................LOS: Reading  14‐e External benefits refer to benefits that accrue to people other than a consumer of a good so the demand curve for the good does not reflect all the benefits that accrue. This will lead to underproduction of the good, this inefficient production level leads to a deadweight loss. This is a loss to society as a whole, which is a social loss.

Reference: CFA® Program Curriculum, Volume 2, pp. 46‐48. 

 

 

 

20.  The efficient quantity produced of a good is least likely to: 

A. maximize the sum of the consumer and producer surpluses.

B. be the quantity to which free market forces drive production.

C. be the point where the marginal society benefit equals the marginal society cost.

D. disregard external benefits and external costs to the consumer and producer respectively.

Correct Answer:  D..........................................................................................LOS: Reading  14‐e External costs and benefits will distort the quantity produced and consumed so the efficient quantity is unlikely to be produced if they are disregarded.

Reference: CFA® Program Curriculum, Volume 2, pp. 43‐46. 

Page 149: Cfa Ques Answer

Economics: Microeconomic Analysis   133 

 

21.  If the price of televisions produced by manufacturer is increased by 10% from $500 to $550 then demand  is  forecast  to  fall by  20%,  from  100,000 units  to  80,000 units. The  elasticity  of demand  is closest to: 

A.  0.45. 

B.  2.22. 

C.  2.33. 

D.  2.50. 

 

Note: percentages are calculated on the average price and average quantity.  We also ignore minus signs; we are measuring the magnitude of elasticity. 

 

 

 

22.  The price of ABC Financial News is increased from $2.00 to $2.50, this leads to an increase in the sales of a competing  financial magazine  , XYZ Finance, which now sells 120,000 copies a week, up from 100,000 copies a week. The cross elasticity of demand is closest to:  

A.  0.81. 

B.  1.04. 

C.  1.22. 

D.  1.25. 

Page 150: Cfa Ques Answer

134   Study Session 04:  

 

21.  If the price of televisions produced by manufacturer is increased by 10% from $500 to $550 then demand  is  forecast  to  fall by  20%,  from  100,000 units  to  80,000 units. The  elasticity  of demand  is closest to: 

A. 0.45.

B. 2.22.

C. 2.33.

D. 2.50.

Correct Answer:  C ......................................................................................... LOS: Reading  13‐b 

Price elasticity of demand

where

QDem = quantity demanded

P = price

= percentage change in quantity demanded, this is

Note:

percentages are calculated on the average price and average quantity. We also ignore minus signs; we are measuring the magnitude of elasticity.

Reference: CFA® Program Curriculum, Volume 2, pp. 13‐14. 

P∆%Q∆%

= Dem

DemQ∆% aveQ/Q∆

.330/90

2=525/50

2=

P∆%Q∆% Dem

Page 151: Cfa Ques Answer

Economics: Microeconomic Analysis   135 

 

22.  The price of ABC Financial News is increased from $2.00 to $2.50, this leads to an increase in the sales of a competing  financial magazine  , XYZ Finance, which now sells 120,000 copies a week, up from 100,000 copies a week. The cross elasticity of demand is closest to:  

A. 0.81.

B. 1.04.

C. 1.22.

D. 1.25.

Correct Answer:  A.......................................................................................... LOS: Reading  13‐a 

Cross elasticity of demand

where

QDem = quantity demanded

PC = price of substitute or complement

= percentage change in quantity demanded.

The cross elasticity of demand is:

81.0=2222.01818.0

=25.2$50.0$

110/20=

P∆%Q∆%

C

Dem

Reference: CFA® Program Curriculum, Volume 2, pp. 19‐21. 

 

 

C

Dem

P∆%Q∆%

=

DemQ∆%

Page 152: Cfa Ques Answer

136   Study Session 05:  

 

Study Session 05: Economics: Market Structure and Macroeconomic Analysis

 

This  study  session  first  compares  and  contrasts  the  different  market  structures  in  which  firms operate. The market environment  influences  the price a  firm can demand  for  its goods or services. Among the most  important of these market forms are monopoly and perfect competition, although monopolistic competition and oligopoly are also covered. 

The study session  then  introduces  the macroeconomic concepts  that have an  impact on all  firms  in the  same  environment, be  it  a  country,  a group of  related  countries, or  a particular  industry. The readings explain the business cycle, and how to forecast changes in the business cycle and the impact on, among other things, price levels and profitability. The study session concludes by describing how an economy’s aggregate supply and aggregate demand are determined. 

 

Reading 18: Perfect Competition 

Reading 19: Monopoly 

Reading 20: Monopolistic Competition and Oligopoly 

Reading 21: Demand and Supply in Factor Markets 

Reading 22: Monitoring Cycles, Jobs, and the Price Level 

Reading 23: Aggregate Supply and Aggregate Demand 

 

 

 

Page 153: Cfa Ques Answer

Economics: Microeconomic Market Structure and Macroeconomic Analysis   137 

 

1.  The real wage rate is: 

A.  the growth in wages adjusted for productivity. 

B.  the money wage rate multiplied by the price level.  

C.  the amount of goods that an hour’s work can buy. 

D.  the money wage rate divided by the inflation rate.  

 

 

 

2.  Full employment is: 

A.  when unemployment is zero. 

B.  the level of unemployment after allowing for cyclical conditions in the labor markets. 

C.  the level of unemployment after allowing for cyclical and frictional conditions in the labor markets. 

D.  the level of unemployment after allowing for frictional and structural conditions in the labor markets. 

 

 

Page 154: Cfa Ques Answer

138   Study Session 05:  

 

1.  The real wage rate is: 

A. the growth in wages adjusted for productivity.

B. the money wage rate multiplied by the price level.

C. the amount of goods that an hour’s work can buy.

D. the money wage rate divided by the inflation rate.

Correct Answer:  C ......................................................................................... LOS: Reading  22‐b The real wage is simply the money wage rate divided by the price level, or the quantity of goods that an hour’s work can buy.

Reference: CFA® Program Curriculum, Volume 2, pp. 292‐293.  

 

 

 

2.  Full employment is: 

A. when unemployment is zero.

B. the level of unemployment after allowing for cyclical conditions in the labor markets.

C. the level of unemployment after allowing for cyclical and frictional conditions in the labor markets.

D. the level of unemployment after allowing for frictional and structural conditions in the labor markets.

Correct Answer:  D..........................................................................................LOS: Reading  22‐c Full employment is the level of employment after allowing for the normal/natural rate of unemployment caused by frictional and structural factors.

Reference: CFA® Program Curriculum, Volume 2, pp. 297‐300. 

Page 155: Cfa Ques Answer

Economics: Microeconomic Market Structure and Macroeconomic Analysis   139 

 

3.  One of the reasons single‐price monopolies are said to be inefficient is because:  

A.  costs are not minimized. 

B.  excess goods are produced. 

C.   prices are below marginal revenue.  

D.  prices are kept below production costs. 

 

 

 

4.  Which of the following is the least accurate description of a monopoly? 

A.  A firm which has to decide which price will maximize profits. 

B.  A producer of a good for which there are no attractive substitutes. 

C.  A market in which there are only a small number of producers of a good. 

D.  An industry where there are high barriers to entry for firms looking to enter the business. 

Page 156: Cfa Ques Answer

140   Study Session 05:  

 

3.  One of the reasons single‐price monopolies are said to be inefficient is because:  

A. costs are not minimized.

B. excess goods are produced.

C. prices are below marginal revenue.

D. prices are kept below production costs.

Correct Answer:  A ......................................................................................... LOS: Reading  19‐b In a single-price monopoly a shortage of goods is produced, costs are not minimized. For a monopoly price exceeds marginal revenue, so price also exceeds marginal cost.

Reference: CFA® Program Curriculum, Volume 2, pp. 181‐186. 

 

 

 

4.  Which of the following is the least accurate description of a monopoly? 

A. A firm which has to decide which price will maximize profits.

B. A producer of a good for which there are no attractive substitutes.

C. A market in which there are only a small number of producers of a good.

D. An industry where there are high barriers to entry for firms looking to enter the business.

Correct Answer:  C ..........................................................................................LOS: Reading  19‐a A monopoly is a firm or market where there is only one producer of a product.

Reference: CFA® Program Curriculum, Volume 2, pp. 176‐181. 

Page 157: Cfa Ques Answer

Economics: Microeconomic Market Structure and Macroeconomic Analysis   141 

 

5.  When firms are price takers the short‐run market supply curve is: 

A.  horizontal. 

B.  slopes upwards to the right. 

C.  slopes downwards to the right. 

D.  whether it slopes upwards or downwards to the right, or is horizontal, will depend on the industry. 

 

 

 

6.  Which of the following would be the least sensible regulatory step to consider in order to increase resource allocation of a monopoly? 

A.  Abolish licensing requirements that limit entry to the industry. 

B.  Regulate that the monopoly firm must reduce prices to the marginal cost. 

C.  Break up the monopoly into smaller companies, if it is not a natural monopoly. 

D.  Reduce barriers and quotas that are acting as a barrier to foreign firms competing in the industry. 

Page 158: Cfa Ques Answer

142   Study Session 05:  

 

5.  When firms are price takers the short‐run market supply curve is: 

A. horizontal.

B. slopes upwards to the right.

C. slopes downwards to the right.

D. whether it slopes upwards or downwards to the right, or is horizontal, will depend on the industry.

Correct Answer:  B...........................................................................................LOS: Reading  18‐c Firms will increase supply if the price rises so the supply curve slopes upwards to the right. (Note – the long-run supply curve will not necessarily slope upwards to the right).

Reference: CFA® Program Curriculum, Volume 2, pp. 156‐160. 

 

 

 

6.  Which of the following would be the least sensible regulatory step to consider in order to increase resource allocation of a monopoly? 

A. Abolish licensing requirements that limit entry to the industry.

B. Regulate that the monopoly firm must reduce prices to the marginal cost.

C. Break up the monopoly into smaller companies, if it is not a natural monopoly.

D. Reduce barriers and quotas that are acting as a barrier to foreign firms competing in the industry.

Correct Answer:  B...........................................................................................LOS: Reading  19‐e Regulating that a monopoly must price their product at the marginal cost would lead to the monopoly making losses, it would be better to use the average total cost.

Reference: CFA® Program Curriculum, Volume 2, pp. 198‐190. 

Page 159: Cfa Ques Answer

Economics: Microeconomic Market Structure and Macroeconomic Analysis   143 

 

7.  What is the impact of an increase in foreign exchange rate (strengthening of local currency) and an increase in the expected inflation rate on aggregate demand?   

  Foreign exchange rate  Expected inflation rate 

A.  increase      increase 

B.  increase       decrease 

C.  decrease       increase  

D.  decrease       decrease 

 

 

 

8.  Which of the following statements regarding a monopoly is least accurate? 

A.  There are high barriers to entry. 

B.  The demand curve is horizontal. 

C.  There are no substitutes for a product. 

D.  There is a single seller of a well‐defined product. 

Page 160: Cfa Ques Answer

144   Study Session 05:  

 

7.  What is the impact of an increase in foreign exchange rate (strengthening of local currency) and an increase in the expected inflation rate on aggregate demand?   

Foreign exchange rate Expected inflation rate

A. increase increase

B. increase decrease

C. decrease increase

D. decrease decrease

Correct Answer:  C ......................................................................................... LOS: Reading  23‐b An increase in the foreign exchange rate will decrease aggregate demand since net exports will decline. An increase in the expected inflation rate increases aggregate demand as people choose to buy goods today before prices rise.

Reference: CFA® Program Curriculum, Volume 2, pp. 322‐325. 

 

 

 

8.  Which of the following statements regarding a monopoly is least accurate? 

A. There are high barriers to entry.

B. The demand curve is horizontal.

C. There are no substitutes for a product.

D. There is a single seller of a well-defined product.

Correct Answer:  B...........................................................................................LOS: Reading  19‐a The demand curve for a monopoly is the market demand curve. It is downward sloping, as price increases demand will fall.

Reference: CFA® Program Curriculum, Volume 2, pp. 178‐81. 

Page 161: Cfa Ques Answer

Economics: Microeconomic Market Structure and Macroeconomic Analysis   145 

 

9.  A significant rise in the real wage rate is most likely to indicate that: 

A.  productivity is rising. 

B.  the inflation rate is very high. 

C.  the aggregate hours worked has risen. 

D.  the employment rate has dropped sharply. 

 

 

 

10.  Long‐run economic profits can be made by: 

  Price takers    Firms in monopolistic competition 

A.  Yes     Yes 

B.  Yes     No 

C.  No      Yes 

D.  No      No 

Page 162: Cfa Ques Answer

146   Study Session 05:  

 

9.  A significant rise in the real wage rate is most likely to indicate that: 

A. productivity is rising.

B. the inflation rate is very high.

C. the aggregate hours worked has risen.

D. the employment rate has dropped sharply.

Correct Answer:  A ......................................................................................... LOS: Reading  22‐b When real wages rise, it usually coincides with an increase in productivity.

Reference: CFA® Program Curriculum, Volume 2, pp. 292‐293. 

 

 

 

10.  Long‐run economic profits can be made by: 

Price takers Firms in monopolistic competition

A. Yes Yes

B. Yes No

C. No Yes

D. No No

Correct Answer:  D......................................................................................... LOS: Reading  20‐b Price takers and firms in monopolistic completion are operating in markets where there are no barriers to entry and exit. Therefore in the long run they will make zero economic profits.

Reference: CFA® Program Curriculum, Volume 2, pp. 212‐213. 

Page 163: Cfa Ques Answer

Economics: Microeconomic Market Structure and Macroeconomic Analysis   147 

 

11.  A profit maximizing firm in monopolistic competition will expand output until marginal revenue equals: 

A.  market price. 

B.  marginal cost. 

C.  average total cost. 

D.  average variable cost. 

 

 

 

12.  An increase in the quantity of capital in an economy is likely to: 

A.  have no effect on the aggregate supply curves. 

B.  shift only the long‐run aggregate supply curve to the right. 

C.  shift only the short‐run aggregate supply curve to the right. 

D.  shift both the short‐run and long‐run aggregate supply curves to the right. 

Page 164: Cfa Ques Answer

148   Study Session 05:  

 

11.  A profit maximizing firm in monopolistic competition will expand output until marginal revenue equals: 

A. market price.

B. marginal cost.

C. average total cost.

D. average variable cost.

Correct Answer:  B.......................................................................................... LOS: Reading  20‐b Output will be expanded until marginal revenue equals marginal cost.

Reference: CFA® Program Curriculum, Volume 2, pp. 210‐212. 

 

 

 

12.  An increase in the quantity of capital in an economy is likely to: 

A. have no effect on the aggregate supply curves.

B. shift only the long-run aggregate supply curve to the right.

C. shift only the short-run aggregate supply curve to the right.

D. shift both the short-run and long-run aggregate supply curves to the right.

Correct Answer:  D..........................................................................................LOS: Reading  23‐a The larger the capital base the more productive the labor force and the higher the output shifting both supply curves to the right.

Reference: CFA® Program Curriculum, Volume 2, pp. 316‐319. 

Page 165: Cfa Ques Answer

Economics: Microeconomic Market Structure and Macroeconomic Analysis   149 

 

13.  The lowest possible cost is most likely to be found in a market which is: 

A.  a monopoly.  

B.  an oligopoly.  

C.  in perfect competition. 

D.  in monoplolistic competition. 

 

 

 

14.  Classical economists believe: 

A.  money wage rates are slow to change in a recession. 

B.  fiscal policy can be used to stimulate aggregate demand.  

C.  technological change is a strong influence on aggregate demand.  

D.  the government should use monetary policy to stimulate demand in a recession. 

Page 166: Cfa Ques Answer

150   Study Session 05:  

 

13.  The lowest possible cost is most likely to be found in a market which is: 

A. a monopoly.

B. an oligopoly.

C. in perfect competition.

D. in monoplolistic competition.

Correct Answer:  C ......................................................................................... LOS: Reading  20‐b In monopolistic competition firms produce below the efficient amount which increases costs. Also monopolies and oligopolies set prices above an efficient point which reduces quantity and increases average costs.

Reference: CFA® Program Curriculum, Volume 2, pp. 214‐215. 

 

 

 

14.  Classical economists believe: 

A. money wage rates are slow to change in a recession.

B. fiscal policy can be used to stimulate aggregate demand.

C. technological change is a strong influence on aggregate demand.

D. the government should use monetary policy to stimulate demand in a recession.

Correct Answer:  C ......................................................................................... LOS: Reading  23‐d Classical economists believe technological change is the main driver of aggregate demand and supply, and the economy is effective at self regulation.

Reference: CFA® Program Curriculum, Volume 2, p. 335. 

Page 167: Cfa Ques Answer

Economics: Microeconomic Market Structure and Macroeconomic Analysis   151 

 

15.  If the CPI price index was 75 last year and 82 this year, then the annual inflation rate is closest to: 

A.  7.0%. 

B.  8.2%. 

C.  8.5%. 

D.  9.3%. 

 

 

 

16.  A price‐taker market is least likely to be characterized by: 

A.  identical products. 

B.  high entry barriers. 

C.  a large number of participants. 

D.  each participant has a small market share. 

Page 168: Cfa Ques Answer

152   Study Session 05:  

 

15.  If the CPI price index was 75 last year and 82 this year, then the annual inflation rate is closest to: 

A. 7.0%.

B. 8.2%.

C. 8.5%.

D. 9.3%.

Correct Answer:  D......................................................................................... LOS: Reading  22‐d The inflation rate is [(82 - 75)/75] x 100 = 9.33%

Reference: CFA® Program Curriculum, Volume 2, pp. 300‐304. 

 

 

 

16.  A price‐taker market is least likely to be characterized by: 

A. identical products.

B. high entry barriers.

C. a large number of participants.

D. each participant has a small market share.

Correct Answer:  B...........................................................................................LOS: Reading  18‐a A price-taker market is one with perfect competition. There are no restrictions or barriers to entry.

Reference: CFA® Program Curriculum, Volume 2, p. 150‐151. 

Page 169: Cfa Ques Answer

Economics: Microeconomic Market Structure and Macroeconomic Analysis   153 

 

17.  A profit maximizing price taker will expand output until marginal revenue equals: 

A.  market price. 

B.  marginal cost. 

C.  average total cost. 

D.  average variable cost. 

 

 

 

18.  Which of the following groups of people is least likely to be included in the labor force? 

A.  Retired workers. 

B.  Part‐time workers. 

C.  People looking for a job. 

D.  People waiting to start a new job within two weeks. 

Page 170: Cfa Ques Answer

154   Study Session 05:  

 

17.  A profit maximizing price taker will expand output until marginal revenue equals: 

A. market price.

B. marginal cost.

C. average total cost.

D. average variable cost.

Correct Answer:  B.......................................................................................... LOS: Reading  18‐b Output will be expanded until Price = Marginal revenue = Marginal cost. Marginal revenue always equals market price in a price taker, which is a purely competitive, market.

Reference: CFA® Program Curriculum, Volume 2, pp. 151‐152. 

 

 

 

18.  Which of the following groups of people is least likely to be included in the labor force? 

A. Retired workers.

B. Part-time workers.

C. People looking for a job.

D. People waiting to start a new job within two weeks.

Correct Answer:  A ..........................................................................................LOS: Reading  22‐a The labor force includes all the employed and unemployed, it does not include retired workers.

Reference: CFA® Program Curriculum, Volume 2, pp. 287‐288. 

Page 171: Cfa Ques Answer

Economics: Microeconomic Market Structure and Macroeconomic Analysis   155 

 

19.  If a monopoly uses price discrimination it is least likely to have the effect of:  

A.  minimizing the consumer surplus. 

B.  finding the single price that will maximize the quantity sold. 

C.  bringing the market demand curve close to the marginal revenue curve. 

D.  producing the quantity which is similar to that produced in a perfectly competitive market. 

 

 

 

20.  Firms acting in collusion in an oligopoly would generally attempt to:  

  Output    Prices 

A.  increase     increase  

B.  increase     decrease 

C.  decrease    increase  

D.  decrease    decrease 

Page 172: Cfa Ques Answer

156   Study Session 05:  

 

19.  If a monopoly uses price discrimination it is least likely to have the effect of:  

A. minimizing the consumer surplus.

B. finding the single price that will maximize the quantity sold.

C. bringing the market demand curve close to the marginal revenue curve.

D. producing the quantity which is similar to that produced in a perfectly competitive market.

Correct Answer:  B...........................................................................................LOS: Reading  19‐c Price discrimination has the effect of transferring the consumer surplus to the monopoly, bringing the market demand curve close to the marginal revenue curve and producing a higher quantity. By definition price discrimination is not a single-price monopoly so answer B is not accurate

Reference: CFA® Program Curriculum, Volume 2, pp. 191‐196. 

 

 

 

20.  Firms acting in collusion in an oligopoly would generally attempt to:  

Output Prices

A. increase increase

B. increase decrease

C. decrease increase

D. decrease decrease

Correct Answer:  C ..........................................................................................LOS: Reading  20‐a Collusion is usually with the intention of increasing economic profit for the participants by limiting output and raising pierces.

Reference: CFA® Program Curriculum, Volume 2, pp. 220‐222. 

Page 173: Cfa Ques Answer

Economics: Microeconomic Market Structure and Macroeconomic Analysis   157 

 

21.  Which of the following is a characteristic of monopolistic competition?  

A.  Firms produce identical products. 

B.  The market is purely competitive.  

C.  Firms have downward sloping demand curves. 

D.  Firms cannot sell products above the market price.  

 

 

 

22.  The prisoners’ dilemma can be applied to price‐fixing in an oligopoly. It explains why: 

A.  members of a cartel are tempted to cheat on other members of the cartel. 

B.  the more players in a cartel, the easier it is to maintain price‐fixing agreements. 

C.  the opportunity to make large profits will tempt new members to join the cartel. 

D.  cartels can be maintained as long as there are only a small number of members. 

Page 174: Cfa Ques Answer

158   Study Session 05:  

 

21.  Which of the following is a characteristic of monopolistic competition?  

A. Firms produce identical products.

B. The market is purely competitive.

C. Firms have downward sloping demand curves.

D. Firms cannot sell products above the market price.

Correct Answer:  C ..........................................................................................LOS: Reading  20‐a A, B and D are all characteristics of a price-taker or a perfectly competitive market.

Reference: CFA® Program Curriculum, Volume 2, pp. 208‐210. 

Page 175: Cfa Ques Answer

Economics: Microeconomic Market Structure and Macroeconomic Analysis   159 

 

22.  The prisoners’ dilemma can be applied to price‐fixing in an oligopoly. It explains why: 

A. members of a cartel are tempted to cheat on other members of the cartel.

B. the more players in a cartel, the easier it is to maintain price-fixing agreements.

C. the opportunity to make large profits will tempt new members to join the cartel.

D. cartels can be maintained as long as there are only a small number of members.

Correct Answer:  A..........................................................................................LOS: Reading  20‐d Game theory shows that Nash equilibrium is reached when participants cheat, although this is not the best outcome for participants.

Reference: CFA® Program Curriculum, Volume 2, pp. 226‐234. 

 

 

Page 176: Cfa Ques Answer

160   Study Session 06:  

 

Study Session 06: Economics: Monetary and Fiscal Economics

 

This study session focuses on the monetary sector of an economy. It examines the functions of money and how it is created, highlighting the special role of the central bank within an economy. Supply and demand for resources, such as labor and capital, and goods are strongly interrelated, and this study session describes  circumstances when  this may  lead  to  inflation and  the  transmission mechanisms between the monetary sector and the real part of the economy. Finally, the goals and implications of fiscal and monetary policy are explored by examining some of  the main models of macroeconomic theory (Keynesian, classical, and monetarist).  

 

Reading 24: Money, Banks, and the Federal Reserve 

Reading 25: Money, Interest, Real GDP, and the Price Level 

Reading 26: Inflation 

Reading 27: Fiscal Policy 

Reading 28: Monetary Policy 

 

 

Page 177: Cfa Ques Answer

Economics: Monetary and Fiscal Economics   161 

 

1.  It is important the Fed’s policies to control inflation have credibility when faced with the prospect of rising inflation since it is more likely that:   

A.  inflation will fall without an increase in unemployment.  

B.  inflation will fall and real GDP will fall below potential GDP. 

C.  aggregate demand will be less than expected when the Fed raises interest rates.  

D.  the supply curve shifts more sharply to the left, compared to the shift with an unanticipated policy change.  

 

 

 

2.  A monetarist is most likely to believe that:  

A.  long term GDP growth is only affected by technological change. 

B.  steady money growth will allow the economy to operate at full employment. 

C.  monetary policy should be constantly adjusted to reflect levels of aggregate demand.  

D.  the government should adopt expansionary fiscal policies when real GDP is below potential GDP.  

 

Page 178: Cfa Ques Answer

162   Study Session 06:  

 

1.  It is important the Fed’s policies to control inflation have credibility when faced with the prospect of rising inflation since it is more likely that:   

A. inflation will fall without an increase in unemployment.

B. inflation will fall and real GDP will fall below potential GDP.

C. aggregate demand will be less than expected when the Fed raises interest rates.

D. the supply curve shifts more sharply to the left, compared to the shift with an unanticipated policy change.

Correct Answer:  A ......................................................................................... LOS: Reading  28‐d If the Fed’s policies are anticipated then wage rates will grow more slowly reflecting the expectations of lower aggregate demand so inflation can be reduced without a move towards recession or increased unemployment. If the policy is not anticipated demand is still expected to increase at a higher rate and money wage rates continue to rise at a higher rate, so the supply curve moves to the left slowing GDP.

Reference: CFA® Program Curriculum, Volume 2, pp. 477‐479.  

 

 

 

2.  A monetarist is most likely to believe that:  

A. long term GDP growth is only affected by technological change.

B. steady money growth will allow the economy to operate at full employment.

C. monetary policy should be constantly adjusted to reflect levels of aggregate demand.

D. the government should adopt expansionary fiscal policies when real GDP is below potential GDP.

Correct Answer:  B.......................................................................................... LOS: Reading  28‐b Monetarists believe that the economy is self regulating and it will normally operate at full employment if the pace of money growth is kept steady.

Reference: CFA® Program Curriculum, Volume 2, pp. 468‐470. 

Page 179: Cfa Ques Answer

Economics: Monetary and Fiscal Economics   163 

 

3.  The quantity theory of money says that an increase in money supply will lead to an increase in: 

A.  prices. 

B.  output. 

C.  employment. 

D.  the velocity of money. 

 

 

 

4.  When the government adopts a counter cyclical fiscal policy in response to a threat of recession the government might: 

A.  increase business tax rates. 

B.  reduce its outstanding debt. 

C.  increase infrastructure spending. 

D.  reduce the size of the budget deficit (or increase the surplus). 

Page 180: Cfa Ques Answer

164   Study Session 06:  

 

3.  The quantity theory of money says that an increase in money supply will lead to an increase in: 

A. prices.

B. output.

C. employment.

D. the velocity of money.

Correct Answer:  A ..........................................................................................LOS: Reading  25‐c The quantity theory of money says a change in money supply will lead to the same change in price levels, since velocity and output are unaffected by the quantity of money. This is derived from MV = PY.

Reference: CFA® Program Curriculum, Volume 2, pp. 386‐389. 

 

 

 

4.  When the government adopts a counter cyclical fiscal policy in response to a threat of recession the government might: 

A. increase business tax rates.

B. reduce its outstanding debt.

C. increase infrastructure spending.

D. reduce the size of the budget deficit (or increase the surplus).

Correct Answer:  C ..........................................................................................LOS: Reading  24‐e A countercyclical policy attempts to move the economy in the opposite direction to the business cycle and therefore ahead of a recession the policy would be aimed at stimulating demand.

Reference: CFA® Program Curriculum, Volume 2, pp. 361‐364. 

Page 181: Cfa Ques Answer

Economics: Monetary and Fiscal Economics   165 

 

5.  The demand for money is: 

A.  the amount of money drawn out of cash and savings accounts over one year.  

B.  the amount of money that people wish to hold in cash and highly liquid assets. 

C.  a measure of consumers’ willingness to increase productivity for additional remuneration. 

D.  the interest rate level that will attract individuals to place a greater proportion of their wealth on deposit with banks. 

 

 

 

6.  Which of the following is least likely to be an impact of higher inflation if the higher inflation was anticipated? 

A.  An increase in effective tax rates. 

B.  Real GDP will fall below potential GDP. 

C.  A reduction of investment by businesses. 

D.  Decision‐makers spend more time forecasting the impact of inflation and less time in productive activities. 

Page 182: Cfa Ques Answer

166   Study Session 06:  

 

5.  The demand for money is: 

A. the amount of money drawn out of cash and savings accounts over one year.

B. the amount of money that people wish to hold in cash and highly liquid assets.

C. a measure of consumers’ willingness to increase productivity for additional remuneration.

D. the interest rate level that will attract individuals to place a greater proportion of their wealth on deposit with banks.

Correct Answer:  B...........................................................................................LOS: Reading  25‐a The demand for money is simply the amount of money that people wish to hold in cash and highly liquid assets.

Reference: CFA® Program Curriculum, Volume 2, pp. 376‐380. 

 

 

 

6.  Which of the following is least likely to be an impact of higher inflation if the higher inflation was anticipated? 

A. An increase in effective tax rates.

B. Real GDP will fall below potential GDP.

C. A reduction of investment by businesses.

D. Decision-makers spend more time forecasting the impact of inflation and less time in productive activities.

Correct Answer:  B.......................................................................................... LOS: Reading  26‐d Unanticipated inflation will reduce real GDP below potential GDP, but anticipated inflation should not do so as resource costs will rise in anticipation of higher prices, so the aggregate demand and aggregate supply curves will both shift together.

Reference: CFA® Program Curriculum, Volume 2, pp. 412‐414. 

Page 183: Cfa Ques Answer

Economics: Monetary and Fiscal Economics   167 

 

7.  The impact on demand for money (M1) of financial innovation and rising interest rates in the U.S. are most likely to be:   

  inancial innovation      Rising interest rate 

A.    increase        increase 

B.    increase       decrease 

C.    decrease       increase  

D.     decrease       decrease 

 

 

 

8.  A move by government to adopt a more expansionary monetary policy in response to the threat of an economic slowdown will, in the long run: 

A.  have no effect on the economy. 

B.  be ineffective since it will be too early to adjust policy. 

C.  tend to stabilize the economy as output responds to the change in policy. 

D.  be ineffective in increasing output since decision‐makers will adjust their expectations to reflect the policy. 

Page 184: Cfa Ques Answer

168   Study Session 06:  

 

7.  The impact on demand for money (M1) of financial innovation and rising interest rates in the U.S. are most likely to be:   

inancial innovation Rising interest rate

A. increase increase

B. increase decrease

C. decrease increase

D. decrease decrease

Correct Answer:  D..........................................................................................LOS: Reading  25‐a Financial innovation tends to reduce the demand for money, new products such as credit card and ATMs have reduced the demand for M1. As interest rates rise the opportunity cost of holding money in currency or checking accounts, which give zero interest, increases and money is placed in interest bearing securities such as savings accounts. This decreases the demand of money.

Reference: CFA® Program Curriculum, Volume 2, pp. 376‐380. 

 

 

 

8.  A move by government to adopt a more expansionary monetary policy in response to the threat of an economic slowdown will, in the long run: 

A. have no effect on the economy.

B. be ineffective since it will be too early to adjust policy.

C. tend to stabilize the economy as output responds to the change in policy.

D. be ineffective in increasing output since decision-makers will adjust their expectations to reflect the policy.

Correct Answer:  D......................................................................................... LOS: Reading  25‐b In the long run decision-makers would have time to adjust their policies, so the only impact will be a rise in prices.

Reference: CFA® Program Curriculum, Volume 2, pp. 386‐387. 

Page 185: Cfa Ques Answer

Economics: Monetary and Fiscal Economics   169 

 

9.  In the case that households  increase their holdings of currency and reduce the balances  in their checking accounts by an equal amount, and the Federal Reserve does not take any offsetting action, the impact on the money supply will be: 

A.  to reduce banks’ required reserves, which will increase money supply. 

B.  to increase money supply since the currency is available for immediate use. 

C.  to reduce the amount that can be loaned by banks, which will tend to reduce the money supply. 

D.  there will be no impact on money supply, since the increased holdings in currency will offset the reduction in checking accounts. 

 

 

 

10.  According to the quantity theory of money if the GDP of a country is $50 billion and the velocity of circulation is 8, then the quantity of money is:  

A.  $6.25 billion. 

B.  $160 billion. 

C.  $400 billion. 

D.  $3,200 billion. 

Page 186: Cfa Ques Answer

170   Study Session 06:  

 

9.  In the case that households  increase their holdings of currency and reduce the balances  in their checking accounts by an equal amount, and the Federal Reserve does not take any offsetting action, the impact on the money supply will be: 

A. to reduce banks’ required reserves, which will increase money supply.

B. to increase money supply since the currency is available for immediate use.

C. to reduce the amount that can be loaned by banks, which will tend to reduce the money supply.

D. there will be no impact on money supply, since the increased holdings in currency will offset the reduction in checking accounts.

Correct Answer:  C ..........................................................................................LOS: Reading  24‐a Checking accounts and currency are included in money supply. If the balances in checking accounts are reduced then it will not directly affect the money supply but it will reduce the amount of reserves that the banks need to keep and also reduce the amount that they can loan, which will reduce the money supply.

Reference: CFA® Program Curriculum, Volume 2, pp. 348‐352. 

 

 

 

10.  According to the quantity theory of money if the GDP of a country is $50 billion and the velocity of circulation is 8, then the quantity of money is:  

A. $6.25 billion.

B. $160 billion.

C. $400 billion.

D. $3,200 billion.

Correct Answer:  A ..........................................................................................LOS: Reading  25‐c Use MV = GDP, so M = $50 billion/8 = $6.25 billion.

Reference: CFA® Program Curriculum, Volume 2, pp. 387‐389. 

Page 187: Cfa Ques Answer

Economics: Monetary and Fiscal Economics   171 

 

11.  When aggregate demand drops sharply the Keynesian feedback rule is most likely to lead to the Fed: 

  Interest rates    Quantity of money 

A.  increasing      increasing 

B.  increasing       decreasing 

C.  decreasing      increasing 

D.  decreasing       decreasing 

 

 

 

12.  Intermediate targets of the Fed’s monetary policy are least likely to include:  

A.  M1. 

B.  Treasury‐bill rate. 

C.  the monetary base.  

D.  the federal funds rate. 

Page 188: Cfa Ques Answer

172   Study Session 06:  

 

11.  When aggregate demand drops sharply the Keynesian feedback rule is most likely to lead to the Fed: 

Interest rates Quantity of money

A. increasing increasing

B. increasing decreasing

C. decreasing increasing

D. decreasing decreasing

Correct Answer:  C ..........................................................................................LOS: Reading  28‐c The Fed would decrease the interest rate and increase the quantity of money in order to shift the aggregate demand curve back to the right.

Reference: CFA® Program Curriculum, Volume 2, pp. 470‐472. 

 

 

 

12.  Intermediate targets of the Fed’s monetary policy are least likely to include:  

A. M1.

B. Treasury-bill rate.

C. the monetary base.

D. the federal funds rate.

Correct Answer:  C ..........................................................................................LOS: Reading  28‐a Intermediate targets give the Fed information on whether the monetary policy is having the desired effect. Typically these targets will be monetary aggregates (M1, M2 and/or the monetary base) and the federal funds rate.

Reference: CFA® Program Curriculum, Volume 2, pp. 462‐463. 

Page 189: Cfa Ques Answer

Economics: Monetary and Fiscal Economics   173 

 

13.  After analysis of the economic environment the Fed decides not to change its monetary policy in response to a demand‐driven slowdown in the economy.  This is most likely to be an example of a:  

A.  fixed‐rule policy. 

B.  stabilization policy. 

C.  discretionary policy. 

D.  feedback‐rule policy. 

 

 

 

14.  Which of the following is an automatic stabilizer? 

A.  Interest rates. 

B.  Inflation rates. 

C.  Exchange rates. 

D.  Progressive corporate tax. 

Page 190: Cfa Ques Answer

174   Study Session 06:  

 

13.  After analysis of the economic environment the Fed decides not to change its monetary policy in response to a demand‐driven slowdown in the economy.  This is most likely to be an example of a:  

A. fixed-rule policy.

B. stabilization policy.

C. discretionary policy.

D. feedback-rule policy.

Correct Answer:  C ......................................................................................... LOS: Reading  23‐b Discretionary policies respond to the state of the economy after taking into account all of the available information so C is the best answer.

Reference: CFA® Program Curriculum, Volume 2, pp. 467‐468. 

 

 

 

14.  Which of the following is an automatic stabilizer? 

A. Interest rates.

B. Inflation rates.

C. Exchange rates.

D. Progressive corporate tax.

Correct Answer:  D..........................................................................................LOS: Reading  27‐e Automatic stabilizers will tend to expand a budget deficit during a recession and contract a budget deficit in a boom. They include unemployment benefits, progressive corporate and personal taxes.

Reference: CFA® Program Curriculum, Volume 2, pp. 452‐455. 

Page 191: Cfa Ques Answer

Economics: Monetary and Fiscal Economics   175 

 

15.  Which of the following is least likely to be included in M1? 

A.  Currency. 

B.  Issued checks. 

C.  Traveler’s checks. 

D.  Checking deposits. 

 

 

 

16.  The demand for money curve shows that the quantity of money demanded is:  

A.  positively related to GDP growth. 

B.  inversely related to GDP growth. 

C.  positively related to interest rates. 

D.  inversely related to interest rates. 

Page 192: Cfa Ques Answer

176   Study Session 06:  

 

15.  Which of the following is least likely to be included in M1? 

A. Currency.

B. Issued checks.

C. Traveler’s checks.

D. Checking deposits.

Correct Answer:  B.......................................................................................... LOS: Reading  24‐b M1 is made up of currency, traveler’s checks and checking deposits. Checks are a promise to pay, not a form of money.

Reference: CFA® Program Curriculum, Volume 2, pp. 302‐305. 

 

 

 

16.  The demand for money curve shows that the quantity of money demanded is:  

A. positively related to GDP growth.

B. inversely related to GDP growth.

C. positively related to interest rates.

D. inversely related to interest rates.

Correct Answer:  D..........................................................................................LOS: Reading  25‐a Demand for money measures the relationship between interest rates and the amount of money that people want to hold.

Reference: CFA® Program Curriculum, Volume 2, pp. 376‐380. 

Page 193: Cfa Ques Answer

Economics: Monetary and Fiscal Economics   177 

 

17.  The crowding‐out model says: 

A.  increasing budget deficits to stimulate growth has no effect on economic activity. 

B.  increasing budget deficits to stimulate growth will be lead to a decline in net exports. 

C.  increasing budget deficits to stimulate growth will be financed by higher total savings rates. 

D.  the effect of increasing budget deficits to stimulate growth will be dampened by the negative impact of higher real interest rates on private spending. 

 

 

 

18.  If banks retain 5% of new deposits in reserves and lend out 95%, when a customer puts $100,000 on deposit with a bank, it will lead to a potential increase in loans and deposits of: 

  Loans      Deposits 

A.  $1,900,000      $2,000,000 

B.  $2,000,000      $2,000,000 

C.  $9,400,000       $9,500,000 

D.  $9,500,000       $9,500,000 

Page 194: Cfa Ques Answer

178   Study Session 06:  

 

17.  The crowding‐out model says: 

A. increasing budget deficits to stimulate growth has no effect on economic activity.

B. increasing budget deficits to stimulate growth will be lead to a decline in net exports.

C. increasing budget deficits to stimulate growth will be financed by higher total savings rates.

D. the effect of increasing budget deficits to stimulate growth will be dampened by the negative impact of higher real interest rates on private spending.

Correct Answer:  D......................................................................................... LOS: Reading  27‐b A budge deficit decreases savings supply and therefore increases the real interest rate, this will crowd out private investment.

Reference: CFA® Program Curriculum, Volume 2, pp. 444‐445. 

 

 

 

18.  If banks retain 5% of new deposits in reserves and lend out 95%, when a customer puts $100,000 on deposit with a bank, it will lead to a potential increase in loans and deposits of: 

Loans Deposits

A. $1,900,000 $2,000,000

B. $2,000,000 $2,000,000

C. $9,400,000 $9,500,000

D. $9,500,000 $9,500,000

Correct Answer:  A ......................................................................................... LOS: Reading  24‐d The bank keeps $5,000 as reserves and can lend out $95,000. This money is in turn placed on deposit at another bank (after being used as payment in a transaction) and the next bank keeps $4,750 as reserves and lends out $90,250. If we continue the exercise we can see that the total increase in deposits is

$100,000 + $95,000 + $90,250 + …. = $100,000(1/0.05) = $2,000,000

The potential deposit expansion multiplier is 1/0.05 = 20.

The increase in loans will be $2,000,000, less the original $100,000.

Reference: CFA® Program Curriculum, Volume 2, pp. 356‐359. 

Page 195: Cfa Ques Answer

Economics: Monetary and Fiscal Economics   179 

 

19.  A generational imbalance created by fiscal policy refers to: 

A.  different generations are likely to pay different proportions of any fiscal imbalance. 

B.  the difference between the present value of expected tax revenues and social security obligations. 

C.  the difference between the current year’s forecast tax revenues and social security obligations. 

D.  the imbalance between the amount of tax collected from the younger generation and the amount collected from the older generation in any one calendar year. 

 

 

 

20.  The slope of the long‐run Phillips curve shows that: 

A.  unemployment and inflation are inversely related. 

B.  unemployment and expected inflation are inversely related. 

C.  at the natural unemployment rate any anticipated inflation rate is possible. 

D.  an increase in aggregate demand lowers unemployment and increases inflation. 

Page 196: Cfa Ques Answer

180   Study Session 06:  

 

19.  A generational imbalance created by fiscal policy refers to: 

A. different generations are likely to pay different proportions of any fiscal imbalance.

B. the difference between the present value of expected tax revenues and social security obligations.

C. the difference between the current year’s forecast tax revenues and social security obligations.

D. the imbalance between the amount of tax collected from the younger generation and the amount collected from the older generation in any one calendar year.

Correct Answer:  A ..........................................................................................LOS: Reading  27‐c The generational imbalance refers to how the load of the fiscal imbalance will be spread among different generations. In the U.S. it looks likely that part of the imbalance will have to be paid for by future generations.

Reference: CFA® Program Curriculum, Volume 2, pp. 445‐447. 

 

 

 

20.  The slope of the long‐run Phillips curve shows that: 

A. unemployment and inflation are inversely related.

B. unemployment and expected inflation are inversely related.

C. at the natural unemployment rate any anticipated inflation rate is possible.

D. an increase in aggregate demand lowers unemployment and increases inflation.

Correct Answer:  C ..........................................................................................LOS: Reading  26‐e The LRPC is the relationship between inflation and unemployment when the inflation rate equals the expected rate. It is vertical since GDP equals potential GDP when inflation is anticipated and unemployment is at its natural rate.

Reference: CFA® Program Curriculum, Volume 2, pp. 416‐417. 

Page 197: Cfa Ques Answer

Economics: Monetary and Fiscal Economics   181 

 

21.  Which  of  the  following  is  the  most  accurate  description  of  what  generational  accounting measures? 

 

A.  life expectancy and pension costs of each generation. 

B.  the tax burden and benefits taken by each generation. 

C.  the division of the fiscal imbalance between the different generations. 

D.  present value of a government’s obligation to pay benefits and the present value of its tax revenues.  

 

 

 

22.  The year‐end price data for a country was:  

Year‐end  Price level 

2005  110 

2006  113 

2007  117 

 

The annual inflation rate in 2006 and the compound annual rate for the 2005‐2007 period were closest to:   

  2006 inflation rate     2005‐2007 annual rate  

A.  2.7%      3.1% 

B.  2.7%      6.2% 

C.  3.0%      7.0% 

D.  3.5%      3.1%  

Page 198: Cfa Ques Answer

182   Study Session 06:  

 

21.  Which  of  the  following  is  the  most  accurate  description  of  what  generational  accounting measures? 

A. life expectancy and pension costs of each generation.

B. the tax burden and benefits taken by each generation.

C. the division of the fiscal imbalance between the different generations.

D. present value of a government’s obligation to pay benefits and the present value of its tax revenues.

Correct Answer:  B...........................................................................................LOS: Reading  27‐c Generational accounting is the method used to measure tax (income and social security) paid by each generation versus the social security benefit they receive in retirement. D refers to fiscal balances across all generations and C refers to the generational imbalance.

Reference: CFA® Program Curriculum, Volume 2, pp. 445‐447. 

Page 199: Cfa Ques Answer

Economics: Monetary and Fiscal Economics   183 

 

22.  The year‐end price data for a country was:  

Year‐end  Price level 

2005  110 

2006  113 

2007  117 

 The annual inflation rate in 2006 and the compound annual rate for the 2005-2007 period were closest to:

2006 inflation rate 2005-2007 annual rate

A. 2.7% 3.1%

B. 2.7% 6.2%

C. 3.0% 7.0%

D. 3.5% 3.1%

Correct Answer:  A.......................................................................................... LOS: Reading  26‐a 

 

100×= yearlast level Price

yearlast level Price - yearthis level Price rateInflation

 

%7.2=100×110

110113=2006 rateInflation

 

%5.3=100×113

113117=2007 rateInflation

 

The compound annual rate is  ( )( ) %1.3=1035.1027.1   

Reference: CFA® Program Curriculum, Volume 2, pp. 400‐401. 

 

 

Page 200: Cfa Ques Answer

184   Study Session 7 Introduction 

 

Study Session 7: Introduction Introductory Readings

Financial  Accounting,  8th  edition,  Belverd  E.  Needles,  Jr.,  and  Marian  Powers,  (Houghton Mifflin, 2004) “Measuring Business Income,” Ch. 3 “Financial Reporting and Analysis,” Ch. 5, pp. 246–258 “Inventories,” Ch. 8 “Current Liabilities and the Time Value of Money,” Ch. 9, pp. 412–426 “Contributed Capital,” Ch. 12, pp. 543‐553 “The Corporate Income Statement and the Statement of Stockholders’ Equity,” Ch.13, pp. 584‐591 

Measuring Business Income Introduction

This  section  looks  at  the  concept  of  net  income  which  is  used  to  measure  a  company’s profitability.  Candidates  need  to  be  familiar  with  the  accrual  basis  of  accounting  and  the matching principle and why they are used, rather than accounting simply on a cash basis. 

Accounting methods Financial statements are prepared at the end of regular accounting periods to make comparisons between different accounting periods easier. A fiscal year refers to the twelve‐month period used by a company (which in many cases is not the same as the calendar year). Although revenues and expenses can be accounted  for on a cash basis  for  tax purposes,  this  is generally regarded as unsatisfactory since revenues are often earned  in a different period  from which  the payments are received, and expenses paid  in a different period  from which  they are incurred.  The matching  rule  states  that  revenues must  be  assigned  to  the  periods when  the services are performed or  the goods are  sold, and expenses must be assigned  to  the period  in which they produce the revenue. Accrual accounting refers  to methods  that accountants use  to apply  the matching rule,  i.e. that revenues are recognized when they are earned and expenses when they are incurred rather than when  they are actually paid. Accounts are  likely  to need adjustment. For example  transactions may occur in one accounting period but the benefits may spread over more than one accounting period.  

Page 201: Cfa Ques Answer

Introductory Readings   185 

 

Financial Reporting and Analysis Introduction

In this section Candidates learn the definition of each category or component on the balance sheet and income statement. We will refer to balance sheet and income statement items throughout the rest  of  the  Financial  Statement Analysis  notes  so  it  is  essential  you  are  comfortable with  the structure, presentation and contents of financial statements. 

Balance Sheet The major categories of a balance sheet are as follows: 

ASSETS  LIABILITIES 

Current Assets  Current Liabilities 

Investments  Long‐term Liabilities 

Property, Plant and Equipment   

Intangible Assets  STOCKHOLDERS’ EQUITY   Contributed Capital  

  Retained Earnings Current Assets – cash and other assets which can reasonably be expected to be realized in cash or used within one year, or within the normal operating cycle of the business, whichever is longer. Investments  –  assets  that  are  not  used  in  the  normal  operation  of  the  business  and  are  not expected to be converted into cash within one year. Property,  Plant  and  Equipment  –  long‐term  assets  used  in  the  continuing  operation  of  the business. These assets are depreciated to allocate the cost of the assets over the period when they are used. Intangible Assets – long‐term assets with no physical substance – e.g. patents, goodwill. Current Liabilities  –  obligations due  to  be paid  or performed within  one  year,  or within  the normal operating cycle of the business, whichever is longer. Long‐Term Liabilities – debts that are due to be paid out in more than one year or beyond the normal operating cycle. Contributed Capital – the par value of issued stock plus the amounts paid‐in in excess of the par value. Retained Earnings – earnings that have been retained by the company, rather than distributed to stockholders.  

Page 202: Cfa Ques Answer

186   Study Session 7 Introduction 

 

Income statement The components of a multi‐step income statement are as follows: 

  Net Sales –    Cost of Goods Sold =  Gross Profit –    Operating Expenses =   Income from Operations +/‐  Other Revenue and Expenses =   Income before Income Taxes –    Income Taxes =   Net Income 

Net Sales – gross proceeds from sales, less sales returned and discounts offered. Cost of Goods Sold – amount paid for the goods that were sold. Operating  Expenses  –  costs  other  than  the  cost  of  goods  sold,  often  broken  down  into  selling expenses and general and administrative expenses. Other Revenue and Expenses – revenues and expenses  that are not a result of operating activities, these include interest income and interest expenses. 

Page 203: Cfa Ques Answer

Introductory Readings   187 

 

Inventories Introduction

Inventory  is another short‐term asset appearing on the balance sheet and  the cost of  inventory will also  affect  the  income  statement.  There  are  four  methods  of  accounting  for  inventory  and  the candidate needs to understand the impact on the balance sheet and income statement of the method used. This Reading is covered in more detail in Study Session 9, Reading 39. 

Inventory Inventory is a current asset and consists of goods held for sale in the normal course of business. The cost of inventory will include the costs of raw materials used, cost of labor and overhead costs. The cost  of  goods  sold  (COGS)  is measured  as  the  cost  of  goods  available  for  sale  less  the  value  of inventory  at  the  end  of  the  accounting  period.  The  valuation method  used  to  value  inventory  is critical since it will decide the COGS and gross profit of the company and the value of inventory on the balance sheet. 

Inventory cost Inventory cost  is the price or consideration paid to acquire an asset. There are three components to inventory cost: 

• Invoice price less purchase discounts. • Freight or transportation including insurance costs when in transit. • Applicable taxes and tariffs.

It  could  be  argued  that  other  costs,  such  as  storage  costs,  should  also  be  included  in  the  cost  of inventory. Allocation of these costs is difficult so they are usually expensed. When  assigning  costs  to  items  that  are  sold, different methods  can be used. These methods make different assumptions on the order in which items are sold. The four most commonly used methods are shown below. 

Specific Identification Method This method can be used when it is possible to match units left in inventory with a specific purchase. This might be used by an art dealer where there are high priced items for sale that are all unique. 

Average-Cost Method Under  this method  inventory  is  priced  at  the  average  cost  of  items  available  for  sale  during  the period. 

First-In, First-Out Method (FIFO) This is based on the assumption that the cost of goods sold is associated with the earliest purchases in inventory. This means that the cost of goods held in inventory is associated with those that have been purchased most recently. 

Last-In, First-Out Method (LIFO) The  assumption  under  LIFO  is  that  the  cost  of  goods  sold  is  associated with  the most  recently purchased,  the cost of goods held  in  inventory are associated with  those  that have been purchased the earliest. 

Page 204: Cfa Ques Answer

188   Study Session 7 Introduction 

 

 

Example Int-1 Calculating cost of inventory

On December 31st a company holds 200 units in inventory with an assigned total value of $1,000, or $5.00 per unit. On January 10th it purchases 30 additional units at a cost of $5.50 per unit and on January 20th it purchases 70 additional units at a cost of $6.00 per unit. 50 units are sold over the month so 250 units are left in inventory at the end of January.

Specific Identification Method This requires additional information. If we are told that the specific cost of the 50 units sold was $4.50 each, then

Cost of Goods Sold (COGS)   =  00.225$50.4$50 =×  January 31st inventory   = Costs of goods available for sale – COGS         = $1,000 + (30 x $5.50) + (70 x $6.00) ‐‐ $225 = $1,360.00 Average‐Cost Method Average unit cost of goods available for sale         = Cost of goods available for sale/units available         = $1,585/(200 + 30 + 70) = $5.28 Cost of goods available  = $1,585.00 less January 31st inventory   28.5$250×=          00.320,1$=   COGS         = $265.00 First‐In, First‐Out Method (FIFO) 

In this case the 50 units sold are those that were purchased first at a cost of $5.00 per unit

Cost of goods available  = $1,585.00 less January 31st inventory  ( ) ( ) ( )00.6$7050.5$3000.5$150 ×+×+×=          00.335,1$=  COGS        = $250.00 Last‐In, First‐Out Method (LIFO) In this case the 50 units sold are those that were purchased  last at a cost of $6.00 per unit. Costs of goods available   = $1,585.00 less January 31st inventory   ( ) ( ) ( )00.6$2050.5$3000.5$200 ×+×+×=          = $1,285.00 COGS         = $300.00 

Effect of inventory accounting method In the above example, where prices are rising, we can see that LIFO gives the highest COGS and FIFO the  lowest COGS,  so LIFO will give  the  lowest gross margin. For  income  statement  items LIFO  is generally  considered  the best method  since  it more  closely  follows  the matching  rule,  the  current value of costs is used. However, looking at the balance sheet, the inventory value recorded is higher 

Page 205: Cfa Ques Answer

Introductory Readings   189 

 

under FIFO than LIFO. In this case FIFO could be considered a better method since inventory values are closer to current values. When the replacement cost of inventory falls below the historic cost, perhaps due to a decline in price levels or obsolescence,  then  the  inventory  should be written down  to  the  lower‐of‐cost‐or‐market. There are two methods for doing this: 

Item-by-Item Method Cost and market values are computed for each item in inventory. Major Category Method Total cost and total market values are computed for each category of goods. 

 

Example Int-2 Calculating inventory values

Item‐by‐Item Method 

  Quantity Cost per Unit 

Market Value per Unit 

Lower of Cost or Market 

Category 1   Item a  100 $4.00 $4.50  $400.00Item b  100 $5.00 $3.50  $350.00Category 2   Item c  100 $2.00 $1.00  $100.00Item d  100 $6.00 $5.00  $500.00

  Inventory at the lower of cost or market  $1,350.00 

Major Category Method 

Category 1  Quantity  Total Cost Total Market Value 

Lower of Cost or Market 

Item a  100 $400.00 $450.00 Item b  100 $500.00 $350.00 Total  $900.00 $800.00  $800.00Category 2   Item c  100 $200.00 $100.00 Item d  100 $600.00 $500.00 Total  $800.00 $600.00  $600.00

  Inventory at the lower of cost or market  $1,400.00  

Page 206: Cfa Ques Answer

190   Study Session 7 Introduction 

 

Current Liabilities and the Time Value of Money Introduction

We now move on  to  look  at  the  liability  side of  the balance  sheet. Candidates need  to be  able  to differentiate  between  current  liabilities  and  long‐term  liabilities  and  know  how  to  treat  liabilities when the amount or likelihood of payment is uncertain. 

Liabilities A liability is a legal obligation to make a future payment of assets, or perform a service in the future, as a result of a past transaction. Current liabilities are liabilities that are expected to be paid within a year, or within a normal operating cycle, whichever is longer. Long‐term liabilities are due after one year  or  operating  cycle. Generally,  current  liabilities  are paid  from  the  current  assets  or  from  the proceeds  of  current  operations whereas  long‐term  liabilities  represent  the  financing  of  long‐term assets. Liabilities are usually valued at the amount of money need to pay the debt or the value of goods or services  to  be  delivered. Whilst  in  some  cases  the  amount  is  definitely  known  in  other  cases  an estimate  is made, perhaps based on past experience. An example  is when a sale  is made but  there remains a liability to service the item sold. Current liabilities can be broken down into: 

(i) Definitely Determinable Liabilities • Accounts payable • Bank loans and commercial paper • Notes payable • Accrued liabilities • Dividends payable • Sales and excise tax payable • Current portion of long-term debt • Payroll liabilities • Unearned revenues (ii) Estimated Liabilities

The existence of a  liability  is clear but often  the amount will not be known until  later. These  items include: 

• Income taxes – a company’s managers will usually only know the profits after the year-end so an estimated figure must be used.

• Property tax payable – these are usually paid to the local governments in the U.S., and the assessment dates are unlikely to match a firm’s year-end. Therefore an estimated figure should be used.

• Product warranty liability – whilst a warranty or guarantee is outstanding on a product there is a liability and a firm should estimate how much the warranty is likely to cost.

• Vacation pay liability – not all employees will collect vacation pay so this should also be recorded as an estimated liability.

Page 207: Cfa Ques Answer

Introductory Readings   191 

 

(iii)Contingent liabilities A contingent liability is not an existing liability but a potential liability in the sense that it depends on the outcome of a future event that arises because of a past transaction. It  should be entered  in  the accounts  if  it  is both  (i) probable and  (ii)  can be  reasonably estimated. Potential  liabilities  that  do  not meet  these  conditions  should  be  referred  to  in  the  notes  to  the accounts. 

Contributed Capital Introduction

Candidates are expected  to know  the breakdown of contributed capital and  the difference between preferred  stock and ordinary  stock. We also  look at payments  to  stockholders  in  the  form of  cash dividends or  repurchase of  stock  and  the  impact on  the balance  sheet. Dividends  are  revisited  in Study Session 13. 

Contributed capital This is made up of three components: 

1. Preferred Stock – par value, amount authorized and amount issued and outstanding. 2. Common Stock – par value, amount authorized and amount issued and outstanding. 3. Paid‐in Capital in excess of par value. Accounting for dividends

There are three dates to consider: 1. Date of declaration  – when  the directors declare  that  a dividend  is going  to be paid,  the 

issuer will record a cash dividend payable item as a liability. 2. Date of  record – when ownership of  the  stock entitles  the owner  to  receive  the dividend. 

After this, and prior to payment, the stock is ex‐dividend. No accounting entries are needed. 3. Date of payment – the date the dividend is paid, and the liability is settled. Common stock

This is the residual equity which means that the holders have the last claim on assets in the case that the company goes  into  liquidation.  It  is usually  the only stock  that carries voting rights. Dividends may be paid to stockholders which means that part of the stockholders’ equity, usually earnings, is being paid out to stockholders. 

Preferred stock This has preference over common stock, usually in terms of both dividends and claim on assets in the case of  liquidation. Dividends are often quoted as a percentage of par value and  in  this  sense  the characteristics of preferred stock are similar to those of a fixed‐income instrument. Different types of preferred stock are: Cumulative preferred stock – if a dividend is missed then it accumulates and must be paid before a dividend can be paid to common stock holders. Convertible preferred stock – can be exchanged into common stock. Callable preferred stock – the issuer can redeem the stock at a pre‐specified price. 

Page 208: Cfa Ques Answer

192   Study Session 7 Introduction 

 

Stock issuance Par value stock – the par value is credited to the Common Stock (or Preferred Stock) account and any surplus, or deficit, to the ‘Paid‐in Capital in Excess of Par Value’ account, or debited to the ‘Discount on Capital Stock’ account respectively. No‐par stock – the proceeds of the issue are credited to the Common Stock account. 

Treasury stock This is stock that has been bought back by the issuer, usually in the stock market, and not been resold or retired. It is treated as stock that has been issued but is no longer outstanding, and therefore does not have voting rights, rights to dividends etc. 

The Corporate Income Statement and the Statement of Stockholders’ Equity Introduction

Here we consider stock dividends and stock splits which are corporate actions  that do not affect a company’s income or asset value. 

Retained earnings Retained  earnings  are  the  part  of  stockholders’  equity  that  represents  the  stockholders’  claim  on assets generated by the firm’s earnings. It is the profits (or losses) since inception less any dividends paid to stockholders or transfers to contributed capital. 

Accounting for stock dividends and stock splits Stock dividends

This  is  a distribution of  shares  to  existing  common  stockholders  in proportion  to  the  size of  their holding. This does not  involve a cash payment and  leads only  to a  transfer of  funds  from retained earnings  to  the  contributed  capital  account. The  amount  transferred  is determined  by  the market value of the shares issued. 

Stock splits This  is  when  a  corporation  increases  the  number  of  issued  shares  and  reduces  the  par  value accordingly. Again  this does not  involve a  cash payment. The motivation  is usually  to  reduce  the market price and increase liquidity of the shares. 

Page 209: Cfa Ques Answer

Introductory Readings   193 

 

Introduction Concept Check Questions

1.  In a firm’s financial statements expenses should be recorded:  

A. when they occur.

B. when they are paid.

C. once they can be reasonably estimated.

D. in the same period that related revenues are recorded.

 

 

 

 

2.  When inventory costs are increasing due to inflation, the decision to use LIFO rather than FIFO will lead to: 

A. lower net income and lower inventory balances.

B. lower net income and higher inventory balances.

C. higher net income and lower inventory balances.

D. higher net income and higher inventory balances.

Page 210: Cfa Ques Answer

194   Study Session 7 Introduction 

 

3.  A stock dividend: 

A. Has no impact on the financial statements.

B. Reduces the par value of stock outstanding.

C. Transfers an amount from retained earnings to contributed capital.

D. On the declaration date reduces stockholders’ equity by the size of the dividend payable.

 

 

 

 

4.  If Treasury  stock  is  included  in a  firm’s balance  sheet as part of  stockholders’ equity  it means that:  

A. the firm has issued preferred shares.

B. the firm has repurchased its own stock.

C. the firm has invested in Treasury notes or Treasury bonds.

D. the firm has issued stock where the dividends are linked to Treasury bill yields.

Page 211: Cfa Ques Answer

Introductory Readings   195 

 

Introduction Concept Check Answers  1.  In a firm’s financial statements expenses should be recorded:  

A. when they occur.

B. when they are paid.

C. once they can be reasonably estimated.

D. in the same period that related revenues are recorded.

Correct Answer 1:   D 

The matching rule says that related revenues and expenses should be recorded in the same accounting period.

 

 

 

2.  When inventory costs are increasing due to inflation, the decision to use LIFO rather than FIFO will lead to: 

A. lower net income and lower inventory balances.

B. lower net income and higher inventory balances.

C. higher net income and lower inventory balances.

D. higher net income and higher inventory balances.

Correct Answer 2:   A 

COGS will be higher under LIFO reducing net income. Ending inventory will be lower since it will include ‘old’ inventory bought at lower prices.

Page 212: Cfa Ques Answer

196   Study Session 7 Introduction 

 

3.  A stock dividend: 

A. has no impact on the financial statements.

B. reduces the par value of stock outstanding.

C. transfers an amount from retained earnings to contributed capital.

D. on the declaration date reduces stockholders’ equity by the size of the dividend payable.

Correct Answer 3:   C 

B refers to a stock split. D is not true since no cash is paid out.

 

 

 

4.  If Treasury  stock  is  included  in a  firm’s balance  sheet as part of  stockholders’ equity  it means that:  

A. the firm has issued preferred shares.

B. the firm has repurchased its own stock.

C. the firm has invested in Treasury notes or Treasury bonds.

D. the firm has issued stock where the dividends are linked to Treasury bill yields.

3.

Correct Answer 4:   B 

Treasury stock refers to repurchased stock. This is likely to have happened because the managers believed the stock was undervalued in the market or as a defense against a takeover.

 

 

Page 213: Cfa Ques Answer

Economics: Monetary and Fiscal Economics   197 

 

 

 

 

Page 214: Cfa Ques Answer

198   Study Session 07:  

 

Study Session 07: Financial Statement Analysis: An Introduction

The  readings  in  this  study  session discuss  the general principles of  the  financial  reporting  system, underscoring the critical role of the analysis of financial reports in investment decision making. 

The  first  reading  introduces  the  range  of  information  that  an  analyst may  use  in  analyzing  the financial  performance  of  a  company,  including  the  principal  financial  statements  (the  income statement, balance sheet, cash flow statement, and statement of changes in owners’ equity), notes to those  statements,  and management  discussion  and  analysis  of  results.  A  general  framework  for addressing most financial statement analysis tasks is also presented. 

A  company’s  financial  statements  are  the  end‐products  of  a  process  for  recording  the  business transactions  of  the  company.  The  second  reading  illustrates  this  process,  introducing  such  basic concepts as the accounting equation and accounting accruals. 

The presentation of financial information to the public by a company must conform to the governing set of financial reporting standards applying in the jurisdiction in which the information is released. The  final  reading  in  this  study  explores  the  role  of  financial  reporting  standard‐setting  bodies worldwide and the International Financial Reporting Standards framework promulgated by one key body,  the  International  Accounting  Standards  Board.  The  movement  towards  worldwide convergence of financial reporting standards is also introduced. 

Note:  New rulings and/or pronouncements issued after the publication of the readings in Study Sessions 7 through 10  in  financial statement analysis may cause some of  the  information  in  these  readings  to become  dated.  Candidates  are  expected  to  be  familiar  with  the  overall  analytical  framework contained in the study session readings, as well as the implications of alternative accounting methods for financial analysis and valuation, as provided in the assigned readings. For the purpose of Level I questions  on  financial  statement  analysis,  when  a  ratio  is  defined  and  calculated  differently  in various texts, candidates should use the definitions given in the CFA Institute copyrighted readings by Robinson, et al. Variations in ratio definitions are part of the nature of practical financial analysis.  

 

Reading 29: Financial Statement Analysis: An Introduction 

Reading 30: Financial Reporting Mechanics 

Reading 31: Financial Reporting Standards 

 

Page 215: Cfa Ques Answer

Financial Statement Analysis: Introduction   199 

 

1.  When  a  firm  records  an  allowance  for  uncollectible  receivables  as  a  deduction  against  the receivables account this is: 

A.  a contra account. 

B.  an adjunct account. 

C.  a non‐recurring item. 

D.  an extraordinary item. 

 

 

 

2.  A general journal records all: 

A.  business transactions by account. 

B.  business transactions in date order. 

C.  account balances, usually prepared on a daily basis. 

D.  account balances, usually prepared at the end of each accounting period.  

 

Page 216: Cfa Ques Answer

200   Study Session 07:  

 

1.  When  a  firm  records  an  allowance  for  uncollectible  receivables  as  a  deduction  against  the receivables account this is: 

A. a contra account.

B. an adjunct account.

C. a non-recurring item.

D. an extraordinary item.

Correct Answer:  A ......................................................................................... LOS: Reading  30‐b An account which reduces the value of an asset, to a new estimate of its net realizable value, is a contra account.

Reference: CFA® Program Curriculum, Volume 3, pp. 38‐39. 

 

 

 

2.  A general journal records all: 

A. business transactions by account.

B. business transactions in date order.

C. account balances, usually prepared on a daily basis.

D. account balances, usually prepared at the end of each accounting period.

Correct Answer:  B...........................................................................................LOS: Reading  30‐g The general journal is the fist step in the flow of information through a financial reporting system. It is where all transactions are recorded showing which accounts they affect, they are recorded in date order.

Reference: CFA® Program Curriculum, Volume 3, pp. 68‐69. 

Page 217: Cfa Ques Answer

Financial Statement Analysis: Introduction   201 

 

3.  Income that is independent of a firm’s capital structure is: 

A.  net income. 

B.  pre‐tax income. 

C.  operating income. 

D.  comprehensive income. 

 

 

 

4.  Which of the following is least likely to be important to an analyst when using financial reports? 

 

A.  Detail. 

B.  Relevance. 

C.  Materiality. 

D.  Consistency. 

Page 218: Cfa Ques Answer

202   Study Session 07:  

 

3.  Income that is independent of a firm’s capital structure is: 

A. net income.

B. pre-tax income.

C. operating income.

D. comprehensive income.

Correct Answer:  C ......................................................................................... LOS: Reading  29‐b Operating income is revenues less costs of goods sold, SGA, and research and development. Therefore it is prior to interest costs which reflect the cost of debt financing.

Reference: CFA® Program Curriculum, Volume 3, p. 13. 

 

 

 

4.  Which of the following is least likely to be important to an analyst when using financial reports? 

A. Detail.

B. Relevance.

C. Materiality.

D. Consistency.

Correct Answer:  A ..........................................................................................LOS: Reading  31‐e If the financial reports are to be relevant and timely it is not practical for them to include every detail of the financial data.

Reference: CFA® Program Curriculum, Volume 3, pp. 109‐110. 

Page 219: Cfa Ques Answer

Financial Statement Analysis: Introduction   203 

 

5.  A company sells $50,000 of goods but under credit terms given to customers, payment will not be received  for 30 days.   The  cost of  the goods  is  recorded as $35,000. As a  result of  this  transaction assets will:  

A.  decrease by $35,000. 

B.  decrease by $50,000. 

C.  increase by $15,000. 

D.  increase by $50,000. 

 

 

 

6.  Two  firms  buy  new  photocopiers.    The  first  firm  is  a  retailer  of  office  equipment  and  the photocopier will be resold, and the second firm is a bank planning to use the photocopier in its head office.  How will the purchases be classified? 

  Retailer      Bank 

A.  investing activity    investing activity 

B.  operating activity    investing activity 

C.  operating activity    financing activity 

D.  operating activity    operating activity 

Page 220: Cfa Ques Answer

204   Study Session 07:  

 

5.  A company sells $50,000 of goods but under credit terms given to customers, payment will not be received  for 30 days.   The  cost of  the goods  is  recorded as $35,000. As a  result of  this  transaction assets will:  

A. decrease by $35,000.

B. decrease by $50,000.

C. increase by $15,000.

D. increase by $50,000.

Correct Answer:  C ......................................................................................... LOS: Reading  30‐d The accounting entries will be to increase accounts receivable (an asset) by $50,000 and decrease inventory by $35,000, a net increase in assets of $15,000. Additionally revenue of $50,000 and COGS of $35,000 are recorded.

Reference CFA® Program Curriculum, Volume 3, pp. 50-58.

 

 

 

6.  Two  firms  buy  new  photocopiers.    The  first  firm  is  a  retailer  of  office  equipment  and  the photocopier will be resold, and the second firm is a bank planning to use the photocopier in its head office.  How will the purchases be classified? 

Retailer Bank

A. investing activity investing activity

B. operating activity investing activity

C. operating activity financing activity

D. operating activity operating activity

Correct Answer:  B...........................................................................................LOS: Reading  30‐a The purchase will be part of the day-to-day activity of the retailer so it will be classed as an operating activity. For the bank the photocopier will be a long-term asset to support its operations so it will be an investing activity.

Reference: CFA® Program Curriculum, Volume 3, pp. 36‐37. 

Page 221: Cfa Ques Answer

Financial Statement Analysis: Introduction   205 

 

7.  When new accounting standards are  issued which will be  implemented  in  the  future, which of the following best describes the requirements of a company under International Financial Reporting Standards (IFRS)?  

A.  The company must disclose whether the new standards will or will not apply to its financial statements. 

B.  At present a company is not obliged to comment on new standards until they are actually implemented. 

C.  The company must provide detailed information on the likely impact of the standards on the financial statements. 

D.  The company must provide a discussion on whether there will be any impact from the new standards on the financial statements. 

 

 

 

8.  A manufacturing company receives dividends on a long‐term investment it has made in another company’s shares and pays a dividend  to  its own stockholders.   These are  likely  to be classified as which types of activity?  

  Receives dividend  Pays dividend 

A.  Investing    Investing 

B.  Investing    Financing 

C.  Financing    Investing  

D.  Financing    Financing 

Page 222: Cfa Ques Answer

206   Study Session 07:  

 

7.  When new accounting standards are  issued which will be  implemented  in  the  future, which of the following best describes the requirements of a company under International Financial Reporting Standards (IFRS)?  

A. The company must disclose whether the new standards will or will not apply to its financial statements.

B. At present a company is not obliged to comment on new standards until they are actually implemented.

C. The company must provide detailed information on the likely impact of the standards on the financial statements.

D. The company must provide a discussion on whether there will be any impact from the new standards on the financial statements.

Correct Answer:  D......................................................................................... LOS: Reading  31‐h Companies are expected to provide a discussion on the impending regulations, the conclusions can be anything from ‘does not apply’ to ‘management is still evaluating the impact’ to ‘the impact of adoption is discussed’.

Reference: CFA® Program Curriculum, Volume 3, pp. 127‐128. 

 

 

 

8.  A manufacturing company receives dividends on a long‐term investment it has made in another company’s shares and pays a dividend  to  its own stockholders.   These are  likely  to be classified as which types of activity?  

Receives dividend Pays dividend

A. Investing Investing

B. Investing Financing

C. Financing Investing

D. Financing Financing

Correct Answer:  B...........................................................................................LOS: Reading  30‐a Receiving dividends from a long-term investment, if this is not the company’s main business, will be an investing activity. Paying dividends to its own stockholders is a financing activity.

Reference: CFA® Program Curriculum, Volume 3, pp. 36‐37. 

Page 223: Cfa Ques Answer

Financial Statement Analysis: Introduction   207 

 

9.  Under U.S. GAAP minority interest appears in the: 

A.  balance sheet as an asset. 

B.  balance sheet as a liability. 

C.  income statement as a revenue. 

D.  income statement as an expense. 

 

 

 

10.  A bank lends money and receives interest from the borrowers; it also takes deposits on which it pays interest.  These activities are likely to be classified as which types of activity?  

  Receives interest   Pays interest 

A.  Financing    Operating 

B.  Financing    Financing 

C.  Investing    Financing 

D.  Operating    Operating 

Page 224: Cfa Ques Answer

208   Study Session 07:  

 

9.  Under U.S. GAAP minority interest appears in the: 

A. balance sheet as an asset.

B. balance sheet as a liability.

C. income statement as a revenue.

D. income statement as an expense.

Correct Answer:  B.......................................................................................... LOS: Reading  30‐b Minority interest refers to equity held by other investors in a subsidiary that is consolidated in the accounts; it is reported as a liability under U.S. GAAP and is a balance sheet item.

Reference: CFA® Program Curriculum, Volume 3, pp. 38‐40. 

 

 

 

10.  A bank lends money and receives interest from the borrowers; it also takes deposits on which it pays interest.  These activities are likely to be classified as which types of activity?  

Receives interest Pays interest

A. Financing Operating

B. Financing Financing

C. Investing Financing

D. Operating Operating

Correct Answer:  D..........................................................................................LOS: Reading  30‐a A bank’s business is taking deposits and making loans so these are both operating activities.

Reference: CFA® Program Curriculum, Volume 3, pp. 36‐37. 

Page 225: Cfa Ques Answer

Financial Statement Analysis: Introduction   209 

 

11.  Owners’ equity is least likely to be referred to as: 

A.  net book value. 

B.  a residual claim. 

C.  total assets less total liabilities. 

D.  contributed capital less retained earnings. 

 

 

 

12.  The International Accounting Standards Board (IASB) is: 

A.  a standard‐setting body which was set up to harmonize accounting standards internationally. 

B.  responsible for developing international financial reporting and accounting standards outside the U.S. 

C.  a regulatory body with responsibility for ensuring that companies adhere to International Financial Reporting Standards. 

D.  responsible for ensuring that investors receive financial information from companies that best assists them in making investment decisions.  

Page 226: Cfa Ques Answer

210   Study Session 07:  

 

11.  Owners’ equity is least likely to be referred to as: 

A. net book value.

B. a residual claim.

C. total assets less total liabilities.

D. contributed capital less retained earnings.

Correct Answer:  D..........................................................................................LOS: Reading  30‐c Owners’ equity is total assets less total liabilities, which is net book value. This is equivalent to contributed capital plus retained earnings. Owners’ equity is also referred to as a residual claim since it is what remains after all the liabilities are settled.

Reference: CFA® Program Curriculum, Volume 3, pp. 40‐43. 

 

 

 

12.  The International Accounting Standards Board (IASB) is: 

A. a standard-setting body which was set up to harmonize accounting standards internationally.

B. responsible for developing international financial reporting and accounting standards outside the U.S.

C. a regulatory body with responsibility for ensuring that companies adhere to International Financial Reporting Standards.

D. responsible for ensuring that investors receive financial information from companies that best assists them in making investment decisions.

Correct Answer:  A ......................................................................................... LOS: Reading  31‐b The IASB is a standard-setting organization responsible for developing international financial reporting and accounting standards in major countries including the U.S. One of its objectives is to achieve convergence in accounting standards, so A is the best answer.

Reference: CFA® Program Curriculum, Volume 3, p. 101. 

Page 227: Cfa Ques Answer

Financial Statement Analysis: Introduction   211 

 

13.  Information about directors’ compensation is most likely to be included in a firm’s 

A.  auditor’s report. 

B.  proxy statement. 

C.  income statement. 

D.  Management’s Discussion and Analysis. 

 

 

 

14.  Which of the following statements is most accurate in describing the measurement of assets and liabilities?   

A.  Whenever possible fair values of assets should be used. 

B.  Assets should generally be stated at historical cost and liabilities at present value. 

C.  A company is permitted to use a number of different methods to calculate asset and liability values.  

D.  A company must select whether to use historical or current cost methods and apply the same method to all assets and liabilities.  

Page 228: Cfa Ques Answer

212   Study Session 07:  

 

13.  Information about directors’ compensation is most likely to be included in a firm’s 

A. auditor’s report.

B. proxy statement.

C. income statement.

D. Management’s Discussion and Analysis.

Correct Answer:  B...........................................................................................LOS: Reading  29‐e Proxy statements relate to shareholder meetings. They provide information on board members and management, executive compensation, stock options and major stockholders.

Reference: CFA® Program Curriculum, Volume 3, p. 25. 

 

 

 

14.  Which of the following statements is most accurate in describing the measurement of assets and liabilities?   

A. Whenever possible fair values of assets should be used.

B. Assets should generally be stated at historical cost and liabilities at present value.

C. A company is permitted to use a number of different methods to calculate asset and liability values.

D. A company must select whether to use historical or current cost methods and apply the same method to all assets and liabilities.

Correct Answer:  C ......................................................................................... LOS: Reading  31‐d Companies can use different combinations of historical cost, current cost, realizable value, present vale and fair value, so C is the best answer.

Reference: CFA® Program Curriculum, Volume 3, p. 113. 

Page 229: Cfa Ques Answer

Financial Statement Analysis: Introduction   213 

 

15.  Which of the following statements is least accurate regarding U.S. GAAP?  

A.  Relevance and reliability are considered primary qualities in the U.S. GAAP framework. 

B.  U.S. GAAP includes standards established by a number of different organizations. 

C.  U.S. GAAP has a hierarchical structure with standards issued by the FASB holding the highest position.  

D.  The target is to replace U.S. GAAP by International Financial Reporting Standards by 2012.  

 

 

 

16.  A  manufacturing  company  records  a  depreciation  expense  associated  with  the  purchase  of machinery  and  the  company  pays  income  tax. These  are  likely  to  be  classified  as which  types  of activity?  

  Depreciation  Income tax 

A.  Financing    Operating 

B.  Operating    Financing 

C.  Investing    Financing 

D.  Operating    Operating 

Page 230: Cfa Ques Answer

214   Study Session 07:  

 

15.  Which of the following statements is least accurate regarding U.S. GAAP?  

A. Relevance and reliability are considered primary qualities in the U.S. GAAP framework.

B. U.S. GAAP includes standards established by a number of different organizations.

C. U.S. GAAP has a hierarchical structure with standards issued by the FASB holding the highest position.

D. The target is to replace U.S. GAAP by International Financial Reporting Standards by 2012.

Correct Answer:  D......................................................................................... LOS: Reading  31‐b The objective is to harmonize U.S. GAAP and IFRS, not to replace one with the other.

Reference: CFA® Program Curriculum, Volume 3, pp. 118‐119. 

 

 

 

16.  A  manufacturing  company  records  a  depreciation  expense  associated  with  the  purchase  of machinery  and  the  company  pays  income  tax. These  are  likely  to  be  classified  as which  types  of activity?  

Depreciation Income tax

A. Financing Operating

B. Operating Financing

C. Investing Financing

D. Operating Operating

Correct Answer:  D..........................................................................................LOS: Reading  30‐a Depreciation is a cost of using machinery which is likely to be used to manufacture goods for sale, therefore it is an operating item. Paying income tax is an operating item, since it results from the operations of the company.

Reference: CFA® Program Curriculum, Volume 3, pp. 36‐37. 

Page 231: Cfa Ques Answer

Financial Statement Analysis: Introduction   215 

 

17.  The Statement of Changes in Owners‘ Equity would be least likely to include information on: 

A.  net assets. 

B.  net income. 

C.  issue of new shares. 

D.  dividends distributed. 

 

 

 

18.  Accruals appear in financial statements when: 

A.  a company uses a cash–based accounting method. 

B.  there is uncertainty whether revenue is going to be realized or an expense paid. 

C.  there is an adjustment to values of assets and liabilities on the balance sheet to reflect fair values. 

D.  there is a difference between the timing of cash movements and the recognition of a revenue or expense. 

Page 232: Cfa Ques Answer

216   Study Session 07:  

 

17.  The Statement of Changes in Owners‘ Equity would be least likely to include information on: 

A. net assets.

B. net income.

C. issue of new shares.

D. dividends distributed.

Correct Answer:  A ......................................................................................... LOS: Reading  29‐b The Statement of Changes in Owners‘ Equity contains information on the composition and changes in owners’ equity over the reporting period. This would include information on new shares issued and retained income. It would not explicitly include information on net assets; this would be in the balance sheet.

Reference: CFA® Program Curriculum, Volume 3, p. 19. 

 

 

 

18.  Accruals appear in financial statements when: 

A. a company uses a cash–based accounting method.

B. there is uncertainty whether revenue is going to be realized or an expense paid.

C. there is an adjustment to values of assets and liabilities on the balance sheet to reflect fair values.

D. there is a difference between the timing of cash movements and the recognition of a revenue or expense.

Correct Answer:  D..........................................................................................LOS: Reading  30‐e Accrual accounting requires that revenue is recognized when it is earned and expenses when they are incurred. This may be before or after the cash is received or paid which gives rise to accrual entries.

Reference: CFA® Program Curriculum, Volume 3, pp. 65‐67. 

Page 233: Cfa Ques Answer

Financial Statement Analysis: Introduction   217 

 

19.  Which of the following is least likely to be a constraint in preparing useful financial statements? 

A.  The time taken to ensure information is accurate. 

B.  The cost of providing information that is accurate and useful. 

C.  Many nonquantifiable items are not included in the financial statements.   

D.  The requirement to include information regardless of whether it is relevant or not. 

 

 

 

20.  Historically  the  approaches  of  the  International  Financial Reporting  Standards  (IFRS)  and  the Financial Accounting Standards Board (FASB) have been best described as:  

  IFRS    FASB 

A.  Rules‐based   Rules‐based  

B.  Rules‐based   Principles‐based 

C.  Principles‐based  Rules‐based  

D.  Principles‐based  Principles‐based 

Page 234: Cfa Ques Answer

218   Study Session 07:  

 

19.  Which of the following is least likely to be a constraint in preparing useful financial statements? 

A. The time taken to ensure information is accurate.

B. The cost of providing information that is accurate and useful.

C. Many nonquantifiable items are not included in the financial statements.

D. The requirement to include information regardless of whether it is relevant or not.

Correct Answer:  D......................................................................................... LOS: Reading  31‐d Information that is not relevant, in terms of being out of date or would not influence decision makers, is not useful and does not automatically need to be included in financial statements. D is the best answer.

Reference: CFA® Program Curriculum, Volume 3, pp. 110‐111. 

 

 

 

20.  Historically  the  approaches  of  the  International  Financial Reporting  Standards  (IFRS)  and  the Financial Accounting Standards Board (FASB) have been best described as:  

IFRS FASB

A. Rules-based Rules-based

B. Rules-based Principles-based

C. Principles-based Rules-based

D. Principles-based Principles-based

Correct Answer:  C ..........................................................................................LOS: Reading  31‐g IFRS adopt a principles–based approach which gives more flexibility on the preparation of financial reports. The FASB has been rules-based which means there are specific rules for each element or transaction. This difference in approach has created a barrier to establishing a single coherent framework for financial reporting.

Reference: CFA® Program Curriculum, Volume 3, pp. 122‐123. 

Page 235: Cfa Ques Answer

Financial Statement Analysis: Introduction   219 

 

21.  The following information is provided on a company: 

Retained earnings at beginning of year  $220 million 

Estimated income for year       $35 million 

Estimated revenue for year   $345 million 

Repurchase of stock over year     $60 million 

Estimated dividends paid over year     $20 million 

 

The estimated retained earnings at end of year are: 

A.  $155 million. 

B.  $215 million. 

C.  $255 million. 

D.  $525 million. 

 

 

 

22.  The following information is provided on a company 

Retained earnings at end of previous year     $28 million 

Net income over year      $6 million 

Contributed capital at year‐end    $35 million 

Dividends paid over year       $2 million 

Expenses over year       $3 million 

 

Owners’ equity at year end will be: 

A.  $32 million. 

B.  $63 million. 

C.  $64 million. 

D.  $67 million.  

Page 236: Cfa Ques Answer

220   Study Session 07:  

 

21.  The following information is provided on a company: 

Retained earnings at beginning of year $220 million

Estimated income for year $35 million

Estimated revenue for year $345 million

Repurchase of stock over year $60 million

Estimated dividends paid over year $20 million

The estimated retained earnings at end of year are:

A. $155 million.

B. $215 million.

C. $255 million.

D. $525 million.

Correct Answer:  B...........................................................................................LOS: Reading  30‐c Estimated retained earnings at end of year

= retained earnings at beginning of year + net income – dividends paid

= $220 million + $35 million - $20 million = £215 million

The repurchase of stock affects paid-up equity rather than retained earnings.

Reference: CFA® Program Curriculum, Volume 3, pp. 40‐46. 

Page 237: Cfa Ques Answer

Financial Statement Analysis: Introduction   221 

 

22.  The following information is provided on a company 

Retained earnings at end of previous year   $28 million 

Net income over year    $6 million 

Contributed capital at year‐end  $35 million 

Dividends paid over year     $2 million 

Expenses over year     $3 million 

Owners’ equity at year end will be:

A. $32 million.

B. $63 million.

C. $64 million.

D. $67 million.

Correct Answer:  D ......................................................................................... LOS: Reading  30‐c Owners’ equity = contributed capital + retained earnings

= $35 million + $28 million + ($6 million – $2 million) = $67 million

Reference: CFA® Program Curriculum, Volume 3, pp. 40‐46. 

 

 

Page 238: Cfa Ques Answer

222   Study Session 08:  

 

Study Session 08: Financial Statement Analysis: The Income Statement, Balance Sheet, and Cash Flow Statement

 

Each reading in this study session focuses on one of the three major financial statements: the balance sheet, the income statement, and the statement of cash flows. For each financial statement, the chapter details  its purpose,  construction, pertinent  ratios,  and  common‐size  analysis. Understanding  these concepts  allows  a  financial  analyst  to  evaluate  trends  in  performance  over  several measurement periods and to compare the performance of different companies over the same period(s). Additional analyst tools such as the earnings per share calculation are also described. 

 

Reading 32: Understanding the Income Statement 

Reading 33: Understanding the Balance Sheet 

Reading 34: Understanding the Cash Flow Statement 

 

 

 

 

Page 239: Cfa Ques Answer

Financial Statement Analysis: Income Statement, Balance Sheet, & Cash Flow   223 

 

1.  A manufacturing company acquires a smaller competitor. The acquirer combines the inventory of the  two  companies which will  be  sold  to  generate  future  sales.  This  is most  likely  to  have  the following effect on the acquiring company’s cash flows:  

  Operating cash flow   Financing cash flow 

A.  No effect       Understated. 

B.  Overstated      No effect. 

C.  Understated    No effect.. 

D.  Understated    Overstated. 

 

 

 

2.  If  a  firm’s  quick  ratio  is  above  the  industry  average  but  the  cash  ratio  is  below  the  industry average this might be explained by: 

A.  the firm prefers to hold marketable securities rather than cash. 

B.  the firm has higher inventory levels compared with the rest of the industry. 

C.  the firm has a large number of ‘other short‐term assets’ in the balance sheet. 

D.  the firm has extended to their customers more favorable credit terms than their competitors. 

 

Page 240: Cfa Ques Answer

224   Study Session 08:  

 

1.  A manufacturing company acquires a smaller competitor. The acquirer combines the inventory of the  two  companies which will  be  sold  to  generate  future  sales.  This  is most  likely  to  have  the following effect on the acquiring company’s cash flows:  

Operating cash flow Financing cash flow

A. No effect Understated.

B. Overstated No effect.

C. Understated No effect..

D. Understated Overstated.

Correct Answer:  B...........................................................................................LOS: Reading  34‐e The acquirer will not be recognizing the cost of the inventory as an operating cash outflow; it will be recorded as an investment activity. Therefore cash flow from operations will be overstated. There is no direct impact on cash flow from financing activities.

Reference CFA® Program Curriculum, Volume 3, pp. 279-283.

 

 

 

2.  If  a  firm’s  quick  ratio  is  above  the  industry  average  but  the  cash  ratio  is  below  the  industry average this might be explained by: 

A. the firm prefers to hold marketable securities rather than cash.

B. the firm has higher inventory levels compared with the rest of the industry.

C. the firm has a large number of ‘other short-term assets’ in the balance sheet.

D. the firm has extended to their customers more favorable credit terms than their competitors.

Correct Answer:  D.......................................................................................... LOS: Reading  33‐i The quick ratio includes cash, marketable securities and receivables in the numerator whereas the cash ratio only includes cash and marketable securities. The relatively high quick ratio therefore indicates high levels of receivables, or customers owing money.

Reference: CFA® Program Curriculum, Volume 3, pp. 239‐240. 

Page 241: Cfa Ques Answer

Financial Statement Analysis: Income Statement, Balance Sheet, & Cash Flow   225 

 

3.  Which of the following ratios are correctly described as liquidity and solvency ratios? 

  Liquidity       Solvency 

A.  cash      current 

B.  current       debt to equity 

C.  debt to equity    financial leverage 

D.  financial leverage    quick ratio 

 

 

 

4.  Under  U.S.  GAAP  which  of  the  following  is  the  least  accurate  statement  concerning extraordinary items? An extraordinary item: 

A.  is reported net of tax. 

B.  is not part of operating income. 

C.  is included in comprehensive income. 

D.  must be both unusual and infrequent. 

Page 242: Cfa Ques Answer

226   Study Session 08:  

 

3.  Which of the following ratios are correctly described as liquidity and solvency ratios? 

Liquidity Solvency

A. cash current

B. current debt to equity

C. debt to equity financial leverage

D. financial leverage quick ratio

Correct Answer:  B........................................................................................... LOS: Reading  33‐i Liquidity ratios measure a company’s ability to meet its short-term obligations; solvency ratios measure the ability to meet long-term and other obligations.

Reference: CFA® Program Curriculum, Volume 3, pp. 239‐240. 

 

 

 

4.  Under  U.S.  GAAP  which  of  the  following  is  the  least  accurate  statement  concerning extraordinary items? An extraordinary item: 

A. is reported net of tax.

B. is not part of operating income.

C. is included in comprehensive income.

D. must be both unusual and infrequent.

Correct Answer:  C ..........................................................................................LOS: Reading  32‐g An extraordinary item will be reported separately on the income statement, below discontinued operations, and net of tax. Therefore it will impact on net income and is not reported separately in comprehensive income.

Reference: CFA® Program Curriculum, Volume 3, p. 168. 

Page 243: Cfa Ques Answer

Financial Statement Analysis: Income Statement, Balance Sheet, & Cash Flow   227 

 

5.  If a company decides to change from a policy of leasing plant and equipment through operating leases to purchasing its own plant and equipment this will:  

A.  increase investing cash flows. 

B.  increase operating cash flows. 

C.  decrease operating cash flows. 

D.  have no impact on operating cash flows. 

 

 

 

6.  Which  inventory accounting method usually gives a valuation of  inventory that  is closest to  its economic value?  

A.  FIFO. 

B.  LIFO. 

C.  Lower of cost or market. 

D.  Weighted average cost method. 

Page 244: Cfa Ques Answer

228   Study Session 08:  

 

5.  If a company decides to change from a policy of leasing plant and equipment through operating leases to purchasing its own plant and equipment this will:  

A. increase investing cash flows.

B. increase operating cash flows.

C. decrease operating cash flows.

D. have no impact on operating cash flows.

Correct Answer:  B...........................................................................................LOS: Reading  34‐a Lease rentals are classified as an operating cash flow and the purchase of equipment is classified as an investing cash flow. The decision to switch from leasing to purchasing plant and equipment will increase operating cash flows but decrease investing cash flows.

Reference: CFA® Program Curriculum, Volume 3, pp. 265‐273. 

 

 

 

6.  Which  inventory accounting method usually gives a valuation of  inventory that  is closest to  its economic value?  

A. FIFO.

B. LIFO.

C. Lower of cost or market.

D. Weighted average cost method.

Correct Answer:  A ......................................................................................... LOS: Reading  32‐d Since FIFO leaves the most recently purchased goods in inventory the valuation will usually be the closest to current values.

Reference: CFA® Program Curriculum, Volume 3, pp. 157‐161. 

Page 245: Cfa Ques Answer

Financial Statement Analysis: Income Statement, Balance Sheet, & Cash Flow   229 

 

7.  Using  common‐size  statement  analysis  of  the  balance  sheet  of  a  company will  help  identify changes in: 

A.  total assets. 

B.  profitability. 

C.  financial leverage. 

D.  efficiency of use of assets  

 

 

 

8.  When a company recognizes a warranty expense in the same accounting period as the sale of a good this is an application of: 

A.  accrual accounting. 

B.  the matching principle.  

C.  contingency accounting. 

D.  the cost recovery method. 

Page 246: Cfa Ques Answer

230   Study Session 08:  

 

7.  Using  common‐size  statement  analysis  of  the  balance  sheet  of  a  company will  help  identify changes in: 

A. total assets.

B. profitability.

C. financial leverage.

D. efficiency of use of assets

Correct Answer:  C .......................................................................................... LOS: Reading  33‐i Common-size statement analysis of the balance sheet will help identify changes in ratios which use balance sheet items; this includes financial leverage which is assets/equity. Total assets will also be read off the balance sheet, but in common-size statements assets will always be 100%.

Reference: CFA® Program Curriculum, Volume 3, pp. 239‐240. 

 

 

 

8.  When a company recognizes a warranty expense in the same accounting period as the sale of a good this is an application of: 

A. accrual accounting.

B. the matching principle.

C. contingency accounting.

D. the cost recovery method.

Correct Answer:  B...........................................................................................LOS: Reading  32‐c Under matching principles expenses should be recognized in the same period as the associated revenues, so the company should recognize estimated warranty expenses in the same period as the sale.

Reference: CFA® Program Curriculum, Volume 3, p. 162. 

Page 247: Cfa Ques Answer

Financial Statement Analysis: Income Statement, Balance Sheet, & Cash Flow   231 

 

9.  The sale of a property which cost $3,000,000 is agreed.  The purchaser makes a down payment of $1,500,000 and agrees to pay further installments of $600,000 per year at the end of each of the next five years. The profit that will be recognized as a result of the down payment, using the installment method and cost recovery method is closest to:  

  Installment method   Cost recovery method  

A.  $0        $0 

B.  $750,000      $0 

C.  $750,000      $750,000 

D.  $1,000,000      $750,000 

 

 

 

10.  The Comprehensive Income Statement contains:  

A.  details of income from subsidiaries. 

B.  information on changes in stockholders’ equity. 

C.  details on income from joint ventures and associates.   

D.  a detailed breakdown of cost of goods sold and other expenses. 

Page 248: Cfa Ques Answer

232   Study Session 08:  

 

9.  The sale of a property which cost $3,000,000 is agreed.  The purchaser makes a down payment of $1,500,000 and agrees to pay further installments of $600,000 per year at the end of each of the next five years. The profit that will be recognized as a result of the down payment, using the installment method and cost recovery method is closest to:  

Installment method Cost recovery method

A. $0 $0

B. $750,000 $0

C. $750,000 $750,000

D. $1,000,000 $750,000

Correct Answer:  B.......................................................................................... LOS: Reading  32‐b Installment method: the profit to sales ratio is ($4,500,000 - $3,000,000)/$3,000,000 equals 50%. The profit recognized from the down payment is 50% x $1,500,000, equals $750,000.

Cost recovery method: no profit is recognized until the costs have been recovered.

Reference: CFA® Program Curriculum, Volume 3, pp. 152‐154. 

 

 

 

10.  The Comprehensive Income Statement contains:  

A. details of income from subsidiaries.

B. information on changes in stockholders’ equity.

C. details on income from joint ventures and associates.

D. a detailed breakdown of cost of goods sold and other expenses.

Correct Answer:  B........................................................................................... LOS: Reading  32‐l Comprehensive income is the change in stockholders’ equity as a result of net income and other revenue and expense items that have been excluded from the income statement. These items include foreign currency translation adjustments, pension liability adjustments, unrealized gains and losses on derivatives contracts and investments.

Reference: CFA® Program Curriculum, Volume 3, pp. 185‐186. 

Page 249: Cfa Ques Answer

Financial Statement Analysis: Income Statement, Balance Sheet, & Cash Flow   233 

 

11.  Jones Construction takes on a new project which it anticipates will take two years to complete. It agrees a total contract price of $4,000,000 and its expected operating costs are $3,000,000. In the first year it incurs costs of $1,800,000 and receives payments of $2,000,000. If it is using the percentage‐of‐completion method to recognize revenue then the operating income recorded in the first year will be: 

A.  $0. 

B.  $200,000. 

C.  $600,000. 

D.  $1,800,000. 

 

 

 

12.  I.W.S. Inc.’s cash flow from investing was (ref. Question 41): 

A.  ($19 million). 

B.  ($12 million).  

C.  ($9 million). 

D.  ($6 million). 

Page 250: Cfa Ques Answer

234   Study Session 08:  

 

11.  Jones Construction takes on a new project which it anticipates will take two years to complete. It agrees a total contract price of $4,000,000 and its expected operating costs are $3,000,000. In the first year it incurs costs of $1,800,000 and receives payments of $2,000,000. If it is using the percentage‐of‐completion method to recognize revenue then the operating income recorded in the first year will be: 

A. $0.

B. $200,000.

C. $600,000.

D. $1,800,000.

Correct Answer:  C ......................................................................................... LOS: Reading  32‐b The revenue in the first year is: $1,800,000 x $4,000,000 = $2,400,000

$3,000,000

Costs = $1,800,000

Operating income = $2,400,000 - $1,800,000 = $600,000

Reference: CFA® Program Curriculum, Volume 3, pp. 148‐152. 

 

 

 

12.  I.W.S. Inc.’s cash flow from investing was (ref. Question 41): 

A. ($19 million).

B. ($12 million).

C. ($9 million).

D. ($6 million).

Correct Answer:  C ..........................................................................................LOS: Reading  34‐a Cash flow from investing activities

Cash flow from investing activitiesPurchase of land  (15)Sale of equipment  6  (9)

Reference: CFA® Program Curriculum, Volume 3, pp. 271‐273. 

Page 251: Cfa Ques Answer

Financial Statement Analysis: Income Statement, Balance Sheet, & Cash Flow   235 

 

13.  I.W.S. Inc.’s cash flow from financing was (ref. Question 41): 

A.  ($35 million). 

B.  ($23 million). 

C.  ($20 million). 

D.  ($12 million). 

 

 

 

14.  A U.S. consultancy company discovers that a transaction that took place last year was incorrectly recorded and in fact a client paid an additional $10,000 of fees over that recorded as a bonus based on the benefits he received from the consultancy provided in that period. The company should:  

A.  adjust the reported income in the previous year’s accounts. 

B.  increase the income received in the current year’s accounts. 

C.  adjust the retained earnings in the previous year’s balance sheet. 

D.  recognize the $10,000 as an extraordinary profit in the current year. 

Page 252: Cfa Ques Answer

236   Study Session 08:  

 

13.  I.W.S. Inc.’s cash flow from financing was (ref. Question 41): 

A. ($35 million).

B. ($23 million).

C. ($20 million).

D. ($12 million).

Correct Answer:  C ..........................................................................................LOS: Reading  34‐a Cash flow from financing activities

Cash flow from financing Retirement of common stock  (12)Dividend payment  (8)  (20)

Reference: CFA® Program Curriculum, Volume 3, pp. 273‐275. 

 

 

 

14.  A U.S. consultancy company discovers that a transaction that took place last year was incorrectly recorded and in fact a client paid an additional $10,000 of fees over that recorded as a bonus based on the benefits he received from the consultancy provided in that period. The company should:  

A. adjust the reported income in the previous year’s accounts.

B. increase the income received in the current year’s accounts.

C. adjust the retained earnings in the previous year’s balance sheet.

D. recognize the $10,000 as an extraordinary profit in the current year.

Correct Answer:  B...........................................................................................LOS: Reading  32‐g Adjustments to prior years’ accounts through adjustment to the retained earnings is only to be used for accounting errors. The adjustment should be reported in the current year’s income statement. Although it is a non recurring item it is not an extraordinary item.

Reference: CFA® Program Curriculum, Volume 3, pp. 169‐170. 

Page 253: Cfa Ques Answer

Financial Statement Analysis: Income Statement, Balance Sheet, & Cash Flow   237 

 

15.  Bishop  Steel Manufacturing  reported  little  change  in  net  income whereas  the  operating  cash flows rose sharply. This might be explained by the company: 

A.  extending the payment period for customers. 

B.  raising cash through the issue of long term bonds. 

C.  selling land that was not being used for a substantial sum of money. 

D.  using inventory to meet their customers’ orders and minimizing raw material purchases.  

 

 

 

16.  The costs of long‐lived assets are usually allocated: 

A.  over 5 years. 

B.  at the time of disposal. 

C.  over a period that is chosen by the firm. 

D.  when the impairment of the assets takes place. 

Page 254: Cfa Ques Answer

238   Study Session 08:  

 

15.  Bishop  Steel Manufacturing  reported  little  change  in  net  income whereas  the  operating  cash flows rose sharply. This might be explained by the company: 

A. extending the payment period for customers.

B. raising cash through the issue of long term bonds.

C. selling land that was not being used for a substantial sum of money.

D. using inventory to meet their customers’ orders and minimizing raw material purchases.

Correct Answer:  D..........................................................................................LOS: Reading  34‐e A is not correct since this would increase receivables and reduce operating cash flow.

B is not correct since the issue of bonds would be classified as a financing cash flow.

C is not correct since the sale of land would be classified as an investment cash flow.

D is correct since this would raise cash from the sale of inventory.

Reference: CFA® Program Curriculum, Volume 3, pp. 279‐281. 

 

 

 

16.  The costs of long‐lived assets are usually allocated: 

A. over 5 years.

B. at the time of disposal.

C. over a period that is chosen by the firm.

D. when the impairment of the assets takes place.

Correct Answer:  C ......................................................................................... LOS: Reading  32‐d  Usually the management has discretion over the period, or useful life, that assets are depreciated.

Reference: CFA® Program Curriculum, Volume 3, pp. 162‐167. 

Page 255: Cfa Ques Answer

Financial Statement Analysis: Income Statement, Balance Sheet, & Cash Flow   239 

 

17.  The  indirect method  of  reporting  cash  flows  calculates  operating  cash  flow  by which  of  the following methods? 

A.  Start with cash collections for the period and deduct cash outflows incurred in collecting this cash.  

B.  Start with net income for the period and adjust for all non cash expenses/revenues and adjust for non‐operating items included in net income.  

C.  Start with cash collections for the period and deduct cash outflows incurred in collecting this cash, and then adjust for changes in the balance of operating asset and liability accounts.  

D.  Start with net income for the period, adjust for all non cash expenses/revenues, adjust for non‐operating items included in net income and then adjust for changes in the balance of operating asset and liability accounts. 

 

 

 

18.  Which of the following is least likely to be included in the Statement of Stockholders’ Equity? 

A.  Capital leases. 

B.  Retained earnings. 

C.  Cumulative foreign exchange adjustments. 

D.  Additional paid in capital above the par value of common stock. 

Page 256: Cfa Ques Answer

240   Study Session 08:  

 

17.  The  indirect method  of  reporting  cash  flows  calculates  operating  cash  flow  by which  of  the following methods? 

A. Start with cash collections for the period and deduct cash outflows incurred in collecting this cash.

B. Start with net income for the period and adjust for all non cash expenses/revenues and adjust for non-operating items included in net income.

C. Start with cash collections for the period and deduct cash outflows incurred in collecting this cash, and then adjust for changes in the balance of operating asset and liability accounts.

D. Start with net income for the period, adjust for all non cash expenses/revenues, adjust for non-operating items included in net income and then adjust for changes in the balance of operating asset and liability accounts.

Correct Answer:  D......................................................................................... LOS: Reading  34‐d The indirect method starts at the net income figure and makes adjustments whereas the direct method works through the cash items staring with cash collections.

Reference CFA® Program Curriculum, Volume 3, pp. 265-271.

 

 

 

18.  Which of the following is least likely to be included in the Statement of Stockholders’ Equity? 

A. Capital leases.

B. Retained earnings.

C. Cumulative foreign exchange adjustments.

D. Additional paid in capital above the par value of common stock.

Correct Answer:  A ......................................................................................... LOS: Reading  33‐h Capital leases are reported in the assets and liabilities section of the balance sheet. All the other items appear as part of shareholders’ equity.

Reference: CFA® Program Curriculum, Volume 3, pp. 226‐230. 

Page 257: Cfa Ques Answer

Financial Statement Analysis: Income Statement, Balance Sheet, & Cash Flow   241 

 

19.  The cash flow from investments in 2007 for North Company is (ref. question 28):   

A.  ($85,000). 

B.  ($70,000). 

C.  $30,000. 

D.  $130,000. 

 

 

 

20.  The cash flow from financing in 2007 for North Company is (ref. question 28): 

A.  ($170,000). 

B.  ($115,000). 

C.  ($100,000). 

D.  ($15,000). 

Page 258: Cfa Ques Answer

242   Study Session 08:  

 

19.  The cash flow from investments in 2007 for North Company is (ref. question 28):   

A. ($85,000).

B. ($70,000).

C. $30,000.

D. $130,000.

Correct Answer:  C ..........................................................................................LOS: Reading  34‐a 

Investing cash flows 

Sale of old machine  30

Net from investment  30

 

 

Reference: CFA® Program Curriculum, Volume 3, pp. 271‐273. 

 

 

 

20.  The cash flow from financing in 2007 for North Company is (ref. question 28): 

A. ($170,000).

B. ($115,000).

C. ($100,000).

D. ($15,000).

Correct Answer:  B...........................................................................................LOS: Reading  34‐a 

Financing cash flows:   

Bank notes  (100)

Dividends paid  (15)

Net from financing  (115)

 

Reference: CFA® Program Curriculum, Volume 3, pp. 273‐275. 

Page 259: Cfa Ques Answer

Financial Statement Analysis: Income Statement, Balance Sheet, & Cash Flow   243 

 

21.  W.S. Inc uses U.S. GAAP and supplied the following financial data: 

 

$ million 

Cash payment for salaries  23  

Purchase of land  15  

Cash payment for interest   3 

Retirement of common stock  12 

Cash collection from customers  115 

Cash payment to suppliers  43 

Depreciation expense  10 

Dividend payment  8 

Sale of equipment  6 

 

Cash flow from operating activities was: 

A.  $36 million. 

B.  $39 million. 

C.  $46 million. 

D.  $49 million. 

Page 260: Cfa Ques Answer

244   Study Session 08:  

 

21.  W.S. Inc uses U.S. GAAP and supplied the following financial data: 

$ million

Cash payment for salaries 23

Purchase of land 15

Cash payment for interest 3

Retirement of common stock 12

Cash collection from customers 115

Cash payment to suppliers 43

Depreciation expense 10

Dividend payment 8

Sale of equipment 6

Cash flow from operating activities was:

A. $36 million.

B. $39 million.

C. $46 million.

D. $49 million.

Correct Answer:  C ..........................................................................................LOS: Reading  34‐a 

  $ millionCash flow from operating activitiesCash collection from customers  115Cash payment to suppliers  (43)Cash payment for salaries  (23)Cash payment for interest  (3)  46

Reference: CFA® Program Curriculum, Volume 3, pp. 265‐271. 

Page 261: Cfa Ques Answer

Financial Statement Analysis: Income Statement, Balance Sheet, & Cash Flow   245 

 

22.  North Company uses U.S. GAAP and provides the following financial statements: 

2007 Income statement (in $’000)Sales  5,400COGS  (4,600)Depreciation  (200)Interest expense  (55)Gain on sale of old machinery 30Income before taxes  575Income tax expense  (175)Net income after taxes 400

 

Balance sheet (in $’000)  end  beg Assets: Cash  895 100Accounts receivable 260 200Inventory  500 800Property,  plant  &  300 500Total assets  1,955 1,600  Liabilities: Accounts payable 370 350Bank notes  0 100Deferred taxes 90 40Common stock 1,000 1,000 Retained Earnings 495 110Total liabilities 1,955 1,600 

Total dividends of $15,000 were paid. 

Old machinery was sold; the machinery had already been fully depreciated. 

The cash flow from operations in 2007 is:   

A.  $580,000. 

B.  $830,000. 

C.  $880,000. 

D.  $910,000. 

Page 262: Cfa Ques Answer

246   Study Session 08:  

 

22.  North Company uses U.S. GAAP and provides the following financial statements: 

2007 Income statement (in $’000)    Balance sheet (in $’000) 

Sales   5,400      end 2007 beg 2007 COGS  (4,600)    Assets:   Depreciation   (200)    Cash  895 100 Interest expense  (55)    Accounts receivable  260 200 Gain on sale of old  30    Inventory  500 800 Income before taxes  575    Property, plant &  300 500 Income tax expense  (175)    Total assets  1,955 1,600 Net income after taxes  400             Liabilities:         Accounts payable  370 350       Bank notes  0 100       Deferred taxes  90 40       Common stock  1,000 1,000       Retained Earnings  495 110       Total liabilities  1,955 1,600 Total dividends of $15,000 were paid. 

Old machinery was sold; the machinery had already been fully depreciated. 

The cash flow from operations in 2007 is:   

A. $580,000.

B. $830,000.

C. $880,000.

D. $910,000.

Correct Answer:  C ..........................................................................................LOS: Reading  34‐a 

Page 263: Cfa Ques Answer

Financial Statement Analysis: Income Statement, Balance Sheet, & Cash Flow   247 

 

Cash flow from operations:

Net income 400

Adjust for

Depreciation 200

Gain on sale of machinery (30)

Deferred taxes 50

Change in accounts receivable (60)

Decrease in inventory 300

Change in accounts payable 20

Net from operations 880

Reference: CFA® Program Curriculum, Volume 3, pp. 265‐271. 

 

Page 264: Cfa Ques Answer

248   Study Session 09:  

 

Study Session 09: Financial Statement Analysis: Inventories, Long-Term Assets, Deferred Taxes, and On- and Off-Balance Sheet Debt

The  readings  in  this  study  session  examine  specific  categories  of  assets  and  liabilities  that  are particularly susceptible to the impact of alternative accounting policies and estimates. Analysts must understand the effects of alternative policies on financial statements and ratios, and be able to execute appropriate adjustments to enhance comparability between companies. In addition, analysts must be alert to differences between a companyʹs reported financial statements and economic reality. The description and measurement of inventories require careful attention because the investment in inventories is frequently the largest current asset for merchandizing and manufacturing companies. For these companies, the measurement of inventory cost (i.e., cost of goods sold) is a critical factor in determining gross profit and other measures of company profitability. Long‐term operating assets are often  the  largest  category of  assets on  a  companyʹs balance  sheet. The  analyst needs  to  scrutinize managementʹs  choices with  respect  to  recognizing  expenses  associated with  the  operating  assets because of the potentially large impact such choices can have on reported earnings. A  companyʹs  accounting  policies  (such  as  depreciation  choices)  can  cause  differences  in  taxes reported in financial statements and taxes reported on tax returns. The reading “Analysis of Income Taxes” discusses several issues that arise relating to deferred taxes. Both  on‐  and  off‐balance‐sheet  debt  affect  a  companyʹs  liquidity  and  solvency,  and  have consequences  for  its  long‐term growth and viability. The notes of  the  financial statements must be carefully  reviewed  to  ensure  that  all  potential  liabilities  (e.g.,  leasing  arrangements  and  other contractual  commitments)  are  appropriately  evaluated  for  their  conformity  to  economic  reality. Adjustments to the financial statements may be required to achieve comparability when evaluating several companies, and may also be required to improve credit and investment decision‐making. 

Reading 35: Analysis of Inventories 

Reading 36: Analysis of Long‐Lived Assets: 

  Part I—The Capitalization Decision 

Reading 37: Analysis of Long‐Lived Assets: 

  Part II—Analysis of Depreciation and Impairment 

Reading 38: Analysis of Income Taxes 

Reading 39: Analysis of Financing Liabilities 

Reading 40: Leases and Off‐Balance‐Sheet Debt 

Page 265: Cfa Ques Answer

Financial Statement Inventories, Assets, Taxes & Debt   249 

 

1.  When a company reports a  large LIFO reserve, has  it  (1) been using LIFO or FIFO  to calculate inventory balances and has it (2) overstated or understated the value of inventory compared with the current market value?  

  Inventory balances     Overstated/understated 

A.  LIFO       overstated  

B.  FIFO       overstated  

C.  LIFO       understated  

D.  FIFO       understated 

 

 

 

2.  The average age of Manufacturers Corporation’s machinery is lower than other companies in the same industry. Which of the following is the least likely to explain this? 

A.  The company has just made major write downs of impaired assets. 

B.  The company uses a shorter depreciation life for machinery than its competitors. 

C.  The company has just finished a major capital expenditure program to replace machinery.  

D.  The company has just acquired another manufacturing company and is making use of its machinery, at the time of acquisition the depreciation on this machinery was set at zero. 

 

Page 266: Cfa Ques Answer

250   Study Session 09:  

 

1.  When a company reports a  large LIFO reserve, has  it  (1) been using LIFO or FIFO  to calculate inventory balances and has it (2) overstated or understated the value of inventory compared with the current market value?  

Inventory balances Overstated/understated

A. LIFO overstated

B. FIFO overstated

C. LIFO understated

D. FIFO understated

Correct Answer:  C ..........................................................................................LOS: Reading  35‐c The LIFO reserve is reported by companies using LIFO. Accounting using LIFO usually leads to an understatement of the inventory balance compared with current values, so information on the LIFO reserve is provided.

Reference: CFA® Program Curriculum, Volume 3, pp. 308‐315. 

 

 

2.  The average age of Manufacturers Corporation’s machinery is lower than other companies in the same industry. Which of the following is the least likely to explain this? 

A. The company has just made major write downs of impaired assets.

B. The company uses a shorter depreciation life for machinery than its competitors.

C. The company has just finished a major capital expenditure program to replace machinery.

D. The company has just acquired another manufacturing company and is making use of its machinery, at the time of acquisition the depreciation on this machinery was set at zero.

Correct Answer:  A ......................................................................................... LOS: Reading  37‐d 

expense depreciat.ondepreciati accumul.

=age average

Impairment of assets would lead to lower current depreciation charges; this will lead to a higher average asset life. B, C and D will lead to lower average age.

Reference: CFA® Program Curriculum, Volume 3, pp. 402‐404. 

Page 267: Cfa Ques Answer

Financial Statement Inventories, Assets, Taxes & Debt   251 

 

3.  A company has just issued a bond with covenants. Which of the following clauses is least likely to be included in the bond covenants? 

A.  A cap on leverage ratios at specified dates over the life of the bond. 

B.  A requirement that return on capital should exceed a specified percentage. 

C.  A restriction on dividend payments to shareholders if they would reduce stockholders’ equity below a specified level. 

D.  A requirement that no future debt can be issued that ranks higher than the original bonds giving the holders a prior claim on assets.  

 

 

 

4.  Which of the following statements is most accurate regarding cash flow impact for a lessee of a capital lease? 

A.  Total cash outflow each year is the same as the lease payment. 

B.  All of the rental payments are treated as cash flow from operations. 

C.  The rental payments are partly allocated as cash flow from operations and partly as cash flow from investing. 

D.  The present value of the lease payments is classified as an investing cash outflow at the beginning of the lease. 

Page 268: Cfa Ques Answer

252   Study Session 09:  

 

3.  A company has just issued a bond with covenants. Which of the following clauses is least likely to be included in the bond covenants? 

A. A cap on leverage ratios at specified dates over the life of the bond.

B. A requirement that return on capital should exceed a specified percentage.

C. A restriction on dividend payments to shareholders if they would reduce stockholders’ equity below a specified level.

D. A requirement that no future debt can be issued that ranks higher than the original bonds giving the holders a prior claim on assets.

Correct Answer:  B...........................................................................................LOS: Reading  39‐e Covenants tend to focus on clauses that restrict leverage, issuance of new debt or spending (on dividends or investment) that would limit the funds available to repay bond holders. Therefore a return on capital requirement is least likely to be specified.

Reference: CFA® Program Curriculum, Volume 3, p. 500. 

 

 

 

4.  Which of the following statements is most accurate regarding cash flow impact for a lessee of a capital lease? 

A. Total cash outflow each year is the same as the lease payment.

B. All of the rental payments are treated as cash flow from operations.

C. The rental payments are partly allocated as cash flow from operations and partly as cash flow from investing.

D. The present value of the lease payments is classified as an investing cash outflow at the beginning of the lease.

Correct Answer:  A ......................................................................................... LOS: Reading  40‐b There is no cash outflow at the beginning of the lease, other than lease payments, since the asset is not purchased.

The rental payments are partly allocated to cash flow from operations and partly to cash flow from financing.

Reference: CFA® Program Curriculum, Volume 3, pp. 521‐524. 

Page 269: Cfa Ques Answer

Financial Statement Inventories, Assets, Taxes & Debt   253 

 

5.  A mining company is required by regulation to restore land to its original state when mining has finished. Under SFAS 143 the company is required to: 

A.  calculate the undiscounted cost of restoring the land to its original state and depreciate this cost over the life of the mine. 

B.  calculate the present value of restoring the land to its original state and deduct this from the carrying value of the assets. 

C.  calculate the present value of restoring the land to its original state and recognize this as an expense in the first year of operation of the mine.  

D.  calculate the present value of restoring the land to its original state at the end of each year and recognize the difference from the previous year is an accretion expense. 

 

 

 

6.  A U.S. company reviews the value of its fixed assets and finds that an asset that had been written down due to  impairment  in the previous accounting period now has a carrying value  less than the future cash flows expected to be received from the use of the asset.  The company should, under U.S. GAAP: 

A.  revise the value of the asset and record the gain as ‘other income’.  

B.  revise the value of the asset and record the gain as an unusual item. 

C.  revise the value of the asset and record the gain as an extraordinary income. 

D.  not do anything, an increase in the value of assets cannot be recognized until they are sold.  

Page 270: Cfa Ques Answer

254   Study Session 09:  

 

5.  A mining company is required by regulation to restore land to its original state when mining has finished. Under SFAS 143 the company is required to: 

A. calculate the undiscounted cost of restoring the land to its original state and depreciate this cost over the life of the mine.

B. calculate the present value of restoring the land to its original state and deduct this from the carrying value of the assets.

C. calculate the present value of restoring the land to its original state and recognize this as an expense in the first year of operation of the mine.

D. calculate the present value of restoring the land to its original state at the end of each year and recognize the difference from the previous year is an accretion expense.

Correct Answer:  D..........................................................................................LOS: Reading  37‐e A is not correct; the cost should be discounted.

B is not correct; present value should be added to the asset value.

C is not correct; the cost should be added to the asset value and be depreciated over the useful life of the mine.

Reference: CFA® Program Curriculum, Volume 3, pp. 408‐411. 

 

 

 

6.  A U.S. company reviews the value of its fixed assets and finds that an asset that had been written down due to  impairment  in the previous accounting period now has a carrying value  less than the future cash flows expected to be received from the use of the asset.  The company should, under U.S. GAAP: 

A. revise the value of the asset and record the gain as ‘other income’.

B. revise the value of the asset and record the gain as an unusual item.

C. revise the value of the asset and record the gain as an extraordinary income.

D. not do anything, an increase in the value of assets cannot be recognized until they are sold.

Correct Answer:  D......................................................................................... LOS: Reading  37‐d Under U.S. GAAP an increase in the value of an asset cannot be recognized until sale and also previous impairments cannot be restored.

Reference: CFA® Program Curriculum, Volume 3, pp. 402‐406. 

Page 271: Cfa Ques Answer

Financial Statement Inventories, Assets, Taxes & Debt   255 

 

7.  A franchisee who buys the rights to use a name, a product and management expertise of another company should: 

A.  expense the costs of purchasing these rights. 

B.  capitalize the cost of purchasing these rights.  

C.  capitalize the cost of purchasing the rights to use a name and expense the cost of the right to use a product and management expertise. 

D.  capitalize the cost of purchasing the rights to use a name and a product and expense the cost of the right to use management expertise. 

 

 

 

8.  If  a  company  decided  to  change  the  estimated  life  of  an  asset  that  is  already  owned  by  the company it must: 

A.  restate prior years’ accounts.  

B.  not make any retroactive adjustment.  

C.  disclose it as a change in accounting principle.  

D.  calculate the cumulative effect of the change and report net of taxes. 

Page 272: Cfa Ques Answer

256   Study Session 09:  

 

7.  A franchisee who buys the rights to use a name, a product and management expertise of another company should: 

A. expense the costs of purchasing these rights.

B. capitalize the cost of purchasing these rights.

C. capitalize the cost of purchasing the rights to use a name and expense the cost of the right to use a product and management expertise.

D. capitalize the cost of purchasing the rights to use a name and a product and expense the cost of the right to use management expertise.

Correct Answer:  B.......................................................................................... LOS: Reading  36‐b All the costs of purchase should be capitalized.

Reference: CFA® Program Curriculum, Volume 3, p. 360. 

 

 

 

8.  If  a  company  decided  to  change  the  estimated  life  of  an  asset  that  is  already  owned  by  the company it must: 

A. restate prior years’ accounts.

B. not make any retroactive adjustment.

C. disclose it as a change in accounting principle.

D. calculate the cumulative effect of the change and report net of taxes.

Correct Answer:  B.......................................................................................... LOS: Reading  37‐b A, C and D are not correct since changing an estimate of life of an asset is not a change in accounting principle.

Since this is only a change in accounting estimate there is no need to make retroactive adjustments, only current and future depreciation expense will be affected.

Reference: CFA® Program Curriculum, Volume 3, pp. 394‐397. 

Page 273: Cfa Ques Answer

Financial Statement Inventories, Assets, Taxes & Debt   257 

 

9.  If the Cost of Goods Sold under LIFO is $20 million, the increase in LIFO reserve over the same period is $5 million, and the closing inventory under LIFO is $80 million, then the Cost of Goods Sold under FIFO is:  

A.  $15 million. 

B.  $25 million. 

C.  $75 million. 

D.  $85 million. 

 

 

 

10.  Tax rates are reduced at the beginning of a company’s financial year. In the company’s financial statements the tax rate reduction will lead to: 

A.  a restatement of prior years’ accounts. 

B.  a reduction in the value of deferred tax assets. 

C.  an increase in the value of deferred tax liabilities. 

D.  an extraordinary gain which is stated net of tax on the income statement. 

Page 274: Cfa Ques Answer

258   Study Session 09:  

 

9.  If the Cost of Goods Sold under LIFO is $20 million, the increase in LIFO reserve over the same period is $5 million, and the closing inventory under LIFO is $80 million, then the Cost of Goods Sold under FIFO is:  

A. $15 million.

B. $25 million.

C. $75 million.

D. $85 million.

Correct Answer:  A ......................................................................................... LOS: Reading  35‐b COGS under FIFO = COGS under LIFO – increase in LIFO reserve.

COGS under FIFO = $20 million - $5 million = $15 million

Reference: CFA® Program Curriculum, Volume 3, pp. 312‐315. 

 

 

 

10.  Tax rates are reduced at the beginning of a company’s financial year. In the company’s financial statements the tax rate reduction will lead to: 

A. a restatement of prior years’ accounts.

B. a reduction in the value of deferred tax assets.

C. an increase in the value of deferred tax liabilities.

D. an extraordinary gain which is stated net of tax on the income statement.

Correct Answer:  B...........................................................................................LOS: Reading  38‐g The lower tax rate reduces the value of the tax benefit when the deferred tax asset is realized.

Reference: CFA® Program Curriculum, Volume 3, pp. 427‐432. 

Page 275: Cfa Ques Answer

Financial Statement Inventories, Assets, Taxes & Debt   259 

 

11.  Market rates are 5% per annum and a 6% semi annual coupon bond is issued at $1,100 with a face value of  $1,000.  If  an  investor buys  1,000 bonds  at  issue  and  receives  a  coupon payment  after  six months how much of this coupon payment will be recorded as principal repayment by the issuer? 

A.  $2,500. 

B.  $5,000. 

C.  $5,500. 

D.  $30,000. 

 

 

 

12.  A  company  enters  into  an  agreement  to  lease  equipment  (treated  as  a  capital  lease) over  two years and will pay lease payments of $50,000 at the end of each year. If the discount rate used is 8% the closing liability at the end of the first year is closest to: 

A.  $44,582. 

B.  $46,296. 

C.  $50,000. 

D.  $58,000. 

Page 276: Cfa Ques Answer

260   Study Session 09:  

 

11.  Market rates are 5% per annum and a 6% semi annual coupon bond is issued at $1,100 with a face value of  $1,000.  If  an  investor buys  1,000 bonds  at  issue  and  receives  a  coupon payment  after  six months how much of this coupon payment will be recorded as principal repayment by the issuer? 

A. $2,500.

B. $5,000.

C. $5,500.

D. $30,000.

Correct Answer:  A ..........................................................................................LOS: Reading  39‐a The coupon paid will be 3% multiplied by $1,000,000, which is $30,000. The interest the investor earns is 2.5% multiplied by $1,100,000, which is $27,500, so the remainder is principal repayment of $2,500.

Reference: CFA® Program Curriculum, Volume 3, pp. 466‐471. 

 

 

 

12.  A  company  enters  into  an  agreement  to  lease  equipment  (treated  as  a  capital  lease) over  two years and will pay lease payments of $50,000 at the end of each year. If the discount rate used is 8% the closing liability at the end of the first year is closest to: 

A. $44,582.

B. $46,296.

C. $50,000.

D. $58,000.

Correct Answer:  B.......................................................................................... LOS: Reading  40‐b At the beginning of the lease period the liability will be the present value of the lease payments, which is $89,163. The first lease payment is divided between interest (8% x $89,163 = $7,133) and principal repayment. Therefore the liability is reduced by $42,867 to $46,296.

Reference: CFA® Program Curriculum, Volume 3, pp. 521‐523. 

Page 277: Cfa Ques Answer

Financial Statement Inventories, Assets, Taxes & Debt   261 

 

13.  Which of the following is an example of a take‐or‐pay contract? 

A.  A firm agrees to buy a certain amount of oil from an oil producer at the market price each year for the next five years. 

B.  A firm has the option to buy a certain amount of oil from an oil producer at the market price each year for the next five years. 

C.  An oil producer has the option to sell a certain amount of oil to another firm at a fixed price each year for the next five years. 

D.  An oil producer has the option to sell a certain amount of oil to another firm at the market price each year for the next five years. 

 

 

 

14.  If a deferred tax liability is very unlikely to be paid then it should be treated as: 

A.  a long‐term liability. 

B.  stockholders’ equity. 

C.  a short‐term liability. 

D.  an extraordinary item. 

Page 278: Cfa Ques Answer

262   Study Session 09:  

 

13.  Which of the following is an example of a take‐or‐pay contract? 

A. A firm agrees to buy a certain amount of oil from an oil producer at the market price each year for the next five years.

B. A firm has the option to buy a certain amount of oil from an oil producer at the market price each year for the next five years.

C. An oil producer has the option to sell a certain amount of oil to another firm at a fixed price each year for the next five years.

D. An oil producer has the option to sell a certain amount of oil to another firm at the market price each year for the next five years.

Correct Answer:  A ..........................................................................................LOS: Reading  40‐c A take-or-pay contract is a firm commitment, not an option, at either a fixed or market price, and can provide security of supply of a raw material for the buyer and security of income for the seller. These contracts are an example of off-balance-sheet financing since future commitments are not treated as debt.

Reference: CFA® Program Curriculum, Volume 3, pp. 532‐533. 

 

 

 

14.  If a deferred tax liability is very unlikely to be paid then it should be treated as: 

A. a long-term liability.

B. stockholders’ equity.

C. a short-term liability.

D. an extraordinary item.

Correct Answer:  B.......................................................................................... LOS: Reading  38‐d If the factors that created the deferred tax liability are unlikely to be reversed, than it is considered part of stockholders’ equity.

Reference: CFA® Program Curriculum, Volume 3, pp. 439‐440. 

Page 279: Cfa Ques Answer

Financial Statement Inventories, Assets, Taxes & Debt   263 

 

15.  In a direct  financing  lease  the  sale value  reported at  the beginning of  the  lease on  the  lessor’s financial statements is: 

A.  the sum of the lease payments. 

B.  the present value of lease payments. 

C.  zero, there is no sale value reported. 

D.  the present value of lease payments plus the present value of the residual value. 

 

 

 

16.  If a company actively uses operating leases, which of the following statements is least accurate? 

A.  Total cash flow is understated. 

B.  Off‐balance‐sheet financing is significant. 

C.  Minimum lease payments for the next five years must be disclosed in the company’s accounts. 

D.  Return on assets should be adjusted downwards to give a clearer view of the company’s efficiency. 

Page 280: Cfa Ques Answer

264   Study Session 09:  

 

15.  In a direct  financing  lease  the  sale value  reported at  the beginning of  the  lease on  the  lessor’s financial statements is: 

A. the sum of the lease payments.

B. the present value of lease payments.

C. zero, there is no sale value reported.

D. the present value of lease payments plus the present value of the residual value.

Correct Answer:  C ......................................................................................... LOS: Reading  40‐d With a direct financing lease only financing or interest income is reported. There is no ‘manufacturing’ profit and no sale is recorded.

Reference: CFA® Program Curriculum, Volume 3, pp. 550‐551. 

 

 

 

16.  If a company actively uses operating leases, which of the following statements is least accurate? 

A. Total cash flow is understated.

B. Off-balance-sheet financing is significant.

C. Minimum lease payments for the next five years must be disclosed in the company’s accounts.

D. Return on assets should be adjusted downwards to give a clearer view of the company’s efficiency.

Correct Answer:  A ......................................................................................... LOS: Reading  40‐b Total cash flow is not understated but cash flow from operations is lower than if the lease were capitalized.

Reference:  CFA® Program Curriculum, Volume 3, pp. 541‐543. 

Page 281: Cfa Ques Answer

Financial Statement Inventories, Assets, Taxes & Debt   265 

 

17.  A company manufactures a machine and agrees to lease the machine under a sales‐type capital lease agreement. The company will record the gross investment in the machine as:  

A.  the sum of the minimum lease payments. 

B.  the sum of the present values of the minimum lease payments. 

C.  the sum of the minimum lease payments plus the residual value of the machine. 

D.  the sum of the present values of the minimum lease payments plus the present value of the residual value of the machine. 

 

 

 

18.  When market interest rates decline, a company that has fixed‐rate debt outstanding will: 

A.  lose in economic terms. 

B.  gain in economic terms. 

C.  the debt on the balance sheet is automatically adjusted to current market value so the economic loss will be reflected as an accounting loss. 

D.  the debt on the balance sheet is automatically adjusted to current market value so the economic gain will be reflected as an accounting gain. 

Page 282: Cfa Ques Answer

266   Study Session 09:  

 

17.  A company manufactures a machine and agrees to lease the machine under a sales‐type capital lease agreement. The company will record the gross investment in the machine as:  

A. the sum of the minimum lease payments.

B. the sum of the present values of the minimum lease payments.

C. the sum of the minimum lease payments plus the residual value of the machine.

D. the sum of the present values of the minimum lease payments plus the present value of the residual value of the machine.

Correct Answer:  C ......................................................................................... LOS: Reading  40‐d Net investment looks at the present value of the payments, the difference between the gross investment and net investment is the interest component of the revenue represented by unearned income.

Reference: CFA® Program Curriculum, Volume 3, pp. 547‐550. 

 

 

 

18.  When market interest rates decline, a company that has fixed‐rate debt outstanding will: 

A. lose in economic terms.

B. gain in economic terms.

C. the debt on the balance sheet is automatically adjusted to current market value so the economic loss will be reflected as an accounting loss.

D. the debt on the balance sheet is automatically adjusted to current market value so the economic gain will be reflected as an accounting gain.

Correct Answer:  A .......................................................................................... LOS: Reading  39‐f The debt on the balance sheet reflects the market rate at issuance and therefore if market interest rates decline the market value of the debt will increase which leads to an economic loss.

Reference: CFA® Program Curriculum, Volume 3, pp. 489‐493. 

Page 283: Cfa Ques Answer

Financial Statement Inventories, Assets, Taxes & Debt   267 

 

19.  The financial ratios for a company, which uses operating leases compared to a company that uses capital leases, will be as follows: 

  Interest cover    Return on assets 

A.  lower       lower 

B.  lower       higher  

C.  higher       lower  

D.  higher       higher 

 

 

 

20.  An analyst notices that a company’s net deferred tax liability has declined; this is most likely to be explained by: 

A.  the company has made a profit. 

B.  a new tax law has increased tax rates. 

C.  it is a growing company with rising capital expenditure. 

the company has written down asset values due to impairment. 

Page 284: Cfa Ques Answer

268   Study Session 09:  

 

19.  The financial ratios for a company, which uses operating leases compared to a company that uses capital leases, will be as follows: 

Interest cover Return on assets

A. lower lower

B. lower higher

C. higher lower

D. higher higher

Correct Answer:  D......................................................................................... LOS: Reading  40‐b Interest cover will be higher as interest payments will be lower.

Return on assets will be higher since assets will be lower.

Reference: CFA® Program Curriculum, Volume 3, pp. 521‐524. 

 

 

 

20.  An analyst notices that a company’s net deferred tax liability has declined; this is most likely to be explained by: 

A. the company has made a profit.

B. a new tax law has increased tax rates.

C. it is a growing company with rising capital expenditure.

the company has written down asset values due to impairment.

Correct Answer:  D..........................................................................................LOS: Reading  37‐a A would not reduce deferred tax liabilities.

B is not correct since this would increase deferred tax liabilities

C is not correct since this would to lead to increasing deferred tax liabilities if accelerated tax depreciation methods are used for tax reporting.

Reference: CFA® Program Curriculum, Volume 3, pp. 425‐452. 

Page 285: Cfa Ques Answer

Financial Statement Inventories, Assets, Taxes & Debt   269 

 

21.  The following financial data is provided by Sportswear Corporation: 

$ million (using LIFO)  2006  2007 

Inventory    105 125

Current assets  300 350

LIFO reserve    45 55

Current liabilities  110 160

COGS  1450 1700

 

The current ratios using FIFO are closest to: 

  2006    2007 

A.  1.36   1.13 

B.  2.32    1.84 

C.  2.73    2.19 

D.  3.14    2.53 

Page 286: Cfa Ques Answer

270   Study Session 09:  

 

21.  The following financial data is provided by Sportswear Corporation: 

$ million (using LIFO)   2006  2007 

Inventory    105 125

Current assets  300 350

LIFO reserve    45 55

Current liabilities  110 160

COGS  1450 1700

The current ratios using FIFO are closest to:

2006 2007

A. 1.36 1.13

B. 2.32 1.84

C. 2.73 2.19

D. 3.14 2.53

Correct Answer:  D......................................................................................... LOS: Reading  35‐d Adjust current assets by adding the LIFO reserve to get current assets under FIFO of 345 in 2006 and 405 in 2007.

Current ratio is current assets /current liabilities = 345/110 and 405/160 respectively which equal 3.14 and 2.53 respectively.

Reference: CFA® Program Curriculum, Volume 3, pp. 312‐315. 

Page 287: Cfa Ques Answer

Financial Statement Inventories, Assets, Taxes & Debt   271 

 

22.  The following information is given regarding a company’s activities: 

Tax rate is 30%. 

The only expense is depreciation. 

A new machine is purchased at a cost of $10,000. 

Annual revenues of $40,000 are generated from the new machine. 

The company, in its financial reports, depreciates the machine by using the straight‐line method over four years and the salvage value is estimated to be $2,000. 

For tax purposes the machine is depreciated using the straight‐line method over two years with the same salvage value. 

The deferred tax expense in year 2 will be: 

A.  $600. 

B.  $1,200. 

C.  $10,800. 

D.  $11,400. 

Page 288: Cfa Ques Answer

272   Study Session 09:  

 

22.  The following information is given regarding a company’s activities: 

Tax rate is 30%.

The only expense is depreciation.

A new machine is purchased at a cost of $10,000.

Annual revenues of $40,000 are generated from the new machine.

The company, in its financial reports, depreciates the machine by using the straight-line method over four years and the salvage value is estimated to be $2,000.

For tax purposes the machine is depreciated using the straight‐line method over two years with the same salvage value. 

The deferred tax expense in year 2 will be: 

A. $600.

B. $1,200.

C. $10,800.

D. $11,400.

Correct Answer:  A .......................................................................................... LOS: Reading  38‐f 

 

Tax reporting – straight-line depreciation over two Financial statement reporting – straight-line

  Year 2  Year 1 Year 1 Year 2 Revenue  $40,000  $40,000 Revenue $40,000  $40,000Depreciation  $4,000  $4,000 Depreciation $2,000  $2,000Taxable income  $36,000  $36,000 Pre‐tax income   $38,000  $38,000Taxes payable  $10,800  $10,800 Tax expense $11,400  $11,400    of which:       Taxes payable $10,800   $10,800    Deferred tax  $600  $600 

Reference: CFA® Program Curriculum, Volume 3, pp. 425‐470. 

Page 289: Cfa Ques Answer

Financial Statement Inventories, Assets, Taxes & Debt   273 

 

 

 

 

 

Page 290: Cfa Ques Answer

274   Study Session 10:  

 

Study Session 10: Financial Statement Analysis: Techniques, Applications, and International Standards Convergence

 

The readings in this study session discuss financial analysis techniques, financial statement analysis applications, and the international convergence of accounting standards. 

The first reading presents the most frequently used tools and techniques used to evaluate companies, including  common  size  analysis,  cross‐sectional  analysis,  trend  analysis,  and  ratio  analysis.  The second  reading  then  shows  the  application  of  financial  analysis  techniques  to major  analyst  tasks including  the evaluation of past and  future  financial performance, credit risk, and  the screening of potential equity  investments. The reading also discusses analyst adjustments  to reported  financials. Such adjustments are often needed to put companies’ reported results on a comparable basis. 

This  study  session  concludes with  a  reading on  convergence of  international  and U.S.  accounting standards. Although there has been much progress in harmonizing accounting standards globally, as this  reading discusses,  there  are  still  significant  variations  between  generally  accepted  accounting principles from one country to another. 

 

Reading 41: Financial Analysis Techniques 

Reading 42: Financial Statement Analysis: Applications 

Reading 43: International Standards Convergence 

 

 

Page 291: Cfa Ques Answer

Financial Statement Techniques, Applications, and International Standards Convergence   275 

 

1.  If the debt‐to‐capital ratio has decreased for a firm this is most likely to be explained by: 

A.  the firm has just completed a new issue of equity to finance its expansion. 

B.  the book value of debt has not changed but the market value of long‐term debt has decreased. 

C.  the book value of debt has not changed but the market value of long‐term debt has increased. 

D.  the firm has switched from using long‐term debt to using more short‐term borrowing to finance its activities. 

 

 

 

2.  Some  analysts  prefer  to  use  EBITDA  rather  than  earnings  per  share  to  value  a  company’s performance, which of the following would support this preference?  

A.  EBITDA is more sensitive to the gearing of the company. 

B.  EBITDA is a more volatile number than earning per share. 

C.  EBITDA does not take into account the cost of using fixed assets. 

D.  EBITDA gives a better indication of profits available to equity shareholders.  

 

Page 292: Cfa Ques Answer

276   Study Session 10:  

 

1.  If the debt‐to‐capital ratio has decreased for a firm this is most likely to be explained by: 

A. the firm has just completed a new issue of equity to finance its expansion.

B. the book value of debt has not changed but the market value of long-term debt has decreased.

C. the book value of debt has not changed but the market value of long-term debt has increased.

D. the firm has switched from using long-term debt to using more short-term borrowing to finance its activities.

Correct Answer:  A ......................................................................................... LOS: Reading  41‐d Debt-to-capital ratio is total debt divided by total debt plus shareholders’ equity. B and C are not correct since the book value of debt is used in the calculation. Usually total interest bearing debt is used in the calculation of capital so D is not correct.

A would lead to an increase in total equity so would explain a fall in the debt-to-capital ratio.

Reference: CFA® Program Curriculum, Volume 3, pp. 593‐597. 

 

 

 

2.  Some  analysts  prefer  to  use  EBITDA  rather  than  earnings  per  share  to  value  a  company’s performance, which of the following would support this preference?  

A. EBITDA is more sensitive to the gearing of the company.

B. EBITDA is a more volatile number than earning per share.

C. EBITDA does not take into account the cost of using fixed assets.

D. EBITDA gives a better indication of profits available to equity shareholders.

Correct Answer:  C ..........................................................................................LOS: Reading  41‐g Usually earnings per share are more volatile than EBITDA, since they are after interest which is effectively a fixed charge. EBITDA is profit available to all providers of capital, since it is before interest, and is not directly sensitive to the level of gearing.

Since EBITDA is before depreciation and capital expenditure it does not take into account the cost of using fixed assets, so might be used of compare companies with very different levels of fixed asset investment.

Reference: CFA® Program Curriculum, Volume 3, pp. 610‐613. 

Page 293: Cfa Ques Answer

Financial Statement Techniques, Applications, and International Standards Convergence   277 

 

3.  An analyst is evaluating the impact of changes in oil prices on an airline’s profitability; another analyst is projecting the cash flows for a retailer based on different economic forecasts.  The analysts are most likely to use which type of analysis? 

  Airline    Retailer 

A.  scenario     scenario 

B.  scenario    sensitivity  

C.  sensitivity     scenario 

D.  sensitivity     sensitivity  

 

 

 

4.  Under International Financial Reporting Standards (IFRS) impairment of goodwill: 

A.  is not permitted, amortization is still used. 

B.  it charged as a valuation allowance against the acquired assets. 

C.  leads to a reduction in net income in the period when the impairment is recognized. 

D.  is charged directly to stockholders’ equity and does not impact on the income statement. 

Page 294: Cfa Ques Answer

278   Study Session 10:  

 

3.  An analyst is evaluating the impact of changes in oil prices on an airline’s profitability; another analyst is projecting the cash flows for a retailer based on different economic forecasts.  The analysts are most likely to use which type of analysis? 

Airline Retailer

A. scenario scenario

B. scenario sensitivity

C. sensitivity scenario

D. sensitivity sensitivity

Correct Answer:  C ......................................................................................... LOS: Reading  41‐h Sensitivity analysis will give a range of possible outcomes if a variable changes, so would be applicable to measuring the sensitivity of the airlines profits to an oil price move. In scenario analysis forecasts are made based on specific assumptions or outcomes, so this would accommodate using different economic forecasts to predict cash flows for the retailer.

Reference: CFA® Program Curriculum, Volume 3, pp. 622‐623. 

 

 

 

4.  Under International Financial Reporting Standards (IFRS) impairment of goodwill: 

A. is not permitted, amortization is still used.

B. it charged as a valuation allowance against the acquired assets.

C. leads to a reduction in net income in the period when the impairment is recognized.

D. is charged directly to stockholders’ equity and does not impact on the income statement.

Correct Answer:  C ......................................................................................... LOS: Reading  43‐b Impairment reduces income in the year it is recorded, reduces the value of net asset and reduces stockholder’s equity. Under IFRS goodwill must be checked annually for impairment, amortization is not permitted.

Reference: CFA® Program Curriculum, Volume 3, pp. 685‐689. 

Page 295: Cfa Ques Answer

Financial Statement Techniques, Applications, and International Standards Convergence   279 

 

5.  National Telecoms Corporation’s interest coverage is closest to (ref. financial data in question 34):  

A.  1.11. 

B.  2.11. 

C.  2.67. 

D.  6.00. 

 

 

 

6.  The  gross  profit margin  of  a  company  has declined  relative  to  its  competitors. This  could  be explained by: 

A.  the company has recognized losses on plant and equipment that has become obsolete. 

B.  the company’s credit quality has declined pushing up the interest rate on short‐term debt. 

C.  the company has increased its marketing efforts and hired additional marketing executives. 

D.  the company has switched to using LIFO accounting rather than FIFO accounting in an inflationary environment. 

Page 296: Cfa Ques Answer

280   Study Session 10:  

 

5.  National Telecoms Corporation’s interest coverage is closest to (ref. financial data in question 34):  

A. 1.11.

B. 2.11.

C. 2.67.

D. 6.00.

Correct Answer:  C ......................................................................................... LOS: Reading  41‐d Interest coverage = EBIT = 120 = 2.67

interest expense 45

Reference: CFA® Program Curriculum, Volume 3, pp. 593‐597. 

 

 

 

6.  The  gross  profit margin  of  a  company  has declined  relative  to  its  competitors. This  could  be explained by: 

A. the company has recognized losses on plant and equipment that has become obsolete.

B. the company’s credit quality has declined pushing up the interest rate on short-term debt.

C. the company has increased its marketing efforts and hired additional marketing executives.

D. the company has switched to using LIFO accounting rather than FIFO accounting in an inflationary environment.

Correct Answer:  D......................................................................................... LOS: Reading  41‐d Gross profit margin is (revenue – COGS)/revenue.

Interest expense, losses on sale of plant and equipment and marketing expense are not part of COGS. LIFO accounting would increase the cost of inventory sold thereby reducing the gross margin.

Reference: CFA® Program Curriculum, Volume 3, pp. 597‐601. 

Page 297: Cfa Ques Answer

Financial Statement Techniques, Applications, and International Standards Convergence   281 

 

7.  A company has a low fixed asset base relative to its sector average. It is generally the case that: 

A.  its sales variability is higher than that of other companies in the sector. 

B.  the company’s financial risk is lower than that of other companies in the sector. 

C.  operating profits will be relatively volatile compared to other companies in the sector.  

D.  investors will tolerate a higher debt‐to‐equity relative to other companies in the sector. 

 

 

 

8.  The return on total capital of a company has increased. This could be explained by: 

A.  the company’s operating profit has been deteriorating as competitive pressure increases.  

B.  the company has sold unprofitable business units and used the proceeds to reduce debt. 

C.  the company has become less risky and therefore a higher return on total invested capital is required. 

D.  the company has increased borrowing to expand its operating capacity but this has not yet been reflected in a rise in earnings. 

Page 298: Cfa Ques Answer

282   Study Session 10:  

 

7.  A company has a low fixed asset base relative to its sector average. It is generally the case that: 

A. its sales variability is higher than that of other companies in the sector.

B. the company’s financial risk is lower than that of other companies in the sector.

C. operating profits will be relatively volatile compared to other companies in the sector.

D. investors will tolerate a higher debt-to-equity relative to other companies in the sector.

Correct Answer:  D......................................................................................... LOS: Reading  41‐d A low fixed asset base generally leads to lower operating leverage, i.e. operating profits are less sensitive to changes in sales revenue. Investors will usually accept greater financial risk if operating risk is lower. One of the measures of financial risk is the debt-to-equity ratio.

Reference: CFA® Program Curriculum, Volume 3, pp. 593‐594. 

 

 

 

8.  The return on total capital of a company has increased. This could be explained by: 

A. the company’s operating profit has been deteriorating as competitive pressure increases.

B. the company has sold unprofitable business units and used the proceeds to reduce debt.

C. the company has become less risky and therefore a higher return on total invested capital is required.

D. the company has increased borrowing to expand its operating capacity but this has not yet been reflected in a rise in earnings.

Correct Answer:  B.......................................................................................... LOS: Reading  41‐d The total capital will have been reduced by the repayment of debt, thereby increasing the return on total capital.

Reference: CFA® Program Curriculum, Volume 3, pp. 597‐601. 

Page 299: Cfa Ques Answer

Financial Statement Techniques, Applications, and International Standards Convergence   283 

 

9.  A major competitor of a company has the same net profit margin and total asset turnover as the company. The competitor’s return on equity is higher; this is explained by the competitor having:  

A.  a lower tax rate. 

B.  higher financial leverage. 

C.  lower interest costs as a percentage of sales. 

D.  lower interest costs as a percentage of operating profit. 

 

 

 

10.  A company provides the following information:  

 

  2006  2007 

Return on equity  8.9% 9.4%

Return on total assets  4.5% 4.2%

Total asset turnover  1.5 1.7

 

The numbers could be explained by: 

  Financial leverage    Net profit margin 

A.  increased      increased 

B.  increased      decreased 

C.  decreased      increased 

D.  decreased      decreased 

Page 300: Cfa Ques Answer

284   Study Session 10:  

 

9.  A major competitor of a company has the same net profit margin and total asset turnover as the company. The competitor’s return on equity is higher; this is explained by the competitor having:  

A. a lower tax rate.

B. higher financial leverage.

C. lower interest costs as a percentage of sales.

D. lower interest costs as a percentage of operating profit.

Correct Answer:  B.......................................................................................... LOS: Reading  41‐d Return on equity = net profit margin x total asset turnover x financial leverage

Reference: CFA® Program Curriculum, Volume 3, pp. 597‐601. 

 

 

10.  A company provides the following information:  

  2006  2007 

Return on equity  8.9% 9.4%

Return on total assets  4.5% 4.2%

Total asset turnover  1.5 1.7

The numbers could be explained by: 

Financial leverage Net profit margin

A. increased increased

B. increased decreased

C. decreased increased

D. decreased decreased

Correct Answer:  B........................................................................................... LOS: Reading  41‐f Return on equity = return on assets x financial leverage. Return on equity has increased when return on assets fell, so financial leverage must have increased.

Return on assets = net profit margin x total asset turnover. If return on assets fell and total asset turnover increased in must be because net profit margins fell.

Reference: CFA® Program Curriculum, Volume 3, pp. 84‐89. 

Page 301: Cfa Ques Answer

Financial Statement Techniques, Applications, and International Standards Convergence   285 

 

11.  Which of  the  following  is  least  likely  to be  a  limitation of using  ratio  analysis  to  evaluate  the financial performance of a company?  

A.  Ratios are more applicable for equity analysis and are less useful for credit analysis. 

B.  Ratios based on financial statements using different accounting methods may not be comparable.  

C.  There are potential inconsistencies between the signals different ratios are giving about a company’s financial health.  

D.  A company’s operations may be diverse and it is not always easy to find companies that can provide useful industry comparisons. 

 

 

 

12.  When Moody’s calculates  that a  firm’s  retained cash  flow  to debt  is 0.25,  this  is most  likely  to mean that: 

A.  the firm could pay off its debt from retained cash flow in about three months. 

B.  unless the firm has a heavy capital expenditure program it could pay off its debt from retained cash flow in about four years. 

C.  the cash flows required to pay interest on the debt are approximately a quarter of the firm’s total cash flows, after capital expenditure.  

D.  the firm could increase debt financing by a factor of four times and still have sufficient cash flow to cover interest payments on the debt. 

Page 302: Cfa Ques Answer

286   Study Session 10:  

 

11.  Which of  the  following  is  least  likely  to be  a  limitation of using  ratio  analysis  to  evaluate  the financial performance of a company?  

A. Ratios are more applicable for equity analysis and are less useful for credit analysis.

B. Ratios based on financial statements using different accounting methods may not be comparable.

C. There are potential inconsistencies between the signals different ratios are giving about a company’s financial health.

D. A company’s operations may be diverse and it is not always easy to find companies that can provide useful industry comparisons.

Correct Answer:  A ......................................................................................... LOS: Reading  41‐b Ratios are widely used by both equity analysts and credit analysts so A is not correct. B, C and D are all potential limitations of using ratios for financial analysis.

Reference: CFA® Program Curriculum, Volume 3, pp. 572‐573. 

 

 

 

12.  When Moody’s calculates  that a  firm’s  retained cash  flow  to debt  is 0.25,  this  is most  likely  to mean that: 

A. the firm could pay off its debt from retained cash flow in about three months.

B. unless the firm has a heavy capital expenditure program it could pay off its debt from retained cash flow in about four years.

C. the cash flows required to pay interest on the debt are approximately a quarter of the firm’s total cash flows, after capital expenditure.

D. the firm could increase debt financing by a factor of four times and still have sufficient cash flow to cover interest payments on the debt.

Correct Answer:  B...........................................................................................LOS: Reading  42‐c Retained cash flow is defined as operating cash flow before working capital changes, less dividends. If cash flows and debt remains stable and there were no capital expenditures then the company could pay off its debt from retained cash flow in 1/0.25, or 4 years.

Reference: CFA® Program Curriculum, Volume 3, pp. 649‐652 

Page 303: Cfa Ques Answer

Financial Statement Techniques, Applications, and International Standards Convergence   287 

 

13.  When a firm cannot confidently forecast the outcome, in terms or total revenues and costs, of a construction project the appropriate accounting treatments for recognizing revenue under U.S. GAAP and International Financial Reporting Standards (IFRS) are:  

  U.S. GAAP        IFRS 

A.  completed contract       percentage of completion 

B.  completed contract       revenue if contract costs can be recovered 

C.  percentage of completion    completed contract   

D.  percentage of completion    revenue if contract costs can be recovered  

 

 

 

14.  A firm has a higher return on equity to its competitors; this is least likely to be explained by:  

A.  a lower tax burden. 

B.  higher asset turnover. 

C.  a lower interest burden. 

D.  lower financial leverage. 

Page 304: Cfa Ques Answer

288   Study Session 10:  

 

13.  When a firm cannot confidently forecast the outcome, in terms or total revenues and costs, of a construction project the appropriate accounting treatments for recognizing revenue under U.S. GAAP and International Financial Reporting Standards (IFRS) are:  

U.S. GAAP IFRS

A. completed contract percentage of completion

B. completed contract revenue if contract costs can be recovered

C. percentage of completion completed contract

D. percentage of completion revenue if contract costs can be recovered

 

Correct Answer:  B.......................................................................................... LOS: Reading  43‐b Under IFRS, revenue can be recognised to the extent that contract costs can be recovered. Under U.S. GAAP, the completed contract method must be used.

Reference: CFA® Program Curriculum, Volume 3, p. 684. 

 

 

 

14.  A firm has a higher return on equity to its competitors; this is least likely to be explained by:  

A. a lower tax burden.

B. higher asset turnover.

C. a lower interest burden.

D. lower financial leverage.

Correct Answer:  D.......................................................................................... LOS: Reading  41‐f Return on equity, using DuPont analysis, can be decomposed into

total asset turnover x leverage x tax burden x interest burden x EBIT margin.

Therefore lower financial leverage would tend to reduce return on equity.

Reference: CFA® Program Curriculum, Volume 3, pp. 604‐609. 

Page 305: Cfa Ques Answer

Financial Statement Techniques, Applications, and International Standards Convergence   289 

 

15.  Under  International  Financial  Reporting  Standards  (IFRS)  unrealized  gains  on which  type  of marketable securities are recorded in the income statement?  

A.  held‐for‐trading securities only. 

B.  available‐for‐sale securities only. 

C.  held‐for‐trading securities and available‐for sale securities only. 

D.  held‐for‐trading securities, available‐for sale securities and held‐to‐maturity securities. 

 

 

 

16.  A company has employed a new financial controller who has installed a new system to improve the  efficiency  of  inventory management,  and who has written  off  a  large  amount  of uncollectible receivables.  This is likely to: 

  Inventory turnover    Receivables turnover 

A.  increase      increase 

B.  increase      decrease 

C.  decrease      increase 

D.  decrease      decrease 

Page 306: Cfa Ques Answer

290   Study Session 10:  

 

15.  Under  International  Financial  Reporting  Standards  (IFRS)  unrealized  gains  on which  type  of marketable securities are recorded in the income statement?  

A. held-for-trading securities only.

B. available-for-sale securities only.

C. held-for-trading securities and available-for sale securities only.

D. held-for-trading securities, available-for sale securities and held-to-maturity securities.

Correct Answer:  A ..........................................................................................LOS: Reading  43‐a Unrealized gains are not recognized for held-to-maturity securities. Available-for sale securities are recorded at market value but any unrealized gains flow straight through to shareholders’ equity. Only unrealized gains on held-for-trading securities are recorded in the income statement.

Reference: CFA® Program Curriculum, Volume 3, pp. 681‐683. 

 

 

 

16.  A company has employed a new financial controller who has installed a new system to improve the  efficiency  of  inventory management,  and who has written  off  a  large  amount  of uncollectible receivables.  This is likely to: 

Inventory turnover Receivables turnover

A. increase increase

B. increase decrease

C. decrease increase

D. decrease decrease

Correct Answer:  A ......................................................................................... LOS: Reading  41‐d More efficient management of inventory would be expected to increase the inventory turnover, by reducing the number of days goods were held as inventory. Writing off receivables would reduce the denominator in the receivables turnover, thereby increasing receivables turnover.

Reference: CFA® Program Curriculum, Volume 3, pp. 586‐589. 

Page 307: Cfa Ques Answer

Financial Statement Techniques, Applications, and International Standards Convergence   291 

 

17.  Which  of  the  following  actions  would  be  least  likely  to  improve  a  firm’s  working  capital turnover?  

A.  Reducing inventory levels. 

B.  Tightening credit terms given to customers.   

C.  Reducing the fixed assets used in the business. 

D.  Maintaining working capital at current levels whilst achieving steady revenue growth.  

 

 

 

18.  South Inc.’s breakdown of current assets and liabilities is as follows: 

Current assets  ($ ‘000) Current liabilities  ($ ‘000)

Cash and cash equivalents  40  Short term debt  25 

Receivables  650  Accounts payable  420 

Inventories  350  Taxes payable  120 

Other current assets  70  Other current liabilities  45 

Total  1,110 Total  610

 

  The quick ratio is closest to:  

A.  0.07. 

B.  1.13. 

C.  1.55. 

D.  1.82. 

Page 308: Cfa Ques Answer

292   Study Session 10:  

 

17.  Which  of  the  following  actions  would  be  least  likely  to  improve  a  firm’s  working  capital turnover?  

A. Reducing inventory levels.

B. Tightening credit terms given to customers.

C. Reducing the fixed assets used in the business.

D. Maintaining working capital at current levels whilst achieving steady revenue growth.

Correct Answer:  C ......................................................................................... LOS: Reading  41‐d Working capital turnover is revenue/average working capital, where working capital is current assets less current liabilities. Reducing working capital by, for example reducing inventory levels would improve the ratio. Increasing revenue collection from customers would reduce receivables and as long as the cash collected was not held in cash equivalents this would improve the ratio. Fixed assets are not part of working capital so reducing fixed assets would not directly affect working capital turnover.

Reference: CFA® Program Curriculum, Volume 3, pp. 583‐589. 

 

 

18.  South Inc.’s breakdown of current assets and liabilities is as follows: 

Current assets  ($ ‘000) Current liabilities ($ ‘000) Cash and cash equivalents  40 Short term debt 25 Receivables  650 Accounts payable 420 Inventories  350 Taxes payable 120 Other current assets  70 Other current liabilities 45 Total  1,110 Total 610 

The quick ratio is closest to:

A. 0.07.

B. 1.13.

C. 1.55.

D. 1.82.

Correct Answer:  B.......................................................................................... LOS: Reading  41‐d Quick ratio = cash + market securities + receivables

current liabilities

= 690 = 1.13

610

Reference: CFA® Program Curriculum, Volume 3, pp. 590‐593. 

Page 309: Cfa Ques Answer

Financial Statement Techniques, Applications, and International Standards Convergence   293 

 

19.  Which of the following ratios is least likely to be used to measure profitability? 

A.  Return on assets. 

B.  Total asset turnover. 

C.  Operating profit margin. 

D.  Return on common equity. 

 

 

 

20.  A computer manufacturing company has very short days of  inventory on hand compared with its competitors, this could be explained by:  

A.  the company runs a just‐in‐time manufacturing system.  

B.  a significant proportion of the company’s inventory is obsolete. 

C.  the company maintains high inventory levels in order to meet customer orders promptly.  

D.  the resources of the company tied up in inventory are relatively high compared with its competitors.  

Page 310: Cfa Ques Answer

294   Study Session 10:  

 

19.  Which of the following ratios is least likely to be used to measure profitability? 

A. Return on assets.

B. Total asset turnover.

C. Operating profit margin.

D. Return on common equity.

Correct Answer:  B.......................................................................................... LOS: Reading  41‐d Total asset turnover is a measure of activity rather than profitability.

Reference: CFA® Program Curriculum, Volume 3, pp. 597‐598. 

 

 

 

20.  A computer manufacturing company has very short days of  inventory on hand compared with its competitors, this could be explained by:  

A. the company runs a just-in-time manufacturing system.

B. a significant proportion of the company’s inventory is obsolete.

C. the company maintains high inventory levels in order to meet customer orders promptly.

D. the resources of the company tied up in inventory are relatively high compared with its competitors.

Correct Answer:  A ......................................................................................... LOS: Reading  41‐d Short DOH indicates high inventory turnover (cost of goods sold/inventory) so it is unlikely the inventory is obsolete and the resources of the company tied up in inventory is smaller than its competitors. A just-in-time manufacturing system means that computers are manufactured in response to customer demand which will reduce inventory levels, so A is correct.

Reference: CFA® Program Curriculum, Volume 3, pp. 583‐589. 

Page 311: Cfa Ques Answer

Financial Statement Techniques, Applications, and International Standards Convergence   295 

 

21.  ABC Corporation provides you with the following information: 

Income statement $  million 

Balance sheet Average over period                $ million 

Sales  150  Cash  90 Accounts payable  30

COGS    (75)  Accounts receivable  10 Short‐term bank notes  25

Gross profit  75  Inventory  70 Long‐term debt           60

SGA expenses  (20)  Property, P & E  150  

Op. profit  55  Depreciation  (70) Common stock    50

Interest  expense          (15)  Investment  30 Retained earnings  115

Tax  (10)     

Net income  30  Total assets  280 Total liabilities & equity 

280

 

Inventory turnover is closest to: 

A.  0.47. 

B.  0.93. 

C.  1.07. 

D.  2.14. 

Page 312: Cfa Ques Answer

296   Study Session 10:  

 

21.  ABC Corporation provides you with the following information: 

Income statement $  million 

Balance sheet Average over period                $ million 

Sales  150  Cash  90 Accounts payable  30

COGS    (75)  Accounts receivable  10 Short‐term bank notes  25

Gross profit  75  Inventory  70 Long‐term debt           60

SGA expenses  (20)  Property, P & E  150  

Op. profit  55  Depreciation  (70) Common stock    50

Interest  expense          (15)  Investment  30 Retained earnings  115

Tax  (10)     

Net income  30  Total assets  280 Total liabilities & equity 

280

Inventory turnover is closest to:

A. 0.47.

B. 0.93.

C. 1.07.

D. 2.14.

Correct Answer:  C ......................................................................................... LOS: Reading  41‐d 

Inventory turnover =

Reference: CFA® Program Curriculum, Volume 3, pp. 583‐586. 

07.17075

inventory averageCOGS

==

Page 313: Cfa Ques Answer

Financial Statement Techniques, Applications, and International Standards Convergence   297 

 

22.  National Telecoms Corporation provides the following information: 

Income statement  Balance sheet $ million  Average over period    $ million 

Sales  1,050 Cash  90 Accounts payable  50COGS  (780) Accounts  170 Notes payable  125Gross profit  270 Inventory*  200 Long‐term debt  300SGA exp.  (150) Property, P&E  650  

Op. profit  120 Depreciation  (430) Common stock  150Interest exp.  (45)   Retained earnings  55Tax  (25)    

Net income  50  Total assets  680  Total liabilities & equity 

680 

  *Inventory is unchanged over the period 

 

National Telecoms Corporation’s cash conversion cycle is closest to:  

A.  58 days. 

B.  130 days. 

C.  153 days. 

D.  176 days. 

Page 314: Cfa Ques Answer

298   Study Session 10:  

 

22.  National Telecoms Corporation provides the following information: 

Income statement  Balance sheet $ million  Average over period    $ million 

Sales  1,050  Cash  90 Accounts payable  50COGS  (780)  Accounts  170 Notes payable  125Gross profit  270  Inventory*  200 Long‐term debt  300SGA exp.  (150)  Property, P&E  650  

Op. profit  120  Depreciation  (430) Common stock  150Interest exp.  (45)    Retained earnings  55Tax  (25)     

Net income  50  Total assets  680  Total liabilities & equity

680 

*Inventory is unchanged over the period

National Telecoms Corporation’s cash conversion cycle is closest to:

A. 58 days.

B. 130 days.

C. 153 days.

D. 176 days.

Correct Answer:  B.......................................................................................... LOS: Reading  41‐d Days of sales outstanding = 365/annual receivables turnover = 365/(1050/170) = 59 days

Days of inventory on hand = 365/inventory turnover = 365/(780/200) = 94 days

Days of payables = 365/payables turnover = 365/(780/50) = 23 days

Cash conversion cycle =

Receivables collection period + inventory processing period – payables payment period= 130 days

Reference: CFA® Program Curriculum, Volume 3, pp. 583‐589. 

Page 315: Cfa Ques Answer

Financial Statement Techniques, Applications, and International Standards Convergence   299 

 

 

 

Page 316: Cfa Ques Answer

300   Study Session 11:  

 

Study Session 11: Corporate Finance:  

This study session covers the principles that corporations use to make their investing and financing decisions. Capital budgeting  is  the process of making decisions about which  long‐term projects  the corporation should accept for  investment, and which  it should reject. Both the expected return of a project and the financing cost should be taken into account. The cost of capital, or the rate of return required for a project, must be developed using economically sound methods. 

Corporate managers  are  concerned with  liquidity  and  solvency,  and  use  financial  statements  to evaluate performance as well as to develop and communicate future plans. The final reading in this study session is on corporate governance practices, which can expose the firm to a heightened risk of ethical lapses. Although these practices may not be inherently unethical, they create the potential for conflicts of  interest  to develop between shareholders and managers, and  the extent of  that conflict affects the firm’s valuation.  

 

Reading 44: Capital Budgeting 

Reading 45: Cost of Capital 

Reading 46: Working Capital Management 

Reading 47: Financial Statement Analysis 

Reading 48: The Corporate Governance of Listed Companies: 

  A Manual for Investors 

 

Page 317: Cfa Ques Answer

Corporate Finance   301 

 

1.  A  company has  trade  credit which  is  2/10, net  30.   The  company pays on  the  20th day.   The effective borrowing cost of not paying on the 10th day is closest to: 

A.  36.5%. 

B.  44.6%. 

C.  106.0%. 

D.  109.0%. 

 

 

 

2.  A company has trade credit with its suppliers and receives a discount if it pays within a specified number  of  days.  It works  out  the  cost  of  trade  credit  is  50%  if  it  pays  on  the  net  day.      If  the company’s short term cost of funds is 18% then the company should pay: 

A.  on the net day. 

B.  on the day or purchase. 

C.  on the last day of the discount period 

D.  just after the discount period has ended. 

 

Page 318: Cfa Ques Answer

302   Study Session 11:  

 

1.  A  company has  trade  credit which  is  2/10, net  30.   The  company pays on  the  20th day.   The effective borrowing cost of not paying on the 10th day is closest to: 

A. 36.5%.

B. 44.6%.

C. 106.0%.

D. 109.0%.

Correct Answer:  D......................................................................................... LOS: Reading  46‐d The cost of trade credit is calculated as the implicit rate of return which is represented by the trade discount offer.

Cost =

Reference: CFA® Program Curriculum, Volume 4, pp. 120‐121. 

 

 

 

2.  A company has trade credit with its suppliers and receives a discount if it pays within a specified number  of  days.  It works  out  the  cost  of  trade  credit  is  50%  if  it  pays  on  the  net  day.      If  the company’s short term cost of funds is 18% then the company should pay: 

A. on the net day.

B. on the day or purchase.

C. on the last day of the discount period

D. just after the discount period has ended.

Correct Answer:  C ......................................................................................... LOS: Reading  46‐d The discount offers a higher return for the company compared to its borrowing rate so it should take advantage of the discount offered and pay on the last day of the discount period. Losing the discount is more costly than delaying payment to the net day.

Reference: CFA® Program Curriculum, Volume 4, pp. 120‐121. 

%109=02.01

02.0+1

20/365

1 - -

Page 319: Cfa Ques Answer

Corporate Finance   303 

 

3.  Which of the following is least likely to be a pull on liquidity for a company?  

A.  paying vendors early.  

B.  low liquidity positions.  

C.  uncollected receivables. 

D.  limited short‐term lines of credit.  

 

 

 

4.  A  company  is offered  two projects with  the net  cash  flows,  in  $ million,  from  each project  as shown below. The cost of project A is $2 million and the cost of project B is $10 million. The cost of capital  for project A  is 10% and  for project B  is 12%. Which of  the projects should be accepted  for inclusion in the capital budget? 

End of year  Project A  Project B 

1  1.1  2.0 

2  0.8  4.0 

3  0.4  7.0 

A.  Both projects should be rejected. 

B.  Both projects should be accepted. 

C.  A should be accepted and B rejected.  

D.  B should be accepted and A rejected. 

Page 320: Cfa Ques Answer

304   Study Session 11:  

 

3.  Which of the following is least likely to be a pull on liquidity for a company?  

A. paying vendors early.

B. low liquidity positions.

C. uncollected receivables.

D. limited short-term lines of credit.

Correct Answer:  C ..........................................................................................LOS: Reading  46‐a A pull on liquidity refers to payments that are made too early or when credit availability is limited forcing the company to pay out funds before they receive money from sales or other sources. Uncollected receivables or not a pull, but a drag, on liquidity.

Reference: CFA® Program Curriculum, Volume 4, pp. 88‐89. 

 

 

4.  A  company  is offered  two projects with  the net  cash  flows,  in  $ million,  from  each project  as shown below. The cost of project A is $2 million and the cost of project B is $10 million. The cost of capital  for project A  is 10% and  for project B  is 12%. Which of  the projects should be accepted  for inclusion in the capital budget? 

End of year  Project A  Project B1  1.1  2.02  0.8  4.03  0.4  7.0

A. Both projects should be rejected.

B. Both projects should be accepted.

C. A should be accepted and B rejected.

D. B should be accepted and A rejected.

Correct Answer:  A ......................................................................................... LOS: Reading  44‐d The present value of the cash flows for project A, discounted at 10%, is $1.96 million (1.00 + 0.66 + 0.30) which is less than the cost, therefore the project should be rejected.

The present value of the cash flows for project B, discounted at 12%, is $9.96 million (1.79 + 3.19 + 4.98) which is less than the cost; therefore the project should be rejected.

Reference: CFA® Program Curriculum, Volume 4, pp. 12‐14. 

Page 321: Cfa Ques Answer

Corporate Finance   305 

 

5.  To enable a board to act in the best  long‐term interests of shareowners, it  is least appropriate  it possesses: 

A.  resources. 

B.  independence. 

C.  political affiliation. 

D.  experience and expertise. 

 

 

 

6.  North Company has a common stock price of $72. The  latest reported earnings per share were $4.80 and dividends per share were $1.60. The return on equity is 8%. The cost of equity is closest to: 

A.  5.33%. 

B.  7.46%. 

C.  7.55%. 

D.  7.67%. 

Page 322: Cfa Ques Answer

306   Study Session 11:  

 

5.  To enable a board to act in the best  long‐term interests of shareowners, it  is least appropriate  it possesses: 

A. resources.

B. independence.

C. political affiliation.

D. experience and expertise.

Correct Answer:  C ......................................................................................... LOS: Reading  48‐d The major factors that enable a board to act in the best long-term interests of shareowners are independence, experience and expertise, and the resources to support the independent work. Political affiliation may work in the short term but may well be detrimental in the long term.

Reference: CFA® Program Curriculum, Volume 4, pp. 164‐168. 

 

 

 

6.  North Company has a common stock price of $72. The  latest reported earnings per share were $4.80 and dividends per share were $1.60. The return on equity is 8%. The cost of equity is closest to: 

A. 5.33%.

B. 7.46%.

C. 7.55%.

D. 7.67%.

Correct Answer:  D......................................................................................... LOS: Reading  45‐h The growth rate is the earnings retention rate multiplied by the ROE, this is 5.33%.

Next year’s dividends will be $1.60 x 1.0533 = $1.685

%67.7=%33.5+72$685.1$

=g+PD

=k 1e

Reference: CFA® Program Curriculum, Volume 4, pp. 54‐55. 

Page 323: Cfa Ques Answer

Corporate Finance   307 

 

7.  A firm is looking at borrowing for two months and could issue $500,000 nominal of commercial paper at 7.5%.  Dealer’s commission would be 0.25% and backup line costs would be 0.3% based on the $500,000 issued.  The cost of borrowing is closest to: 

A.  7.50% 

B.  7.85% 

C.  8.05%. 

D.  8.15% 

 

 

 

8  The case that the board chair also holds the title of chief executive, from the corporate governance point of view, is: 

A.  unacceptable because it is universally prohibited in all jurisdictions. 

B.  acceptable, because combining the two positions will save costs and hence enhance shareholder value. 

C.  acceptable, because the effectiveness of the chief executive would be enhanced by his or her position as the chairperson of the board. 

D.  unacceptable because combining the two positions may reduce the ability and willingness of independent board members to exercise their independent judgment. 

Page 324: Cfa Ques Answer

308   Study Session 11:  

 

7.  A firm is looking at borrowing for two months and could issue $500,000 nominal of commercial paper at 7.5%.  Dealer’s commission would be 0.25% and backup line costs would be 0.3% based on the $500,000 issued.  The cost of borrowing is closest to: 

A. 7.50%

B. 7.85%

C. 8.05%.

D. 8.15%

Correct Answer:  D..........................................................................................LOS: Reading  46‐g cost = (interest + dealer’s commission + back-up costs)/net proceeds x 12

= [(0.075 + 0.0025 + 0.003) x $500,000 x 2/12] x 6 = 0.0805/0.9875 = 8.15%

$500,000 – (0.075 x $500,000 x 2/12)

Reference: CFA® Program Curriculum, Volume 4, pp. 126‐127. 

 

 

8  The case that the board chair also holds the title of chief executive, from the corporate governance point of view, is: 

A. unacceptable because it is universally prohibited in all jurisdictions.

B. acceptable, because combining the two positions will save costs and hence enhance shareholder value.

C. acceptable, because the effectiveness of the chief executive would be enhanced by his or her position as the chairperson of the board.

D. unacceptable because combining the two positions may reduce the ability and willingness of independent board members to exercise their independent judgment.

Correct Answer:  D..........................................................................................LOS: Reading  48‐c Choice A is incorrect because there are some jurisdictions that allow the combining of the two positions.

Choice B is incorrect, because the saving in cost is not the main issue from the corporate governance point of view.

Choice C is incorrect because it is not the effectiveness of the chief executive that is being questioned.

The main issue is whether the board’s independence would be compromised.

Reference: CFA® Program Curriculum, Volume 4, pp. 165‐166. 

Page 325: Cfa Ques Answer

Corporate Finance   309 

 

9  Which of the following ratios would be useful in evaluating a company’s internal liquidity? 

A.  Quick ratio. 

B.  Equity turnover. 

C.  Interest coverage. 

D.  Debt to equity ratio. 

 

 

 

10  Which  of  the  following  statements  is  most  accurate  concerning  the  different  methods  for evaluating projects? 

A.  The IRR method can give multiple answers if there are nonconventional cash flows. 

B.  If the profitability index is positive it indicates that a project should be accepted. 

C.  The IRR method is preferred since it assumes reinvestment at the project’s cost of capital. 

D.  The average accounting rate of return is attractive because it does not need a present value calculation.   

Page 326: Cfa Ques Answer

310   Study Session 11:  

 

9  Which of the following ratios would be useful in evaluating a company’s internal liquidity? 

A. Quick ratio.

B. Equity turnover.

C. Interest coverage.

D. Debt to equity ratio.

Correct Answer:  A ..........................................................................................LOS: Reading  46‐a Quick ratio = cash + marketable securities + receivables

current liabilities

The ratio measures the current assets that can be quickly liquidated to meet current liabilities.

Reference: CFA® Program Curriculum, Volume 4, pp. 89‐91. 

 

 

 

10  Which  of  the  following  statements  is  most  accurate  concerning  the  different  methods  for evaluating projects? 

A. The IRR method can give multiple answers if there are nonconventional cash flows.

B. If the profitability index is positive it indicates that a project should be accepted.

C. The IRR method is preferred since it assumes reinvestment at the project’s cost of capital.

D. The average accounting rate of return is attractive because it does not need a present value calculation.

Correct Answer:  A ..........................................................................................LOS: Reading  44‐e When a project has nonconventional cash flows (such as more than one cash inflow over the project’s life) the IRR method can give more than one solution, or multiple IRRs. A profitability index of over 1 means a project should be accepted, the IRR method discounts back at the IRR, andthe lack of adjustment for time value of money is a disadvantage of the AAR method.

Reference: CFA® Program Curriculum, Volume 4, pp. 25‐28. 

Page 327: Cfa Ques Answer

Corporate Finance   311 

 

11.  The following financial information is given for a company: 

Net profit margin   = 4% 

Operating profit margin  = 16% 

Equity turnover    = 3.2 

Total asset turnover  = 2.5 

The return on equity is closest to:  

 

A.  10.0%. 

B.  12.8%. 

C.  51.2%. 

D.  there is insufficient information given to calculate the return on equity. 

 

 

 

12.  In order to secure short‐term funding a company factors its accounts receivable.  This means that:  

A.  receivables are being used as collateral for a loan.  

B.  it pays over any receivables collected to the factor in return for short‐term funding. 

C.  sells the receivables to the factor who takes responsibility for collecting the receivables. 

D.  it takes out protection from the factor against its customers defaulting on payment of money owed. 

Page 328: Cfa Ques Answer

312   Study Session 11:  

 

11.  The following financial information is given for a company: 

Net profit margin = 4%

Operating profit margin = 16%

Equity turnover = 3.2

Total asset turnover = 2.5

The return on equity is closest to:

A. 10.0%.

B. 12.8%.

C. 51.2%.

D. there is insufficient information given to calculate the return on equity.

Correct Answer:  B...........................................................................................LOS: Reading  47‐a return on equity = net profit margin x sales turnover x financial leverage

= net profit margin x equity turnover

= 0.04 x 3.2

= 12.8%

Reference: CFA® Program Curriculum, Volume 4, pp. 136‐145. 

 

 

12.  In order to secure short‐term funding a company factors its accounts receivable.  This means that:  

A. receivables are being used as collateral for a loan.

B. it pays over any receivables collected to the factor in return for short-term funding.

C. sells the receivables to the factor who takes responsibility for collecting the receivables.

D. it takes out protection from the factor against its customers defaulting on payment of money owed.

Correct Answer:  C ..........................................................................................LOS: Reading  46‐g When a company factors its accounts receivables it sells its receivables to the factor and transfers credit granting and collection to the factor. This is different to when receivables are used as collateral for a loan which is an assignment of the accounts receivable.

Reference: CFA® Program Curriculum, Volume 4, pp. 125‐126. 

Page 329: Cfa Ques Answer

Corporate Finance   313 

 

13.  In proforma analysis which of the following items is usually considered to be a fixed rather than a sales‐driven burden? 

A.  tax rate. 

B.  depreciation.  

C.  selling expenses. 

D.  employees’ salaries. 

 

 

 

14.  Which one of the following is least likely to be considered good corporate governance of a listed company? 

A.  The board members’ actions and decisions represent the best interests of shareowners. 

B.  Appropriate controls and procedures are in place covering management’s activities in running the day‐to‐day operations of the company. 

C.  The board and its committees are structured to act independently from management and other parties that might influence the management. 

D.  All shareowners have the same right to participate in the governance of the company, with founding shareowners normally given the right to veto certain resolutions. 

Page 330: Cfa Ques Answer

314   Study Session 11:  

 

13.  In proforma analysis which of the following items is usually considered to be a fixed rather than a sales‐driven burden? 

A. tax rate.

B. depreciation.

C. selling expenses.

D. employees’ salaries.

Correct Answer:  A ......................................................................................... LOS: Reading  47‐b Income items with the exception of interest costs and tax are usually assumed to grow in line with sales. Unless there is a change in government tax rates the tax rate is assumed to be constant.

Reference: CFA® Program Curriculum, Volume 4, pp. 149‐153. 

 

 

 

14.  Which one of the following is least likely to be considered good corporate governance of a listed company? 

A. The board members’ actions and decisions represent the best interests of shareowners.

B. Appropriate controls and procedures are in place covering management’s activities in running the day-to-day operations of the company.

C. The board and its committees are structured to act independently from management and other parties that might influence the management.

D. All shareowners have the same right to participate in the governance of the company, with founding shareowners normally given the right to veto certain resolutions.

Correct Answer:  D..........................................................................................LOS: Reading  48‐g In a listed company, all shareowners should have equal rights to participate in the governance of the company. Differentiating between economic and voting rights should be avoided.

Reference: CFA® Program Curriculum, Volume 43, pp. 183‐186. 

Page 331: Cfa Ques Answer

Corporate Finance   315 

 

15.  Islington Corporation provides you with the following information:  

Income Statement 

$  million 

Balance Sheet 

Average over period                $ million 

Sales on credit  298        Cash  50  Accounts Payable  80 

COGS    (200)  Accounts Receivable 

60  Accrued Expenses  65 

Gross Profit  98  Inventory  100  Long‐term Debt   150 

SGA Expenses  (60)  Property, P & E  375     

Op. Profit  38  Depreciation  (85)  Common Stock    140 

Interest  expense        (6)      Retained Earnings  65 

Tax  (13)         

Net Income  19  Total Assets  500  Total Liabilities & Equity 

500 

 

The average receivables collection period for Islington Corporation is closest to: 

A.  1.4 days. 

B.  5.0 days. 

C.  73.5 days. 

D.  109.5 days. 

Page 332: Cfa Ques Answer

316   Study Session 11:  

 

15.  Islington Corporation provides you with the following information:  

Income Statement 

$  million 

Balance Sheet 

Average over period                $ million 

Sales on credit  298        Cash  50  Accounts Payable  80 

COGS    (200)  Accounts Receivable 

60  Accrued Expenses  65 

Gross Profit  98  Inventory  100  Long‐term Debt         150 

SGA Expenses  (60)  Property, P & E  375     

Op. Profit  38  Depreciation  (85)  Common Stock    140 

Interest  expense        (6)      Retained Earnings  65 

Tax  (13)         

Net Income  19  Total Assets  500  Total Liabilities & Equity 

500 

The average receivables collection period for Islington Corporation is closest to:

A. 1.4 days.

B. 5.0 days.

C. 73.5 days.

D. 109.5 days.

Correct Answer:  C  .........................................................................................LOS: Reading  46‐a Receivables turnover = credit sales/average receivables

= 298/60

= 4.966

Average receivables collection period = 365/receivables turnover

= 73.5 days

Reference: CFA® Program Curriculum, Volume 4, pp. 89‐91. 

Page 333: Cfa Ques Answer

Corporate Finance   317 

 

16.  A company provides the following information  

 

Credit sales   $125 million 

Cost of goods sold  $80 million 

Accounts receivable  $15 million 

Beginning Inventory  $16 million 

Ending Inventory  $22 million 

Accounts payable  $13 million 

 

The operating cycle and net operating cycle are closest to: 

  Operating cycle    Net operating cycle 

A.  108.2      48.9 

B.  108.2      53.1 

C.  144.2       84.9 

D.  144.2      89.0 

Page 334: Cfa Ques Answer

318   Study Session 11:  

 

16.  A company provides the following information  

Credit sales   $125 million 

Cost of goods sold  $80 million 

Accounts receivable  $15 million 

Beginning Inventory  $16 million 

Ending Inventory  $22 million 

Accounts payable  $13 million 

The operating cycle and net operating cycle are closest to:

Operating cycle Net operating cycle

A. 108.2 48.9

B. 108.2 53.1

C. 144.2 84.9

D. 144.2 89.0

Correct Answer:  D..........................................................................................LOS: Reading  46‐a Operating cycle

= number of days of receivables + number of days of inventory

= (15/125) x 365 + (22/80) x 365 = 43.8 + 100.38 = 144.18 days

Net operating cycle or cash conversion cycle

= number of days of receivables + number of days of inventory – number of days of payables

To calculate number of days of payables, we need to calculate purchases

= $80 million + ( $22 - $16) million = $86 million

= 144.18 days - (13/86) x 365 = 144.18 – 55.18 = 89.00 days

Reference: CFA® Program Curriculum, Volume 4, pp. 87‐95. 

Page 335: Cfa Ques Answer

Corporate Finance   319 

 

17.  Islington Corporation provides you with the following information:  

Income Statement 

$  million 

Balance Sheet 

Average over period                $ million 

Sales on credit  298  Cash  50  Accounts Payable  80 

COGS  (200)  Accounts Receivable 

60  Accrued Expenses  65 

Gross Profit  98  Inventory  100  Long‐term Debt  150 

SGA Expenses  (60)  Property, P & E  375     

Op. Profit  38  Depreciation  (85)  Common Stock  140 

Interest  expense  (6)      Retained Earnings  65 

Tax  (13)         

Net Income  19  Total Assets  500  Total Liabilities & Equity 

500 

The quick ratio is closest to: 

A.  0.34. 

B.  0.63. 

C.  0.76. 

D.  1.45. 

Page 336: Cfa Ques Answer

320   Study Session 11:  

 

17.  Islington Corporation provides you with the following information:  

Income Statement 

$  million 

Balance Sheet 

Average over period                $ million 

Sales on credit  298  Cash  50  Accounts Payable  80 

COGS  (200)  Accounts Receivable 

60  Accrued Expenses  65 

Gross Profit  98  Inventory  100  Long‐term Debt  150 

SGA Expenses  (60)  Property, P & E  375     

Op. Profit  38  Depreciation  (85)  Common Stock  140 

Interest  expense  (6)      Retained Earnings  65 

Tax  (13)         

Net Income  19  Total Assets  500  Total Liabilities & Equity 

500 

The quick ratio is closest to:

A. 0.34.

B. 0.63.

C. 0.76.

D. 1.45.

Correct Answer:  C ..........................................................................................LOS: Reading  46‐a Quick ratio = (cash + marketable securities + receivables)/current liabilities

= (50 + 60)/145

= 0.76

Reference: CFA® Program Curriculum, Volume 4, pp. 89‐90. 

Page 337: Cfa Ques Answer

Corporate Finance   321 

 

18.  National Telecoms Corporation provides the following information: 

Income Statement 2007 Balance Sheet end 2007

$ million Average over period $ million

Credit Sales  1050    Cash   90 Accounts Payable  50COGS  (780)    Accounts Receivable  170 Notes payable   125Gross Profit    270    Inventory  200 Long term debt  300SGA exp.   (150)    Property, P & E  650  

Op. Profit    120    Depreciation  (430) Common Stock  150Interest exp.  (45)      Retained Earnings  55Tax  (25)        

Net Income     50    Total Assets  680 Total Liabilities & Equity 

680

Dividends Paid  12 

Inventory level unchanged from 2006 levels. 

National Telecoms Corporation’s net operating cycle in 2007 is closest to:  

A.  58 days.  

B.  130 days. 

C.  153 days. 

D.  176 days.   

Page 338: Cfa Ques Answer

322   Study Session 11:  

 

18.  National Telecoms Corporation provides the following information: 

Income Statement 2007 Balance Sheet end 2007

$ million Average over period $ million

Credit Sales  1050    Cash   90 Accounts Payable  50COGS  (780)    Accounts Receivable  170 Notes payable   125Gross Profit    270    Inventory  200 Long term debt  300SGA exp.   (150)    Property, P & E  650  

Op. Profit    120    Depreciation  (430) Common Stock  150Interest exp.  (45)      Retained Earnings  55Tax  (25)        

Net Income     50    Total Assets  680 Total Liabilities & Equity 

680

Dividends Paid  12 

Inventory level unchanged from 2006 levels. 

National Telecoms Corporation’s net operating cycle in 2007 is closest to:  

A. 58 days.

B. 130 days.

C. 153 days.

D. 176 days.

Correct Answer:  B.......................................................................................... LOS: Reading  46‐b number of days of receivables = 365/annual receivables turnover =365/(1050/170) = 59 days

number of days of inventory = 365/inventory turnover = 365/(780/200) = 94 days

= number of days of receivables + number of days of inventory = 153 days

Reference: CFA® Program Curriculum, Volume 4, pp. 89‐95. 

Page 339: Cfa Ques Answer

Corporate Finance   323 

 

19.  National Telecoms Corporation provides the following information: 

Income Statement 

$ million Credit Sales 1050 COGS (780) Gross Profit 270 SGA exp. (150) Op. Profit 120 Interest exp. (45) Tax (25) Net Income 50

It  is assumed  that  sales and  related  costs are growing at 5%per annum and  there  is no  change  in capital structure, then National Telecom Corporation’s proforma net income in 2008 is closest to: 

A.  $ 51.25 million. 

B.  $ 52.50 million. 

C.  $ 54.00 million. 

D.  $ 56.00 million. 

Page 340: Cfa Ques Answer

324   Study Session 11:  

 

19.  National Telecoms Corporation provides the following information: 

Income Statement

$ million Credit Sales 1050 COGS (780) Gross Profit 270 SGA exp. (150) Op. Profit 120 Interest exp. (45) Tax (25) Net Income 50

It is assumed that sales and related costs are growing at 5%per annum and there is no change in capital structure, then National Telecom Corporation’s proforma net income in 2008 is closest to:

A. $ 51.25 million.

B. $ 52.50 million.

C. $ 54.00 million.

D. $ 56.00 million.

Correct Answer:  C ......................................................................................... LOS: Reading  47‐b Operating profit will grow at 5% to $126 million, interest expense is assumed constant at $45 million and the tax ate is 33.33% so tax will be $81 million x 33.33% = $27 million, so profoma net income will be $54 million.

Reference: CFA® Program Curriculum, Volume 4, pp. 164‐167. 

Page 341: Cfa Ques Answer

Corporate Finance   325 

 

20.  The  cost  of  a project  is  $150 million  and  the  following  cash  flows  are  anticipated,  the  cost  of capital is 10%. 

Year  Net Cash Flow ($ million) 

0  ‐150 

1   25 

2   50 

3   55 

4   40 

5   60 

Total   

 

The implied decision to accept or reject the project, and the Profitability Index (PI) is closest to:  

  Accept/reject    PI 

A.  Reject       0.13 

B.  Reject      0.53 

C.  Accept      1.13 

D.  Accept      1.70 

Page 342: Cfa Ques Answer

326   Study Session 11:  

 

20.  The  cost  of  a project  is  $150 million  and  the  following  cash  flows  are  anticipated,  the  cost  of capital is 10%. 

Year  Net Cash Flow ($ million)

0  ‐150 1   25 2   50 3   55 4   40 5   60 

Total   

The implied decision to accept or reject the project, and the Profitability Index (PI) is closest to:

Accept/reject PI

A. Reject 0.13

B. Reject 0.53

C. Accept 1.13

D. Accept 1.70

Correct Answer:  C ......................................................................................... LOS: Reading  44‐d First of all calculate the present values of the cash flows:

 

Year  Net Cash Flow ($ million) 

Discounted Net Cash Flow ($ million) 

0  ‐150 ‐150.01   25 22.72   50 41.33   55 41.34   40 27.35   60 37.3  

Total    19.9

Reference: CFA® Program Curriculum, Volume 4, p. 19. 

Page 343: Cfa Ques Answer

Corporate Finance   327 

 

21.  The cost of a project is $150 million and it will be depreciated using the straight line method over 5 years with a zero salvage value.  The following net income is anticipated.  

Year  Net Income ($ million)

1  4 2  20 3  22 4  20 5  – 5 

 

The average accounting rate of return (AAR) of the project is closest to 

A.  4.1% 

B.  8.1% 

C.  12.2% 

D.  16.3% 

 

 

 

22.  A  company  prohibits  itself  from  offering  shares  at  discounted  prices  to management,  board members and other insiders prior to a public offering of its securities. This practice is: 

A.  preferred by the tax office because it avoids imputation of income taxes for the executives. 

B.  preferred by investors because it demonstrates that the company aligns itself with the investors’ interests. 

C.  not preferred by the capital markets regulatory body because it might encourage an opportunity for insiders’ trading. 

D.  not preferred from a corporate governance point of view because it might encourage the executives to give compensation to themselves from short‐term share transactions. 

Page 344: Cfa Ques Answer

328   Study Session 11:  

 

21  The cost of a project is $150 million and it will be depreciated using the straight line method over 5 years with a zero salvage value.  The following net income is anticipated.  

Year  Net Income ($ million) 

1  4 2  20 3  22 4  20 5  – 5 

The average accounting rate of return (AAR) of the project is closest to

A. 4.1%

B. 8.1%

C. 12.2%

D. 16.3%

Correct Answer:  D......................................................................................... LOS: Reading  44‐d Average book value is ($150 million + $0million)/2 = $75 million

Average net income is $(4 + 20 + 22 + 20 – 5)million/5 = $12.2 million

AAR = Average net income = $12.2 million

Average book value $75 million

= 16.3%

Reference: CFA® Program Curriculum, Volume 4, p. 18. 

Page 345: Cfa Ques Answer

Corporate Finance   329 

 

22.  A  company  prohibits  itself  from  offering  shares  at  discounted  prices  to management,  board members and other insiders prior to a public offering of its securities. This practice is: 

A. preferred by the tax office because it avoids imputation of income taxes for the executives.

B. preferred by investors because it demonstrates that the company aligns itself with the investors’ interests.

C. not preferred by the capital markets regulatory body because it might encourage an opportunity for insiders’ trading.

D. not preferred from a corporate governance point of view because it might encourage the executives to give compensation to themselves from short-term share transactions.

Correct Answer:  B ..........................................................................................LOS: Reading  48‐g This is a preferred practice from a corporate governance point of view, which indicates it is beneficial for the long-term interests of the investors.

Reference: CFA® Program Curriculum, Volume 4, pp. 173‐175. 

 

Page 346: Cfa Ques Answer

330   Study Session 12:  

 

Study Session 12: Portfolio Management:  

As  the  first  discussion within  the  CFA  curriculum  on  portfolio management,  this  study  session provides the critical framework and context for subsequent Level I study sessions covering equities, fixed income, derivatives, and alternative investments. Furthermore, this study session provides the underlying theories and tools for portfolio management at Levels II and III. 

The first reading discusses the asset allocation decision and the portfolio management process—they are  an  integrated  set  of  steps  undertaken  in  a  consistent  manner  to  create  and  maintain  an appropriate portfolio (combination of assets) to meet clients’ stated goals. The last two readings focus on the design of a portfolio and introduces the capital asset pricing model (CAPM), a centerpiece of modern financial economics that relates the risk of an asset to its expected return. 

 

Reading 49: The Asset Allocation Decision 

Reading 50: An Introduction to Portfolio Management 

Reading 51: An Introduction to Asset Pricing Models 

 

 

Page 347: Cfa Ques Answer

Portfolio Management   331 

 

1.  The  asset  allocation of  investors between  equities  and  fixed  income  in  the major  international capital markets is: 

A.  very similar, since investors are using the same optimization models. 

B.  quite different, since investors are making decisions in different economic and social environments. 

C.  very similar, since asset allocation is the most important step in portfolio construction in all major markets.  

D.  very similar, since long term equity and fixed income returns have been consistent across different markets. 

 

 

 

2.  The beta of an asset: 

A.  lies between ‐1 and 1. 

B.  is a measure of unsystematic risk. 

C.  is 1 if the asset is the market portfolio. 

D.  is the covariance of the asset with the market. 

 

Page 348: Cfa Ques Answer

332   Study Session 12:  

 

1.  The  asset  allocation of  investors between  equities  and  fixed  income  in  the major  international capital markets is: 

A. very similar, since investors are using the same optimization models.

B. quite different, since investors are making decisions in different economic and social environments.

C. very similar, since asset allocation is the most important step in portfolio construction in all major markets.

D. very similar, since long term equity and fixed income returns have been consistent across different markets.

Correct Answer:  B...........................................................................................LOS: Reading  49‐e The different social and economic environments, in addition to political and tax issues, have led to different weightings in equities and fixed income in portfolios in different countries.

Reference: CFA® Program Curriculum, Volume 4, pp. 223‐224. 

 

 

 

2.  The beta of an asset: 

A. lies between -1 and 1.

B. is a measure of unsystematic risk.

C. is 1 if the asset is the market portfolio.

D. is the covariance of the asset with the market.

Correct Answer:  C ......................................................................................... LOS: Reading  51‐d A. is not correct since beta does not lie in any fixed range.

B. is not correct, it is a measure of systematic risk.

D. is not correct because beta is the covariance divided by the variance of the market.

By definition the beta of the market itself is one.

Reference: CFA® Program Curriculum, Volume 4, pp. 264‐269. 

Page 349: Cfa Ques Answer

Portfolio Management   333 

 

3.  Which of the following is least likely to be a step in the investment process? 

A.  Asset allocation. 

B.  Monitoring the portfolio. 

C.  Forecasting market returns. 

D.  Submitting regulatory reports to the appropriate authority.  

 

 

 

4.  The stock analyst  in your  firm  recommends  that you buy shares  in Mayfair Corp. The current share price is $26 and she forecasts that a year from now the share price will have risen to $30. There is no dividend payment  expected. You note  that  the beta of  the  stock  is  0.8,  the  expected market return over  the next year  is 15% and  the risk‐free rate  is 5%. On  the basis of  the analyst’s  forecast, Mayfair Corp.’s stock is: 

A.  overvalued. 

B.  undervalued. 

C.  correctly valued. 

D.  the question needs to provide data on the market risk premium to be able to decide whether the stock is fairly valued. 

Page 350: Cfa Ques Answer

334   Study Session 12:  

 

3.  Which of the following is least likely to be a step in the investment process? 

A. Asset allocation.

B. Monitoring the portfolio.

C. Forecasting market returns.

D. Submitting regulatory reports to the appropriate authority.

Correct Answer:  D..........................................................................................LOS: Reading  49‐a The portfolio management process consists of 4 steps: construct the policy statement, forecast future economic and market trends, construct the portfolio and continually monitor and evaluate performance. Submitting regulatory reports is not part of the investment decision-making process so D is the best answer.

Reference: CFA® Program Curriculum, Volume 4, pp. 202‐203. 

 

 

 

4.  The stock analyst  in your  firm  recommends  that you buy shares  in Mayfair Corp. The current share price is $26 and she forecasts that a year from now the share price will have risen to $30. There is no dividend payment  expected. You note  that  the beta of  the  stock  is  0.8,  the  expected market return over  the next year  is 15% and  the risk‐free rate  is 5%. On  the basis of  the analyst’s  forecast, Mayfair Corp.’s stock is: 

A. overvalued.

B. undervalued.

C. correctly valued.

D. the question needs to provide data on the market risk premium to be able to decide whether the stock is fairly valued.

Correct Answer:  B...........................................................................................LOS: Reading  51‐e Using CAPM the estimated return is:

( )[ ] ( ) %13%5%158.0%5RRERR fmfx =−+=−β+= The analyst is forecasting a return of 15.4% so the stock looks undervalued.

Reference: CFA® Program Curriculum, Volume 4, pp. 263‐266. 

Page 351: Cfa Ques Answer

Portfolio Management   335 

 

5.  If a stock lies above the security market line (SML) this would indicate that the stock: 

A.  is overvalued. 

B.  is undervalued. 

C.  has a higher expected return than the market. 

D.  has a lower expected return than the market. 

 

 

 

6.  The characteristic line is used to estimate: 

A.  the risk‐free rate. 

B.  the beta of a stock. 

C.  the risk aversion of an investor. 

D.  the standard deviation of a portfolio. 

Page 352: Cfa Ques Answer

336   Study Session 12:  

 

5.    If a stock lies above the security market line (SML) this would indicate that the stock: 

A. is overvalued.

B. is undervalued.

C. has a higher expected return than the market.

D. has a lower expected return than the market.

Correct Answer:  B...........................................................................................LOS: Reading  51‐e The stock is undervalued because the expected rate of return is higher than the required rate of return to compensate for its beta risk.

Reference: CFA® Program Curriculum, Volume 4, pp. 263‐267. 

 

 

 

6.  The characteristic line is used to estimate: 

A. the risk-free rate.

B. the beta of a stock.

C. the risk aversion of an investor.

D. the standard deviation of a portfolio.

Correct Answer:  B.......................................................................................... LOS: Reading  51‐d The characteristic line is the regression line of best fit through a scatter diagram of points representing a stock’s return against the market return; the slope gives the stock beta.

Reference: CFA® Program Curriculum, Volume 4, pp. 267‐269. 

Page 353: Cfa Ques Answer

Portfolio Management   337 

 

7.  A portfolio is 70% invested in an index fund and 30% in a risk‐free asset. The index fund has a variance of returns of 0.0027, the variance for the total portfolio is closest to: 

A.  0.0013. 

B.  0.0019. 

C.  0.0027. 

D.  0.0039. 

 

 

 

8.  In capital market theory the Market Portfolio can be least accurately described as: 

A.  the portfolio where systematic risk has been completely diversified away. 

B.  it is the point where the Capital Market Line touches the efficient frontier. 

C.  the portfolio which contains all risky assets in proportion to their market value. 

D.  the point where the tangent from the risk‐free rate touches the efficient frontier. 

Page 354: Cfa Ques Answer

338   Study Session 12:  

 

7.  A portfolio is 70% invested in an index fund and 30% in a risk‐free asset. The index fund has a variance of returns of 0.0027, the variance for the total portfolio is closest to: 

A. 0.0013.

B. 0.0019.

C. 0.0027.

D. 0.0039.

Correct Answer:  A ..........................................................................................LOS: Reading  51‐a The variance and standard deviation of the risk-free asset are zero. Therefore the variance of the portfolio is:

( )0013.0

0027.07.0

wwr2ww2

21211222

22

21

21

2port

==

σσ+σ+σ=σ

Reference: CFA® Program Curriculum, Volume 4, pp. 256‐258. 

 

 

 

8.  In capital market theory the Market Portfolio can be least accurately described as: 

A. the portfolio where systematic risk has been completely diversified away.

B. it is the point where the Capital Market Line touches the efficient frontier.

C. the portfolio which contains all risky assets in proportion to their market value.

D. the point where the tangent from the risk-free rate touches the efficient frontier.

Correct Answer:  A ......................................................................................... LOS: Reading  51‐b A is not correct – it is the unsystematic risk that can be diversified away.

Reference: CFA® Program Curriculum, Volume 4, p. 259. 

Page 355: Cfa Ques Answer

Portfolio Management   339 

 

9.  If an investor has steep utility curves it is likely to indicates that: 

A.  the investor is aggressive. 

B.  the investor is conservative. 

C.  the investor has a long time horizon. 

D.  the investor has a short time horizon. 

 

 

 

10.  A U.S. based investment manager is concerned that the volatility of an international equity fund he is managing is too high. Which of the following would be an appropriate course of action? 

A.  Increase the weighting in high beta stocks. 

B.  Sell the international holdings and only hold U.S. stocks. 

C.  Consider investing in markets which have a low correlation with the existing assets in the portfolio. 

D.  Sell small capitalization stocks and concentrate the portfolio in a small number of big market capitalization stocks. 

Page 356: Cfa Ques Answer

340   Study Session 12:  

 

9.  If an investor has steep utility curves it is likely to indicates that: 

A. the investor is aggressive.

B. the investor is conservative.

C. the investor has a long time horizon.

D. the investor has a short time horizon.

Correct Answer:  B...........................................................................................LOS: Reading  50‐g A steep utility curve shows that the investor needs to be compensated for taking a small amount of additional risk by receiving a significantly higher return, indicating he is conservative with a low appetite for risk.

Reference: CFA® Program Curriculum, Volume 4, pp. 248‐249. 

 

 

 

10.  A U.S. based investment manager is concerned that the volatility of an international equity fund he is managing is too high. Which of the following would be an appropriate course of action? 

A. Increase the weighting in high beta stocks.

B. Sell the international holdings and only hold U.S. stocks.

C. Consider investing in markets which have a low correlation with the existing assets in the portfolio.

D. Sell small capitalization stocks and concentrate the portfolio in a small number of big market capitalization stocks.

Correct Answer:  C .......................................................................................... LOS: Reading  50‐f In order to reduce volatility, which is the standard deviation of returns, the manager could include assets which have a low correlation with the existing assets in the portfolio.

Reference: CFA® Program Curriculum, Volume 4, pp. 256‐259. 

Page 357: Cfa Ques Answer

Portfolio Management   341 

 

11.  Which of the following is a least accurate description of an assumption of Capital Market Theory? 

A.  Capital markets are in equilibrium.  

B.  All investors have the same time horizon. 

C.  Investors can borrow or lend at the risk‐free rate. 

D.  Investors are not able to correctly anticipate inflation. 

 

 

 

12.  The correlation coefficient between the returns of two assets is 0.6 and the standard deviations of returns of the two assets are 7% and 12%. The covariance of returns is closest to: 

A.  14.0. 

B.  50.4. 

C.  71.4. 

D.  140.0. 

Page 358: Cfa Ques Answer

342   Study Session 12:  

 

11.  Which of the following is a least accurate description of an assumption of Capital Market Theory? 

A. Capital markets are in equilibrium.

B. All investors have the same time horizon.

C. Investors can borrow or lend at the risk-free rate.

D. Investors are not able to correctly anticipate inflation.

Correct Answer:  D..........................................................................................LOS: Reading  51‐a Capital Market Theory makes the assumption that there is no inflation or change in interest rates, or any inflation is fully anticipated.

Reference: CFA® Program Curriculum, Volume 4, pp. 254‐255. 

 

 

12.  The correlation coefficient between the returns of two assets is 0.6 and the standard deviations of returns of the two assets are 7% and 12%. The covariance of returns is closest to: 

A. 14.0.

B. 50.4.

C. 71.4.

D. 140.0.

Correct Answer:  B.......................................................................................... LOS: Reading  50‐d 

4.501276.0ariancecov

ariancecovr

yx

xyxy

=××=

σσ=

where:

rxy = correlation between the returns of x and y

xσ = standard deviation of returns of x

yσ = standard deviation of returns of y

Reference: CFA® Program Curriculum, Volume 4, pp. 236‐237. 

Page 359: Cfa Ques Answer

Portfolio Management   343 

 

13.   The efficient frontier represents portfolios that: 

A.  offer the highest return for a given level of total risk. 

B.  offer the highest return for a given level of systematic risk. 

C.  are equally attractive to an investor with a specified level of total risk tolerance. 

D.  are equally attractive to an investor with a specified level of systematic risk tolerance. 

 

 

14.  The following data is provided on the expected return of an asset under different scenarios: 

Probability   Return

0.20  12% 

0.60  15% 

0.20  18% 

The standard deviation of the returns is closest to: 

A.  1.2%. 

B.  1.9%. 

C.  2.3%. 

D.  3.6%. 

Page 360: Cfa Ques Answer

344   Study Session 12:  

 

13.   The efficient frontier represents portfolios that: 

A. offer the highest return for a given level of total risk.

B. offer the highest return for a given level of systematic risk.

C. are equally attractive to an investor with a specified level of total risk tolerance.

D. are equally attractive to an investor with a specified level of systematic risk tolerance.

Correct Answer:  A .......................................................................................... LOS: Reading  50‐f The efficient frontier represents the portfolios which offer the highest return for any given level of risk so A is correct.

Reference: CFA® Program Curriculum, Volume 4, pp. 247‐249. 

 

 

14.  The following data is provided on the expected return of an asset under different scenarios: 

Probability   Return 

0.20  12% 

0.60  15% 

0.20  18% 

The standard deviation of the returns is closest to:

A. 1.2%.

B. 1.9%.

C. 2.3%.

D. 3.6%.

Correct Answer:  B...........................................................................................LOS: Reading  50‐c The expected return is (0.20 x 12%) + (0.60 x 15%) + (0.20 x 18%) = 15%.

The standard deviation is given by �, where

( ) ( )1.9%or 01897.0

00036.015.018.02.015.012.02.0 222

=σ=−+−=σ

Reference: CFA® Program Curriculum, Volume 4, pp. 230‐231. 

Page 361: Cfa Ques Answer

Portfolio Management   345 

 

15.  The expected return from a market is 12% and the risk‐free rate is 5% and a stock has a beta of 0.5. The required rate of return from the stock is: 

A.  6.0%. 

B.  8.5%. 

C.  11.0%. 

D.  14.5%. 

 

 

 

16.  The beta of a stock can be computed from the:  

A.  relative volatility of the stock returns to the market returns. 

B.  correlation of the stock with the market and the market return. 

C.  correlation of the stock with the market and the variance of the stock returns. 

D.  covariance of the stock with the market and the variance of the market returns. 

Page 362: Cfa Ques Answer

346   Study Session 12:  

 

15.  The expected return from a market is 12% and the risk‐free rate is 5% and a stock has a beta of 0.5. The required rate of return from the stock is: 

A. 6.0%.

B. 8.5%.

C. 11.0%.

D. 14.5%.

Correct Answer:  B.......................................................................................... LOS: Reading  51‐d Using CAPM we can calculate the required return from the stock:

( )[ ] ( ) %5.8%5%125.0%5RRERR fmfx =−+=−β+= .

where:

Rf = the risk-free rate

RM = market return

�i = beta of stock

Reference: CFA® Program Curriculum, Volume 4, pp. 263‐267. 

 

 

 

16.  The beta of a stock can be computed from the:  

A. relative volatility of the stock returns to the market returns.

B. correlation of the stock with the market and the market return.

C. correlation of the stock with the market and the variance of the stock returns.

D. covariance of the stock with the market and the variance of the market returns.

Correct Answer:  D......................................................................................... LOS: Reading  51‐d The beta of an asset is the covariance of the assets returns with market returns divided by the variance of the market returns.

Reference: CFA® Program Curriculum, Volume 4, pp. 267 ‐221. 

Page 363: Cfa Ques Answer

Portfolio Management   347 

 

17.  In  a  rapidly  rising  market,  a  stock  with  a  beta  of  0.5  is  expected,  relative  to  the  market performance, to: 

  Performance      Direction  

A.  outperform       Same 

B.  outperform       Opposite 

C.  underperform       Same 

D.  underperform       Opposite 

 

 

 

18.  The characteristic line: 

A.  is a regression line used to estimate a stock’s systematic risk. 

B.  is a regression line used to estimate a stock’s standard deviation. 

C.  indicates the required rate of return of a stock given its systematic risk. 

D.  indicates the required rate of return of a stock given its standard deviation. 

Page 364: Cfa Ques Answer

348   Study Session 12:  

 

17.  In  a  rapidly  rising  market,  a  stock  with  a  beta  of  0.5  is  expected,  relative  to  the  market performance, to: 

Performance Direction

A. outperform Same

B. outperform Opposite

C. underperform Same

D. underperform Opposite

Correct Answer:  C ......................................................................................... LOS: Reading  51‐d A low beta means the stock will move by less than the market if the market moves upwards by more than the risk-free rate. The positive beta means it will move in the same direction as the market.

Reference: CFA® Program Curriculum, Volume 4, pp. 263‐266. 

 

 

 

18.  The characteristic line: 

A. is a regression line used to estimate a stock’s systematic risk.

B. is a regression line used to estimate a stock’s standard deviation.

C. indicates the required rate of return of a stock given its systematic risk.

D. indicates the required rate of return of a stock given its standard deviation.

Correct Answer:  A ......................................................................................... LOS: Reading  51‐d Systematic risk is often calculated by using regression analysis to examine the return of an asset against the return of the market. The slope of the line is the beta which measures systematic risk.

Reference: CFA® Program Curriculum, Volume 4, pp. 267‐271. 

Page 365: Cfa Ques Answer

Portfolio Management   349 

 

19.  An investor policy statement is important because it: 

A.  clarifies the investors’ objectives. 

B.  provides the investor with the portfolio manager’s market outlook. 

C.  provides details on the stocks that will be purchased for the portfolio. 

D.  means that the investor takes responsibility for investment performance. 

 

 

 

20.  The optimal portfolio for an investor is represented by the point where the: 

A.  investor utility curves intersect each other. 

B.  security market line is tangent to the efficient frontier. 

C.  investor utility curve is tangent to the efficient frontier. 

D.  investor utility curve is tangent to the security market line. 

Page 366: Cfa Ques Answer

350   Study Session 12:  

 

19.  An investor policy statement is important because it: 

A. clarifies the investors’ objectives.

B. provides the investor with the portfolio manager’s market outlook.

C. provides details on the stocks that will be purchased for the portfolio.

D. means that the investor takes responsibility for investment performance.

Correct Answer:  A ......................................................................................... LOS: Reading  49‐b A policy statement helps the investor specify realistic goals and in setting the goals become more aware of the risks of investing. It is a valuable way of communicating with the portfolio manager. It also helps set the benchmark against which performance can be measured.

Reference: CFA® Program Curriculum, Volume 4, pp. 203‐206. 

 

 

 

20.  The optimal portfolio for an investor is represented by the point where the: 

A. investor utility curves intersect each other.

B. security market line is tangent to the efficient frontier.

C. investor utility curve is tangent to the efficient frontier.

D. investor utility curve is tangent to the security market line.

Correct Answer:  C ..........................................................................................LOS: Reading  50‐g The utility curves represent the trade-off between risk and return. The optimal portfolio will be where the highest utility curve touches the efficient frontier.

Reference: CFA® Program Curriculum, Volume 4, pp. 238‐249. 

Page 367: Cfa Ques Answer

Portfolio Management   351 

 

21.  A portfolio is invested equally between two assets, the assets have standard deviations of 4% and 8%,  and  the  correlation  between  the  two  assets  is  0.3.    The  standard  deviation  of  the  combined portfolio is closest to: 

 

A.  4.98%. 

B.  6.00%. 

C.  6.64%. 

D.  24.80%. 

 

 

 

22.  Two assets have zero correlation.  If a portfolio is invested with 30% in the first asset that has a variance of 12, and 70%  in  the second asset  that has a variance of 8,  the variance of  the combined portfolio is closest to:  

 

A.  2.2.   

B.  5.0. 

C.  6.7. 

D.  9.2. 

Page 368: Cfa Ques Answer

352   Study Session 12:  

 

21.  A portfolio is invested equally between two assets, the assets have standard deviations of 4% and 8%,  and  the  correlation  between  the  two  assets  is  0.3.    The  standard  deviation  of  the  combined portfolio is closest to: 

A. 4.98%.

B. 6.00%.

C. 6.64%.

D. 24.80%.

Correct Answer:  A ..........................................................................................LOS: Reading  50‐c 

( ) ( ) ( ) ( ) ( )( )( )( )( )

0498.000248.0

00048.00016.00004.008.05.004.05.03.0208.05.004.05.0

wwr2ww2222

21211222

22

21

21

2port

=σ=

++=++=

σσ+σ+σ=σ

where:

iσ = standard deviation of returns of asset i

jσ = standard deviation of returns of asset j

wi = weighting of asset i in the portfolio

rij = correlation between the returns of assets i and j

Reference: CFA® Program Curriculum, Volume 4, pp. 238‐241. 

Page 369: Cfa Ques Answer

Portfolio Management   353 

 

22.  Two assets have zero correlation.  If a portfolio is invested with 30% in the first asset that has a variance of 12, and 70%  in  the second asset  that has a variance of 8,  the variance of  the combined portfolio is closest to:  

A. 2.2.

B. 5.0.

C. 6.7.

D. 9.2.

Correct Answer:  B .......................................................................................... LOS: Reading  50‐c Variance is given by:

( ) ( )

592.308.1

87.0123.0

wwr2ww22

21211222

22

21

21

2port

=+=

+=

σσ+σ+σ=σ

where:

iσ = standard deviation of returns of asset i

wi = weighting of asset i in the portfolio

rij = correlation between the returns of assets i and j

Reference: CFA® Program Curriculum, Volume 4, pp. 238‐241. 

 

Page 370: Cfa Ques Answer

354   Study Session 13:  

 

Study Session 13: Equity Investments: Securities Markets

 

This  study  session  addresses  how  securities  are  bought  and  sold  and  what  constitutes  a  well‐functioning securities market. The reading on market indexes gives an understanding of how indexes are constructed and calculated and the biases inherent in each of the weighting schemes used. 

Some  of  the most  interesting  and  important work  in  the  investment  field during  the past  several decades revolves around the efficient market hypothesis (EMH) and its implications for active versus passive equity portfolio management. The readings on this subject provide an understanding of the EMH and  the  seemingly persistent anomalies  to  the  theory, an understanding  that  is necessary  to judge the value of fundamental or technical security analysis.  

 

Reading 52: Organization and Functioning of Securities Markets 

Reading 53: Security‐Market Indexes 

Reading 54: Efficient Capital Markets 

Reading 55: Market Efficiency and Anomalies 

Page 371: Cfa Ques Answer

Equity Investments: Securities Markets   355 

 

1.  A corporation looking to raise funds may decide to do a private placement because: 

A.  it will reduce issuing costs. 

B.  it will avoid using an investment bank. 

C.  it will provide greater liquidity in the secondary market. 

D.  the issue can be sold at a higher price than if it was sold in a public offering. 

 

 

 

2.  When a stock in the Dow Jones Industrial Average has a stock split this will lead to: 

A.  the divisor increasing. 

B.  the divisor decreasing. 

C.  there being no change in the divisor. 

D.  the divisor increasing if the stock is a high‐priced stock and falling if it is a low‐priced stock. 

 

Page 372: Cfa Ques Answer

356   Study Session 13:  

 

1.  A corporation looking to raise funds may decide to do a private placement because: 

A. it will reduce issuing costs.

B. it will avoid using an investment bank.

C. it will provide greater liquidity in the secondary market.

D. the issue can be sold at a higher price than if it was sold in a public offering.

Correct Answer:  A ......................................................................................... LOS: Reading  52‐b A private placement will have lower issue costs, mainly since the requirement for lengthy registration documentation is reduced. However the pricing will be lower than for a public offer since the purchasers will need to be compensated for the lack of liquidity in the secondary market.

Reference: CFA® Program Curriculum, Volume 5, p. 9. 

 

 

 

2.  When a stock in the Dow Jones Industrial Average has a stock split this will lead to: 

A. the divisor increasing.

B. the divisor decreasing.

C. there being no change in the divisor.

D. the divisor increasing if the stock is a high-priced stock and falling if it is a low-priced stock.

Correct Answer:  B...........................................................................................LOS: Reading  53‐a When there is a stock split the stock price will fall, there is no immediate impact on the index so the divisor must also decrease.

Reference: CFA® Program Curriculum, Volume 5, pp. 42‐42. 

Page 373: Cfa Ques Answer

Equity Investments: Securities Markets   357 

 

3.  A short seller of a stock will generally: 

A.  sell the stock when the price is falling. 

B.  benefit from a rise in price of the stock. 

C.  deposit collateral when he borrows stock. 

D.  benefit from the price fall when a stock goes ex‐dividend. 

 

 

 

4.  There are two stocks ABC and XYZ included in an unweighted index and the following data is provided: 

 

Stock  Number of shares  Price at end  Price at end 

ABC  10,000  $25.00  $30.00 XYZ  50,000  $35.00  $36.75 

 

If the index is computed using geometric averages, the increase in the index over 2007 is closest to: 

A.  8.33%. 

B.  11.25%. 

C.  12.25%. 

D.  12.50%. 

Page 374: Cfa Ques Answer

358   Study Session 13:  

 

3.  A short seller of a stock will generally: 

A. sell the stock when the price is falling.

B. benefit from a rise in price of the stock.

C. deposit collateral when he borrows stock.

D. benefit from the price fall when a stock goes ex-dividend.

Correct Answer:  C .......................................................................................... LOS: Reading  52‐f A is not correct since he must sell on an uptick trade, B is not correct since he will benefit when the price falls and he can buy back the stock at a cheaper price. D is not correct since he must pay the dividends to the lender of the stock. Usually the lender of stock will require collateral as protection against the borrower failing to return the stock.

Reference: CFA® Program Curriculum, Volume 5, pp. 25‐26. 

 

 

 

4.  There are two stocks ABC and XYZ included in an unweighted index and the following data is provided: 

Stock  Number of shares  Price at end  Price at end 

ABC  10,000  $25.00  $30.00 XYZ  50,000  $35.00  $36.75 

If the index is computed using geometric averages, the increase in the index over 2007 is closest to:

A. 8.33%.

B. 11.25%.

C. 12.25%.

D. 12.50%.

Correct Answer:  C ..........................................................................................LOS: Reading  53‐a An unweighted index is computed on the basis that an equal dollar amount is invested in each of stocks ABC and XYZ. ABC rose by 20% and XYZ by 5% so the index performance is given by (1.20 x 1.05)1/2 – 1 = 0.1225 = 12.25%.

Reference: CFA® Program Curriculum, Volume 5, pp. 44‐46. 

Page 375: Cfa Ques Answer

Equity Investments: Securities Markets   359 

 

5.  An investor calls a broker on the first day of the month to find out the price of ABC Inc.’s shares and is quoted $103 bid – 103½ ask. The shares are very liquid. The share price then moves to $98 – 98½ before rising to $115 –115¾ at the end of the month. If the investor had (i) placed a market order to buy the shares on the first day and then sold them on the last day (ii) a limit order to buy at $100 and sell at the end of the month, his profit before transaction costs would be closest to: 

A.  (i) 11.11% (ii) 15.00%. 

B.  (i) 11.11% (ii) 15.75%. 

C.  (i) 12.39% (ii) 15.00%. 

D.  (i) 12.39% (ii) 15.75%. 

 

 

 

6.  An informationally efficient market is one where: 

A.  information is available on a timely basis to all investors. 

B.  investors allocate their funds to the companies that can make the best use of them. 

C.  security prices adjust slowly to the arrival of new information giving investors time to take advantage of positive news. 

D.  security prices adjust rapidly to the arrival of new information and therefore security prices reflect all information about the security. 

Page 376: Cfa Ques Answer

360   Study Session 13:  

 

5.  An investor calls a broker on the first day of the month to find out the price of ABC Inc.’s shares and is quoted $103 bid – 103½ ask. The shares are very liquid. The share price then moves to $98 – 98½ before rising to $115 –115¾ at the end of the month. If the investor had (i) placed a market order to buy the shares on the first day and then sold them on the last day (ii) a limit order to buy at $100 and sell at the end of the month, his profit before transaction costs would be closest to: 

A. (i) 11.11% (ii) 15.00%.

B. (i) 11.11% (ii) 15.75%.

C. (i) 12.39% (ii) 15.00%.

D. (i) 12.39% (ii) 15.75%.

Correct Answer:  A ..........................................................................................LOS: Reading  52‐e (i) If it is a market order he will pay the ask price to buy the shares, i.e. $103 ½ and he will sell at the bid price of $115 making a profit of 11.11%. (ii) A limit order to buy at $100 will be executed and he will sell at $115 giving a profit of 15%.

Reference: CFA® Program Curriculum, Volume 5, pp. 25‐26. 

 

 

 

6.  An informationally efficient market is one where: 

A. information is available on a timely basis to all investors.

B. investors allocate their funds to the companies that can make the best use of them.

C. security prices adjust slowly to the arrival of new information giving investors time to take advantage of positive news.

D. security prices adjust rapidly to the arrival of new information and therefore security prices reflect all information about the security.

Correct Answer:  D..........................................................................................LOS: Reading  54‐a An informationally efficient market is one where security prices adjust very rapidly to the arrival of new information.

Reference: CFA® Program Curriculum, Volume 5, pp. 62‐63. 

Page 377: Cfa Ques Answer

Equity Investments: Securities Markets   361 

 

7.  Buying on margin means that: 

A.  if the margin requirement is 60%, the investor may borrow 60% of the cost of buying shares, allowing him to leverage his transaction. 

B.  the returns from buying on margin will only be higher than paying in full for shares when the share price moves significantly up or down. 

C.  if an investor has bought shares on margin and the share price rises he will have made a more attractive return on his investment than if he had fully paid for the shares. 

D.  if an investor has bought shares on margin he will be required to keep a maintenance margin with the broker; if the share price falls he will immediately receive a margin call. 

 

 

 

8.  A call market refers to: 

A.  a market where trades are done by open outcry. 

B.  a market which specializes in providing derivatives trading. 

C.  a market where it is attempted to match all the bids and asks at a specified time. 

D.  a market where all the trading is done by computer rather than on a trading floor. 

Page 378: Cfa Ques Answer

362   Study Session 13:  

 

7.  Buying on margin means that: 

A. if the margin requirement is 60%, the investor may borrow 60% of the cost of buying shares, allowing him to leverage his transaction.

B. the returns from buying on margin will only be higher than paying in full for shares when the share price moves significantly up or down.

C. if an investor has bought shares on margin and the share price rises he will have made a more attractive return on his investment than if he had fully paid for the shares.

D. if an investor has bought shares on margin he will be required to keep a maintenance margin with the broker; if the share price falls he will immediately receive a margin call.

Correct Answer:  C ..........................................................................................LOS: Reading  52‐g A is not true since if the margin requirement is 60% the investor can only borrow 40%.

B is not true since if the share moves down the losses will be higher if the investor has bought on margin.

D is not true since if the maintenance margin is lower than the initial margin the investor will only receive a margin call after the shares have fallen to a level where the proportion of equity to the total value of stock is below the maintenance margin.

C is true due to the leverage effect.

Reference: CFA® Program Curriculum, Volume 5, pp. 26‐29. 

 

 

 

8.  A call market refers to: 

A. a market where trades are done by open outcry.

B. a market which specializes in providing derivatives trading.

C. a market where it is attempted to match all the bids and asks at a specified time.

D. a market where all the trading is done by computer rather than on a trading floor.

Correct Answer:  C ..........................................................................................LOS: Reading  52‐c In a call market all the bids and asks for a stock are collected and a stock price that will best match buyers and sellers is decided.

Reference: CFA® Program Curriculum, Volume 5, p. 15. 

Page 379: Cfa Ques Answer

Equity Investments: Securities Markets   363 

 

9.  Which of the following is least likely to be a member of a U.S. securities exchange? 

A.  A specialist. 

B.  A floor broker. 

C.  An underwriter. 

D.  A registered trader. 

 

 

 

10.  The quotations for the price of Brown and Co. are given by 3 dealers as shown below: 

 

  Bid AskDealer 1  17 ¾ 18 ¼Dealer 2  17 ½ 18Dealer 3  17 ⅝ 18

 

If Dealer 3 has excess inventory of Brown and Co. stock, changing his quote to which of the following would be the most effective in reducing his inventory?  

A.  17 ⅝ ‐ 18. 

B.  17 ⅝ ‐ 18 ¼. 

C.  17 ¼ ‐ 18 ¼. 

D.  17 ¼ ‐ 17 ¾. 

Page 380: Cfa Ques Answer

364   Study Session 13:  

 

9.  Which of the following is least likely to be a member of a U.S. securities exchange? 

A. A specialist.

B. A floor broker.

C. An underwriter.

D. A registered trader.

Correct Answer:  C ..........................................................................................LOS: Reading  52‐e The four major categories of membership are:

1. Specialist.

2. Commission broker.

3. Floor broker.

4. Registered trader.

Reference: CFA® Program Curriculum, Volume 5, pp. 24‐25. 

 

 

10.  The quotations for the price of Brown and Co. are given by 3 dealers as shown below: 

  Bid  AskDealer 1  17 ¾  18 ¼Dealer 2  17 ½  18Dealer 3  17 ⅝  18

If Dealer 3 has excess inventory of Brown and Co. stock, changing his quote to which of the following would be the most effective in reducing his inventory?

A. 17 � - 18.

B. 17 � - 18 ¼.

C. 17 ¼ - 18 ¼.

D. 17 ¼ - 17 ¾.

Correct Answer:  D..........................................................................................LOS: Reading  52‐e Moving the ask price lower than the other dealers will mean the dealer is able to sell stock, and moving the bid price lower will mean he is unlikely to have to buy stock, until the other dealers move their prices.

Reference: CFA® Program Curriculum, Volume 5, pp. 20‐23. 

Page 381: Cfa Ques Answer

Equity Investments: Securities Markets   365 

 

11.  When  investors  prefer  to  read  research  that  agrees with  their  own  views,  rather  than  read  a contrary opinion, this is referred to in behavioral finance as: 

A.  escalation bias. 

B.  prospect theory. 

C.  overconfidence. 

D.  confirmation bias. 

 

 

 

12.  A call market is one in which: 

A.  short selling is not permitted. 

B.  dealers are making markets in stocks. 

C.  stock prices are determined by auction. 

D.  individual stocks are traded at specified times. 

Page 382: Cfa Ques Answer

366   Study Session 13:  

 

11.  When  investors  prefer  to  read  research  that  agrees with  their  own  views,  rather  than  read  a contrary opinion, this is referred to in behavioral finance as: 

A. escalation bias.

B. prospect theory.

C. overconfidence.

D. confirmation bias.

Correct Answer:  D......................................................................................... LOS: Reading  54‐d Research shows that investors seek out information that confirms their own views, this is called confirmation bias.

Reference: CFA® Program Curriculum, Volume 5, pp. 83‐84. 

 

 

 

12.  A call market is one in which: 

A. short selling is not permitted.

B. dealers are making markets in stocks.

C. stock prices are determined by auction.

D. individual stocks are traded at specified times.

Correct Answer:  D..........................................................................................LOS: Reading  52‐c B and C are types of continuous market. Short selling may or may not be permitted in a call market.

In a call market the exchange usually sets the price and all trades are executed at a specified time at this price.

Reference: CFA® Program Curriculum, Volume 5, pp. 14‐15. 

Page 383: Cfa Ques Answer

Equity Investments: Securities Markets   367 

 

13.  A pricing anomaly is best described as:  

A.  a price move that reflects a factor that is specific to the individual stock. 

B.  a predictable deviation between expected and actual returns for a stock.  

C.  out or underperformance of a stock against the market average performance. 

D.  a short‐term random movement in a stock prices giving an opportunity to make a profit. 

 

 

 

14.  If a market is perfectly efficient and a portfolio manager does not have superior analysts then the portfolio manager should: 

A.  only hold cash. 

B.  select a portfolio of securities at random. 

C.  increase the systematic risk of the portfolio. 

D.  diversify away unsystematic risk and adjust the systematic risk of each portfolio to meet clients’ objectives. 

Page 384: Cfa Ques Answer

368   Study Session 13:  

 

13.  A pricing anomaly is best described as:  

A. a price move that reflects a factor that is specific to the individual stock.

B. a predictable deviation between expected and actual returns for a stock.

C. out or underperformance of a stock against the market average performance.

D. a short-term random movement in a stock prices giving an opportunity to make a profit.

Correct Answer:  B.......................................................................................... LOS: Reading  55‐b A key element of an anomaly is that it is a predictable, rather than a random, move.

Reference: CFA® Program Curriculum, Volume 5, p. 100. 

 

 

 

14.  If a market is perfectly efficient and a portfolio manager does not have superior analysts then the portfolio manager should: 

A. only hold cash.

B. select a portfolio of securities at random.

C. increase the systematic risk of the portfolio.

D. diversify away unsystematic risk and adjust the systematic risk of each portfolio to meet clients’ objectives.

Correct Answer:  D..........................................................................................LOS: Reading  54‐c Portfolio managers who do not have superior analysts should do the following:

(i) Quantify their clients’ risk tolerance.

(ii) Construct a portfolio which has the appropriate risk profile and maintain this risk level on an ongoing basis.

(iii) Diversify to eliminate any unsystematic risk by investing globally.

(iv) Minimize costs including taxes, transaction costs and avoid holding illiquid stocks.

Reference: CFA® Program Curriculum, Volume 5, pp. 87‐89. 

Page 385: Cfa Ques Answer

Equity Investments: Securities Markets   369 

 

15.  Which of  the  following  is  least  likely  to be a  reason why we  should be  skeptical of  claims by analysts to have discovered an anomaly?  

A.  Data mining. 

B.  Survivorship bias. 

C.  Nonsynchronous trading. 

D.  Limited capital of arbitrageurs. 

 

 

 

16.  If an investor buys 1,000 shares at $50 and the initial margin is 60% and the maintenance margin is 30%, he/she will receive the first margin call when the stock price falls below: 

A.  $14.00. 

B.  $21.00. 

C.  $28.57. 

D.  $49.99. 

Page 386: Cfa Ques Answer

370   Study Session 13:  

 

15.  Which of  the  following  is  least  likely  to be a  reason why we  should be  skeptical of  claims by analysts to have discovered an anomaly?  

A. Data mining.

B. Survivorship bias.

C. Nonsynchronous trading.

D. Limited capital of arbitrageurs.

Correct Answer:  D......................................................................................... LOS: Reading  55‐d Limited capital can constrain the ability of arbitrageurs to take advantage of anomalies, but it is not a reason to be skeptical that they exist.

Reference: CFA® Program Curriculum, Volume 5, pp. 100‐104. 

 

 

 

16.  If an investor buys 1,000 shares at $50 and the initial margin is 60% and the maintenance margin is 30%, he/she will receive the first margin call when the stock price falls below: 

A. $14.00.

B. $21.00.

C. $28.57.

D. $49.99.

Correct Answer:  C ..........................................................................................LOS: Reading  52‐g The amount borrowed would be $20,000. We need to calculate when the value of the equity equals 30% of the total value of the stock. This is given by:

1,000P - $20,000 = 0.30(1,000P)

P = $28.57

Reference: CFA® Program Curriculum, Volume 5, pp. 26‐29. 

Page 387: Cfa Ques Answer

Equity Investments: Securities Markets   371 

 

17.  Two indexes contain exactly the same stocks; one is a value‐weighted index which increased by 12% whereas the other is an unweighted index which increased by 5% over the same period. This is explained by: 

A.  there were a large number of stock splits over the period. 

B.  there were a small number of stock splits over the period. 

C.  small capitalization stocks outperformed large capitalization stocks. 

D.  large capitalization stocks outperformed small capitalization stocks. 

 

 

 

18.  The initial margin requirement for a stock purchase is: 

A.  the market value of the stock less the amount borrowed. 

B.  the market value of the stock less the amount paid in cash. 

C.  the percentage of the transaction value that can be borrowed. 

D.  the percentage of the transaction value that must be paid for in cash. 

Page 388: Cfa Ques Answer

372   Study Session 13:  

 

17.  Two indexes contain exactly the same stocks; one is a value‐weighted index which increased by 12% whereas the other is an unweighted index which increased by 5% over the same period. This is explained by: 

A. there were a large number of stock splits over the period.

B. there were a small number of stock splits over the period.

C. small capitalization stocks outperformed large capitalization stocks.

D. large capitalization stocks outperformed small capitalization stocks.

Correct Answer:  D..........................................................................................LOS: Reading  53‐a Stock splits will not affect either index since in the market-value-weighted index when there is a stock split the number of shares outstanding will increase but the share price will fall by a corresponding amount. An unweighted index will be computed on an equal amount of money invested in each stock regardless of price or market value.

In a value-weighted index companies with a larger market capitalization will have a higher weighting so D is the correct answer.

Reference: CFA® Program Curriculum, Volume 5, pp. 44‐46. 

 

 

 

18.  The initial margin requirement for a stock purchase is: 

A. the market value of the stock less the amount borrowed.

B. the market value of the stock less the amount paid in cash.

C. the percentage of the transaction value that can be borrowed.

D. the percentage of the transaction value that must be paid for in cash.

Correct Answer:  D..........................................................................................LOS: Reading  52‐g The initial margin is simply the percentage or proportion of the transaction value that must be paid for in cash, rather than borrowed.

Reference: CFA® Program Curriculum, Volume 5, pp. 26‐29. 

Page 389: Cfa Ques Answer

Equity Investments: Securities Markets   373 

 

19.  The Efficient Market Hypothesis and the tests done to prove the Hypothesis imply doing which of the following is the least useful for portfolio managers? 

A.  Using technical analysis. 

B.  Minimizing total transaction costs. 

C.  Focusing on analyzing neglected companies. 

D.  Using analysts who have the ability to estimate economic variables that have an impact on security prices. 

 

 

 

20.  Which of the following supports the semistrong‐form of the Efficient Market Hypothesis? 

A.  The January effect. 

B.  The neglected firm effect. 

C.  The performance of stocks with a low price/book value ratio. 

D.  The performance of stocks that have announced a change in accounting methods. 

Page 390: Cfa Ques Answer

374   Study Session 13:  

 

19.  The Efficient Market Hypothesis and the tests done to prove the Hypothesis imply doing which of the following is the least useful for portfolio managers? 

A. Using technical analysis.

B. Minimizing total transaction costs.

C. Focusing on analyzing neglected companies.

D. Using analysts who have the ability to estimate economic variables that have an impact on security prices.

Correct Answer:  A ..........................................................................................LOS: Reading  54‐c The weak-form of the Efficient Market Hypothesis implies that technical analysis based on past trading data does not have any value.

Reference: CFA® Program Curriculum, Volume 5, p. 85. 

 

 

 

20.  Which of the following supports the semistrong‐form of the Efficient Market Hypothesis? 

A. The January effect.

B. The neglected firm effect.

C. The performance of stocks with a low price/book value ratio.

D. The performance of stocks that have announced a change in accounting methods.

Correct Answer:  D......................................................................................... LOS: Reading  54‐b The January effect, the neglected firm effect and the outperformance of low price/book value stocks are all anomalies of the semistrong-form of the EMH. The performance of stocks that have announced a change in accounting methods supports the semistrong-form of the EMH. Studies find that analysts look at the true value of companies rather than the effects of a change in the way financial performance is reported.

Reference: CFA® Program Curriculum, Volume 5, pp. 67‐79. 

Page 391: Cfa Ques Answer

Equity Investments: Securities Markets   375 

 

21.  Based on  historic data the correlation between the S&P 500 and the Russell Small‐Cap index and the IFC Emerging Market index are as shown in the table below.  

Index  Correlation with S&P 500 Russell Small‐Cap index  0.783 IFC Emerging Market index  0.392 

On the basis of the data given, in order to diversify a portfolio that is invested in S&P 500 stocks, from which index is it likely to be most effective to include stocks, and which is the most important factor in explaining the correlation figures?   

  Include stocks from     Correlation explained by  

A.  Russell Small‐Cap index    Index construction 

B.  Russell Small‐Cap index    Index constituents 

C.  IFC Emerging Market index   Index construction  

D.  IFC Emerging Market index   Index constituents 

 

 

 

22.  A value‐weighted index is made up of two stocks, X and Y, and the following data is provided: 

  December 31st 2006  December 31st 2007 Stock  Price  Shares  outstanding  Price  Shares  outstanding X  $25  10,000  $15  20,000* Y  $50  6,000  $65  6,000 

* after a 2 for 1 stock split 

The base index is set at 100 on December 31st 2006. The index on December 31st 2007 is closest to: 

A.  98.18. 

B.  106.67. 

C.  125.45. 

D.  126.67. 

Page 392: Cfa Ques Answer

376   Study Session 13:  

 

21.  Based on  historic data the correlation between the S&P 500 and the Russell Small‐Cap index and the IFC Emerging Market index are as shown in the table below.  

Index  Correlation with S&P 500 

Russell Small‐Cap index  0.783 

IFC Emerging Market index  0.392 

On the basis of the data given, in order to diversify a portfolio that is invested in S&P 500 stocks, from which index is it likely to be most effective to include stocks, and which is the most important factor in explaining the correlation figures?

Include stocks from Correlation explained by

A. Russell Small-Cap index Index construction

B. Russell Small-Cap index Index constituents

C. IFC Emerging Market index Index construction

D. IFC Emerging Market index Index constituents

Correct Answer:  D..........................................................................................LOS: Reading  53‐c A low correlation indicates that there are stronger diversification benefits, so including stocks from the IFC Emerging Market index would be most effective, based on historic data. Most of the major indexes (except for the Nikkei) are value weighted, the most important factor in explaining low correlation is the different constituent stocks in the indexes.

Reference: CFA® Program Curriculum, Volume 5, pp. 56‐58. 

Page 393: Cfa Ques Answer

Equity Investments: Securities Markets   377 

 

22.  A value‐weighted index is made up of two stocks, X and Y, and the following data is provided: 

  December 31st 2006  December 31st 2007 

Stock  Price  Shares outstanding

Price  Shares outstanding 

X  $25  10,000  $15  20,000* 

Y  $50  6,000  $65  6,000 

* after a 2 for 1 stock split

The base index is set at 100 on December 31st 2006. The index on December 31st 2007 is closest to:

A. 98.18.

B. 106.67.

C. 125.45.

D. 126.67.

Correct Answer:  C.......................................................................................... LOS: Reading  53‐a Total market value on Dec. 31st 2006 is ($25 x 10,000) + ($50 x 6,000) = $550,000

Total market value on Dec. 31st 2007 is ($15 x 20,000) + ($65 x 6,000) = $690,000

The index = ($690,000/$550,000) x100 = 125.45

Reference: CFA® Program Curriculum, Volume 5, pp. 44‐45. 

 

 

Page 394: Cfa Ques Answer

378   Study Session 14:  

 

Study Session 14: Equity Investments: Industry and Company Analysis

 

This study session focuses on industry and company analysis and describes the tools used in forming an opinion about investing in a particular stock or group of stocks. 

This  study  session  begins with  the  essential  tools  of  equity  valuation:  the  discounted  cash  flow technique  and  the  relative  valuation  approach.  These  techniques  provide  the means  to  estimate reasonable  price  for  a  stock.  The  readings  on  industry  analysis  are  an  important  element  in  the valuation process, providing the top–down context crucial to estimating a company’s potential. Also addressed is estimating a company’s earnings per share by forecasting sales and profit margins. 

The last reading in this study session focuses on price multiples, one of the most familiar and widely used  tools  in estimating  the value of a company, and  introduces  the application of  four commonly used price multiples to valuation. 

 

Reading 56: An Introduction to Security Valuation: Part I 

Reading 57: Industry Analysis 

Reading 58: Equity: Concepts and Techniques 

Reading 59: Company Analysis and Stock Valuation 

Reading 60: An Introduction to Security Valuation: Part II 

Reading 61: Introduction to Price Multiples 

 

 

Page 395: Cfa Ques Answer

Equity Investments: Industry and Company Analysis   379 

 

1.  If  an  investor’s  required  rate of  return  is  12%  and  a  company’s  stock price  is  $45.00,  the next dividend is estimated to be $3.60 and the growth rate of dividends is a constant 4.5%, then the stock is: 

A.  overvalued. 

B.  undervalued. 

C.  fairly valued. 

D.  information is needed on the dividend payout ratio to answer the question. 

 

 

 

2.  Which of the following would be a positive factor for producers’ profitability? 

A.  Low entry barriers. 

B.  Strong buyer power. 

C.  Weak supplier power. 

D.  Availability of substitutes. 

 

Page 396: Cfa Ques Answer

380   Study Session 14:  

 

1.  If  an  investor’s  required  rate of  return  is  12%  and  a  company’s  stock price  is  $45.00,  the next dividend is estimated to be $3.60 and the growth rate of dividends is a constant 4.5%, then the stock is: 

A. overvalued.

B. undervalued.

C. fairly valued.

D. information is needed on the dividend payout ratio to answer the question.

Correct Answer:  B.......................................................................................... LOS: Reading  60‐b The stock price using the DDM should be 3.6/(0.12 – 0.045) = $48, if the price is $45 the stock is undervalued.

Reference: CFA® Program Curriculum, Volume 5, pp. 177‐182. 

 

 

 

2.  Which of the following would be a positive factor for producers’ profitability? 

A. Low entry barriers.

B. Strong buyer power.

C. Weak supplier power.

D. Availability of substitutes.

Correct Answer:  C ..........................................................................................LOS: Reading  58‐e Weak supplier power means that the suppliers to the industry are in a weak negotiating position and are unable to squeeze the producers’ profits; this is a positive factor for the producers’ profitability.

Low entry barriers will increase competition, reducing profit margins. Strong buyer power means the buyers can demand low prices reducing profitability. Availability of substitutes restrains producers who wish to raise prices.

Reference: CFA® Program Curriculum, Volume 5, pp. 141‐144. 

Page 397: Cfa Ques Answer

Equity Investments: Industry and Company Analysis   381 

 

3.  Which of the following factors is least likely to directly determine the level of competition in an industry? 

A.  Entry barriers. 

B.  Unit labor costs. 

C.  Bargaining power of buyers. 

D.  Availability of substitute products. 

 

 

 

4.  An analyst forecasts a company will pay dividends of $2 in the first year, $3 in the second year and $3.50 in the third year. After the third year dividends are forecast to grow at 4% per annum. If an investor’s required rate of return is 12% the value of the stock is closest to: 

A.  $36.56. 

B.  $37.81.   

C.  $39.05 

D.  $51.69. 

Page 398: Cfa Ques Answer

382   Study Session 14:  

 

3.  Which of the following factors is least likely to directly determine the level of competition in an industry? 

A. Entry barriers.

B. Unit labor costs.

C. Bargaining power of buyers.

D. Availability of substitute products.

Correct Answer:  B...........................................................................................LOS: Reading  58‐e Unit labor costs impact the profitability of an industry, and although they indirectly affect how attractive a business is for participants they are not a direct factor determining competition.

Reference: CFA® Program Curriculum, Volume 5, pp. 141‐144. 

 

 

4.  An analyst forecasts a company will pay dividends of $2 in the first year, $3 in the second year and $3.50 in the third year. After the third year dividends are forecast to grow at 4% per annum. If an investor’s required rate of return is 12% the value of the stock is closest to: 

A. $36.56.

B. $37.81.

C. $39.05

D. $51.69.

Correct Answer:  C ......................................................................................... LOS: Reading  60‐b First, calculate the value of the stock at the end of year three,

( )( )

( ) 50.45$04.012.0

04.150.3$gk

DValue 4 =

−=

−=

The value of the stock today is the sum of the present values of the first three years’ dividends plus the present value of the end-year-three value.

( ) ( ) ( ) ( ) ( ) ( )05.39$

12.150.3$50.45$

12.13$

12.12$

k1DP

k1D

k1DValue 323

32

21 =+

++=+

++

++

+=

Reference: CFA® Program Curriculum, Volume 5, pp. 176‐182. 

Page 399: Cfa Ques Answer

Equity Investments: Industry and Company Analysis   383 

 

5.  A company is operating at the peak of the business cycle. Which of the following EPS calculations is likely to give the highest P/E ratio? 

A.  Leading P/E. 

B.  Trailing P/E. 

C.  Current P/E. 

D.  P/E using normalized earnings. 

 

 

 

6.  Estimating a company earnings multiplier using macroanalysis involves: 

A.  examining the relationship between the company earnings multiplier and the economy. 

B.  examining the relationship between the company earnings multiplier and the market earnings multiplier. 

C.  examining the relationship between the historic earnings multiplier for the company and company performance. 

D.  examining the specific variables that influence the earnings multiplier, including the dividend payout ratio and required rates of return. 

Page 400: Cfa Ques Answer

384   Study Session 14:  

 

5.  A company is operating at the peak of the business cycle. Which of the following EPS calculations is likely to give the highest P/E ratio? 

A. Leading P/E.

B. Trailing P/E.

C. Current P/E.

D. P/E using normalized earnings.

Correct Answer:  D..........................................................................................LOS: Reading  61‐a Normalized earnings will calculate the earnings at the midpoint of the cycle so is likely to be lower than current/trailing earnings or prospective earnings; hence the P/E will be higher on this basis.

Reference CFA® Program Curriculum, Volume 5, pp. 205-210.

 

 

 

6.  Estimating a company earnings multiplier using macroanalysis involves: 

A. examining the relationship between the company earnings multiplier and the economy.

B. examining the relationship between the company earnings multiplier and the market earnings multiplier.

C. examining the relationship between the historic earnings multiplier for the company and company performance.

D. examining the specific variables that influence the earnings multiplier, including the dividend payout ratio and required rates of return.

Correct Answer:  B.......................................................................................... LOS: Reading  59‐b Macroanalysis is estimating a P/E or earnings multiplier by looking at the relationship between the company and its market and industry. Answer D refers to microanalysis.

Reference: CFA® Program Curriculum, Volume 5, pp. 160‐166. 

Page 401: Cfa Ques Answer

Equity Investments: Industry and Company Analysis   385 

 

7.  A company whose earnings are very sensitive to the business cycle is a: 

A.  growth stock. 

B.  cyclical stock. 

C.  growth company. 

D.  cyclical company. 

 

 

 

8.  Value chain analysis in return expectations analysis is important because it studies: 

A.  co‐opetition risks. 

B.  competition between rival firms in an industry. 

C.  the government’s ability to raise value added taxes. 

D.  how goods are transformed from raw materials through to the finished good and profits are shared between firms along the chain. 

Page 402: Cfa Ques Answer

386   Study Session 14:  

 

7.  A company whose earnings are very sensitive to the business cycle is a: 

A. growth stock.

B. cyclical stock.

C. growth company.

D. cyclical company.

Correct Answer:  D..........................................................................................LOS: Reading  59‐a A cyclical company is one whose sales and earnings are very sensitive to the business cycle such as auto or steel companies. A cyclical stock is defined differently; it is one which has a high beta so it has changes in rates of return which are larger than the changes in rates of return of the market.

Reference: CFA® Program Curriculum, Volume 5, pp. 150‐151. 

 

 

 

8.  Value chain analysis in return expectations analysis is important because it studies: 

A. co-opetition risks.

B. competition between rival firms in an industry.

C. the government’s ability to raise value added taxes.

D. how goods are transformed from raw materials through to the finished good and profits are shared between firms along the chain.

Correct Answer:  D......................................................................................... LOS: Reading  58‐b Value chain analysis in return expectations analysis looks at how returns are split between the participants in a value chain. Co-opetition risks refer to risks, not returns, in the value chain and competition between rival firms is not referring to firms in the same value chain.

Reference: CFA® Program Curriculum, Volume 5, pp. 136‐140. 

Page 403: Cfa Ques Answer

Equity Investments: Industry and Company Analysis   387 

 

9.  An analyst is valuing a company where he suspects that the management has been manipulating the accounts by capitalizing expenses  that  should have been  treated as operating expenses. Which would be an appropriate approach to take when valuing the company? 

A.  Use normalized earnings rather than current earnings in the price to earnings calculation. 

B.  Use price to cash flow as the main relative measure using free cash flows in the calculation. 

C.  Not attempt to value the company if there are concerns about the quality of the reported accounts. 

D.  Focus on price to book value since book value will not have been affected by any manipulation of the accounts.  

 

 

 

10.  Which of  the  following  is most  likely  to  lead  to a  company’s earnings multiplier being higher than the market multiplier? 

A.  The company has a high beta. 

B.  The risk of the company’s earnings is higher than the market. 

C.  Low return on equity for the company relative to the market. 

D.  High earnings growth rates for the company relative to the market. 

Page 404: Cfa Ques Answer

388   Study Session 14:  

 

9.  An analyst is valuing a company where he suspects that the management has been manipulating the accounts by capitalizing expenses  that  should have been  treated as operating expenses. Which would be an appropriate approach to take when valuing the company? 

A. Use normalized earnings rather than current earnings in the price to earnings calculation.

B. Use price to cash flow as the main relative measure using free cash flows in the calculation.

C. Not attempt to value the company if there are concerns about the quality of the reported accounts.

D. Focus on price to book value since book value will not have been affected by any manipulation of the accounts.

Correct Answer:  B...........................................................................................LOS: Reading  61‐a Normalized earnings will look at earnings over a business cycle so will not resolve the issue of earnings manipulation. Book value will also reflect a decision to capitalize rather than expense earnings. Cash flow will reflect the actual cash going in and out of the company so will be less subject to manipulation.

Answer D is possible but analysts may not have the choice to ignore companies where they suspect that the accounts have been manipulated. They will need to attempt to value them by adjusting the accounts and/or using cash flow methods.

Reference: CFA® Program Curriculum, Volume 5, pp. 204‐210. 

 

 

10.  Which of  the  following  is most  likely  to  lead  to a  company’s earnings multiplier being higher than the market multiplier? 

A. The company has a high beta.

B. The risk of the company’s earnings is higher than the market.

C. Low return on equity for the company relative to the market.

D. High earnings growth rates for the company relative to the market.

Correct Answer:  D......................................................................................... LOS: Reading  59‐b High growth generally leads to a high multiplier, so D is the correct answer.

A high beta would lead to a high required rate of return and therefore a lower multiplier so A is not correct.

Higher risk means a higher required rate of return, and therefore a lower multiplier so B is not correct.

Low return on equity implies low growth rates which would lead to a lower multiplier so C is not correct.

Reference: CFA® Program Curriculum, Volume 5, pp. 160‐166. 

Page 405: Cfa Ques Answer

Equity Investments: Industry and Company Analysis   389 

 

11.  ABC Commodities  is  sensitive  to  the economic  cycle and an analyst decides  that  the  six years ending 2007 reflect a business cycle  for  the company.   He collects  the  following data: earnings per share (EPS), book value per share ( BVPS) and return on equity (ROE):. 

  2002  2003  2004  2005  2006  2007 Adjusted* EPS, $   1.30  2.65  5.50  4.30  3.25  1.00 ROE* %  0.04  0.13  0.22  0.18  0.12  0.03 BVPS, $            32.00 

* Adjusted for non‐recurring items 

The current share price of ABC Commodities is $30.00 

The P/E of ABC commodities based on the average historical EPS is closest to: 

A.  7.81. 

B.  9.37. 

C.  10.00. 

D.  33.33. 

 

 

12.  Which of the following is least likely to be an adjustment made to book value in order to make it better reflect the value of shareholders’ investment? 

 

A.  Use tangible book value rather than total book value.  

B.  Adjust book value for off‐balance‐sheet assets and liabilities. 

C.  Use historic average of past book values over a business cycle. 

D.  Adjust book value for differences in fair value and historic cost of assets. 

Page 406: Cfa Ques Answer

390   Study Session 14:  

 

11.  ABC Commodities  is  sensitive  to  the economic  cycle and an analyst decides  that  the  six years ending 2007 reflect a business cycle  for  the company.   He collects  the  following data: earnings per share (EPS), book value per share ( BVPS) and return on equity (ROE):. 

  2002 2003 2004 2005 2006 2007Adjusted* EPS, $   1.30 2.65 5.50 4.30 3.25 1.00ROE* %  0.04 0.13 0.22 0.18 0.12 0.03BVPS, $    32.00 

* Adjusted for non-recurring items

The current share price of ABC Commodities is $30.00

The P/E of ABC commodities based on the average historical EPS is closest to:

A. 7.81.

B. 9.37.

C. 10.00.

D. 33.33.

Correct Answer:  C ......................................................................................... LOS: Reading  61‐b The average EPS = (1.30 + 2.65 + 5.50 + 4.30 + 3.25 + 1.00)/6 = 3.00

P/E = 30.00/3.00 = 10.00

Reference: CFA® Program Curriculum, Volume 5, pp. 206‐210. 

 

 

12.  Which of the following is least likely to be an adjustment made to book value in order to make it better reflect the value of shareholders’ investment? 

A. Use tangible book value rather than total book value.

B. Adjust book value for off-balance-sheet assets and liabilities.

C. Use historic average of past book values over a business cycle.

D. Adjust book value for differences in fair value and historic cost of assets.

Correct Answer:  C ..........................................................................................LOS: Reading  61‐a Since book value is a cumulative number, reflecting retained earnings, it is a more stable number than one year earnings, so it is less likely that an average historic figure would be used.

Reference: CFA® Program Curriculum, Volume 5, pp. 210‐218. 

Page 407: Cfa Ques Answer

Equity Investments: Industry and Company Analysis   391 

 

13.  If a company has an earnings retention ratio of 60%, earnings are growing at 5% per annum and investors’ required rate of return is 12%, the P/E of the stock is closest to: 

A.  3.33. 

B.  4.44. 

C.  5.71. 

D.  8.57. 

 

 

 

14.  Which of the following factors will encourage new entrants to an industry?  

 

A.  High exit costs. 

B.  A steep cost curve. 

C.  Excess industry capacity. 

D.  High industry profitability. 

Page 408: Cfa Ques Answer

392   Study Session 14:  

 

13.  If a company has an earnings retention ratio of 60%, earnings are growing at 5% per annum and investors’ required rate of return is 12%, the P/E of the stock is closest to: 

A. 3.33.

B. 4.44.

C. 5.71.

D. 8.57.

Correct Answer:  C ..........................................................................................LOS: Reading  60‐c 

 

Reference: CFA® Program Curriculum, Volume 5, pp. 186‐188. 

 

 

14.  Which of the following factors will encourage new entrants to an industry?  

A. High exit costs.

B. A steep cost curve.

C. Excess industry capacity.

D. High industry profitability.

Correct Answer:  D..........................................................................................LOS: Reading  58‐e High profitability will attract new entrants whereas a steep cost curve means it is difficult to enter the industry with small volumes. High exit costs, such as irreversibility of required capital investment, discourages new entrants, and excess capacity means existing manufacturers may be willing to cut prices to deter new entrants.

Reference: CFA® Program Curriculum, Volume 5, pp. 143‐144. 

Page 409: Cfa Ques Answer

Equity Investments: Industry and Company Analysis   393 

 

15.  Which of the following is least likely to be a reason for using a price/sales (P/S) ratio to value a company? 

A.  Sales are generally more stable than earnings per share. 

B.  Sales numbers are less subject to accounting manipulation. 

B.  P/S is viewed as an appropriate valuation method for cyclical companies. 

D.  P/S valuation methods can be used comparing companies with different cost structures. 

 

 

 

16.  Which stage of the industry cycle is characterized by fast sales growth and high profit margins? 

A.  Mature growth. 

B.  Market maturity. 

C.  Pioneering development. 

D.  Rapid accelerating growth. 

Page 410: Cfa Ques Answer

394   Study Session 14:  

 

15.  Which of the following is least likely to be a reason for using a price/sales (P/S) ratio to value a company? 

A. Sales are generally more stable than earnings per share.

B. Sales numbers are less subject to accounting manipulation.

B. P/S is viewed as an appropriate valuation method for cyclical companies.

D. P/S valuation methods can be used comparing companies with different cost structures.

Correct Answer:  D..........................................................................................LOS: Reading  61‐a P/S is not a good method for comparing companies with different cost structures because it doesn’t take into account a company’s profitability. It is generally only useful for comparing companies operating in the same industry.

Reference: CFA® Program Curriculum, Volume 5, pp. 218‐221. 

 

 

 

16.  Which stage of the industry cycle is characterized by fast sales growth and high profit margins? 

A. Mature growth.

B. Market maturity.

C. Pioneering development.

D. Rapid accelerating growth.

Correct Answer:  D..........................................................................................LOS: Reading  58‐c Rapid accelerating growth is the second stage when the industry is seeing rapid growth in sales and the potential for high profit margins because new entrants and therefore competition may be limited.

Reference: CFA® Program Curriculum, Volume 5, pp. 137‐138. 

Page 411: Cfa Ques Answer

Equity Investments: Industry and Company Analysis   395 

 

17.  If the expected inflation rate in a country is 3% and the real risk‐free rate is 6% then the required nominal risk‐free rate is closest to: 

A.  2.9%. 

B.  9.0%. 

C.  9.2%. 

D.  18.0%. 

 

 

 

18.  An investor who follows an industry rotation strategy believes the economy has past its trough and there is confirmation of recovery. Which strategy would he/she be likely to follow? 

A.  Sell property. 

B.  Purchase stocks. 

C.  Purchase bonds. 

D.  Sell commodities. 

Page 412: Cfa Ques Answer

396   Study Session 14:  

 

17.  If the expected inflation rate in a country is 3% and the real risk‐free rate is 6% then the required nominal risk‐free rate is closest to: 

A. 2.9%.

B. 9.0%.

C. 9.2%.

D. 18.0%.

Correct Answer:  C ......................................................................................... LOS: Reading  60‐d NRFR = [1 +RRFR] [1 + E(I)] – 1

where:

NRFR = nominal risk-free rate

RRFR = real risk-free rate

E(I) = expected rate of inflation

NRFR = (1.06)(1.03 – 1) = 9.18%

Reference: CFA® Program Curriculum, Volume 5, pp. 191‐193. 

 

 

 

18.  An investor who follows an industry rotation strategy believes the economy has past its trough and there is confirmation of recovery. Which strategy would he/she be likely to follow? 

A. Sell property.

B. Purchase stocks.

C. Purchase bonds.

D. Sell commodities.

Correct Answer:  B...........................................................................................LOS: Reading  58‐a If the economy is in the recovery stage the best performance would be expected to come from cyclical stocks and commodities, and then property, so B is the best answer.

Reference: CFA® Program Curriculum, Volume 5, pp. 130‐132. 

Page 413: Cfa Ques Answer

Equity Investments: Industry and Company Analysis   397 

 

19.  Which  of  the  following would  be  least  likely  to  be  a  result  of  structural  change  in  the U.S. economy? 

A.  Rising consumer confidence leading to a surge in auto sales. 

B.  A rise in the minimum wage increasing costs for fast‐food outlets. 

C.  Increasing demand for residential nursing care as the population ages. 

D.  The move in population away from cities leading to stronger demand for catalogue and online shopping. 

 

 

 

20.  Which  of  the  following  is  one  of  the  reasons why  price  to  book  value  is  a  useful  valuation measure? 

A.  It can be used to value loss‐making companies. 

B.  It is useful for comparing the value of stocks across industries. 

C.  Book value is not usually distorted by the accounting methods used. 

D.  Book value has proved to be a good indicator of the market value of a company’s assets. 

Page 414: Cfa Ques Answer

398   Study Session 14:  

 

19.  Which  of  the  following would  be  least  likely  to  be  a  result  of  structural  change  in  the U.S. economy? 

A. Rising consumer confidence leading to a surge in auto sales.

B. A rise in the minimum wage increasing costs for fast-food outlets.

C. Increasing demand for residential nursing care as the population ages.

D. The move in population away from cities leading to stronger demand for catalogue and online shopping.

Correct Answer:  A ............................................................................................. LOS: Reading  57 Rising consumer confidence leading to a surge in auto sales would usually be the result of a cyclical move in the economy. The other choices reflect longer-term structural changes in the economy, so A is the best answer.

Reference: CFA® Program Curriculum, Volume 5, pp. 125‐127. 

 

 

 

20.  Which  of  the  following  is  one  of  the  reasons why  price  to  book  value  is  a  useful  valuation measure? 

A. It can be used to value loss-making companies.

B. It is useful for comparing the value of stocks across industries.

C. Book value is not usually distorted by the accounting methods used.

D. Book value has proved to be a good indicator of the market value of a company’s assets.

Correct Answer:  A ..........................................................................................LOS: Reading  61‐a Loss making companies cannot be valued using current P/E and will often also have negative cash flow so P/B, which is based on book value representing cumulative earnings and paid up capital, is a method that can usually be applied.

Reference: CFA® Program Curriculum, Volume 5, pp. 210‐218. 

Page 415: Cfa Ques Answer

Equity Investments: Industry and Company Analysis   399 

 

21.  When a  fund manager uses  top‐down analysis  to manage equity portfolios he  is  least  likely  to consider: 

A.  impact of economic environment on different industries.   

B.  impact of industry environment on individual companies. 

C.  the impact of monetary and fiscal policies on the market environment. 

D.  technical factors that will affect supply and demand for shares in the market.   

 

 

 

22.  An analyst calculates the price to sales ratio for a company and finds it is significantly less than the market average, this could be explained by: 

A.  sales per share are higher than for the average company. 

B.  the company has a low profit margin relative to the average for the market. 

C.  the company has exhibited rapid sales growth relative to the market average. 

D.  sales growth has been consistent and the risk of sales growth faltering is small. 

Page 416: Cfa Ques Answer

400   Study Session 14:  

 

21.  When a  fund manager uses  top‐down analysis  to manage equity portfolios he  is  least  likely  to consider: 

A. impact of economic environment on different industries.

B. impact of industry environment on individual companies.

C. the impact of monetary and fiscal policies on the market environment.

D. technical factors that will affect supply and demand for shares in the market.

Correct Answer:  D............................................................................................. LOS: Reading  56 Top-down approaches usually focusing on fundamental rather than technical analysis. The first step is to analyze alternative economies and securities markets. The objective is to decide the country allocation and the allocation between bonds, equities and cash in each market. Analysts of economies will study a government’s fiscal and monetary policies, inflation, interest rates, corporate and consumer expenditure, and exchange rates. The next step is to use economic and market analysis to decide which industries will benefit and which will suffer in the expected environment. Individual companies are analyzed within the industries selected. This will include analyzing each company’s past performance and forecasting its future prospects and then determining its value.

Reference: CFA® Program Curriculum, Volume 5, pp. 117‐121. 

Page 417: Cfa Ques Answer

Equity Investments: Industry and Company Analysis   401 

 

22.  An analyst calculates the price to sales ratio for a company and finds it is significantly less than the market average, this could be explained by: 

A. sales per share are higher than for the average company.

B. the company has a low profit margin relative to the average for the market.

C. the company has exhibited rapid sales growth relative to the market average.

D. sales growth has been consistent and the risk of sales growth faltering is small.

Correct Answer:  B ..........................................................................................LOS: Reading  61‐b P/S is the same as P/E multiplied by the profit margin, so a low profit margin could explain a low P/S ratio. Answers C and D would push up the P/S ratio. High sales per share are not going to necessarily lead to a low P/S ratio.

Reference: CFA® Program Curriculum, Volume 5, pp. 218‐221. 

 

 

Page 418: Cfa Ques Answer

402   Study Session 15:  

 

Study Session 15: Fixed Income Investments: Basic Concepts

 

This  study  session  presents  the  foundation  for  fixed  income  investments,  one  of  the  largest  and fastest  growing  segments  of  global  financial markets.  It  begins with  an  introduction  to  the  basic features  and  characteristics  of  fixed  income  securities  and  the  associated  risks.  The  session  then builds  by  describing  the  primary  issuers,  sectors,  and  types  of  bonds.  Finally,  the  study  session concludes with an introduction to yields and spreads and the effect of monetary policy on financial markets.  These  readings  combined  are  the  primary  building  blocks  for  mastering  the  analysis, valuation, and management of fixed income securities.   

 

Reading 62: Features of Debt Securities 

Reading 63: Risks Associated with Investing in Bonds 

Reading 64: Overview of Bond Sectors and Instruments 

Reading 65: Understanding Yield Spreads 

Reading 66: Monetary Policy in an Environment of Global Financial Markets 

 

Page 419: Cfa Ques Answer

Fixed Income Investments: Basic Concepts   403 

 

1.  Which  of  the  following  is  least  likely  to  be  an  example  of  embedded  options  that might  be granted to bondholders? 

A.  A cap on a floater. 

B.  A floor on a floater. 

C.  Conversion privileges. 

D.  The right to put the issue. 

 

 

 

2.  Which of the following is an example of a negative covenant? 

A.  The issuer must submit periodic statements to the bond trustee. 

B.  Mortgage holders are not permitted to prepay mortgages ahead of the scheduled date. 

C.  A floating rate security has a minimum coupon rate that must be paid if the reference rate declines below a certain level. 

D.  The issuer cannot secure any of its assets to a new debt issue without giving equal treatment to the existing debt holders. 

 

Page 420: Cfa Ques Answer

404   Study Session 15:  

 

1.  Which  of  the  following  is  least  likely  to  be  an  example  of  embedded  options  that might  be granted to bondholders? 

A. A cap on a floater.

B. A floor on a floater.

C. Conversion privileges.

D. The right to put the issue.

Correct Answer:  A ......................................................................................... LOS: Reading  62‐b A cap on a floater is a benefit to the issuer if interest rates rise, not to the bondholders.

Reference: CFA® Program Curriculum, Volume 5, pp. 242‐246. 

 

 

 

2.  Which of the following is an example of a negative covenant? 

A. The issuer must submit periodic statements to the bond trustee.

B. Mortgage holders are not permitted to prepay mortgages ahead of the scheduled date.

C. A floating rate security has a minimum coupon rate that must be paid if the reference rate declines below a certain level.

D. The issuer cannot secure any of its assets to a new debt issue without giving equal treatment to the existing debt holders.

Correct Answer:  D..........................................................................................LOS: Reading  62‐a A negative covenant is a restriction on the issuer to protect investors in the bond. An example is when unsecured debt holders are protected from the assets of the issuer being given as collateral in a subsequent debt issue.

Reference: CFA® Program Curriculum, Volume 5, p. 239. 

Page 421: Cfa Ques Answer

Fixed Income Investments: Basic Concepts   405 

 

3.  When the issuer of a bond agrees to retire a certain proportion of a bond issue each year, this is an example of: 

A.  a callable bond. 

B.  a refundable bond. 

C.  a prepayment option. 

D.  a sinking fund provision. 

 

 

 

4.  An investor pays tax at a marginal rate of 40% and holds a tax‐exempt issue that has a yield of 6%. The taxable‐equivalent yield is: 

A.  2.4%. 

B.  3.6%. 

C.  10.0%. 

D.  15.0%. 

Page 422: Cfa Ques Answer

406   Study Session 15:  

 

3.  When the issuer of a bond agrees to retire a certain proportion of a bond issue each year, this is an example of: 

A. a callable bond.

B. a refundable bond.

C. a prepayment option.

D. a sinking fund provision.

Correct Answer:  D..........................................................................................LOS: Reading  62‐e A sinking fund provision allows the issuer of a bond to retire a certain proportion of a bond issue each year.

Reference: CFA® Program Curriculum, Volume 5, pp. 251. 

 

 

 

4.  An investor pays tax at a marginal rate of 40% and holds a tax‐exempt issue that has a yield of 6%. The taxable‐equivalent yield is: 

A. 2.4%.

B. 3.6%.

C. 10.0%.

D. 15.0%.

Correct Answer:  C .......................................................................................... LOS: Reading  65‐i taxable-equivalent yield = tax-exempt yield/(1 – marginal tax rate)

= 0.06/0.6 = 0.1 or 10%

Reference: CFA® Program Curriculum, Volume 5, pp. 359‐361. 

Page 423: Cfa Ques Answer

Fixed Income Investments: Basic Concepts   407 

 

5.  A floating‐rate note has the following coupon formula: 

Six‐month Treasury bill rate + 60 basis points with a cap of 7% and a floor of 6.5% 

The 6‐month Treasury bill rates are as follows: 

  6‐month  Treasury bill rate 

First reset date  6.5% Second reset date  5.8% Third reset date  6.3% Fourth reset date  6.1% 

 

What would be the coupon rates at the first and the second reset dates, respectively? 

A.  6.5%    5.8% 

B.  6.5%    6.5% 

C.  7.0%    6.5% 

D.  7.6%    7.1% 

 

 

 

 

6.  Liquidity risk in bond markets can be measured by: 

A.  recovery rates. 

B.   price volatility. 

C.  the bid‐ask spread. 

D.  the slope of the yield curve. 

Page 424: Cfa Ques Answer

408   Study Session 15:  

 

5.  A floating‐rate note has the following coupon formula: 

Six‐month Treasury bill rate + 60 basis points with a cap of 7% and a floor of 6.5% 

The 6‐month Treasury bill rates are as follows: 

  6‐month  Treasury bill rate 

First reset date  6.5% Second reset date  5.8% Third reset date  6.3% Fourth reset date  6.1% 

What would be the coupon rates at the first and the second reset dates, respectively? 

A. 6.5% 5.8%

B. 6.5% 6.5%

C. 7.0% 6.5%

D. 7.6% 7.1%

Correct Answer:  C ......................................................................................... LOS: Reading  62‐b The coupon rate at the first reset date is 6.5% + 0.6% = 7.1% which is higher than the cap rate. Therefore it takes on the cap rate, which is 7.0%.

The coupon rate at the second reset date is 5.8% + 0.6% = 6.4% which is lower than the floor rate. Therefore it takes on the floor rate, which is 6.5%.

Reference: CFA® Program Curriculum, Volume 5, pp. 242‐246. 

 

 

6.  Liquidity risk in bond markets can be measured by: 

A. recovery rates.

B. price volatility.

C. the bid-ask spread.

D. the slope of the yield curve.

Correct Answer:  C ......................................................................................... LOS: Reading  63‐k The bid-ask spread quoted by dealers reflects the liquidity of an issue.

Reference: CFA® Program Curriculum, Volume 5, pp. 281‐283. 

Page 425: Cfa Ques Answer

Fixed Income Investments: Basic Concepts   409 

 

7.  Which one of the following is the least likely to be an example of a securitized bond? 

A.  Mortgage bond. 

B.  Mortgage‐backed security. 

C.  Collateralized Mortgage Obligation. 

D.  Credit Card Receivables Asset‐Backed security. 

 

 

 

8.  The  difference  in  yield  between  bonds  issued  by  industrial  companies  and  bonds  issued  by information technology companies is an example of a: 

A.  credit spread. 

B.  quality spread. 

C.  intermarket sector spread. 

D.  intramarket sector spread. 

Page 426: Cfa Ques Answer

410   Study Session 15:  

 

7.  Which one of the following is the least likely to be an example of a securitized bond? 

A. Mortgage bond.

B. Mortgage-backed security.

C. Collateralized Mortgage Obligation.

D. Credit Card Receivables Asset-Backed security.

Correct Answer:  A ..........................................................................................LOS: Reading  64‐e Mortgage bonds are loans secured by mortgages to the company’s assets. They are not structured with securitization techniques, such as the use of bankruptcy-remote special purpose vehicles.

Reference: CFA® Program Curriculum, Volume 5, p. 305. 

 

 

 

8.  The  difference  in  yield  between  bonds  issued  by  industrial  companies  and  bonds  issued  by information technology companies is an example of a: 

A. credit spread.

B. quality spread.

C. intermarket sector spread.

D. intramarket sector spread.

Correct Answer:  C ..........................................................................................LOS: Reading  65‐e The credit or quality spread is the difference in yield between Treasury securities and non-Treasury securities. The intramarket sector spread is the difference in yield between two bonds with the same maturity, in the same sector in a market. Industrial companies are in a distinct sector to information technology companies.

Reference: CFA® Program Curriculum, Volume 5, pp. 355‐356. 

Page 427: Cfa Ques Answer

Fixed Income Investments: Basic Concepts   411 

 

9.  If the yield spread of a bond has widened it means that: 

A.  it is a low‐grade bond. 

B.  it is a high‐grade bond. 

C.  the bond price has risen relative to that of an equivalent Treasury bond issue. 

D.  the bond price has declined relative to that of an equivalent Treasury bond issue. 

 

 

 

10.  Medium‐term notes (MTNs) differ from corporate bonds since: 

A.  MTNs have a maximum maturity of 5 years. 

B.  MTNs have a maximum maturity of 10 years. 

C.  MTNs are not rated by any of the major credit rating organizations. 

D.  MTNs are offered to investors on a continuous basis by the issuer or its agent. 

Page 428: Cfa Ques Answer

412   Study Session 15:  

 

9.  If the yield spread of a bond has widened it means that: 

A. it is a low-grade bond.

B. it is a high-grade bond.

C. the bond price has risen relative to that of an equivalent Treasury bond issue.

D. the bond price has declined relative to that of an equivalent Treasury bond issue.

Correct Answer:  D.......................................................................................... LOS: Reading  63‐j The credit or yield spread is the difference between the yield on the bond and the yield on a Treasury bond. If the spread has widened it means that the yield demanded by an investor in the bond has increased, so the price has fallen, relative to a Treasury bond.

Reference: CFA® Program Curriculum, Volume 5, pp. 277‐278. 

 

 

 

10.  Medium‐term notes (MTNs) differ from corporate bonds since: 

A. MTNs have a maximum maturity of 5 years.

B. MTNs have a maximum maturity of 10 years.

C. MTNs are not rated by any of the major credit rating organizations.

D. MTNs are offered to investors on a continuous basis by the issuer or its agent.

Correct Answer:  D......................................................................................... LOS: Reading  64‐h The unique feature of MTNs is that they are offered to investors on a continuous basis by the issuer or its agent.

Reference: CFA® Program Curriculum, Volume 5, pp. 322‐325. 

Page 429: Cfa Ques Answer

Fixed Income Investments: Basic Concepts   413 

 

11.  From the equity markets, central banks find it most useful to derive information on: 

A.  future economic activity. 

B.  international trade activity. 

C.  market expectations of interest rates. 

D.  ownership of equities by individuals. 

 

 

 

12.  Which  of  the  following  embedded  options  is  increasingly  valuable  to  an  investor  in  a  rising interest rate environment? 

A.  Put option. 

B.  Call option. 

C.  Interest rate cap. 

D.  Accelerated sinking fund provision. 

Page 430: Cfa Ques Answer

414   Study Session 15:  

 

11.  From the equity markets, central banks find it most useful to derive information on: 

A. future economic activity.

B. international trade activity.

C. market expectations of interest rates.

D. ownership of equities by individuals.

Correct Answer:  A ......................................................................................... LOS: Reading  66‐b Equity markets provide information on anticipated economic activity.

Reference: CFA® Program Curriculum, Volume 5, pp. 379‐383. 

 

 

 

12.  Which  of  the  following  embedded  options  is  increasingly  valuable  to  an  investor  in  a  rising interest rate environment? 

A. Put option.

B. Call option.

C. Interest rate cap.

D. Accelerated sinking fund provision.

Correct Answer:  A ..........................................................................................LOS: Reading  62‐e From the investor perspective, a put option is beneficial when interest rates rise. The investor benefits from being able to sell back the bond at a specified price that is higher than the prevailing market price.

Reference: CFA® Program Curriculum, Volume 5, pp. 252‐254. 

Page 431: Cfa Ques Answer

Fixed Income Investments: Basic Concepts   415 

 

13.  The legal entity that is used in asset securitization for bankruptcy remoteness is: 

A.  a rating agency. 

B.  an indenture trustee. 

C.  an investment bank. 

D.  a special purpose vehicle. 

 

 

 

14.  A bond  is initially priced at 97.99. If the yield declines by 50 basis points, the price  increases to 99.99. If the yield rises by 50 basis points, the price decreases to 94.89. The duration of the bond is: 

A.  0.50. 

B.  1.00. 

C.  5.25. 

D.  10.50. 

Page 432: Cfa Ques Answer

416   Study Session 15:  

 

13.  The legal entity that is used in asset securitization for bankruptcy remoteness is: 

A. a rating agency.

B. an indenture trustee.

C. an investment bank.

D. a special purpose vehicle.

Correct Answer:  D.......................................................................................... LOS: Reading  64‐i A special purpose vehicle is the legal entity that the assets are sold to, so the assets used as collateral are separate from the firm that is requiring funding.

Reference: CFA® Program Curriculum, Volume 5, pp. 328‐329. 

 

 

 

14.  A bond  is initially priced at 97.99. If the yield declines by 50 basis points, the price  increases to 99.99. If the yield rises by 50 basis points, the price decreases to 94.89. The duration of the bond is: 

A. 0.50.

B. 1.00.

C. 5.25.

D. 10.50.

Correct Answer:  C .......................................................................................... LOS: Reading  63‐f The duration = (99.99 – 94.89)/(2 x 97.99 x 0.005) = 5.25

Reference: CFA® Program Curriculum, Volume 5, pp. 269‐271. 

Page 433: Cfa Ques Answer

Fixed Income Investments: Basic Concepts   417 

 

15.  A bond  is initially priced at 97.99. If the yield declines by 50 basis points, the price  increases to 99.99. If the yield rises by 50 basis points, the price decreases to 94.89. If the par value of the bond is $100,000 then the dollar duration is: 

A.  $500. 

B.  $1,000. 

C.  $5,250. 

D.  $10,500. 

 

 

 

16.  A sinking fund provision refers to when:  

A.  an issuer is required to retire a pre‐specified portion of a bond each year. 

B.  an issuer has made an arrangement to buy back a bond at par in the case of deterioration in the quality of collateral for a bond.  

C.  an investor has the right to sell the bonds back to the issuer prior to maturity if the market price has fallen below a prespecified level.  

D.  mortgagees are prepaid ahead of schedule and the cash received is set aside into a fund which will be used to pay back investors at a later date.  

Page 434: Cfa Ques Answer

418   Study Session 15:  

 

15.  A bond  is initially priced at 97.99. If the yield declines by 50 basis points, the price  increases to 99.99. If the yield rises by 50 basis points, the price decreases to 94.89. If the par value of the bond is $100,000 then the dollar duration is: 

A. $500.

B. $1,000.

C. $5,250.

D. $10,500.

Correct Answer:  C .......................................................................................... LOS: Reading  63‐f The duration = (99.99 – 94.89)/(2 x 97.99 x 0.005) = 5.25

The dollar duration will be 5.25% x $100,000 = $5,250

Reference: CFA® Program Curriculum, Volume 5, pp. 269‐271. 

 

 

 

16.  A sinking fund provision refers to when:  

A. an issuer is required to retire a pre-specified portion of a bond each year.

B. an issuer has made an arrangement to buy back a bond at par in the case of deterioration in the quality of collateral for a bond.

C. an investor has the right to sell the bonds back to the issuer prior to maturity if the market price has fallen below a prespecified level.

D. mortgagees are prepaid ahead of schedule and the cash received is set aside into a fund which will be used to pay back investors at a later date.

Correct Answer:  A ......................................................................................... LOS: Reading  62‐d A sinking fund requirement is usually put in place to reduce credit risk because the issuer has to retire a portion of the bond issue prior to maturity, either by buying bonds in the market or providing cash for the trustee to call bonds using a lottery system.

Reference: CFA® Program Curriculum, Volume 5, pp. 251‐255. 

Page 435: Cfa Ques Answer

Fixed Income Investments: Basic Concepts   419 

 

17.  Which a firm issues bonds that are nonrefundable this means that:   

A.  the bonds cannot be called prior to the maturity date. 

B.  the bonds can be called but the investor cannot be offered a replacement bond with a similar yield.  

C.  an investor cannot exercise an embedded put option whilst they are in a prespecified nonrefunding period. 

D.  the bonds cannot be redeemed using proceeds of another debt issue that has provided a lower cost source of funds. 

 

 

 

18.  A credible and transparent monetary policy of the ECB is reflected in:   

A.   stable exchange rates. 

B.  a strong euro against the dollar. 

C.  insignificant overnight interest rate moves following a policy announcement. 

D.  an increased level of implied volatility of the bond markets over the last few years.  

Page 436: Cfa Ques Answer

420   Study Session 15:  

 

17.  Which a firm issues bonds that are nonrefundable this means that:   

A. the bonds cannot be called prior to the maturity date.

B. the bonds can be called but the investor cannot be offered a replacement bond with a similar yield.

C. an investor cannot exercise an embedded put option whilst they are in a prespecified nonrefunding period.

D. the bonds cannot be redeemed using proceeds of another debt issue that has provided a lower cost source of funds.

Correct Answer:  D..........................................................................................LOS: Reading  62‐e When a callable bond is issued it may have restrictions on when it can be called, nonrefundable is such a restriction and means that the firm cannot issue a new bond on a lower yield to pay back the original bond holders.

Reference: CFA® Program Curriculum, Volume 5, pp. 247‐251. 

 

 

 

18.  A credible and transparent monetary policy of the ECB is reflected in:   

A. stable exchange rates.

B. a strong euro against the dollar.

C. insignificant overnight interest rate moves following a policy announcement.

D. an increased level of implied volatility of the bond markets over the last few years.

Correct Answer:  C ..........................................................................................LOS: Reading  66‐c A credible and transparent policy means that market participants can forecast very accurately any policy announcements so there is little change in rates following an announcement.

Reference: CFA® Program Curriculum, Volume 5, pp. 379‐381. 

Page 437: Cfa Ques Answer

Fixed Income Investments: Basic Concepts   421 

 

19.  The clean price of a bond is: 

A.  the agreed bond price without accrued interest. 

B.  the accrued interest component of the bond price. 

C.  the agreed bond price plus the accrued interest due to the seller of the bond. 

D.  the agreed bond price for the bond less the accrued interest due to the buyer of the bond.. 

 

 

 

20.  Which  of  the  following  is  least  likely  to  be  an  example  of  embedded  options  that might  be granted to the issuer of a bond? 

A.  A floor on a floater. 

B.  The right to call the issue. 

C.  An accelerated sinking fund provision. 

D.  The right to prepay an amount above the scheduled principal repayment. 

Page 438: Cfa Ques Answer

422   Study Session 15:  

 

19.  The clean price of a bond is: 

A. the agreed bond price without accrued interest.

B. the accrued interest component of the bond price.

C. the agreed bond price plus the accrued interest due to the seller of the bond.

D. the agreed bond price for the bond less the accrued interest due to the buyer of the bond.

Correct Answer:  A ......................................................................................... LOS: Reading  62‐b The clean price of a bond is the quoted price of a bond where the accrued interest is excluded.

Reference: CFA® Program Curriculum, Volume 5, p. 246. 

 

 

 

20.  Which  of  the  following  is  least  likely  to  be  an  example  of  embedded  options  that might  be granted to the issuer of a bond? 

A. A floor on a floater.

B. The right to call the issue.

C. An accelerated sinking fund provision.

D. The right to prepay an amount above the scheduled principal repayment.

Correct Answer:  A ......................................................................................... LOS: Reading  62‐b A floor on a floater is a benefit to the holder of the bond if interest rates fall, not to the issuer.

Reference: CFA® Program Curriculum, Volume 5, pp. 242‐246. 

Page 439: Cfa Ques Answer

Fixed Income Investments: Basic Concepts   423 

 

21.  The following information is given: 

 

Issue Yield10‐year on‐the‐run Treasury  6% coupon 6.15%10‐year ABG Corporation Series J  5% coupon 7.85%10‐year IBN Limited 8.0% coupon 8.00%

 

The absolute yield spread between IBN Limited and ABG Corporation bonds is: 

A.  0.015%. 

B.  3.000%. 

C.  3 basis points. 

D.  15 basis points. 

 

 

 

22.  The following information is given: 

 

Issue Yield10‐year on‐the‐run Treasury  6% coupon 6.15%10‐year ABG Corporation Series J  5% coupon 7.85%10‐year IBN Limited 8.0% coupon 8.00%

 

The yield ratio between the 10‐year ABG Corp and the 10‐year Treasury is closest to: 

A.  0.78. 

B.  0.83. 

C.  1.20. 

D.  1.28. 

Page 440: Cfa Ques Answer

424   Study Session 15:  

 

21.  The following information is given: 

Issue  Yield10‐year on‐the‐run Treasury  6% coupon 6.15%10‐year ABG Corporation Series J  5% coupon 7.85%10‐year IBN Limited 8.0% coupon  8.00%

The absolute yield spread between IBN Limited and ABG Corporation bonds is:

A. 0.015%.

B. 3.000%.

C. 3 basis points.

D. 15 basis points.

Correct Answer:  D..........................................................................................LOS: Reading  65‐e The absolute yield spread = 8.0% - 7.85% = 0.15% or 15 basis points.

Do not use the coupon rates.

Reference: CFA® Program Curriculum, Volume 5, pp. 352‐354. 

Page 441: Cfa Ques Answer

Fixed Income Investments: Basic Concepts   425 

 

22.  The following information is given: 

Issue Yield10‐year on‐the‐run Treasury  6% coupon 6.15%10‐year ABG Corporation Series J  5% coupon 7.85%10‐year IBN Limited 8.0% coupon 8.00%

The yield ratio between the 10-year ABG Corp and the 10-year Treasury is closest to:

A. 0.78.

B. 0.83.

C. 1.20.

D. 1.28.

Correct Answer:  D ......................................................................................... LOS: Reading  65‐e The yield ratio 7.85%/6.15% = 1.28.

Again do not use the coupon rates.

Reference: CFA® Program Curriculum, Volume 5, pp. 352‐354. 

 

Page 442: Cfa Ques Answer

426   Study Session 16:  

 

Study Session 16: Fixed Income Investments: Analysis and Valuation

 

This study session illustrates the primary tools for valuation and analysis of fixed income securities and markets. It begins with a study of basic valuation theory and techniques for bonds and concludes with a more in‐depth explanation of the primary tools for fixed income investment valuation, specifically, interest rate and yield valuation and interest rate risk measurement and analysis.  

 

Reading 67: Introduction to the Valuation of Debt Securities 

Reading 68: Yield Measures, Spot Rates, and Forward Rates 

Reading 69: Introduction to the Measurement of Interest Rate Risk 

 

Page 443: Cfa Ques Answer

Fixed Income Investments: Analysis and Valuation   427 

 

1.  If the duration of a bond portfolio is 5 and the average yield of bonds in the portfolio rises by 100 basis points then: 

A.  the market value of the portfolio will fall, but by less than 5%. 

B.  the market value of the portfolio will fall by approximately 5%. 

C.  the market value of the portfolio will rise by approximately 5%. 

D.  we do not have sufficient information to calculate the impact on the market value of the portfolio. 

 

 

 

2.  The Macaulay duration of a bond is: 

A.  either negative or positive. 

B.  the same as the maturity of the bond. 

C.  always positive and less than, or equal to, the term to maturity of a bond. 

D.  always positive and more than, or equal to, the term to maturity of a bond.. 

 

 

Page 444: Cfa Ques Answer

428   Study Session 16:  

 

1.  If the duration of a bond portfolio is 5 and the average yield of bonds in the portfolio rises by 100 basis points then: 

A. the market value of the portfolio will fall, but by less than 5%.

B. the market value of the portfolio will fall by approximately 5%.

C. the market value of the portfolio will rise by approximately 5%.

D. we do not have sufficient information to calculate the impact on the market value of the portfolio.

Correct Answer:  D.......................................................................................... LOS: Reading  69‐f The yield of each bond in the portfolio must rise by 100 basis points, namely there should be a parallel shift in the yield curve, for the portfolio duration to be an indicator of the price move of the total portfolio. We need to know information on whether there was a parallel shift in the yield curve or the curve changed shape (e.g. steepened or flattened), so D is the correct answer.

Reference: CFA® Program Curriculum, Volume 5, pp. 499‐501. 

 

 

 

2.  The Macaulay duration of a bond is: 

A. either negative or positive.

B. the same as the maturity of the bond.

C. always positive and less than, or equal to, the term to maturity of a bond.

D. always positive and more than, or equal to, the term to maturity of a bond.

Correct Answer:  C ..........................................................................................LOS: Reading  69‐e The unit of Macaulay duration is years. It is a measure of average time to the present value of cash flow receipts. Mathematically it means that Macaulay duration will always be positive and less than, or equal to, the term to maturity of a bond. It is only the same as the maturity when it is a zero coupon bond. Other measures of duration such as effective duration, can take any number either positive or negative because it better measures the interest rate sensitivity of bonds and takes into account changes in cash flows due to interest rate changes.

Reference: CFA® Program Curriculum, Volume 5, p. 497. 

Page 445: Cfa Ques Answer

Fixed Income Investments: Analysis and Valuation   429 

 

3.  The Macaulay duration of a semiannual bond with a 12% coupon is 6.0 and the yield to maturity is 8%. The modified duration is closest to: 

A.  5.66. 

B.  5.77. 

C.  6.24. 

D.  6.36. 

 

 

 

4.  If the semiannual yield to maturity of a bond is 5%, the bond‐equivalent yield is closest to: 

A.  5.1%. 

B.  10.0%. 

C.  10.3%. 

D.  10.6%. 

Page 446: Cfa Ques Answer

430   Study Session 16:  

 

3.  The Macaulay duration of a semiannual bond with a 12% coupon is 6.0 and the yield to maturity is 8%. The modified duration is closest to: 

A. 5.66.

B. 5.77.

C. 6.24.

D. 6.36.

Correct Answer:  B...........................................................................................LOS: Reading  69‐e Semiannual bonds mean k = 2

Modified duration = Macaulay duration/[(1 + yield/k)]

= 6.0/(1.04)

= 5.77

Reference: CFA® Program Curriculum, Volume 5, p. 497. 

 

 

 

4.  If the semiannual yield to maturity of a bond is 5%, the bond‐equivalent yield is closest to: 

A. 5.1%.

B. 10.0%.

C. 10.3%.

D. 10.6%.

Correct Answer:  B.......................................................................................... LOS: Reading  68‐d The bond-equivalent yield is simply double the semiannual yield.

Reference: CFA® Program Curriculum, Volume 5, pp. 424‐425. 

Page 447: Cfa Ques Answer

Fixed Income Investments: Analysis and Valuation   431 

 

5.  The total dollar return from a bond is: 

A.  the current yield multiplied by the price of the bond. 

B.  the yield to maturity multiplied by the price of the bond. 

C.  the coupon income plus the reinvestment income plus the capital gain/loss on the bond. 

D.  the coupon income plus the value of the quoted margin plus the capital gain/loss on the bond. 

 

 

 

6.  An 8% bond makes semiannual coupon payments, there are two further coupons to be paid and the bond matures at $100 in 273 days (there are 182 days in a coupon period). If the yield to maturity is 6% then the full price of the bond is closest to: 

A.  $96.12. 

B.  $98.27. 

C.  $101.91. 

D.  $103.42. 

Page 448: Cfa Ques Answer

432   Study Session 16:  

 

5.  The total dollar return from a bond is: 

A. the current yield multiplied by the price of the bond.

B. the yield to maturity multiplied by the price of the bond.

C. the coupon income plus the reinvestment income plus the capital gain/loss on the bond.

D. the coupon income plus the value of the quoted margin plus the capital gain/loss on the bond.

Correct Answer:  C ..........................................................................................LOS: Reading  68‐a The total dollar return is the coupon income plus the reinvestment income plus the capital gain/loss on the bond. Multiplying by yield to maturity may be inaccurate since it assumes that coupons can be reinvested at the rate equivalent to the yield to maturity.

Reference: CFA® Program Curriculum, Volume 5, pp. 42‐423. 

 

 

 

6.  An 8% bond makes semiannual coupon payments, there are two further coupons to be paid and the bond matures at $100 in 273 days (there are 182 days in a coupon period). If the yield to maturity is 6% then the full price of the bond is closest to: 

A. $96.12.

B. $98.27.

C. $101.91.

D. $103.42.

Correct Answer:  D..........................................................................................LOS: Reading  67‐c 273 days is equivalent to 1 ½ coupon periods so:

PV = $4/(1.03)1/2 + $104/(1.03)3/2

= $3.94 + $99.49

= $103.42

Reference: CFA® Program Curriculum, Volume 5, pp. 392‐395. 

Page 449: Cfa Ques Answer

Fixed Income Investments: Analysis and Valuation   433 

 

7.  The appropriate duration measure to use for a high coupon callable bond is: 

A.  effective duration. 

B.  modified duration. 

C.  Macaulay duration. 

D.  option‐adjusted duration. 

 

 

 

8.  If the yield to maturity on an annual‐pay bond is 12% its bond‐equivalent yield is closest to: 

A.  11.66%. 

B.  12.00%. 

C.  12.36%. 

D.  12.72%. 

Page 450: Cfa Ques Answer

434   Study Session 16:  

 

7.  The appropriate duration measure to use for a high coupon callable bond is: 

A. effective duration.

B. modified duration.

C. Macaulay duration.

D. option-adjusted duration.

Correct Answer:  A ..........................................................................................LOS: Reading  69‐e In a callable bond, the cash flows might change as interest rates change. The most appropriate duration measure is effective duration.

Reference: CFA® Program Curriculum, Volume 5, p. 496. 

 

 

 

8.  If the yield to maturity on an annual‐pay bond is 12% its bond‐equivalent yield is closest to: 

A. 11.66%.

B. 12.00%.

C. 12.36%.

D. 12.72%.

Correct Answer:  A ......................................................................................... LOS: Reading  68‐d bond-equivalent yield = 2[(1+ yield on annual-pay bond)1/2 –1] = 11.66%

Reference: CFA® Program Curriculum, Volume 5, p. 431. 

Page 451: Cfa Ques Answer

Fixed Income Investments: Analysis and Valuation   435 

 

9.  A non‐callable bond has: 

A.  positive convexity throughout the yield range. 

B.  negative convexity throughout the yield range. 

C.  positive convexity at low yields and negative convexity at high yields. 

D.  negative convexity at low yields and positive convexity at high yields. 

 

 

 

10.  When a dealer purchases a package of Treasury strips  to create a synthetic Treasury bond,  the procedure is called: 

A.  hedging. 

B.  stripping. 

C.  reconstitution. 

D.  bootstrapping. 

Page 452: Cfa Ques Answer

436   Study Session 16:  

 

9.  A non‐callable bond has: 

A. positive convexity throughout the yield range.

B. negative convexity throughout the yield range.

C. positive convexity at low yields and negative convexity at high yields.

D. negative convexity at low yields and positive convexity at high yields.

Correct Answer:  A ......................................................................................... LOS: Reading  69‐b The presence of embedded options, such as a call option in a bond, would make the convexity negative in certain yield ranges. Non-callable bonds have positive convexity throughout the yield range.

Reference: CFA® Program Curriculum, Volume 5, pp. 484‐488. 

 

 

 

10.  When a dealer purchases a package of Treasury strips  to create a synthetic Treasury bond,  the procedure is called: 

A. hedging.

B. stripping.

C. reconstitution.

D. bootstrapping.

Correct Answer:  C .......................................................................................... LOS: Reading  67‐f Reconstitution occurs when a Treasury bond’s price is greater than its arbitrage-free value, and a dealer can purchase a package of Treasury strips to create a synthetic Treasury bond.

Reference: CFA® Program Curriculum, Volume 5, p. 410. 

Page 453: Cfa Ques Answer

Fixed Income Investments: Analysis and Valuation   437 

 

11.  Which of the following is the least accurate statement regarding reinvestment risk? 

A.  For zero‐coupon bonds, the reinvestment income is important to produce the yield promised at purchase date. 

B.  The longer the time to maturity the more the bond is dependent on the reinvestment income to produce the yield promised at the purchase date. 

C.  The higher the coupon rate the more the bond is dependent on the reinvestment income to produce the yield promised at the purchase date. 

D.  For bonds bought at a premium, rather than at a discount, the reinvestment income is more important to produce the yield promised at the purchase date. 

 

 

 

12.  Given the following information on a bond: 

Duration  =    7.5670. 

Convexity  =  41.0935. 

Yields fall by:    250 basis points. 

 

The total estimated price change in percentage terms is closest to: 

A.  2.57%. 

B.  7.57%. 

C.  18.92%. 

D.  21.49%. 

Page 454: Cfa Ques Answer

438   Study Session 16:  

 

11.  Which of the following is the least accurate statement regarding reinvestment risk? 

A. For zero-coupon bonds, the reinvestment income is important to produce the yield promised at purchase date.

B. The longer the time to maturity the more the bond is dependent on the reinvestment income to produce the yield promised at the purchase date.

C. The higher the coupon rate the more the bond is dependent on the reinvestment income to produce the yield promised at the purchase date.

D. For bonds bought at a premium, rather than at a discount, the reinvestment income is more important to produce the yield promised at the purchase date.

Correct Answer:  A ..........................................................................................LOS: Reading  68‐c Zero-coupon bonds have no reinvestment risk since they have no interim income from coupons to be reinvested.

Reference: CFA® Program Curriculum, Volume 5, pp. 425‐430. 

 

 

 

12.  Given the following information on a bond: 

Duration = 7.5670.

Convexity = 41.0935.

Yields fall by: 250 basis points.

The total estimated price change in percentage terms is closest to:

A. 2.57%.

B. 7.57%.

C. 18.92%.

D. 21.49%.

Correct Answer:  D......................................................................................... LOS: Reading  69‐d Estimated change using duration 7.5670% x 0.025 x 100 = 18.9175

Convexity adjustment: 41.0935 x (0.025)2 x 100 = 2.5684

Total estimated percentage price change = 21.4858

Reference: CFA® Program Curriculum, Volume 5, pp. 488‐492. 

Page 455: Cfa Ques Answer

Fixed Income Investments: Analysis and Valuation   439 

 

13.  An option‐free bond has a  remaining  life of  three years and carries an 8% annual coupon  rate payable annually and has a yield to maturity of 9%. If the one‐ and two‐year spot rates are 6.0% and 6.5%, respectively, then the three‐year spot rate is closest to: 

A.  7.0%. 

B.  8.1%. 

C.  9.2%. 

D.  14.5%. 

 

 

 

14.  Which of the following is least likely to be a characteristic of bonds with positive convexity? 

A.  The percentage price change for a given yield change is not the same for all bonds. 

B.  For a large change in yields, the percentage price decrease is greater than the percentage price increase. 

C.  For a small change in yields, the percentage changes are roughly the same, whether the yield increases or decreases. 

D.  For a large change in yields, the percentage price change is not the same for an increase in yields as it is for a decrease in yields. 

Page 456: Cfa Ques Answer

440   Study Session 16:  

 

13.  .  An option‐free bond has a remaining  life of  three years and carries an 8% annual coupon rate payable annually and has a yield to maturity of 9%. If the one‐ and two‐year spot rates are 6.0% and 6.5%, respectively, then the three‐year spot rate is closest to: 

A. 7.0%.

B. 8.1%.

C. 9.2%.

D. 14.5%.

Correct Answer:  C ......................................................................................... LOS: Reading  68‐h The following relationship must hold, where the three-year spot rate is z:

8/(1.09)+8/(1.09)2+108/(1.09)3=8/(1.06)+8/(1.065)2+108/(1+z)3 97.47 = 14.60+108/(1+z)3 (1+z)3 = 108/82.87 1+z = (1.3032)1/3 z = 9.23%  

Reference: CFA® Program Curriculum, Volume 5, pp. 458‐461. 

 

 

 

14.  Which of the following is least likely to be a characteristic of bonds with positive convexity? 

A. The percentage price change for a given yield change is not the same for all bonds.

B. For a large change in yields, the percentage price decrease is greater than the percentage price increase.

C. For a small change in yields, the percentage changes are roughly the same, whether the yield increases or decreases.

D. For a large change in yields, the percentage price change is not the same for an increase in yields as it is for a decrease in yields.

Correct Answer:  B...........................................................................................LOS: Reading  69‐c For a large change in yields, the percentage price increase is greater than the percentage price decrease, when a bond has positive convexity.

Reference: CFA® Program Curriculum, Volume 5, pp. 501‐503. 

Page 457: Cfa Ques Answer

Fixed Income Investments: Analysis and Valuation   441 

 

15.  An  analyst  uses  a model  to  calculate  effective  duration  and  finds  a  collateralized mortgage obligation (CMO) which has duration which is longer than the life of the underlying mortgage loans. Which of the following statements is most accurate? 

A.  Effective duration is an inappropriate measure of duration to use since there are no embedded options in a CMO. 

B.  The model looks to have produced an incorrect estimate of duration since CMOs would be expected to have negative duration. 

C.  It is inappropriate to use a model to estimate effective duration; inputs should be taken from a market source, such as dealers’ prices. 

D.  Longer duration of the CMO than the mortgage loans is possible and simply indicates the sensitivity of the CMO to interest rate moves.  

 

 

 

16.  When interest rates change, it is observed that the price of a callable bond with a coupon of 5% moves as follows:  

 

Rates down 50 basis points  Rates up 50 basis points Price + 5%  Price ‐ 3% 

 

The bond is likely to have 

  Convexity    Yield 

A.  Positive    Less than 5% 

B.  Positive    More than 5% 

C.  Negative    Less than 5% 

D.  Negative    More than 5% 

Page 458: Cfa Ques Answer

442   Study Session 16:  

 

15.  An  analyst  uses  a model  to  calculate  effective  duration  and  finds  a  collateralized mortgage obligation (CMO) which has duration which is longer than the life of the underlying mortgage loans. Which of the following statements is most accurate? 

A. Effective duration is an inappropriate measure of duration to use since there are no embedded options in a CMO.

B. The model looks to have produced an incorrect estimate of duration since CMOs would be expected to have negative duration.

C. It is inappropriate to use a model to estimate effective duration; inputs should be taken from a market source, such as dealers’ prices.

D. Longer duration of the CMO than the mortgage loans is possible and simply indicates the sensitivity of the CMO to interest rate moves.

Correct Answer:  D..........................................................................................LOS: Reading  69‐e Effective duration is the correct method to use since cash flows from the CMO are influenced by interest rates. It is reasonable to use a model to calculate effective duration and effective duration can be positive (and longer then the maturity of the underlying loans) or negative depending on the structure and terms of the CMO.

Reference: CFA® Program Curriculum, Volume 5, pp. 496‐499. 

 

16.  When interest rates change, it is observed that the price of a callable bond with a coupon of 5% moves as follows:  

Rates down 50 basis points  Rates up 50 basis points Price + 5%  Price ‐ 3% 

The bond is likely to have

Convexity Yield

A. Positive Less than 5%

B. Positive More than 5%

C. Negative Less than 5%

D. Negative More than 5%

Correct Answer:  B...........................................................................................LOS: Reading  69‐c The bond has positive convexity since the gain is more than the loss for an equal move in interest rates. The bond’s coupon is probably less than market yields meaning it is unlikely to be called in the near future. As yields fall the bond is more likely to be called and starts to exhibit negative convexity.

Reference: CFA® Program Curriculum, Volume 5, pp. 484‐487. 

Page 459: Cfa Ques Answer

Fixed Income Investments: Analysis and Valuation   443 

 

17.  A zero‐coupon bond with two years remaining to maturity  is currently trading at $82.65. If the par value is $100, the yield to maturity is closest to: 

A.  8.7%. 

B.  9.0%. 

C.  9.8%. 

D.  11.0%. 

 

 

 

18.  A bond without any embedded options has a remaining life of three years, carries an 8% coupon rate payable annually, and has a yield to maturity of 7%. If the one‐ and three‐year spot rates are 8.0% and 7.0%, respectively, then the two‐year spot rate is closest to: 

A.  6.0%. 

B.  6.5%. 

C.  7.5%. 

D.  13.5%. 

Page 460: Cfa Ques Answer

444   Study Session 16:  

 

17.  A zero‐coupon bond with two years remaining to maturity  is currently trading at $82.65. If the par value is $100, the yield to maturity is closest to: 

A. 8.7%.

B. 9.0%.

C. 9.8%.

D. 11.0%.

Correct Answer:  C ..........................................................................................LOS: Reading  67‐e Let x be the semiannual yield we are looking for, then:

( )( )

0488.0x21.165.82100x1

x110065.82

4

4

===+

+=

or 9.8% annual yield

Reference: CFA® Program Curriculum, Volume 5, pp. 398‐399. 

 

18.  A bond without any embedded options has a remaining life of three years, carries an 8% coupon rate payable annually, and has a yield to maturity of 7%. If the one‐ and three‐year spot rates are 8.0% and 7.0%, respectively, then the two‐year spot rate is closest to: 

A. 6.0%.

B. 6.5%.

C. 7.5%.

D. 13.5%.

Correct Answer:  B.......................................................................................... LOS: Reading  68‐h The following relationship must hold, where z is the two-year spot rate:

8/(1.07) + 8/(1.07)2 + 108/(1.07)3 = 8/(1.08) + 8/(1+ z)2 + 108/(1.07)3

14.46 = 7.41 + 8/(1+z)2

(1 + z)2 = 8/7.06

1 + z = (1.1337)1/2

z = 6.45%

Reference: CFA® Program Curriculum, Volume 5, pp. 458‐461. 

Page 461: Cfa Ques Answer

Fixed Income Investments: Analysis and Valuation   445 

 

19.  The price value of a basis point is: 

A.  the absolute change in price of a bond if there is a one basis point change in yield. 

B.  the percentage change in price of a bond if there is a one basis point change in yield. 

C.  the absolute change in dollar duration of a bond if there is a one hundred basis point change in yield. 

D.  the percentage change in dollar duration of a bond if there is a one hundred basis point change in yield. 

 

 

 

20.  The Z‐spread is: 

A.  the option‐adjusted spread if a bond has embedded options. 

B.  the adjustment to the option‐adjusted spread as volatility changes. 

C.  the spread between spot and forward rates over the life of a bond. 

D.  the spread earned on a bond over the Treasury spot rate curve if it is held to maturity. 

Page 462: Cfa Ques Answer

446   Study Session 16:  

 

19.  The price value of a basis point is: 

A. the absolute change in price of a bond if there is a one basis point change in yield.

B. the percentage change in price of a bond if there is a one basis point change in yield.

C. the absolute change in dollar duration of a bond if there is a one hundred basis point change in yield.

D. the percentage change in dollar duration of a bond if there is a one hundred basis point change in yield.

Correct Answer:  A .......................................................................................... LOS: Reading  69‐i The price value of a basis point is the absolute change in price of a bond if there is a one basis point change in yield. It is a special case of dollar duration.

Reference: CFA® Program Curriculum, Volume 5, pp. 504‐505. 

 

 

 

20.  The Z‐spread is: 

A. the option-adjusted spread if a bond has embedded options.

B. the adjustment to the option-adjusted spread as volatility changes.

C. the spread between spot and forward rates over the life of a bond.

D. the spread earned on a bond over the Treasury spot rate curve if it is held to maturity.

Correct Answer:  D.......................................................................................... LOS: Reading  68‐f The Z-spread, also called zero volatility or static spread, is the spread that must be added to the Treasury spot rate to give a discount rate which will make the discounted cash flows from a bond equal to its price.

Reference: CFA® Program Curriculum, Volume 5, pp. 445‐450. 

Page 463: Cfa Ques Answer

Fixed Income Investments: Analysis and Valuation   447 

 

21.  The following data is collected:  

Years to  Spot rate 0.5  5.75% 1.0  6.25% 1.5  7.00% 2.0  7.25% 

 

Based on the above data, the six‐month implied forward rate one and a half years from now is closest to: 

A.  4.00%. 

B.  6.25%. 

C.  6.75%. 

D.  8.00%. 

 

 

 

22.  Given: 

 

Period  Years  Annual yield to maturity (BEY) 

Price  Spot rate (BEY) 

1  0.5  4.00    4.0000 2  1.0  4.30    4.3000 3  1.5  4.65  100  4.6606 

 

The 6‐month forward rate, one year from now is closest to: 

A.  4.3000%. 

B.  4.4803%. 

C.  5.3535%. 

D.  5.3837%. 

Page 464: Cfa Ques Answer

448   Study Session 16:  

 

21.  The following data is collected: 

Years to  Spot rate 0.5  5.75% 1.0  6.25% 1.5  7.00% 2.0  7.25% 

 

Based on the above data, the six‐month implied forward rate one and a half years from now is closest to: 

A. 4.00%.

B. 6.25%.

C. 6.75%.

D. 8.00%.

Correct Answer:  D......................................................................................... LOS: Reading  68‐h 

1f3 = [(1+ 0.03625)4/(1 + 0.035)3] – 1 = 4.0% The forward rate is 4.0% x 2 = 8.0%  

Reference: CFA® Program Curriculum, Volume 5, pp. 453‐458. 

Page 465: Cfa Ques Answer

Fixed Income Investments: Analysis and Valuation   449 

 

22.  Given: 

Period  Years  Annual yield to maturity (BEY) 

Price  Spot rate (BEY) 

1  0.5  4.00    4.0000 2  1.0  4.30    4.3000 3  1.5  4.65  100  4.6606 

The 6-month forward rate, one year from now is closest to:

A. 4.3000%.

B. 4.4803%.

C. 5.3535%.

D. 5.3837%.

Correct Answer:  D .........................................................................................LOS: Reading  68‐h 1f2 = (1 + z3)3/(1 + z2)2 -1 = (1.023303)3/(1.02150)2 -1 = 1.0269 – 1 = 2.6919%

The 6-month forward rate, one year from now, is 2 x 2.6919% = 5.3837%

Reference: CFA® Program Curriculum, Volume 5, pp. 453‐458. 

 

Page 466: Cfa Ques Answer

450   Study Session 17:  

 

Study Session 17: Derivative Investments:  

Derivatives—financial instruments that offer a return based on the return of some underlying asset—have become increasingly important and fundamental in effectively managing financial risk and creating synthetic exposures to asset classes. As in other security markets, arbitrage and market efficiency play a critical role in establishing prices and maintaining parity.  

This study session builds the conceptual framework for understanding derivative investments (forwards, futures, options, and swaps), derivative markets, and the use of options in risk management.   

 

Reading 70: Derivative Markets and Instruments 

Reading 71: Forward Markets and Contracts 

Reading 72: Futures Markets and Contracts 

Reading 73: Option Markets and Contracts 

Reading 74: Swap Markets and Contracts 

Reading 75: Risk Management Applications of Option Strategies 

 

Page 467: Cfa Ques Answer

Derivative Investments   451 

 

1.  A  trader writes a European  call option on a  stock. The  stock’s  current price  is $24,  the option price  is $4 and  the exercise price  is $23. At  the expiration of  the option  the stock price  is $30. The profit/loss of the option writer is a: 

A.  loss of $4. 

B.  loss of $3. 

C.  profit of $3. 

D.  profit of $4. 

 

 

 

2.  The notional principal in a plain vanilla interest rate swap is: 

A.  never paid. 

B.  paid at the time that the swap agreement is made. 

C.  paid in equal parts when each swap payment is made. 

D.  paid at the time that the swap agreement is signed and returned when the final swap payment is made. 

 

 

Page 468: Cfa Ques Answer

452   Study Session 17:  

 

1.  A  trader writes a European  call option on a  stock. The  stock’s  current price  is $24,  the option price  is $4 and  the exercise price  is $23. At  the expiration of  the option  the stock price  is $30. The profit/loss of the option writer is a: 

A. loss of $4.

B. loss of $3.

C. profit of $3.

D. profit of $4.

Correct Answer:  B.......................................................................................... LOS: Reading  75‐b The writer has received the premium of $4 but the loss when the option is exercised is $30 minus $23 giving an overall loss of $3.

Reference: CFA® Program Curriculum, Volume 6, pp. 151‐158. 

 

 

 

2.  The notional principal in a plain vanilla interest rate swap is: 

A. never paid.

B. paid at the time that the swap agreement is made.

C. paid in equal parts when each swap payment is made.

D. paid at the time that the swap agreement is signed and returned when the final swap payment is made.

Correct Answer:  A ......................................................................................... LOS: Reading  74‐b The notional principal is the amount on which the interest payments are calculated and does not change hands in an interest rate swap.

Reference: CFA® Program Curriculum, Volume 6, pp. 134‐138. 

Page 469: Cfa Ques Answer

Derivative Investments   453 

 

3.  The value of a put option at expiry is: 

A.  maximum of (i) zero and (ii) exercise price minus stock price. 

B.  maximum of (i) zero and (ii) stock price minus exercise price. 

C.  maximum of (i) zero and (ii) stock price minus exercise price, minus the option premium. 

D.  maximum of (i) zero and (ii) exercise price minus stock price, minus the option premium. 

 

 

 

4.  A  company  has  borrowed  to  finance  its  operations  using  floating‐rate  debt  but  it  is  now concerned  that  short‐term  interest  rates  are  going  to  rise  sharply.  The  company  should  consider entering into: 

A.  a currency swap agreement where they take the pay‐fixed side  

of the transaction. 

B.  a currency rate swap agreement where they take the receive‐fixed side of the transaction. 

C.  a plain vanilla interest rate swap agreement where they take the pay‐fixed side of the transaction. 

D.  a plain vanilla interest rate swap agreement where they take the receive‐fixed side of the transaction. 

Page 470: Cfa Ques Answer

454   Study Session 17:  

 

3.  The value of a put option at expiry is: 

A. maximum of (i) zero and (ii) exercise price minus stock price.

B. maximum of (i) zero and (ii) stock price minus exercise price.

C. maximum of (i) zero and (ii) stock price minus exercise price, minus the option premium.

D. maximum of (i) zero and (ii) exercise price minus stock price, minus the option premium.

Correct Answer:  A ..........................................................................................LOS: Reading  73‐e The holder of a put option will exercise the option if the exercise price is above the stock price. In that case he could theoretically buy the stock in the market and sell it at a higher price. If the stock price is higher than the exercise price he will let the option lapse worthless. The option premium should only be taken into account if the profit/loss on the option is being calculated.

Reference: CFA® Program Curriculum, Volume 6, pp. 96‐99. 

 

 

 

4.  A  company  has  borrowed  to  finance  its  operations  using  floating‐rate  debt  but  it  is  now concerned  that  short‐term  interest  rates  are  going  to  rise  sharply.  The  company  should  consider entering into: 

A. a currency swap agreement where they take the pay-fixed side

of the transaction.

B. a currency rate swap agreement where they take the receive-fixed side of the transaction.

C. a plain vanilla interest rate swap agreement where they take the pay-fixed side of the transaction.

D. a plain vanilla interest rate swap agreement where they take the receive-fixed side of the transaction.

Correct Answer:  C ......................................................................................... LOS: Reading  74‐b In a plain vanilla interest rate swap if the company pays a fixed rate of interest and receives a floating rate of interest, the floating rate interest payment will, if the agreement is correctly structured, offset the interest rate risk of its debt payments.

Reference: CFA® Program Curriculum, Volume 6, pp. 134‐138. 

Page 471: Cfa Ques Answer

Derivative Investments   455 

 

5.  When a trader writes a covered call this will often be with the objective of: 

A.  increasing income. 

B.  insuring his portfolio value. 

C.  reducing the volatility of his return. 

D.  increasing his gain if the stock price rises above the exercise price plus the premium. 

 

 

 

6.  An  investor  believes  that  the  S&P  Index  is  going  to decline  sharply  over  the  next  two  years. Which of the following strategies would be consistent with this view? 

A.  Buy call options on the S&P Index. 

B.  Write put options on the S&P Index. 

C.  Take a long position in futures on the S&P Index. 

D.  Enter into a two‐year equity swap to receive a fixed payment and pay an equity payment based on the performance of the S&P index. 

Page 472: Cfa Ques Answer

456   Study Session 17:  

 

5.  When a trader writes a covered call this will often be with the objective of: 

A. increasing income.

B. insuring his portfolio value.

C. reducing the volatility of his return.

D. increasing his gain if the stock price rises above the exercise price plus the premium.

Correct Answer:  A ......................................................................................... LOS: Reading  75‐b Writing a covered call means that the trader will increase his income by the option premium. If the stock price rises above the exercise price his shares will be called and he will lose the capital gain he would have made if he had not written the option.

Reference: CFA® Program Curriculum, Volume 6, pp. 151‐158. 

 

 

 

6.  An  investor  believes  that  the  S&P  Index  is  going  to decline  sharply  over  the  next  two  years. Which of the following strategies would be consistent with this view? 

A. Buy call options on the S&P Index.

B. Write put options on the S&P Index.

C. Take a long position in futures on the S&P Index.

D. Enter into a two-year equity swap to receive a fixed payment and pay an equity payment based on the performance of the S&P index.

Correct Answer:  D......................................................................................... LOS: Reading  74‐b If the S&P Index falls the investor will receive both the fixed payment and an equity payment so this would be a viable strategy.

Reference: CFA® Program Curriculum, Volume 6, pp. 138‐141. 

Page 473: Cfa Ques Answer

Derivative Investments   457 

 

7.  An  investor deposits an  initial margin of $20,000  for a  futures  trade and  the next day makes a $2,000  loss  on  the  trade.  The  next  day  he  makes  a  further  loss  of  $2,000.  If  the  maintenance requirement is $15,000 then he must deposit a variation margin of: 

A.  $1,000. 

B.  $4,000. 

C.  $5,000. 

D.  there is no requirement to pay a variation margin. 

 

 

 

8.  A trader sells both a call and a put option on a stock with the same exercise price and the same expiration, he will make a profit on the transaction: 

A.  if the stock price rises sharply or falls sharply. 

B.  only if the stock price falls sharply below the exercise price. 

C.  only if the stock price rises sharply above the exercise price. 

D.  if the stock price remains within a narrow range of the exercise price. 

Page 474: Cfa Ques Answer

458   Study Session 17:  

 

7.  An  investor deposits an  initial margin of $20,000  for a  futures  trade and  the next day makes a $2,000  loss  on  the  trade.  The  next  day  he  makes  a  further  loss  of  $2,000.  If  the  maintenance requirement is $15,000 then he must deposit a variation margin of: 

A. $1,000.

B. $4,000.

C. $5,000.

D. there is no requirement to pay a variation margin.

Correct Answer:  D......................................................................................... LOS: Reading  72‐b The variation margin only needs to be paid if the investor’s equity has fallen below the maintenance requirement. This is not the case since the equity is still $16,000 ($20,000 - $2,000 - $2,000).

Reference: CFA® Program Curriculum, Volume 6, pp. 55‐60. 

 

 

 

8.  A trader sells both a call and a put option on a stock with the same exercise price and the same expiration, he will make a profit on the transaction: 

A. if the stock price rises sharply or falls sharply.

B. only if the stock price falls sharply below the exercise price.

C. only if the stock price rises sharply above the exercise price.

D. if the stock price remains within a narrow range of the exercise price.

Correct Answer:  D..........................................................................................LOS: Reading  75‐a This is a straddle (not explicitly covered in the readings) but the candidate can work out that if the stock price moves up sharply the call option would be exercised, or if it moves down sharply the put option would be exercised. If the move is significant the loss made on either option would be greater than the premium income received. His strategy is profitable if the stock price does not move by more than the combined value of the premiums received away from the exercise price.

Reference: CFA® Program Curriculum, Volume 6, pp. 151‐158. 

Page 475: Cfa Ques Answer

Derivative Investments   459 

 

9.  If put options are used to insure a portfolio which of the following statements is  

most accurate? 

A.  The insured portfolio will only report substantial losses in a small number of cases. 

B.  In the majority of cases the insured portfolio will outperform an equivalent uninsured portfolio. 

C.  The probability of the insured portfolio achieving high positive gains is less than for the uninsured portfolio. 

D.  There is a higher probability that the insured portfolio will achieve any given positive return than the uninsured portfolio. 

 

 

 

10.  An option on a futures is best described as: 

A.  an option where the underlying asset is a futures contract. 

B.  a futures contract where the holder agrees to buy a pre‐specified option at a future date. 

C.  a futures contract where the holder has an option at the delivery date to extend the contract. 

D.  a futures contract where the holder has the option to buy a pre‐specified asset at an agreed price at a future date. 

Page 476: Cfa Ques Answer

460   Study Session 17:  

 

9.  If put options are used to insure a portfolio which of the following statements is  

most accurate?

A. The insured portfolio will only report substantial losses in a small number of cases.

B. In the majority of cases the insured portfolio will outperform an equivalent uninsured portfolio.

C. The probability of the insured portfolio achieving high positive gains is less than for the uninsured portfolio.

D. There is a higher probability that the insured portfolio will achieve any given positive return than the uninsured portfolio.

Correct Answer:  C ......................................................................................... LOS: Reading  75‐b Correctly insuring a portfolio using put options should eliminate the possibility of a large loss since the portfolio value will not fall below the exercise price less the premium. But if the return from the assets is positive the uninsured portfolio will outperform the insured portfolio because of the cost of the premium.

Reference: CFA® Program Curriculum, Volume 6, pp. 162‐165. 

 

 

 

10.  An option on a futures is best described as: 

A. an option where the underlying asset is a futures contract.

B. a futures contract where the holder agrees to buy a pre-specified option at a future date.

C. a futures contract where the holder has an option at the delivery date to extend the contract.

D. a futures contract where the holder has the option to buy a pre-specified asset at an agreed price at a future date.

Correct Answer:  A ......................................................................................... LOS: Reading  73‐b An option on a futures is simply an option where the underlying asset is a futures contract. The option holder has the right to enter into a futures contract (call option for a long position, put for a short position) at a fixed price.

Reference: CFA® Program Curriculum, Volume 6, p. 94. 

Page 477: Cfa Ques Answer

Derivative Investments   461 

 

11.  A corporate treasurer knows that his firm will receive a large cash inflow in 180 days’ time. He is concerned that  interest rates are going to fall and he wishes to put the money on 90‐day LIBOR on receipt. He should consider taking a: 

A.  long position in a 6 x 3 FRA. 

B.  short position in a 6 x 3 FRA. 

C.  long position in a 6 x 9 FRA. 

D.  short position in a 6 x 9 FRA. 

 

 

 

12.  An investor buys a call option at a premium of $10 on a stock which has a market price of $60. If the exercise price is $62 the investor will make a: 

A.  loss if the stock price rises above the breakeven point of $70. 

B.  loss if the stock price rises above the breakeven point of $72. 

C.  profit if the stock price rises above the breakeven point of $70. 

D.  profit if the stock price rises above the breakeven point of $72. 

Page 478: Cfa Ques Answer

462   Study Session 17:  

 

11.  A corporate treasurer knows that his firm will receive a large cash inflow in 180 days’ time. He is concerned that  interest rates are going to fall and he wishes to put the money on 90‐day LIBOR on receipt. He should consider taking a: 

A. long position in a 6 x 3 FRA.

B. short position in a 6 x 3 FRA.

C. long position in a 6 x 9 FRA.

D. short position in a 6 x 9 FRA.

Correct Answer:  D.......................................................................................... LOS: Reading  71‐f A short position will generate a profit if interest rates fall which will offset the loss from placing the cash inflow on deposit at a lower rate. A 6 x 9 FRA is required, that is a 180-day forward contract based on deposits that mature in 270 days. He is effectively locking in the rate he will receive in 180 days’ time.

Reference: CFA® Program Curriculum, Volume 6, pp. 40‐43. 

 

 

 

12.  An investor buys a call option at a premium of $10 on a stock which has a market price of $60. If the exercise price is $62 the investor will make a: 

A. loss if the stock price rises above the breakeven point of $70.

B. loss if the stock price rises above the breakeven point of $72.

C. profit if the stock price rises above the breakeven point of $70.

D. profit if the stock price rises above the breakeven point of $72.

Correct Answer:  D..........................................................................................LOS: Reading  73‐e Breakeven is when 0 = maximum [ 0, (S – X)] – premium, when S = $72

Reference: CFA® Program Curriculum, Volume 6, pp. 96‐100. 

Page 479: Cfa Ques Answer

Derivative Investments   463 

 

13.  A trader sells one wheat futures contract, which  is for 5,000 bushels of wheat, at $4 per bushel. The trader posts an initial margin of $1,500. If the required maintenance margin is $1,100 the trader would first receive a maintenance margin call at a wheat price closest to: 

A.  $2.93 per bushel. 

B.  $3.92 per bushel. 

C.  $4.08 per bushel. 

D.  $5.45 per bushel. 

 

 

 

14.  ABC Asset Management  is running a  fund whose returns are  linked  to  the performance of  the S&P  Index.  It has  a U.S.$100 million  cash  inflow  and  it decides  to use  a  swap  agreement  to gain exposure to the market. It enters  into a one‐year agreement with XYZ  investment bank where XYZ agree  to pay  the  return on  the S&P Total Return  Index and ABC will pay a  fixed  rate of 5% on a notional principal of U.S.$100 million. The payments are to be made quarterly with the first payment on the last day in March and the payments will be based on the actual day count/365 basis. 

If the S&P Index rises by 10% in the quarter ending 31 March the first payment made by XYZ will be closest to: 

 

A.  $5,000,000. 

B.  $8,767,123. 

C.  $9,575,343. 

D.  $11,232,877. 

Page 480: Cfa Ques Answer

464   Study Session 17:  

 

13.  A trader sells one wheat futures contract, which  is for 5,000 bushels of wheat, at $4 per bushel. The trader posts an initial margin of $1,500. If the required maintenance margin is $1,100 the trader would first receive a maintenance margin call at a wheat price closest to: 

A. $2.93 per bushel.

B. $3.92 per bushel.

C. $4.08 per bushel.

D. $5.45 per bushel.

Correct Answer:  C ..........................................................................................LOS: Reading  72‐c When he has made a loss of $400 he would receive a margin call, this is equivalent to a price rise of 8 cents, since he sold the contract.

Reference: CFA® Program Curriculum, Volume 6, pp. 55‐60. 

 

14.  ABC Asset Management  is running a  fund whose returns are  linked  to  the performance of  the S&P  Index.  It has  a U.S.$100 million  cash  inflow  and  it decides  to use  a  swap  agreement  to gain exposure to the market. It enters  into a one‐year agreement with XYZ  investment bank where XYZ agree  to pay  the  return on  the S&P Total Return  Index and ABC will pay a  fixed  rate of 5% on a notional principal of U.S.$100 million. The payments are to be made quarterly with the first payment on the last day in March and the payments will be based on the actual day count/365 basis. 

If the S&P Index rises by 10% in the quarter ending 31 March the first payment made by XYZ will be closest to: 

A. $5,000,000.

B. $8,767,123.

C. $9,575,343.

D. $11,232,877.

Correct Answer:  B.......................................................................................... LOS: Reading  74‐b ABC will make a payment of

$100m x 5% x 90/365

= $1,232,877

XYZ pays $10,000,000

The net payment made by XYZ is $8,767,123

Reference: CFA® Program Curriculum, Volume 6, pp. 138‐141. 

Page 481: Cfa Ques Answer

Derivative Investments   465 

 

15.  Which of the following statements concerning futures and forward contracts is most accurate? 

A.  Only forward contracts are guaranteed by a clearinghouse. 

B.  Forward contracts tend to be more heavily regulated than futures contracts. 

C.  A futures contract is a type of forward contract that has standardized contract terms. 

D.  A forward contract is a type of futures contract that is traded on a recognized exchange. 

 

 

 

16.  If an  investor has  taken a  long position  in a  forward contract but  then wishes  to  terminate  the contract prior to expiry he can: 

A.  do nothing; it is impossible to terminate contracts. 

B.  go to the exchange and request an immediate cash settlement of the contract. 

C.  take a short position in a new contract with same expiry and underlying asset as the original contract. 

D.  go back to the counterparty and take a long position in another contract with the same expiry and underlying asset as the original contract. 

Page 482: Cfa Ques Answer

466   Study Session 17:  

 

15.  Which of the following statements concerning futures and forward contracts is most accurate? 

A. Only forward contracts are guaranteed by a clearinghouse.

B. Forward contracts tend to be more heavily regulated than futures contracts.

C. A futures contract is a type of forward contract that has standardized contract terms.

D. A forward contract is a type of futures contract that is traded on a recognized exchange.

Correct Answer:  C ..........................................................................................LOS: Reading  70‐a A futures contract is a type of forward contract that has standardized contract terms and is traded on a regulated exchange.

Reference: CFA® Program Curriculum, Volume 6, pp. 9‐12. 

 

 

 

16.  If an  investor has  taken a  long position  in a  forward contract but  then wishes  to  terminate  the contract prior to expiry he can: 

A. do nothing; it is impossible to terminate contracts.

B. go to the exchange and request an immediate cash settlement of the contract.

C. take a short position in a new contract with same expiry and underlying asset as the original contract.

D. go back to the counterparty and take a long position in another contract with the same expiry and underlying asset as the original contract.

Correct Answer:  C ......................................................................................... LOS: Reading  71‐b Taking a short position will mean that they no longer have any net exposure to price movements in the underlying asset.

Reference: CFA® Program Curriculum, Volume 6, pp. 33‐34. 

Page 483: Cfa Ques Answer

Derivative Investments   467 

 

17.  A dealer quotes on a 90‐day FRA, where the underlying is 180‐day dollar LIBOR, at a rate of 4%. The end user takes a short position, with a notional principal of $1 million. 

At expiration the rate on 180‐day LIBOR is 5%. The payoff for the end user is closest to: 

A.  a loss of $5,000. 

B.  a loss of $4,878. 

C.  a profit of $4,878. 

D.  a profit of $5,000. 

 

 

 

18.  Two  parties X  and  Y  enter  into  a  ten‐year  fixed‐for‐fixed  currency  swap.  Party  X  holds U.S. dollars and wishes  to exchange  these  for Yen, Party Y holds Yen and wishes  to exchange  these  for U.S. dollars. The principal  is  $100 million. The U.S. dollar  equals Yen 110 when  the  agreement  is signed and the fixed rates on U.S. dollars is set at 6% and Yen at 1%. At the end of the first year: 

A.  Party X pays Yen 110 million, and Party Y pays U.S. dollars 6 million. 

B.  Party Y pays Yen 110 million, and Party X pays U.S. dollars 6 million. 

C.  Party X pays Yen 11,110 million, and Party Y pays U.S. dollars 106 million. 

D.  Party Y pays Yen 11,110 million, and Party X pays U.S. dollars 106 million. 

Page 484: Cfa Ques Answer

468   Study Session 17:  

 

17.  A dealer quotes on a 90‐day FRA, where the underlying is 180‐day dollar LIBOR, at a rate of 4%. The end user takes a short position, with a notional principal of $1 million. 

At expiration the rate on 180-day LIBOR is 5%. The payoff for the end user is closest to:

A. a loss of $5,000.

B. a loss of $4,878.

C. a profit of $4,878.

D. a profit of $5,000.

Correct Answer:  B...........................................................................................LOS: Reading  71‐g The payoff is given by:

( )( )( ) 878,4$

025.1005.0000,000,1$

36018005.0136018004.005.0000,000,1$ =⎟

⎠⎞

⎜⎝⎛=⎥

⎤⎢⎣

⎡+−

Since the end user had a short position he must pay $4,878 to the dealer

Reference: CFA® Program Curriculum, Volume 6, pp. 40‐43. 

 

 

18.  Two  parties X  and  Y  enter  into  a  ten‐year  fixed‐for‐fixed  currency  swap.  Party  X  holds U.S. dollars and wishes  to exchange  these  for Yen, Party Y holds Yen and wishes  to exchange  these  for U.S. dollars. The principal  is  $100 million. The U.S. dollar  equals Yen 110 when  the  agreement  is signed and the fixed rates on U.S. dollars is set at 6% and Yen at 1%. At the end of the first year: 

A. Party X pays Yen 110 million, and Party Y pays U.S. dollars 6 million.

B. Party Y pays Yen 110 million, and Party X pays U.S. dollars 6 million.

C. Party X pays Yen 11,110 million, and Party Y pays U.S. dollars 106 million.

D. Party Y pays Yen 11,110 million, and Party X pays U.S. dollars 106 million.

Correct Answer:  A ......................................................................................... LOS: Reading  74‐b Party X pays interest on the Yen borrowed which is:

Yen 11,000 million x 1% = Yen 110 million.

Party Y pays interest on the U.S. dollars borrowed which is:

U.S. dollars 100 million x 6% = U.S. dollars 6 million.

The principals would be exchanged at the beginning of the agreement.

Reference: CFA® Program Curriculum, Volume 6, pp. 130‐134. 

Page 485: Cfa Ques Answer

Derivative Investments   469 

 

19.  An investor writes a put option at a premium of $6 on a stock with an exercise price of $62. If the stock price is $70 at expiration the investor will make a profit of: 

A.  $2. 

B.  $6. 

C.  $8. 

D.  $14. 

 

 

 

20.  A portfolio insurance strategy for a diversified stock portfolio can be implemented by: 

A.  buying a put option on the stock index representing the underlying stock portfolio. 

B.  writing a put option on the stock index representing the underlying stock portfolio. 

C.  buying a call option on the stock index representing the underlying stock portfolio. 

D.  writing a covered call option on the stock index representing the underlying stock portfolio. 

Page 486: Cfa Ques Answer

470   Study Session 17:  

 

19.  An investor writes a put option at a premium of $6 on a stock with an exercise price of $62. If the stock price is $70 at expiration the investor will make a profit of: 

A. $2.

B. $6.

C. $8.

D. $14.

Correct Answer:  B...........................................................................................LOS: Reading  75‐a The put option will lapse worthless since the exercise price is lower than the market price, so the investor makes a profit of the premium that he collected.

Reference: CFA® Program Curriculum, Volume 6, pp. 154‐158. 

 

 

 

20.  A portfolio insurance strategy for a diversified stock portfolio can be implemented by: 

A. buying a put option on the stock index representing the underlying stock portfolio.

B. writing a put option on the stock index representing the underlying stock portfolio.

C. buying a call option on the stock index representing the underlying stock portfolio.

D. writing a covered call option on the stock index representing the underlying stock portfolio.

Correct Answer:  A ......................................................................................... LOS: Reading  75‐b In the situation that the stock market index falls, the losses on the underlying portfolio will be offset by the profits on the put option.

Reference: CFA® Program Curriculum, Volume 6, pp. 162‐165. 

Page 487: Cfa Ques Answer

Derivative Investments   471 

 

21.  The least likely benefit for an investor if a market includes financial derivatives is: 

A.  price discovery. 

B.  trading efficiency. 

C.  regulatory protection. 

D.  instruments available for risk management. 

 

 

 

22.  A  firm  enters  into  a plain vanilla  interest  rate  swap  agreement  to pay  a  fixed  rate of  8%,  the counterparty agrees  to pay one year LIBOR. Annual payments will be made  in arrears. The  swap covers a  five year period and  is based on a notional principal of $100 million. The one year LIBOR rate at the time of agreement is 8.25%. At the end of one year it is 9%, and at the end of the second year it is 9.5%. The net payment that the pay‐fixed firm receives/pays at the end of the second year is: 

A.  pays $1 million. 

B.  pays $1.5 million. 

C.  receives $1 million. 

D.  receives $1.5 million. 

Page 488: Cfa Ques Answer

472   Study Session 17:  

 

21.  The least likely benefit for an investor if a market includes financial derivatives is: 

A. price discovery.

B. trading efficiency.

C. regulatory protection.

D. instruments available for risk management.

Correct Answer:  C ..........................................................................................LOS: Reading  70‐c Price discovery, ability to hedge risk and market efficiency including low transaction costs are all benefits of derivatives. Although exchange-traded derivatives provide some regulatory protection, OTC derivatives often offer little regulatory protection.

Reference: CFA® Program Curriculum, Volume 6, pp. 20‐22. 

Page 489: Cfa Ques Answer

Derivative Investments   473 

 

22.  A  firm  enters  into  a plain vanilla  interest  rate  swap  agreement  to pay  a  fixed  rate of  8%,  the counterparty agrees  to pay one year LIBOR. Annual payments will be made  in arrears. The  swap covers a  five year period and  is based on a notional principal of $100 million. The one year LIBOR rate at the time of agreement is 8.25%. At the end of one year it is 9%, and at the end of the second year it is 9.5%. The net payment that the pay‐fixed firm receives/pays at the end of the second year is: 

A. pays $1 million.

B. pays $1.5 million.

C. receives $1 million.

D. receives $1.5 million.

Correct Answer:  C..........................................................................................LOS: Reading  74‐b The payment received is $100 million x (9% - 8%) = $1 million.

Reference: CFA® Program Curriculum, Volume 6, pp. 134‐138. 

 

 

Page 490: Cfa Ques Answer

474   Study Session 18:  

 

Study Session 18: Alternative Investments:  

Due  to  diversification  benefits  and  higher  expectations  of  investment  returns,  investors  are increasingly  turning  to  alternative  investments. This  study  session describes  the  common  types of alternative investments, methods for their valuation, unique risks and opportunities associated with them, and the relation between alternative investments and traditional investments. 

Although finding a single definition of an “alternative” investment is difficult, certain features (e.g., limited  liquidity,  infrequent  valuations,  and unique  legal  structures)  are  typically  associated with alternative investments. This study session discusses these features and how to evaluate their impact on expected returns and  investment decisions  in more detail. The reading provides an overview of the major categories of alternative investments, including real estate, private equity, venture capital, hedge funds, closely held companies, distressed securities, and commodities.  

Each one of  these  categories has  several unique  characteristics, and  the  readings discuss valuation methods  for  illiquid  assets  (such  as  direct  real  estate  or  closely  held  companies),  performance measures for private equity and venture capital investments, differences between various hedge fund strategies, and implementation vehicles for investments in alternative assets.  

 

 

Reading 76: Alternative Investments 

 

Page 491: Cfa Ques Answer

Alternative Investments   475 

 

1.  Capital provided to a company that is close to going public is: 

A.  first‐stage financing. 

B.  third‐stage financing. 

C.  mezzanine financing. 

D.  second‐stage financing. 

 

 

 

2.  Which of the following statements is the least accurate description of a characteristic of venture capital investing? 

A.  Illiquidity is a feature of venture capital investment. 

B.  Investors often need to make a long‐term commitment. 

C.  Entrepreneurs have strong management skills which increase the probability of companies being successful. 

D.  Investors expect to achieve higher investment returns than they receive from investing in publicly listed securities. 

 

 

Page 492: Cfa Ques Answer

476   Study Session 18:  

 

1.  Capital provided to a company that is close to going public is: 

A. first-stage financing.

B. third-stage financing.

C. mezzanine financing.

D. second-stage financing.

Correct Answer:  C ..........................................................................................LOS: Reading  76‐g Mezzanine or bridge financing is given to companies who are planning to go public in the near term.

Reference: CFA® Program Curriculum, Volume 6, pp. 201‐202. 

 

 

 

2.  Which of the following statements is the least accurate description of a characteristic of venture capital investing? 

A. Illiquidity is a feature of venture capital investment.

B. Investors often need to make a long-term commitment.

C. Entrepreneurs have strong management skills which increase the probability of companies being successful.

D. Investors expect to achieve higher investment returns than they receive from investing in publicly listed securities.

Correct Answer:  C ..........................................................................................LOS: Reading  76‐g Entrepreneurs often have weak management skills so the venture capitalist can help in providing direction and strategic guidance to the company.

Reference: CFA® Program Curriculum, Volume 6, pp. 202‐204. 

Page 493: Cfa Ques Answer

Alternative Investments   477 

 

3.  A hedge  fund manager specializes  in  taking  long positions  in companies  that are being bid  for and taking short positions in the acquiring company. He is likely to be managing a hedge fund that is: 

A.  a futures fund. 

B.  a long/short fund. 

C.  an event‐driven fund. 

D.  a market‐neutral fund. 

 

 

 

4.  One  of  the  reasons  that  the  shares  in  closely  held  companies  usually  trade  at  a  discount  to publicly traded securities is: 

A.  the shares are less liquid. 

B.  they have high market betas. 

C.  investors often hold a majority stake. 

D.  investors often have a greater influence on the company’s management. 

Page 494: Cfa Ques Answer

478   Study Session 18:  

 

3.  A hedge  fund manager specializes  in  taking  long positions  in companies  that are being bid  for and taking short positions in the acquiring company. He is likely to be managing a hedge fund that is: 

A. a futures fund.

B. a long/short fund.

C. an event-driven fund.

D. a market-neutral fund.

Correct Answer:  C .......................................................................................... LOS: Reading  76‐i C is the best answer since the long and short positions are being taken as a result of specific events, in this case an acquisition.

Reference: CFA® Program Curriculum, Volume 6, pp. 211‐213. 

 

 

 

4.  One  of  the  reasons  that  the  shares  in  closely  held  companies  usually  trade  at  a  discount  to publicly traded securities is: 

A. the shares are less liquid.

B. they have high market betas.

C. investors often hold a majority stake.

D. investors often have a greater influence on the company’s management.

Correct Answer:  A ......................................................................................... LOS: Reading  76‐n Closely held companies’ shares are often not publicly traded and lack marketability, so investors are compensated with a liquidity discount.

Reference: CFA® Program Curriculum, Volume 6, pp. 223‐224. 

Page 495: Cfa Ques Answer

Alternative Investments   479 

 

5.  An investor in an Exchange Traded Fund (ETF) is exposed to tracking error risk; this is the risk that: 

A.  the bid‐ask spread for shares in the ETF widens. 

B.  the ETF fails to make dividend payments to investors. 

C.  the currency the fund is denominated in does not track the U.S. dollar. 

D.  the fund does not closely replicate the performance of the index that it is following. 

 

 

 

6.  Which  of  the  following  is  the  least  accurate  description  of  a  characteristic  of  commodity investment, based on historic data?  

A.  It offers inflation protection. 

B.  It has low volatility of returns. 

C.  It produces attractive returns in periods of economic growth. 

D.  The returns have low correlations with bond and equity returns. 

Page 496: Cfa Ques Answer

480   Study Session 18:  

 

5.  An investor in an Exchange Traded Fund (ETF) is exposed to tracking error risk; this is the risk that: 

A. the bid-ask spread for shares in the ETF widens.

B. the ETF fails to make dividend payments to investors.

C. the currency the fund is denominated in does not track the U.S. dollar.

D. the fund does not closely replicate the performance of the index that it is following.

Correct Answer:  D..........................................................................................LOS: Reading  76‐c Tracking error is a measure of the deviations between ETF returns and the index returns.

Reference: CFA® Program Curriculum, Volume 6, pp. 186‐187. 

 

 

 

6.  Which  of  the  following  is  the  least  accurate  description  of  a  characteristic  of  commodity investment, based on historic data?  

A. It offers inflation protection.

B. It has low volatility of returns.

C. It produces attractive returns in periods of economic growth.

D. The returns have low correlations with bond and equity returns.

Correct Answer:  B.......................................................................................... LOS: Reading  76‐q Historically commodities have exhibited higher volatility than equities.

Reference: CFA® Program Curriculum, Volume 6, pp. 225‐227. 

Page 497: Cfa Ques Answer

Alternative Investments   481 

 

7.  Investors in hedge funds are least likely to be motivated by which of the following?  

A.  Consistent returns across the different categories of hedge funds. 

B.  The low volatility of returns from hedge funds compared to equity returns. 

C.  Higher average returns provided by hedge funds compared to other investments. 

D.  The potential to use hedge funds to diversify portfolios that hold other asset classes. 

 

 

 

8.  Which  of  the  following  statements  regarding  trade  sales  of venture  capital  investments  is  the most accurate? 

A.  A trade sale is the first step in the public offering process. 

B.  Trade sales are an unattractive exit strategy for most venture capitalists. 

C.  Trade sales are one of the most common methods of venture capital investors divesting their holdings. 

D.  Trade sales means that a venture capital investment is sold to another company operating in the same industry. 

Page 498: Cfa Ques Answer

482   Study Session 18:  

 

7.  Investors in hedge funds are least likely to be motivated by which of the following?  

A. Consistent returns across the different categories of hedge funds.

B. The low volatility of returns from hedge funds compared to equity returns.

C. Higher average returns provided by hedge funds compared to other investments.

D. The potential to use hedge funds to diversify portfolios that hold other asset classes.

Correct Answer:  A .......................................................................................... LOS: Reading  76‐i Different types of hedge funds (e.g. fixed-income arbitrage versus global macro) have quite different performance records.

Reference: CFA® Program Curriculum, Volume 6, pp. 217‐219. 

 

 

 

8.  Which  of  the  following  statements  regarding  trade  sales  of venture  capital  investments  is  the most accurate? 

A. A trade sale is the first step in the public offering process.

B. Trade sales are an unattractive exit strategy for most venture capitalists.

C. Trade sales are one of the most common methods of venture capital investors divesting their holdings.

D. Trade sales means that a venture capital investment is sold to another company operating in the same industry.

Correct Answer:  C ..........................................................................................LOS: Reading  76‐g Trade sales refer to a venture capital investment being sold to or merged with another company.

Reference: CFA® Program Curriculum, Volume 6, p. 204. 

Page 499: Cfa Ques Answer

Alternative Investments   483 

 

9.  Which of the following is least likely to be used as a method of valuing real estate? 

A.  Cost approach. 

B.  Income approach. 

C.  Balance sheet approach. 

D.  Discounted cash flow approach. 

 

 

 

10.  The prices of hedge funds are often smoothed because: 

A.  they use arbitrage strategies. 

B.  the managers are risk averse. 

C.  they are actively dealing in derivative exchanges. 

D.  they invest in over‐the‐counter instruments whose prices are based on estimates. 

Page 500: Cfa Ques Answer

484   Study Session 18:  

 

9.  Which of the following is least likely to be used as a method of valuing real estate? 

A. Cost approach.

B. Income approach.

C. Balance sheet approach.

D. Discounted cash flow approach.

Correct Answer:  C ..........................................................................................LOS: Reading  76‐e The methods that are covered in the text are the cost approach, sales comparison approach, income approach and the discounted after-tax cash flow approach.

Reference: CFA® Program Curriculum, Volume 6, pp. 190‐198. 

 

 

 

10.  The prices of hedge funds are often smoothed because: 

A. they use arbitrage strategies.

B. the managers are risk averse.

C. they are actively dealing in derivative exchanges.

D. they invest in over-the-counter instruments whose prices are based on estimates.

Correct Answer:  D.......................................................................................... LOS: Reading  76‐l Over the counter instruments do not have market prices and the estimated values are often less volatile than exchange-traded instruments.

Reference: CFA® Program Curriculum, Volume 6, pp. 220‐221. 

Page 501: Cfa Ques Answer

Alternative Investments   485 

 

11.  Investors in a load open‐end fund: 

A.  purchase shares at the net asset value. 

B.  purchase shares at a discount to the net asset value. 

C.  pay an annual management charge which is called a load. 

D.  purchase shares at the net asset value plus an initial charge. 

 

 

 

12.  The objective of a stock market index fund is to:  

A.  track the return from the stock market index. 

B.  outperform the return from the stock market index. 

D.  generate a return which is independent of the stock market return.  

C.  generate a return which is higher then the CPI index by investing in equities. 

Page 502: Cfa Ques Answer

486   Study Session 18:  

 

11.  Investors in a load open‐end fund: 

A. purchase shares at the net asset value.

B. purchase shares at a discount to the net asset value.

C. pay an annual management charge which is called a load.

D. purchase shares at the net asset value plus an initial charge.

Correct Answer:  D..........................................................................................LOS: Reading  76‐a An open-end investment company continues to buy and sell shares after the initial offering. A managed or load fund is when the offering price is the NAV plus an initial charge called a front-end load.

Reference: CFA® Program Curriculum, Volume 6, pp. 177‐180. 

 

 

 

12.  The objective of a stock market index fund is to:  

A. track the return from the stock market index.

B. outperform the return from the stock market index.

D. generate a return which is independent of the stock market return.

C. generate a return which is higher then the CPI index by investing in equities.

Correct Answer:  A ......................................................................................... LOS: Reading  76‐b The objective of an index fund is to perform in line with a specified index, perhaps by owning all the shares in the index with the same weighting as their representation in the index.

Reference: CFA® Program Curriculum, Volume 6, p. 180. 

Page 503: Cfa Ques Answer

Alternative Investments   487 

 

13.  Which  stage of venture capital  financing  is when capital  is provided  for product development and research? 

A.  Seed financing. 

B.  Start‐up financing. 

C.  First‐stage financing. 

D.  Second‐stage financing. 

 

 

 

14.  A  property  which  is  considered  a  lower  risk  investment  than  another  property  investment, assuming they both generate the same net operating income, will be likely to have a: 

  Market capitalization rate  Valuation 

A.    lower      lower 

B.    lower       higher 

C.    higher      lower 

D.    higher       higher 

Page 504: Cfa Ques Answer

488   Study Session 18:  

 

13.  Which  stage of venture capital  financing  is when capital  is provided  for product development and research? 

A. Seed financing.

B. Start-up financing.

C. First-stage financing.

D. Second-stage financing.

Correct Answer:  A ..........................................................................................LOS: Reading  76‐g Seed financing is the first stage of venture capital investing, for product development and market research. The product is still at the ‘idea’ stage.

Reference: CFA® Program Curriculum, Volume 6, pp. 201‐202. 

 

 

 

14.  A  property  which  is  considered  a  lower  risk  investment  than  another  property  investment, assuming they both generate the same net operating income, will be likely to have a: 

Market capitalization rate Valuation

A. lower lower

B. lower higher

C. higher lower

D. higher higher

Correct Answer:  B........................................................................................... LOS: Reading  76‐f The market capitalization rate reflects the investors’ required rate of return from the property, a low risk project will tend to have a low capitalization rate and therefore, for equal net operating income, a higher value.

Reference: CFA® Program Curriculum, Volume 6, pp. 193‐194. 

Page 505: Cfa Ques Answer

Alternative Investments   489 

 

15.  The after‐tax cash flow from a real estate  investment adjusts the net operating  income for all of the following except: 

A.  cost of debt. 

B.  proceeds from sale of property. 

C.  depreciation expense less tax savings. 

D.  taxation on capital gains when the property is sold. 

 

 

 

16.  Investing in a hedge fund is least likely to be attractive because: 

A.  the returns are higher than those available on equity funds. 

B.  the volatility of returns is lower than that of a fund investing in equities. 

C.   the fund will provide greater transparency than a traditional mutual fund.  

D.  the return from the fund is likely to have a low correlation with listed stocks and bonds. 

Page 506: Cfa Ques Answer

490   Study Session 18:  

 

15.  The after‐tax cash flow from a real estate  investment adjusts the net operating  income for all of the following except: 

A. cost of debt.

B. proceeds from sale of property.

C. depreciation expense less tax savings.

D. taxation on capital gains when the property is sold.

Correct Answer:  C .......................................................................................... LOS: Reading  76‐f The net operating income (NOI) is before depreciation, and depreciation is a non-cash item, so no adjustment is made for the depreciation, although the tax saving is included in the calculation.

Reference: CFA® Program Curriculum, Volume 6, pp. 193‐198. 

 

 

 

16.  Investing in a hedge fund is least likely to be attractive because: 

A. the returns are higher than those available on equity funds.

B. the volatility of returns is lower than that of a fund investing in equities.

C. the fund will provide greater transparency than a traditional mutual fund.

D. the return from the fund is likely to have a low correlation with listed stocks and bonds.

Correct Answer:  C .......................................................................................... LOS: Reading  76‐j The most common legal structure is limited partnership (in the U.S.) or an offshore corporation. The legal structure gives the fund managers not only the freedom to implement a variety of strategies but there are less stringent disclosure requirements than for traditional funds. Lack of transparency of hedge funds can be a major drawback for investors.

Reference: CFA® Program Curriculum, Volume 6, pp. 207‐210. 

Page 507: Cfa Ques Answer

Alternative Investments   491 

 

17.  In the U.S. global funds refer to funds that: 

A.  only invest outside the U.S. 

B.  are only marketed outside the U.S. 

C.  invest in both the U.S. and internationally. 

D.  are marketed in both the U.S. and internationally. 

 

 

 

18.  The role of venture capital investors is least likely to include: 

A.  assisting companies to go public. 

B.  providing financing to small privately held companies. 

C.  assisting the companies that they invest in with strategic planning. 

D.  making a market in the shares of their investments that have gone public.  

Page 508: Cfa Ques Answer

492   Study Session 18:  

 

17.  In the U.S. global funds refer to funds that: 

A. only invest outside the U.S.

B. are only marketed outside the U.S.

C. invest in both the U.S. and internationally.

D. are marketed in both the U.S. and internationally.

Correct Answer:  C ......................................................................................... LOS: Reading  76‐b ‘Global funds’ refer to funds that are investing in both the U.S. and international markets.

Reference: CFA® Program Curriculum, Volume 6, p. 177. 

 

 

 

18.  The role of venture capital investors is least likely to include: 

A. assisting companies to go public.

B. providing financing to small privately held companies.

C. assisting the companies that they invest in with strategic planning.

D. making a market in the shares of their investments that have gone public.

Correct Answer:  D..........................................................................................LOS: Reading  76‐g The role of venture capitalists is not just to provide finance but to also work with the management team to develop and expand the business. This would usually include assisting with a company going public as venture capitalists have experience dealing with underwriters and other financial institutions. They would not normally be specialists or market makers.

Reference: CFA® Program Curriculum, Volume 6, pp. 200‐202. 

Page 509: Cfa Ques Answer

Alternative Investments   493 

 

19.  A mutual fund has issued two classes of shares. Each class holds the same underlying portfolio of securities but the expense structures differ, the fees are shown in the table below: 

 

  Class A  Class B Front‐end fees  4%  None Redemption fees  None  5% in the first year but declining 

by 1% point each year thereafter 

Annual expenses       Distribution fees  0.30%  0.30%   Management fees  0.75%  1.00%   Other expenses  0.20%  0.20%   1.25%  1.50% 

 

What would be the return for each of the classes of shares if an investor invests $1 for a period of five years, assuming the fund grows by 8% annually? 

  Fund A    Fund B 

A.  32.46%    34.88% 

B.  32.46%    36.24% 

C.  33.08%    34.88% 

D.  33.08%    37.01% 

Page 510: Cfa Ques Answer

494   Study Session 18:  

 

19.  A mutual fund has issued two classes of shares. Each class holds the same underlying portfolio of securities but the expense structures differ, the fees are shown in the table below: 

  Class A  Class B Front‐end fees  4%  None Redemption fees  None  5% in the first year but declining 

by 1% point each year thereafter 

Annual expenses       Distribution fees  0.30%  0.30%   Management fees  0.75%  1.00%   Other expenses  0.20%  0.20%   1.25%  1.50% 

What would be the return for each of the classes of shares if an investor invests $1 for a period of five years, assuming the fund grows by 8% annually?

Fund A Fund B

A. 32.46% 34.88%

B. 32.46% 36.24%

C. 33.08% 34.88%

D. 33.08% 37.01%

Correct Answer:  B...........................................................................................LOS: Reading  76‐a 

Fund A:  Due to the front‐end fee of 4%, only $0.96 from the $1 will be available to invest.  At the end of five years, $1 invested in the fund will be worth    $0.96 x (1.08)5 x (1 – 0.0125)5 = $1.3246, or a return of 32.46%.  Fund B:   At the end of 5 years, $1 invested in the funds will be worth    $1 x (1.08)5 x (1 – 0.015)5 = $1.3624, or a return of 36.24%.  

Reference: CFA® Program Curriculum, Volume 6, pp. 178‐180. 

Page 511: Cfa Ques Answer

Alternative Investments   495 

 

20.  A mutual fund has issued two classes of shares. Each class holds the same underlying portfolio of securities but the expense structures differ, the fees are shown in the table below: 

 

  Class A  Class B Front‐end fees  4%  None Redemption fees  None  5% in the first year but declining 

by 1% point each year thereafter 

Annual expenses       Distribution fees  0.30%  0.30%   Management fees  0.75%  1.00%   Other expenses  0.20%  0.20%   1.25%  1.50% 

 

What would be the return for each of the classes of shares if an investor invests $1 and redeems his shares after two years, assuming the fund grows by 8% annually? 

  Fund A    Fund B 

A.  9.19%    8.64% 

B.  9.19%    8.89% 

C.  9.40%    8.64% 

D.  9.40%    9.77% 

Page 512: Cfa Ques Answer

496   Study Session 18:  

 

20.  A mutual fund has issued two classes of shares. Each class holds the same underlying portfolio of securities but the expense structures differ, the fees are shown in the table below: 

  Class A  Class B Front‐end fees  4%  None Redemption fees  None  5% in the first year but declining 

by 1% point each year thereafter 

Annual expenses       Distribution fees  0.30%  0.30%   Management fees  0.75%  1.00%   Other expenses  0.20%  0.20%   1.25%  1.50% 

What would be the return for each of the classes of shares if an investor invests $1 and redeems his shares after two years, assuming the fund grows by 8% annually?

Fund A Fund B

A. 9.19% 8.64%

B. 9.19% 8.89%

C. 9.40% 8.64%

D. 9.40% 9.77%

Correct Answer:  A ..........................................................................................LOS: Reading  76‐a 

Fund A:  There is no redemption fee, so the investment in the fund after two years will be worth    $0.96 x (1.08)2 x (1 – 0.0125)2 = $1.0919, or a return of 9.19%.  Fund B:  After two years the redemption fee will be 4%, i.e. a decline of 1% point from 5%.  The investment in the fund after two years will be worth    $1 x (1.08)2 x (1 – 0.015)2 x (1 ‐ 0.04) = $1.0864 or a return of 8.64%.  

Reference: CFA® Program Curriculum, Volume 6, pp. 178‐180. 

Page 513: Cfa Ques Answer

Alternative Investments   497 

 

21.  An  investor  is  considering purchasing  an  office  building  as  an  investment,  and  the  following information has been collected. The figures are on an annual basis. 

Gross potential rental income   $1,000,000 

Estimated vacancy and collection losses  5% 

Insurance and taxes  $80,000 

Utilities  $30,000 

Repairs and maintenance  $60,000 

Depreciation  $70,000 

Interest on proposed financing  $90,000 

 

The net operating income (NOI) per annum is closest to 

A.  $620,000. 

B.  $690,000. 

C.  $710,000. 

D.  $780,000. 

 

 

22.  An  investor  is  looking  at  investing  $1 million  in  a  project, where  the  expected  payout  is  $10 million at the end of five years. The investor’s cost of equity for the project is 10%. However there is a significant risk of  failure and  the probability of  failure  in any year  is given  in  the  table below. The probability is based on the condition that the project has survived the previous year.  

Year  1  2  3  4  5 Probability of failure  0.40  0.35  0.25  0.20  0.20 

The expected net present value (NPV) of the project is closest to 

A.  $159,400. 

B.  $346,400. 

C.  $870,000. 

D.  $972,400. 

Page 514: Cfa Ques Answer

498   Study Session 18:  

 

21.  An  investor  is  considering purchasing  an  office  building  as  an  investment,  and  the  following information has been collected. The figures are on an annual basis. 

Gross potential rental income   $1,000,000 

Estimated vacancy and collection losses  5% 

Insurance and taxes  $80,000 

Utilities  $30,000 

Repairs and maintenance  $60,000 

Depreciation  $70,000 

Interest on proposed financing  $90,000 

The net operating income (NOI) per annum is closest to

A. $620,000.

B. $690,000.

C. $710,000.

D. $780,000.

Correct Answer:  D.......................................................................................... LOS: Reading  76‐f NOI = gross potential rental income minus expenses

  = $1,000,000 – (0.05 x $1,000,000) ‐ $80,000 ‐ $30,000 ‐ $60,000   = $780,000 

Note the expenses for this calculation do not include depreciation (it is assumed that repairs will maintain the building in good condition indefinitely) and interest expense.

Reference: CFA® Program Curriculum, Volume 6, pp. 193‐194. 

Page 515: Cfa Ques Answer

Alternative Investments   499 

 

22.  An  investor  is  looking  at  investing  $1 million  in  a  project, where  the  expected  payout  is  $10 million at the end of five years. The investor’s cost of equity for the project is 10%. However there is a significant risk of  failure and  the probability of  failure  in any year  is given  in  the  table below. The probability is based on the condition that the project has survived the previous year. 

Year  1  2  3  4  5 Probability of failure  0.40  0.35  0.25  0.20  0.20 

The expected net present value (NPV) of the project is closest to 

A. $159,400.

B. $346,400.

C. $870,000.

D. $972,400.

Correct Answer:  A..........................................................................................LOS: Reading  76‐h The probability that the project survives throughout the five years is given by the product of the individual probabilities it survives each year, which is:

(1 – 0.40) (1 – 0.35) (1 – 0.25) (1 – 0.20) (1 – 0.20) = 18.7%  If the project survives the present value is: $10 million/(1.10)5 ‐ $1 million = $5.2 million  If the project fails the present value is: – $1 million.  The expected NPV:   0.187($5.2 million) + 0.813(– $1 million) = $159,400  

Reference: CFA® Program Curriculum, Volume 6, pp. 205‐207. 

 

Page 516: Cfa Ques Answer

500   Study Session 18:  

 

2008 JUNE EXAM 15 February 2008   Second deadline for new CFA Program enrollments and exam registrations to be received by CFA Institute  

March 2008   Online sample exams available  

17 March 2008   Final deadline  for new CFA Program enrollments and exam registrations  to be received by CFA Institute  

17 March 2008   Final deadline  for disability accommodation requests and requests  for religious alternative dates to be received by CFA Institute  

17 March 2008   All test center change requests must be received by CFA Institute  

Late April 2008   Exam admission tickets available online  

7 June 2008   Exam date   and   8 June 2008   Exam date in Eastern Asia and Oceania 

June‐July 2008   Exams graded  

Late July 2008   Exam results available online for Level I candidates  

Late August 2008   Exam results available online for Level II and III candidates 

2008 DECEMBER EXAM 17 March 2008   First deadline  for new CFA Program enrollments and exam  registrations  to be received by CFA Institute  

15 August 2008   Second deadline for new CFA Program enrollments and exam registrations to be received by CFA Institute  

15 September 2008  Final deadline  for new CFA Program enrollments and exam registrations  to be received by CFA Institute  

15 September 2008  Final deadline  for disability accommodation requests and requests  for religious alternative dates to be received by CFA Institute  

15 September 2008  All test center change requests must be received by CFA Institute  

October 2008   Online sample exams available  

Late October 2008   Exam admission tickets available online  

6 December 2008   Exam date    and   7 December 2008        Exam date in Eastern Asia and Oceania 

December 2008   Exams graded  

January 2009   Exam results available online  

Page 517: Cfa Ques Answer

Terminology   501 

 

Terminology: Appraisal – for real estate, the process of estimating the current market value of a property. Comparative sales approach – the value of a real estate is , at the most, the cost of the land and constructing the building at current prices. Income approach – the value of real estate is the present value of it future income. Market capitalization rate – divide a property’s net operating income by the appropriate market capitalization rate to arrive at an estimate for its current market value.  It reflects the rate of return required by investors in such a property. Positive leverage – the return from a real estate investment is higher than the cost of debt, an investor will achieve a higher rate of return if he/she uses leverage to purchase the property. Real Estate Investment Trust (REIT) – a closed‐end investment company that invests in real estate and mortgages on real estate. Real Estate Limited Partnership (RELP) – a real estate syndicate that invests in different types of real estate. Seed financing – venture capital provided for product development and market research, the product is still at the ‘idea’ stage. Start‐up financing – venture capital provided for early stage product development and initial marketing. First‐stage financing – venture capital provided for initial commercial manufacture and sales. Mezzanine (or bridge) financing – venture capital provided for a company that expects to go public in the near future.  Turnarounds  – capital provided to restructure a company that has problems. Leveraged buyouts (LBOs) – capital to fund a management group (a management buyout) or other investors who wish to purchase a business or company. Investment company – a company that sell its own shares and uses the proceeds to buy stocks, bonds or other financial instruments. Closed‐end investment company – an investment company that issues a fixed number of shares, the shares are then traded in the secondary market. Open‐end investment company – a company that offers new shares to investors and redeems shares continuously. Mutual fund – an open‐end investment company. No‐load fund – shares are sold at net asset value, with no sales charge added. Load fund – a fund that makes an initial sales charge, so the offering price is the net asset value plus a load.  

Page 518: Cfa Ques Answer

502   Exhibit 

 

FREE Author Collaberation: With the purchase of this book you also receive the following

communications free!

Now that you have purchased this product you have access to the Instructors/Authors that authored this book.  Send you questions to us and we will answer them for you.  

 

 

 

This is a free Feature for CFA Candidates of all levels. http://www.financialexams.com/ContentPages/DrHarvey/tabid/71/D

efault.aspx

 

Thank you for reading our book  

Page 519: Cfa Ques Answer

Appendix A: 503

Appendix A:

Exhibits

Exhibit 1: Accounting Statements

Exhibit 2: Puts and Calls

Exhibit 3: PE Breakdown

Exhibit 4: Ratios

Page 520: Cfa Ques Answer

504 Exhibit

Exhibit 1: Accounting Statements Decisions Statement of Cash Flows Income Statement Balance Sheet

Sales

Balance Sheet Income Statement Assets

Stk Eqty

Profit

Investors

Assets come into Balance Sheet in SE to get to Assets

Convert assets into profit to return to Inv or Stkhldrs Equity

Income Stmt: Particular moment in the life of an asset

CFF

Sales - COGS Gross Margin - Expenses EBIT - Interest EBT - Tax EAT

CA CL AR AP Inv FA LTD SE

CFO

CFI

Operating

Investment

Financing

OperationCycle

Inventory stored and converted to product

Sale Acquire Inventory

Cash Collection

CEOs look at things from viewpoint of Cash Flow ... to pay bills, liquidity

A growing company never has enough cash collection to grow the inventory.

CEO is always looking for something which will grow cash.

Newspaper cycles daily, Boeing could take years

Page 521: Cfa Ques Answer

Appendix A: 505

Notes:

Page 522: Cfa Ques Answer

506 Exhibit

Exhibit 2: Puts and Calls Interpret the diagrams that depict the expiration-day values of the long call, short call, long put and short put strategies; (X = strike price, S = Stock price, C = cost of Call, P = cost of Put) Long Call Long Put Value V > 0 Value V > 0 Positive positive

0 0 X negative negative

S X X S Short Call Short Put Value V < 0 Value V < 0 Positive Positive

0 0 negative negative

S X X S Interpret the profit/loss diagrams for the long call, short call, long put, and short put strategies; Long Call Long Put Loss < C, Profit > X+C Loss < X+P, Profit < X Profit Profit 0 0 cost of C cost of P Loss Loss X X+C X X+P Short Call Short Put Loss > X+C, Profit < X+C Loss > X, Profit > X Profit Profit Rec’d fm C Rec’d fm C 0 0 Loss Loss X X+C X X+P

Value of the long call will be the amount over strike the stock is trading. Long call will never have a value less than zero.

Value of the short call will be the amount over the strike the stock is trading. Short call will never have a value greater than zero.

Value of the long put will be the amount less than the strike that the stock is trading. Long put will never have a value less than zero.

Value of the short put will be the amount less than the strike the stock is trading. Short put will never have a value greater than zero.

Profit of the long call is the amount over the strike + cost of the call (X+C). Loss on the call is the amount the stock is under the call + strike, not to exceed the cost of the call. stock price stock price

Profit the long put is the amount under the strike and cost of the put. Loss on the put is the amount the stock is over the strike, not to exceed the cost of the put.

Profit of the short call is the amount under the strike + cost of the call (X+C), not to exceed the proceeds received from the call. Loss on the call is the amount the stock is over the call + strike. stock price stock price

Profit the short put is the amount more than the strike of the put, not to exceed the proceeds received from the put. Loss on the put is the amount the stock is less than the strike.

Page 523: Cfa Ques Answer

Appendix A: 507

Notes:

Page 524: Cfa Ques Answer

508 Exhibit

Exhibit 3: PE Breakdown Note: 3 Stage ROE Profit Margin * Total Asset Turnover * Financial Leverage NI * Sales * Total Assets Sales Total Assets Equity Editor’s Review:

Income Statement

Sales

- COGS

Gross

Margin

- Expenses (mgmt hides it’s perks if EAT is

OK)

EBIT

- Interest Bank happy

EBT What mgmt earns for

shareholders

- Taxes Gov’t happy

EAT Stockholders happy

Page 525: Cfa Ques Answer

Appendix A: 509

EAT = EBT - EBT(t) EAT = EBT(1-t) Expected EPS = Expected Sales/share * Expected Net profit Margin Income Stmt: Next year’s projected We know last year’s P & P/E [ (S) (EBDIT) - D - I ] (1-t) Can calc next year’s earnings pending ratio Depr Inter tax then Proportion via P/E to get Pproj EOY sales per shr Exp adj Pproj EOY = (EProj EOY) (P/E)

Earnings, DDM & ROE Calc

P = Div / Earn Dividend pay-out E k – g k - g (Retention) (ROE) k = Real Rate + Interest Prem + Risk Prem Tax * Interest * Oper’g * Asset * Financial Nominal rate should really multiply Reten Burden Profit Turnover Leverage Rate Margin k = Rf + β(Rm - Rf) β is # units of risk in the stock Stockholders want and pay for k, k determines the P NI * EBT * EBIT * Sls * Assets EBT EBIT Sls Assets Equity P0 = D1 k - g Oper Total -Interest Financial * Tax WACC = (weke) + (wdkd) Profit * Asset Expense * Leverage Retention Margin Turnover Rate Multiplier Rate P0 = D1 k - g EBIT * Sls - Int * Assets * (1 - T) Sls Assets Assets Equity Note: tax ret (1-t)

ReturnEstimated = Cash Div + PriceEnd - PriceNow PriceNow P0 = D1 restate: k = D + g = k = Rf + β(Rm - Rf) also k = (1+RR)(1+IP)(1+RP) k - g P real infl risk should be in equilibrium

Bad side Good side

f

Page 526: Cfa Ques Answer

510 Exhibit

Exhibit 4: Ratios calculate the financial ratios in each major category of analysis and discuss the uses of those ratios; Common Size Statements: B/S in percent of Total Assets, I/S in percent of Sales Quickly compare two different size firms, same firm trends over time, structure of firm’s financial

statements Internal Liquidity (Solvency): ability of firm to meet future short term obligations, compare near term

obligations with current assets or cash flows Current Ratio: Current Assets Working Capital = CA - CL Current Liabilities Quick ratio Cash + mkt sec + AR not include. inventory CL Acid Test CA - Inv a.k.a. Quick Ratio CL Cash ratio Cash + mkt sec even more conservative CL Receivables Turnover Net Annual Sales Avg collection: 365 Avg Receivables Annual Turnover Days Receivable 365 Avg # of days to get paid Avg Receivables Working Cap / Sales CA - CL higher % indicates more liquidity Net Sales Payables Turnover COGS @ payable period: 365 @ AP (do they pay their bills) Annual Turnover Operating Performance: How well management is operating the business Operating Efficiency Ratios: How management uses its assets and capital Activity ratios: sales per something Inventory Turnover COGS @ Inventory period 365 @ Inventory Inventory

Turnover Total Asset Turnover Net Sales high or low relative to industry? @ Tot Net Assets low: tie up too much assets Fixed Asset Turnover Net Sales utilization of fixed assets @ Net Fixed Assets hi: old depr equipment

Page 527: Cfa Ques Answer

Appendix A: 511

Equity Turnover Net Sales excludes CL & LT Debt Average Equity Receivables Turnover Net Annual Sales Avg collection period: 365 Avg Receivables Annual Turnover Operating Profitability: rate of profit on sales, % return on capital (How good is mgmt turning profits into sales) Note: Run down Income Statement and ratio to Sales (GP, EBIT, EBT, EAT) EBIT: Mgmt earns Tax: Government earns Interest: Banker’s earn EBT: Stockholder’s earn Gross Profit Margin Gross Profit (GP = Sls - COGS) Net Sales relative cost price position in industry? Operating Profit Margin Operating Profit (EBIT) Net Sales variability is business risk indicator Net Profit Margin Net Income NI = EAT Net Sales EBT Margin EBT before tax profit margin Net Sales Common Size Income Statement lists all expense and income items as a % of sales (Inc Stmt: / Sls) (Bal: / Sls or Tot Assets) Return on Total Capital Net Income + Interest Expense Debt, Pref. Stock, C Stock Average Total Capital return on all capital

employed Return on Total Equity Net Income ROE @ Total Equity Return on Owners Equity Net Income – Preferred Dividend @ Common Equity Return on Equity (ROE): DuPont System duPont formulation ROE = NI / Equity or EAT/Equity

Page 528: Cfa Ques Answer

512 Exhibit

3 Step ROE = NI = Net * Asset * Financial a.k.a.Equity Multiplier Equity Profit Turnover Leverage Margin NI * Sls * Assets Note:(Sls)(Assets) Sls Assets Equity (Sls)(Assets) 4 Step ROE = NI = Tax * Interest * Oper’g * Asset * Financial Equity Reten Burden Profit Turnover Leverage NI * EBT * EBIT * Sls * Assets EBT EBIT Sls Assets Equity 5 Step Equation = Oper * Total - Interest Financial * Tax Profit Asset Expense * Leverage Retention

Margin Turnover Rate Multiplier Rate EBIT * Sls - Int * Assets * (1 - T) Sls Assets Assets Equity Financial Risk: Uncertainty of returns to equity holders due to a firm’s use of fixed obligation debt

securities Debt / Equity remember firm value is Db + Eq Db=1, Eq=2, firm value = 3, D/E = ½ (not D/E=1/3, easy mistake in a hurry) LTD / LT Cap LTD = Long Term Debt LT Cap = Long Term Capital Total Debt / Total Capital Interest Coverage EBIT NI + Tax + Int Exp Interest Expense Int Exp (Note: Look for this on the exam Cash Flow / LTD Cash Flow / Total Debt

Page 529: Cfa Ques Answer

Appendix A: 513

Growth Analysis Retention rate earnings retained / total earnings (remember: growth = retention rate * ROE) ROE see above, profitability Total Asset Turnover see above, operational performance Total Assets / Equity note component of ROE Net Profit Margin see above, profitability Sustainable growth rate g = retention * ROE retention rate = 1 – (Oper Inc after taxes) Risk Analysis: uncertainty of income flows for the total firm and for sources of capital Business Risk: Uncertainty of income caused by a firm’s industry Variability of sales due to products, customers, production methods Business Risk Coefficient of variation of operating income CV = σ/mean Standard Deviation of Operating Earnings (OE) need 5<Thru> 10 yrs Mean Operating Earnings to compute coef of variation Sales Volatility Coefficient of Variation of Sales CV = σ/mean sd of sales prime determinant of earnings variability mean sales Operating Leverage %∆OE / %∆Sls or Σ | [%∆OE] / [%∆Sls] | / N employment of fixed production

costs direction of change not important, but relative size of the change is relevant OL = % change in operating earnings / % change in sales , calc from #’s not %

Page 530: Cfa Ques Answer

514 Practice Exam Features

FinancialExam Features

Using FinancialExams Quizzer In Brief .......................................................................................................................................515

Study Session..................................................................................................................514 Practice Exams..........................................................................................................................516 FinancialExams Quizzer Features ............................................................................................517

Online Help ......................................................................................................................517 Email: ...............................................................................................................................517 Telephone:.......................................................................................................................517 Changing subjects ...........................................................................................................518 Changing the number of questions..................................................................................518 Changing the subcategories............................................................................................518 Setting up your printer .....................................................................................................518 Printing questions ............................................................................................................518 Using FinancialExams Quizzer options...........................................................................519

The Adaptive Exam ...................................................................................................................519 How to take advantage of Adaptive testing: ....................................................................519 Take the Adaptive Exam .................................................................................................519 Reviewing an Adaptive Exam..........................................................................................520

Starting a Study Session ...........................................................................................................520 Checking your score during a test ...................................................................................521 Checking your performance ............................................................................................521 Checking your overall progress .......................................................................................521

Simulated Exam ........................................................................................................................521 Take the Simulated Exam ...............................................................................................521 Reviewing a Simulated Exam..........................................................................................522

FlashCard Option ......................................................................................................................522 Starting a FlashCard Session....................................................................................................523 How to use a FlashCards ..........................................................................................................523 Taking Notes .............................................................................................................................523

Take notes from a study session.....................................................................................523 Export your notes.............................................................................................................523

Page 531: Cfa Ques Answer

Practice Exam Features 515

Using FinancialExams Quizzer

In Brief The FinancialExams Quizzer exam software is designed to identify your personal areas of weakness relative to passing specific exam objectives. Once this software determines just what those weaknesses are, it will force you to continually face them until you can demonstrate mastery over them. Then it will find something else to haunt you with. To use this software successfully, follow these simple steps.

1) Spend some time getting familiar with the program. When you’re ready to settle down to work, clear the history (the button is on the main screen). Do not clear the history again until all of the following steps have been completed.

2) Take four (4) Adaptive Exams. During this process, you have no choices. The software is in control. It is using its internal logic to determine your strengths and weaknesses on a topic-by-topic basis. (Note: You don’t have to take all four at one sitting.)

3) After the four Adaptive Exams, look at your Historical Analysis (the button is on the main screen). This will give you a graphical presentation of how you have done cumulatively in each category. Let this be your guide as to which subcategories to begin studying. Start with your weakest and work your way up.

4) Begin taking Study Sessions on selected topics. Select the Category and Subcategories that you will be focusing on. You have two primary goals in Study Session mode.

a) See and answer every question on that topic at least once. Keep in mind that any question answered incorrectly will be in the very next Study Session you take on that topic. Once you are able to answer it correctly, it will get shuffled to the bottom of the deck. When you have cycled through available questions, correctly answered questions will begin to reappear.

b) Achieve a minimum score of at least 85% on each and every subcategory topic before moving on to the Simulated Exam. If you want to bounce around a little between subcategories a little, that’s okay. The program will remember where you left off. However, it’s important that you don’t cheat yourself on this step.

5) Take the Simulated Exam. Don’t be too disappointed if you don’t pass the first time. The software has been gaining knowledge about your weaknesses and has just done everything in its power to make you fail this exam. Aren’t you glad that wasn’t the real thing?

6) Clear your history and repeat the above steps two more times. That’s right… two more times. But now, achieve a minimum of 90% and 95% respectfully on each pass through before taking the Simulated Exam.

You will find that the number of Study Sessions and the amount of time necessary to achieve these scores will get smaller. The better you know the material, the quicker the whole process becomes. The rest of this document goes into more detail on using specific features of the FinancialExams Quizzer software. Spend some time getting to know this program. It has helped thousands of people achieve their certification goals. It can do the same for you.

Page 532: Cfa Ques Answer

516 Practice Exam Features

Study Session FinancialExams Quizzer tests your knowledge as you learn about new subjects through interactive quiz sessions. Study Session questions are selected from a single database for each session, dependent on the subcategory selected and the number of times each question has been previously answered correctly. In this way, questions you have answered correctly are not repeated until you have answered all the new questions. Questions that you have missed previously will reappear in later sessions and keep coming back to haunt you until you get the question correct. In addition, you can track your progress by displaying the number of questions you have answered with the Historical Analysis option. You can reset the progress tracking by clicking on the Clear History button. Each time a question is presented the answers are randomized so you will not memorize a pattern or letter that goes with the question. You will start to memorize the correct answer that goes with the question concept.

Practice Exams FinancialExams Quizzer also provides Adaptive and Simulated certification exams. Questions are chosen at random from the database. The Simulated Exam is a timed test that presents a similar number of questions as the real exam. A break from testing occurs at the mid-point of the exam. The Adaptive Exam presents a fixed number questions with a maximum time allotment. The Adaptive Exam is most helpful in identifying areas of weakness in the candidate’s knowledge of the exam objectives.

After you finish an exam, FinancialExams Quizzer displays your score and the passing score required for the FinancialExams Quizzer test. You may display the exam results of this specific exam from this menu. You may review each question, display the correct answer, identify a resource, link to an available electronic book and view an explanation for the answer.

Page 533: Cfa Ques Answer

Practice Exam Features 517

FinancialExams Quizzer Features

1. Each database contains 200 to 1200+ questions

2. Easy to install

3. Easy to upgrade 4. Multiple choice Style questions

5. Essay style questions 6. Fill-in-the-blank Style questions

7. Performance based questions 8. Flash Card Style questions 9. Questions randomized 10. Hot Spot Style questions 11. Print a category of questions 12. Drag/Drop/Mix/Match

questions 13. �Single module studies 14. Answers randomized 15. Simulation exam studies 16. Print one question 17. Instant exam feedback 18. Cheat key or Flash Card

option 19. Statistical analysis 20. Instant question feedback 21. Individual exam analysis 22. Adaptive exam studies 23. Font selection 24. Skills assessment 25. Graphics 26. Historical analysis 27. User Notes creation 28. Resizable screen 29. Some Links to Electronic Book

content 30. Most have Explanations

31. Free version updates via e-mail

32. References

Online Help This manual installs on your PC along with the FinancialExams Quizzer. To access it, select Contents from the Help pull-down menu. Additional help can be obtained via:

Email:

[email protected] [email protected] [email protected]

Telephone:

Toll Free: (888) 992-3131 International: (281) 992-3131 Fax: (281) 482-5390

Page 534: Cfa Ques Answer

518 Practice Exam Features

Changing subjects FinancialExams Quizzer provides several practice exams to test your knowledge. To change exams:

1. Click the Change Exam button in the Main window. 2. Select the exam for the test you want to run from the Select Exam window. 3. Click OK to change to the selected exam, or the Cancel to keep the current exam.

Changing the number of questions You can choose the number of questions presented in each quiz session. To change the number of questions:

1. Click on the box to the right of the Number of Questions field in the Main window. 2. Type a number of questions, between 1 and 250, in the Number of Questions field. 3. If the number of questions selected exceeds the number available in the chosen

subcategory, all that are available will be displayed.

Changing the subcategories Each FinancialExams Quizzer subject has a number of categories and subcategories. You can take a test on any one or any combination of the subcategories.

1. From the Subcategories frame, select the desired general category from the Categories drop down list.

2. Select the desired subcategory from the list box. To select multiple subcategories, hold down the CTRL key while clicking items in the list box.

Setting up your printer FinancialExams Quizzer allows you to customize your print jobs.

1. Select Options from the View pull-down menu. 2. Select the Printing Options radial button. The Printing Options window appears. 3. Select the options for your printer. 4. Click OK to exit and save changes.

Printing questions FinancialExams Quizzer allows you to print questions from your tests, with or without the correct answer(s) marked. To print the question(s), select the desired print option from the File pull-down menu in the Question window.

Page 535: Cfa Ques Answer

Practice Exam Features 519

Using FinancialExams Quizzer options FinancialExams Quizzer provides a number of additional options to customize your test. The following options are available:

1 Stop On Wrong Answers 2 Enable Cheat Key 3 Resizable Screen 4 Font Setting

To select an advanced option, select Options from the View pull-down menu. The Exam Preferences window appears.

Adaptive Exam Adaptive testing is a time saving option used to identify the candidate’s strengths and weaknesses. Before using Adaptive testing, clear historical Analysis. Before you learn about your subject using the Quizzer Study Sessions, you should take the Adaptive exams. This exam style does not simulate all of the exam environments that are found on certification exams. You cannot choose specific subcategories for the Adaptive exam and once a question has been answered you cannot go back to a previous question. You have a time limit in which to complete the adaptive exam. This time varies from subject to subject, although it is usually 15 to 25 questions in 30 minutes. When the time limit has been reached, your exam automatically ends.

How to take advantage of Adaptive testing:

1. Clear the Historical Analysis 2. Take four (4) adaptive exams in a row. Study the contents after each adaptive exam. 3. View the historical analysis to identify your strength and weakness at the end of the four

(4) adaptive exams. 4. Go back to reviewing using the Study Sessions by Sub Category 5. Repeat process until you are ready to pass the real exam.

Take the Adaptive Exam

1. Click the Adaptive Exam radial button from the Main window. 2. Click the Start button. The Adaptive Exam window appears. 3. Click the circle to the left of the correct answer. There may be more than one correct

answer. Text in the bottom left corner of the window instructs you to Choose the Best Answer (if there is only one answer) or Mark All Correct Answers (if there is more than one correct answer).

4. Click the Next button to continue.

Page 536: Cfa Ques Answer

520 Practice Exam Features

After the allotted time has elapsed, the exam exits to review mode. To quit the test at any time, click the Finish button. After you have completed the Adaptive exam, FinancialExams Quizzer displays your score and the passing score required for the test. Display your exam results by selecting Details You may review each question, display the correct answer, and view an explanation for the answer (if available).

Reviewing an Adaptive Exam After you have taken an Adaptive exam, you can review the questions, your answers, and the correct answers. You may only review your questions immediately after completing an Adaptive exam. To review your questions:

1. Click the Correct Answer button. 2. To see your answer, click the Your Answer button.

Starting a Study Session After you choose a subcategory to test yourself on, start the Study Session.

To start a study session:

1. Select the Study Session radial button. 2. Click the Start button. The Question window appears. Optionally, you can select Study

Session from the Start pull-down menu. Click the checkbox to the left of the correct answer. There may be more than one correct answer. Text in the bottom left corner of the window instructs you to Choose the Best Answer (if there is only one answer) or Mark All Correct Answers (if there is more than one correct answer).

3. Click the Next button to continue. FinancialExams Quizzer provides immediate feedback on your answer at the bottom of the window.

If you answered the question correctly, a new question appears. If you did not answer the question correctly, you may try to guess again or move to the next question. If you have selected the Prompt on Wrong Answers option, you have the following choices:

1. To try again, click the Try Again button. 2. To move to the next question after an incorrect guess, click the Next Question button. 3. View the correct answer 4. View the Explanation option

To quit the test at any time, click the Finish button.

Page 537: Cfa Ques Answer

Practice Exam Features 521

Checking your score during a test The X% button displays the current percentage of questions that have been answered correctly.

1. Click the X% button. A window appears displaying the number of questions that have been asked and the number of questions that have been answered correctly.

2. Click OK to return to the test.

Checking your performance Click the QID button. A window appears, displaying the number of times you have been asked this question and the number of times you have answered this question correctly.

Checking your overall progress

1. Click the Historical Analysis button from the Main window. The Historical Analysis window appears.

2. FinancialExams Quizzer displays your progress in each test using a graphical progress bar. Hold the cursor over the progress bar and a dialog box will open. The number of times you have answered questions in that category correctly as a percentage of the total number of questions is displayed.

3. To get detailed information on your performance on any subject, click and hold the progress bar for that subject. FinancialExams Quizzer displays the performance details in the bottom left hand corner of the screen.

Simulated Exam After you have learned about your subject using the Study Sessions, you can take a simulated exam. This exam simulates the exam environment that might be found on a certification exam. You cannot choose subcategories for a Simulated Exam. You have a fixed limit equal to that of the real exam to complete the Simulated Exam. When this time limit has been reached, your exam automatically ends.

Take the Simulated Exam

1. Click the Simulated Exam radial button from the Main window. 2. Click the Start button. The Simulated Exam window appears. 3. Click the circle to the left of the correct answer.

There may be more than one correct answer. Text in the bottom left corner of the window instructs you to Choose the Best Answer (if there is only one answer) or Mark All Correct Answers (if there is more than one correct answer).

Page 538: Cfa Ques Answer

522 Practice Exam Features

4. If you are unsure of the answer and wish to mark the question so you can return to it later, check the Mark box in the upper left hand corner. To review which questions you have marked, which you have answered, and which you have not answered, click the Review button.

5. Click the Next button to continue. After the allotted time for testing, the exam exits to review mode. To quit the test at any time, click the Finish button. After you have completed the Simulated Exam, FinancialExams Quizzer displays your score and the passing score required for the test. Display your exam results by selecting Details. You may review each question, display the correct answer, and view an explanation for the answer (if available).

Reviewing a Simulated Exam After you have taken a simulated exam, you can review the questions, your answers, and the correct answers. You may only review your questions immediately after a Simulated Exam.

To review your questions:

1. Click the Correct Answer button. 2. To see your answer, click the Your Answer button.

FlashCard Option After you have learned about your subject using the Study Sessions, Adaptive, and Simulated exam go to the Thinking Option. The FlashCard environment is very different and will help you more that you might believe on a certification exam. The Flash Card Option is very effect when used with Terminology or Glossary “Fill-in-the-Blank” style questions.

Read the displayed question, Think of the answer, Hit the F4 Function key to display the correct answer, Read the Answer, Go to the next question.

1. You should not view the answer until you have thought about the answer. 2. You have an unlimited time limit to think about each question. 3. Use this option a few days before the real exam to go through a lot of questions in a short

period of time. 4. The FlashCard option can be used as a review of all questions in the database. 5. Use the FlashCard option to provide Positive Feed Back with Correct Answers.

Page 539: Cfa Ques Answer

Practice Exam Features 523

Starting a FlashCard Session After you choose a subcategory to test yourself on, start FlashCards.

1. Select the FlashCards Session radial button. 2. Click the Start button. The Question window appears. Optionally, you can select

FlashCards Session from the Start pull-down menu. You will find no answers to check or any visible area to type your answer into.

3. This is the time to THINK of the answer. Pause a moment than hit the F4 function on your keyboard. (CheatKey)

4. Review the displayed answer. Did you get it correct? 5. When you are ready to proceed Click the Next button to continue.

How to use a FlashCards

1. Read the displayed question 2. Think of the answer 3. Hit the F4 Function key to display the correct answer 4. Read the Answer 5. Go to the next question.

To quit the test at any time, click the Finish button. Note: Some questions may ask for more than one answer: Bypass questions that state "Which of the Following" and "Select all that apply" etc.

Taking Notes While you are taking your exam, you may wish to write down notes about a particular question that you can use in later Study Sessions. FinancialExams Quizzer allows you to enter notes about each question. The notes will be permanently stored in the database for the subject you are studying. You can also use this feature to record any comments you have about a particular question and send it in for review. If you would like to print or view your notes, you can export your notes to a standard text editor.

Take notes from a study session Create your own information or reference.

1. Click the Notes button from the Question window. 2. Type your note. 3. Click OK to keep the note, or Cancel to delete the note.

Note: You cannot take notes during a Simulated or Adaptive session until you have finished the exam and have entered the Review mode.

Export your notes Click the Notes button from the Main window and export to Note Pad or any text editor you wish to use.

Page 540: Cfa Ques Answer

526 CFA Exam Forum

Special: Free Financial Forum Access: Visit http://www.financialcertification.com

FREE Demo at www.financialcertification.com/cfademo

Page 541: Cfa Ques Answer

ebooster 527

ebooster limited is the authorized distributor for TotalRecall Publications materials in Europe, the Middle East and Africa. We stock the full range of TRP products and charge at competitive prices, especially for volume orders. Come to us direct or order from your favorite distributors and retailers including Amazon.co.uk, Waterstones, Borders, Books Etc., Gardners and Bertram Books.

e/booster limited FAO: Chris MacNeil

502, trottenham Court Rd

London, England W1T 1JY

Office +44 (0) 207 993 2253

Fax +44 (0) 207 993 2253

email: [email protected]

Web Site www.ebooster.co.uk

Page 542: Cfa Ques Answer

 

 

Download Instructions The Financial Certification Center    (TFCC) version 4.0     Chartered Financial Analyst  (CFA) Level 1 Exam System Requirements: Windows 95 & 98, Windows NT, Windows 2000/XP, and Windows Server 2003 with

a minimum of 40 MB hard disk space and 64 MB RAM

The Center For Financial Certification, Inc. will help you accomplish your CFA Certifications. This state of

art software program is designed to cut your study time in half, and get you to a passing knowledge level in

the easiest and shortest amount of time possible. The program will adapt to you personally, and then lay out

a prioritized study plan that will visually show you your progress on a day to day basis. When the software

has recognized that you are at a passing level in each objective category, you're ready to sit for the exam.

It's really that easy!

Installation Instructions: To obtain the FE practice exam simply visit the engine download link.

TotalRecall CFA Level I Practice Exam and Engine Download * Click on  the  link or copy  it  to your browser   http://snipurl.com/2008CFAL1eWBZ  to download  the software.   When the computer prompts you to open or save, choose ʺsave this file to disk.ʺ Select the location for the download file CFA2008eWorkBook.ZIP License Key 1. FE-JV 2008_ConceptCheckQuestions 400 = 313112131882

2. Financial Terminology Evaluation = 322478807587

Visit:    http://www.financialexams.com/member_signup.php  http://www.financialcertification.com/forum/registration_rules.asp  or contact [email protected] for Download and Installation Instructions:      Please use InsidersChoice Workbook as the Subject.  Starting FinancialExams           To  start  the program  the next  time  (if  it doesn’t  start automatically), select FinancialExams from the Program’s menu. Assistance with running and using FE is available under the Help menu. 

FREE Content Downloads and Financial Forums Call 281-992-3398 888-992-3398

www.financialexams.com www.cfaexams.com

Good Luck with your certification! Your Book Registration Number is IC-94501-3000